You are on page 1of 207

CONSTITUTIONAL LAW 2 Case Digest

[Pick the date]


INTRODUCTION TO CONSTITUTIONAL LAW 2

THE NATURE OF THE CONSTITUTION AND ITS RELATION WITH THE COURTS

FRANCISCO VS. HOUSE OF REPRESENTATIVES


[415 SCRA 44; G.R. No. 160261; 10 Nov 2003]

Facts:

Impeachment proceedings were filed against Supreme Court Chief Justice Hilario Davide. The
justiciable controversy poised in front of the Court was the constitutionality of the subsequent filing
of a second complaint to controvert the rules of impeachment provided for by law

Issue:

Whether or Not the filing of the second impeachment complaint against Chief Justice Hilario G.
Davide, Jr. with the House of Representatives falls within the one year bar provided in the
Constitution and whether the resolution thereof is a political question – has resulted in a political
crisis.

Held:

In any event, it is with the absolute certainty that our Constitution is sufficient to address all the
issues which this controversy spawns that this Court unequivocally pronounces, at the first instance,
that the feared resort to extra-constitutional methods of resolving it is neither necessary nor legally
permissible. Both its resolution and protection of the public interest lie in adherence to, not
departure from, the Constitution.
In passing over the complex issues arising from the controversy, this Court is ever mindful of the
essential truth that the inviolate doctrine of separation of powers among the legislative, executive or
judicial branches of government by no means prescribes for absolute autonomy in the discharge by
each of that part of the governmental power assigned to it by the sovereign people.

At the same time, the corollary doctrine of checks and balances which has been carefully calibrated
by the Constitution to temper the official acts of each of these three branches must be given effect
without destroying their indispensable co-equality. There exists no constitutional basis for the
contention that the exercise of judicial review over impeachment proceedings would upset the
system of checks and balances. Verily, the Constitution is to be interpreted as a whole and "one
section is not to be allowed to defeat another." Both are integral components of the calibrated
system of independence and interdependence that insures that no branch of government act
beyond the powers assigned to it by the Constitution.

When suing as a citizen, the interest of the petitioner assailing the constitutionality of a statute must
be direct and personal. He must be able to show, not only that the law or any government act is
invalid, but also that he sustained or is in imminent danger of sustaining some direct injury as a
result of its enforcement, and not merely that he suffers thereby in some indefinite way. It must
appear that the person complaining has been or is about to be denied some right or privilege to
which he is lawfully entitled or that he is about to be subjected to some burdens or penalties by
reason of the statute or act complained of. In fine, when the proceeding involves the assertion of a
public right, the mere fact that he is a citizen satisfies the requirement of personal interest.

In the case of a taxpayer, he is allowed to sue where there is a claim that public funds are illegally
disbursed, or that public money is being deflected to any improper purpose, or that there is a
wastage of public funds through the enforcement of an invalid or unconstitutional law. Before he can
Page 2
invoke the power of judicial review, however, he must specifically prove that he has sufficient
interest in preventing the illegal expenditure of money raised by taxation and that he would sustain a
direct injury as a result of the enforcement of the questioned statute or contract. It is not sufficient
that he has merely a general interest common to all members of the public.

At all events, courts are vested with discretion as to whether or not a taxpayer's suit should be
entertained. This Court opts to grant standing to most of the petitioners, given their allegation that
any impending transmittal to the Senate of the Articles of Impeachment and the ensuing trial of the
Chief Justice will necessarily involve the expenditure of public funds.

As for a legislator, he is allowed to sue to question the validity of any official action which he claims
infringes his prerogatives as a legislator. Indeed, a member of the House of Representatives has
standing to maintain inviolate the prerogatives, powers and privileges vested by the Constitution in
his office.83

The framers of the Constitution also understood initiation in its ordinary meaning. Thus when a
proposal reached the floor proposing that "A vote of at least one-third of all the Members of the
House shall be necessary… to initiate impeachment proceedings," this was met by a proposal to
delete the line on the ground that the vote of the House does not initiate impeachment proceeding
but rather the filing of a complaint does.

To the argument that only the House of Representatives as a body can initiate impeachment
proceedings because Section 3 (1) says "The House of Representatives shall have the exclusive
power to initiate all cases of impeachment," This is a misreading of said provision and is contrary to
the principle of reddendo singula singulis by equating "impeachment cases" with "impeachment
proceeding."

Having concluded that the initiation takes place by the act of filing and referral or endorsement of
the impeachment complaint to the House Committee on Justice or, by the filing by at least one-third
of the members of the House of Representatives with the Secretary General of the House, the
meaning of Section 3 (5) of Article XI becomes clear. Once an impeachment complaint has been
initiated, another impeachment complaint may not be filed against the same official within a one
year period.

The Court in the present petitions subjected to judicial scrutiny and resolved on the merits only the
main issue of whether the impeachment proceedings initiated against the Chief Justice
transgressed the constitutionally imposed one-year time bar rule. Beyond this, it did not go about
assuming jurisdiction where it had none, nor indiscriminately turn justiciable issues out of decidedly
political questions. Because it is not at all the business of this Court to assert judicial dominance
over the other two great branches of the government.

No one is above the law or the Constitution. This is a basic precept in any legal system which
recognizes equality of all men before the law as essential to the law's moral authority and that of its
agents to secure respect for and obedience to its commands. Perhaps, there is no other
government branch or instrumentality that is most zealous in protecting that principle of legal
equality other than the Supreme Court which has discerned its real meaning and ramifications
through its application to numerous cases especially of the high-profile kind in the annals of
jurisprudence. The Chief Justice is not above the law and neither is any other member of this Court.
But just because he is the Chief Justice does not imply that he gets to have less in law than
anybody else. The law is solicitous of every individual's rights irrespective of his station in life.

Thus, the Rules of Procedure in Impeachment Proceedings which were approved by the House of
Representatives on November 28, 2001 are unconstitutional. Consequently, the second
impeachment complaint against Chief Justice Hilario G. Davide, Jr is barred under paragraph 5,
section 3 of Article XI of the Constitution.

MANILA PRINCE HOTEL VS. GSIS


Page 3
[267 SCRA 408; G.R. No. 122156; 3 Feb 1997]

Facts:

The controversy arose when respondent Government Service Insurance System (GSIS), pursuant
to the privatization program of the Philippine Government under Proclamation No. 50 dated 8
December 1986, decided to sell through public bidding 30% to 51% of the issued and outstanding
shares of respondent Manila Hotel Corporation. In a close bidding held on 18 September 1995 only
two (2) bidders participated: petitioner Manila Prince Hotel Corporation, a Filipino corporation, which
offered to buy 51% of the MHC or 15,300,000 shares at P41.58 per share, and Renong Berhad, a
Malaysian firm, with ITT-Sheraton as its hotel operator, which bid for the same number of shares at
P44.00 per share, or P2.42 more than the bid of petitioner.

Pending the declaration of Renong Berhad as the winning bidder/strategic partner and the
execution of the necessary contracts, matched the bid price of P44.00 per share tendered by
Renong Berhad.
On 17 October 1995, perhaps apprehensive that respondent GSIS has disregarded the tender of
the matching bid and that the sale of 51% of the MHC may be hastened by respondent GSIS and
consummated with Renong Berhad, petitioner came to this Court on prohibition and mandamus.

In the main, petitioner invokes Sec. 10, second par., Art. XII, of the 1987 Constitution and submits
that the Manila Hotel has been identified with the Filipino nation and has practically become a
historical monument which reflects the vibrancy of Philippine heritage and culture. It is a proud
legacy of an earlier generation of Filipinos who believed in the nobility and sacredness of
independence and its power and capacity to release the full potential of the Filipino people . To all
intents and purposes, it has become a part of the national patrimony. 6 Petitioner also argues that
since 51% of the shares of the MHC carries with it the ownership of the business of the hotel which
is owned by respondent GSIS, a government-owned and controlled corporation, the hotel business
of respondent GSIS being a part of the tourism industry is unquestionably a part of the national
economy.

Issue:

Whether or Not the sale of Manila Hotel to Renong Berhad is violative of the Constitutional provision
of Filipino First policy and is therefore null and void.

Held:

The Manila Hotel or, for that matter, 51% of the MHC, is not just any commodity to be sold to the
highest bidder solely for the sake of privatization. The Manila Hotel has played and continues to
play a significant role as an authentic repository of twentieth century Philippine history and culture.
This is the plain and simple meaning of the Filipino First Policy provision of the Philippine
Constitution. And this Court, heeding the clarion call of the Constitution and accepting the duty of
being the elderly watchman of the nation, will continue to respect and protect the sanctity of the
Constitution. It was thus ordered that GSIS accepts the matching bid of petitioner MANILA PRINCE
HOTEL CORPORATION to purchase the subject 51% of the shares of the Manila Hotel Corporation
at P44.00 per share and thereafter to execute the necessary clearances and to do such other acts
and deeds as may be necessary for purpose.

PEOPLE VS. POMAR


[46 Phil 126; G.R. No. L-22008; 3 Nov 1924]

Facts:

Macaria Fajardo was an employee of La Flor de la Isabela, a Tobacco factory. She was granted a
vacation leave, by reason of her pregnancy, which commenced on the 16 th of July 1923. According
to Fajardo, during that time, she was not given the salary due her in violation of the provisions of Act
Page 4
No. 3071. Fajardo filed a criminal complaint based on Section 13 and 15 of said Act against the
manager of the tobacco Factory, Julio Pomar, herein defendant. The latter, on the other hand,
claims that the facts in the complaint did not constitute an offense and further alleges that the
aforementioned provisions of Act No. 3071 was unconstitutional. Section 13, Act No. 3071 provides
that, “Every person, firm or corporation owning or managing a factory, shop or place of labor of any
description shall be obliged to grant to any woman employed by it as laborer who may be pregnant,
thirty days vacation with pay before and another thirty days after confinement: Provided, That the
employer shall not discharge such laborer without just cause, under the penalty of being required to
pay to her wages equivalent to the total of two months counting from the day of her discharge.”
Section 15 of the same Act provides for the penalty of any violation of section 13. The latter was
enacted by the legislature in the exercise of its supposed Police Power with the purpose of
safeguarding the health of pregnant women laborers in "factory, shop or place of labor of any
description," and of insuring to them, to a certain extent, reasonable support for one month before
and one month after their delivery. The trial court rendered a decision in favor of plaintiff,
sentencing the defendant to pay the fine of fifty pesos and in case of insolvency, to suffer subsidiary
imprisonment. Hence, the case was raised to the Court of Appeals which affirmed the former
decision.

Issue:

Whether or not Section 13 of Act No. 3071 is unconstitutional.

Whether or not the promulgation of the questioned provision was a valid exercise of Police Power.

Held:

The Supreme Court declared Section 13 of Act No. 3071 to be unconstitutional for being violative
or restrictive of the right of the people to freely enter into contracts for their affairs. It has been
decided several times, that the right to contract about one's affairs is a part of the liberty of the
individual, protected by the "due process of law" clause of the constitution. The contracting parties
may establish any agreements, terms, and conditions they may deem advisable, provided they are
not contrary to law, morals or public policy

The police power of the state is a very broad and expanding power. The police power may
encompass every law for the restraint and punishment of crimes, for the preservation of the public
peace, health, and morals. But that power cannot grow faster than the fundamental law of the state,
nor transcend or violate the express inhibition of the constitution. The Police Power is subject to and
is controlled by the paramount authority of the constitution of the state, and will not be permitted to
violate rights secured or guaranteed by the latter.

LAMBINO VS. COMELEC


[G.R. No. 174153; 25 Oct 2006]

Facts:

Petitioners (Lambino group) commenced gathering signatures for an initiative petition to change the
1987 constitution, they filed a petition with the COMELEC to hold a plebiscite that will ratify their
initiative petition under RA 6735. Lambino group alleged that the petition had the support of 6M
individuals fulfilling what was provided by art 17 of the constitution. Their petition changes the 1987
constitution by modifying sections 1-7 of Art 6 and sections 1-4 of Art 7 and by adding Art 18. the
proposed changes will shift the present bicameral- presidential form of government to unicameral-
parliamentary. COMELEC denied the petition due to lack of enabling law governing initiative
petitions and invoked the Santiago Vs. Comelec ruling that RA 6735 is inadequate to implement the
initiative petitions.

Issue:

Page 5
Whether or Not the Lambino Group’s initiative petition complies with Section 2, Article XVII of the
Constitution on amendments to the Constitution through a people’s initiative.

Whether or Not this Court should revisit its ruling in Santiago declaring RA 6735 “incomplete,
inadequate or wanting in essential terms and conditions” to implement the initiative clause on
proposals to amend the Constitution.

Whether or Not the COMELEC committed grave abuse of discretion in denying due course to the
Lambino Group’s petition.

Held:

According to the SC the Lambino group failed to comply with the basic requirements for conducting
a people’s initiative. The Court held that the COMELEC did not grave abuse of discretion on
dismissing the Lambino petition.

1. The Initiative Petition Does Not Comply with Section 2, Article XVII of the Constitution on Direct
Proposal by the People
The petitioners failed to show the court that the initiative signer must be informed at the time of
the signing of the nature and effect, failure to do so is “deceptive and misleading” which renders
the initiative void.

2. The Initiative Violates Section 2, Article XVII of the Constitution Disallowing Revision through
Initiatives
The framers of the constitution intended a clear distinction between “amendment” and “revision,
it is intended that the third mode of stated in sec 2 art 17 of the constitution may propose only
amendments to the constitution. Merging of the legislative and the executive is a radical change,
therefore a constitutes a revision.

3. A Revisit of Santiago v. COMELEC is Not Necessary


Even assuming that RA 6735 is valid, it will not change the result because the present petition
violated Sec 2 Art 17 to be a valid initiative, must first comply with the constitution before
complying with RA 6735

Petition is dismissed.

SANTIAGO VS. COMELEC


[270 SCRA 106; G.R. No.127325; 19 Mar 1997]

Facts:

Private respondent Atty. Jesus Delfin, president of People’s Initiative for Reforms, Modernization
and Action (PIRMA), filed with COMELEC a petition to amend the constitution to lift the term limits of
elective officials, through People’s Initiative. He based this petition on Article XVII, Sec. 2 of the
1987 Constitution, which provides for the right of the people to exercise the power to directly
propose amendments to the Constitution. Subsequently the COMELEC issued an order directing
the publication of the petition and of the notice of hearing and thereafter set the case for hearing. At
the hearing, Senator Roco, the IBP, Demokrasya-Ipagtanggol ang Konstitusyon, Public Interest Law
Center, and Laban ng Demokratikong Pilipino appeared as intervenors-oppositors. Senator Roco
filed a motion to dismiss the Delfin petition on the ground that one which is cognizable by the
COMELEC. The petitioners herein Senator Santiago, Alexander Padilla, and Isabel Ongpin filed
this civil action for prohibition under Rule 65 of the Rules of Court against COMELEC and the Delfin
petition rising the several arguments, such as the following: (1) The constitutional provision on
people’s initiative to amend the constitution can only be implemented by law to be passed by
Congress. No such law has been passed; (2) The people’s initiative is limited to amendments to the
Constitution, not to revision thereof. Lifting of the term limits constitutes a revision, therefore it is
outside the power of people’s initiative. The Supreme Court granted the Motions for Intervention.
Page 6
Issue:

Whether or not Sec. 2, Art. XVII of the 1987 Constitution is a self-executing provision.

Whether or not COMELEC Resolution No. 2300 regarding the conduct of initiative on amendments
to the Constitution is valid, considering the absence in the law of specific provisions on the conduct
of such initiative.

Whether the lifting of term limits of elective officials would constitute a revision or an amendment of
the Constitution.

Held:

Sec. 2, Art XVII of the Constitution is not self executory, thus, without implementing legislation the
same cannot operate. Although the Constitution has recognized or granted the right, the people
cannot exercise it if Congress does not provide for its implementation.

The portion of COMELEC Resolution No. 2300 which prescribes rules and regulations on the
conduct of initiative on amendments to the Constitution, is void. It has been an established rule that
what has been delegated, cannot be delegated (potestas delegata non delegari potest). The
delegation of the power to the COMELEC being invalid, the latter cannot validly promulgate rules
and regulations to implement the exercise of the right to people’s initiative.

The lifting of the term limits was held to be that of a revision, as it would affect other provisions of
the Constitution such as the synchronization of elections, the constitutional guarantee of equal
access to opportunities for public service, and prohibiting political dynasties. A revision cannot be
done by initiative. However, considering the Court’s decision in the above Issue, the issue of
whether or not the petition is a revision or amendment has become academic.

GONZALES VS. COMELEC


[21 SCRA 774; G.R. No. L-28196; 9 Nov 1967]

Facts:

The case is an original action for prohibition, with preliminary injunction.

The main facts are not disputed. On March 16, 1967, the Senate and the House of Representatives
passed the following resolutions:

1. R. B. H. (Resolution of Both Houses) No. 1, proposing that Section 5, Article VI, of the
Constitution of the Philippines, be amended so as to increase the membership of the House of
Representatives from a maximum of 120, as provided in the present Constitution, to a maximum of
180, to be apportioned among the several provinces as nearly as may be according to the number
of their respective inhabitants, although each province shall have, at least, one (1) member;

2. R. B. H. No. 2, calling a convention to propose amendments to said Constitution, the convention


to be composed of two (2) elective delegates from each representative district, to be "elected in the
general elections to be held on the second Tuesday of November, 1971;" and

3. R. B. H. No. 3, proposing that Section 16, Article VI, of the same Constitution, be amended so as
to authorize Senators and members of the House of Representatives to become delegates to the
aforementioned constitutional convention, without forfeiting their respective seats in Congress.

Subsequently, Congress passed a bill, which, upon approval by the President, on June 17, 1967,
became Republic Act No. 4913, providing that the amendments to the Constitution proposed in the

Page 7
aforementioned Resolutions No. 1 and 3 be submitted, for approval by the people, at the general
elections which shall be held on November 14, 1967.

Issue:

Whether or Not a Resolution of Congress, acting as a constituent assembly, violates the


Constitution.

Held:

Inasmuch as there are less than eight (8) votes in favor of declaring Republic Act 4913 and R. B. H.
Nos. 1 and 3 unconstitutional and invalid, the petitions in these two (2) cases must be, as they are
hereby, dismiss and the writs therein prayed for denied, without special pronouncement as to costs.
It is so ordered.

As a consequence, the title of a de facto officer cannot be assailed collaterally. It may not be
contested except directly, by quo warranto proceedings. Neither may the validity of his acts be
questioned upon the ground that he is merely a de facto officer. And the reasons are obvious: (1) it
would be an indirect inquiry into the title to the office; and (2) the acts of a de facto officer, if within
the competence of his office, are valid, insofar as the public is concerned.

"The judicial department is the only constitutional organ which can be called upon to determine the
proper allocation of powers between the several departments and among the integral or constituent
units thereof."

Article XV of the Constitution provides:

. . . The Congress in joint session assembled, by a vote of three-fourths of all the


Members of the Senate and of the House of Representatives voting separately, may
propose amendments to this Constitution or call a contention for that purpose. Such
amendments shall be valid as part of this Constitution when approved by a majority
of the votes cast at an election at which the amendments are submitted to the people
for their ratification.

From our viewpoint, the provisions of Article XV of the Constitution are satisfied so long as the
electorate knows that R. B. H. No. 3 permits Congressmen to retain their seats as legislators, even
if they should run for and assume the functions of delegates to the Convention.

SANIDAD VS. COMELEC


[78 SCRA 333; G.R. No. 90878; 29 Jan 1990]

Facts:

This is a petition for certiorari assailing the constitutionality of Section 19 of Comelec Resolution No.
2167 on the ground that it violates the constitutional guarantees of the freedom of expression and of
the press. On October 23, 1989, Republic Act No. 6766, entitled "AN ACT PROVIDING FOR AN
ORGANIC ACT FOR THE CORDILLERA AUTONOMOUS REGION" was enacted into law.
Pursuant to said law, the City of Baguio and the Cordilleras which consist of the provinces of
Benguet, Mountain Province, Ifugao, Abra and Kalinga-Apayao, all comprising the Cordillera
Autonomous Region, shall take part in a plebiscite for the ratification of said Organic Act originally
scheduled last December 27, 1989 which was, however, reset to January 30, 1990 by virtue of
Comelec Resolution No. 2226 dated December 27, 1989. The Commission on Elections, by virtue
of the power vested by the 1987 Constitution, the Omnibus Election Code (BP 881), said R.A. 6766
and other pertinent election laws, promulgated Resolution No. 2167, to govern the conduct of the
plebiscite on the said Organic Act for the Cordillera Autonomous Region. In a petition dated
November 20, 1989, herein petitioner Pablito V. Sanidad, who claims to be a newspaper columnist
of the "OVERVIEW" for the BAGUIO MIDLAND COURIER, a weekly newspaper circulated in the
Page 8
City of Baguio and the Cordilleras, assailed the constitutionality of Section 19 of Comelec
Resolution No. 2167, which provides:

Section 19. Prohibition on columnists, commentators or announcers. — During the


plebiscite campaign period, on the day before and on the plebiscite day, no mass
media columnist, commentator, announcer or personality shall use his column or
radio or television time to campaign for or against the plebiscite Issue.

It is alleged by petitioner that said provision is void and unconstitutional because it violates
the constitutional guarantees of the freedom of expression and of the press enshrined in the
Constitution. Unlike a regular news reporter or news correspondent who merely reports the
news, petitioner maintains that as a columnist, his column obviously and necessarily
contains and reflects his opinions, views and beliefs on any issue or subject about which he
writes. Petitioner likewise maintains that if media practitioners were to express their views,
beliefs and opinions on the issue submitted to a plebiscite, it would in fact help in the
government drive and desire to disseminate information, and hear, as well as ventilate, all
sides of the issue.

Issue:

Whether or not Section 19 of Comelec Resolution No. 2167 is unconstitutional.

Held:

The Supreme Court ruled that Section 19 of Comelec Resolution No. 2167 is unconstitutional. It is
clear from Art. IX-C of the 1987 Constitution that what was granted to the Comelec was the power
to supervise and regulate the use and enjoyment of franchises, permits or other grants issued for
the operation of transportation or other public utilities, media of communication or information to the
end that equal opportunity, time and space, and the right to reply, including reasonable, equal rates
therefor, for public information campaigns and forums among candidates are ensured. The evil
sought to be prevented by this provision is the possibility that a franchise holder may favor or give
any undue advantage to a candidate in terms of advertising space or radio or television time. This is
also the reason why a "columnist, commentator, announcer or personality, who is a candidate for
any elective office is required to take a leave of absence from his work during the campaign period
(2nd par. Section 11(b) R.A. 6646). It cannot be gainsaid that a columnist or commentator who is
also a candidate would be more exposed to the voters to the prejudice of other candidates unless
required to take a leave of absence.

However, neither Article IX-C of the Constitution nor Section 11 (b), 2nd par. of R.A. 6646 can be
construed to mean that the Comelec has also been granted the right to supervise and regulate the
exercise by media practitioners themselves of their right to expression during plebiscite periods.
Media practitioners exercising their freedom of expression during plebiscite periods are neither the
franchise holders nor the candidates. In fact, there are no candidates involved in a plebiscite.
Therefore, Section 19 of Comelec Resolution No. 2167 has no statutory basis.

Plebiscite Issue are matters of public concern and importance. The people's right to be informed
and to be able to freely and intelligently make a decision would be better served by access to an
unabridged discussion of the Issue, including the forum. The people affected by the Issue presented
in a plebiscite should not be unduly burdened by restrictions on the forum where the right to
expression may be exercised. Comelec spaces and Comelec radio time may provide a forum for
expression but they do not guarantee full dissemination of information to the public concerned
because they are limited to either specific portions in newspapers or to specific radio or television
times.

The instant petition is GRANTED. Section 19 of Comelec Resolution No. 2167 is declared null and
void and unconstitutional.

Page 9
BONDOC VS. PINEDA
[201 SCRA 792; G.R. No. 97710; 26 Sep 1991]

Facts:

In the elections held on May 11, 1987, Marciano Pineda of the LDP and Emigdio Bondoc of the NP
were candidates for the position of Representative for the Fourth District of Pampanga. Pineda was
proclaimed winner. Bondoc filed a protest in the House of Representatives Electoral Tribunal
(HRET), which is composed of 9 members, 3 of whom are Justices of the SC and the remaining 6
are members of the House of Representatives (5 members belong to the LDP and 1 member is
from the NP). Thereafter, a decision had been reached in which Bondoc won over Pineda.
Congressman Camasura of the LDP voted with the SC Justices and Congressman Cerilles of the
NP to proclaim Bondoc the winner of the contest.

On the eve of the promulgation of the Bondoc decision, Congressman Camasura


received a letter informing him that he was already expelled from the LDP for allegedly helping to
organize the Partido Pilipino of Eduardo Cojuangco and for allegedly inviting LDP members in
Davao Del Sur to join said political party. On the day of the promulgation of the decision, the
Chairman of HRET received a letter informing the Tribunal that on the basis of the letter from the
LDP, the House of Representatives decided to withdraw the nomination and rescind the election of
Congressman Camasura to the HRET.

Issue:

Whether or not the House of Representatives, at the request of the dominant political party therein,
may change that party’s representation in the HRET to thwart the promulgation of a decision freely
reached by the tribunal in an election contest pending therein

Held:

The purpose of the constitutional convention creating the Electoral Commission was to provide an
independent and impartial tribunal for the determination of contests to legislative office, devoid of
partisan consideration.

As judges, the members of the tribunal must be non-partisan. They must discharge their functions
with complete detachment, impartiality and independence even independence from the political
party to which they belong. Hence, disloyalty to party and breach of party discipline are not valid
grounds for the expulsion of a member of the tribunal. In expelling Congressman Camasura from
the HRET for having cast a “conscience vote” in favor of Bondoc, based strictly on the result of the
examination and appreciation of the ballots and the recount of the votes by the tribunal, the House
of Representatives committed a grave abuse of discretion, an injustice and a violation of the
Constitution. Its resolution of expulsion against Congressman Camasura is, therefore, null and
void.

Another reason for the nullity of the expulsion resolution of the House of Representatives is that it
violates Congressman Camasura’s right to security of tenure. Members of the HRET, as sole judge
of congressional election contests, are entitled to security of tenure just as members of the Judiciary
enjoy security of tenure under the Constitution. Therefore, membership in the HRET may not be
terminated except for a just cause, such as, the expiration of the member’s congressional term of
office, his death, permanent disability, resignation from the political party he represents in the
tribunal, formal affiliation with another political party or removal for other valid cause. A member
may not be expelled by the House of Representatives for party disloyalty, short of proof that he has
formally affiliated with another

MIRASOL VS CA
[351 SCRA 44; G.R. No. 128448; 1 Feb 2001]

Page 10
Facts:

The Mirasols are sugarland owners and planters. Philippine National Bank (PNB) financed the
Mirasols' sugar production venture FROM 1973-1975 under a crop loan financing scheme. The
Mirasols signed Credit Agreements, a Chattel Mortgage on Standing Crops, and a Real Estate
Mortgage in favor of PNB. The Chattel Mortgage empowered PNB to negotiate and sell the latter's
sugar and to apply the proceeds to the payment of their obligations to it.

President Marcos issued PD 579 in November, 1974 authorizing Philippine Exchange Co., Inc.
(PHILEX) to purchase sugar allocated for export and authorized PNB to finance PHILEX's
purchases. The decree directed that whatever profit PHILEX might realize was to be remitted to the
government. Believing that the proceeds were more than enough to pay their obligations, petitioners
asked PNB for an accounting of the proceeds which it ignored. Petitioners continued to avail of
other loans from PNB and to make unfunded withdrawals from their accounts with said bank. PNB
asked petitioners to settle their due and demandable accounts. As a result, petitioners, conveyed to
PNB real properties by way of dacion en pago still leaving an unpaid amount. PNB proceeded to
extrajudicially foreclose the mortgaged properties. PNB still had a deficiency claim.

Petitioners continued to ask PNB to account for the proceeds, insisting that said proceeds, if
properly liquidated, could offset their outstanding obligations. PNB remained adamant in its stance
that under P.D. No. 579, there was nothing to account since under said law, all earnings from the
export sales of sugar pertained to the National Government.

On August 9, 1979, the Mirasols filed a suit for accounting, specific performance, and damages
against PNB.

Issue:

Whether or not the Trial Court has jurisdiction to declare a statute unconstitutional without notice to
the Solicitor General where the parties have agreed to submit such issue for the resolution of the
Trial Court.

Whether PD 579 and subsequent issuances thereof are unconstitutional.

Whether or not said PD is subject to judicial review.


Held:

It is settled that Regional Trial Courts have the authority and jurisdiction to consider the
constitutionality of a statute, presidential decree, or executive order. The Constitution vests the
power of judicial review or the power to declare a law, treaty, international or executive agreement,
presidential decree, order, instruction, ordinance, or regulation not only in this Court, but in all
Regional Trial Courts.

The purpose of the mandatory notice in Rule 64, Section 3 is to enable the Solicitor General to
decide whether or not his intervention in the action assailing the validity of a law or treaty is
necessary. To deny the Solicitor General such notice would be tantamount to depriving him of his
day in court. We must stress that, contrary to petitioners' stand, the mandatory notice requirement is
not limited to actions involving declaratory relief and similar remedies. The rule itself provides that
such notice is required in "any action" and not just actions involving declaratory relief. Where there
is no ambiguity in the words used in the rule, there is no room for construction. 15 In all actions
assailing the validity of a statute, treaty, presidential decree, order, or proclamation, notice to the
Solicitor General is mandatory.

Petitioners contend that P.D. No. 579 and its implementing issuances are void for violating the due
process clause and the prohibition against the taking of private property without just compensation.
Petitioners now ask this Court to exercise its power of judicial review.

Page 11
Jurisprudence has laid down the following requisites for the exercise of this power: First, there must
be before the Court an actual case calling for the exercise of judicial review. Second, the question
before the Court must be ripe for adjudication. Third, the person challenging the validity of the act
must have standing to challenge. Fourth, the question of constitutionality must have been raised at
the earliest opportunity, and lastly, the issue of constitutionality must be the very lis mota of the
case.

DUMLAO VS. COMELEC


[95 SCRA 392; G.R. No.L-52245; 22 Jan 1980]

Facts:

Petitioner Dumlao questions the constitutionality of Sec. 4 of Batas Pambansa Blg 52 as


discriminatory and contrary to equal protection and due process guarantees of the Constitution.
Sec. 4 provides that any retired elective provincial or municipal official who has received payments
of retirement benefits and shall have been 65 years of age at the commencement of the term of
office to which he seeks to be elected, shall not be qualified to run for the same elective local office
from which he has retired. According to Dumlao, the provision amounts to class legislation.
Petitioners Igot and Salapantan Jr. also assail the validity of Sec. 4 of Batas Pambansa Blg 52,
which states that any person who has committed any act of disloyalty to the State, including those
amounting to subversion, insurrection, rebellion, or other similar crimes, shall not be qualified for
any of the offices covered by the act, or to participate in any partisan activity therein: provided that a
judgment of conviction of those crimes shall be conclusive evidence of such fact and the filing of
charges for the commission of such crimes before a civil court or military tribunal after preliminary
investigation shall be prima facie evidence of such fact.

Issue:

Whether or Not the aforementioned statutory provisions violate the Constitution and thus, should be
declared null and void

Whether or not the requisites of judicial review are complied with

Held:

No constitutional question will be heard and decided by the Court unless there is compliance with
the requisites of a judicial inquiry, which are: 1) There must be an actual case or controversy; 2) The
question of constitutionality must be raised by the proper party; 3) The constitutional question must
be raised at the earliest possible opportunity; and 4) The decision of the constitutional question
must be necessary to the determination of the case itself.

As to (1), Dumlao has not been adversely affected by the application of the provision. His question
is posed merely in the abstract, and without the benefit of a detailed factual record. As to (2), neither
Igot nor Salapantan has been charged with acts of loyalty to the State, nor disqualified from being
candidates for local elective positions. They have no personal nor substantial interest at stake. Igot
and Salapantan have institute the case as a taxpayer’s suit, but the institution of a taxpayer’s suit
per se is no assurance of judicial review. As to (4), there is no cause of action in this particular case.
Therefore, the necessity for resolving the issue of constitutionality is absent.

In regards to the unconstitutionality of the provisions, Sec. 4 of BP Blg 52 remains constitutional and
valid. The constitutional guarantee of equal protection of the laws is subject to rational classification.
One class can be treated differently from another class. In this case, employees 65 years of age are
classified differently from younger employees. The purpose of the provision is to satisfy the “need
for new blood” in the workplace. In regards to the second paragraph of Sec. 4, it should be declared
null and void for being violative of the constitutional presumption of innocence guaranteed to an
accused.

Page 12
LACSON VS. PEREZ
[357 SCRA 756; G.R. No. 147780 ;10 May 2001]

Facts:

President Macapagal-Arroyo declared a State of Rebellion (Proclamation No. 38) on May 1, 2001
as well as General Order No. 1 ordering the AFP and the PNP to suppress the rebellion in the NCR.
Warrantless arrests of several alleged leaders and promoters of the “rebellion” were thereafter
effected. Petitioner filed for prohibition, injunction, mandamus and habeas corpus with an
application for the issuance of temporary restraining order and/or writ of preliminary injunction.
Petitioners assail the declaration of Proc. No. 38 and the warrantless arrests allegedly effected by
virtue thereof. Petitioners furthermore pray that the appropriate court, wherein the information
against them were filed, would desist arraignment and trial until this instant petition is resolved.
They also contend that they are allegedly faced with impending warrantless arrests and unlawful
restraint being that hold departure orders were issued against them.

Issue:

Whether or Not Proclamation No. 38 is valid, along with the warrantless arrests and hold departure
orders allegedly effected by the same.

Held:

President Macapagal-Arroyo ordered the lifting of Proc. No. 38 on May 6, 2006, accordingly the
instant petition has been rendered moot and academic. Respondents have declared that the
Justice Department and the police authorities intend to obtain regular warrants of arrests from the
courts for all acts committed prior to and until May 1, 2001. Under Section 5, Rule 113 of the Rules
of Court, authorities may only resort to warrantless arrests of persons suspected of rebellion in
suppressing the rebellion if the circumstances so warrant, thus the warrantless arrests are not
based on Proc. No. 38. Petitioner’s prayer for mandamus and prohibition is improper at this time
because an individual warrantlessly arrested has adequate remedies in law: Rule 112 of the Rules
of Court, providing for preliminary investigation, Article 125 of the Revised Penal Code, providing for
the period in which a warrantlessly arrested person must be delivered to the proper judicial
authorities, otherwise the officer responsible for such may be penalized for the delay of the same. If
the detention should have no legal ground, the arresting officer can be charged with arbitrary
detention, not prejudicial to claim of damages under Article 32 of the Civil Code. Petitioners were
neither assailing the validity of the subject hold departure orders, nor were they expressing any
intention to leave the country in the near future. To declare the hold departure orders null and void
ab initio must be made in the proper proceedings initiated for that purpose. Petitioners’ prayer for
relief regarding their alleged impending warrantless arrests is premature being that no complaints
have been filed against them for any crime, furthermore, the writ of habeas corpus is uncalled for
since its purpose is to relieve unlawful restraint which Petitioners are not subjected to.

Petition is dismissed. Respondents, consistent and congruent with their undertaking earlier
adverted to, together with their agents, representatives, and all persons acting in their behalf, are
hereby enjoined from arresting Petitioners without the required judicial warrants for all acts
committed in relation to or in connection with the May 1, 2001 siege of Malacañang.

SANLAKAS VS. EXECUTIVE SECRETARY


[421 SCRA 656; G.R. No. 159085; 3 Feb 2004]

Facts:

During the wee hours of July 27, 2003, some three-hundred junior officers and enlisted men of the
AFP, acting upon instigation, command and direction of known and unknown leaders have seized
Page 13
the Oakwood Building in Makati. Publicly, they complained of the corruption in the AFP and
declared their withdrawal of support for the government, demanding the resignation of the
President, Secretary of Defense and the PNP Chief. These acts constitute a violation of Article 134
of the Revised Penal Code, and by virtue of Proclamation No. 427 and General Order No. 4, the
Philippines was declared under the State of Rebellion. Negotiations took place and the officers went
back to their barracks in the evening of the same day. On August 1, 2003, both the Proclamation
and General Orders were lifted, and Proclamation No. 435, declaring the Cessation of the State of
Rebellion was issued.

In the interim, however, the following petitions were filed: (1) SANLAKAS AND PARTIDO NG
MANGGAGAWA VS. EXECUTIVE SECRETARY, petitioners contending that Sec. 18 Article VII of
the Constitution does not require the declaration of a state of rebellion to call out the AFP, and that
there is no factual basis for such proclamation. (2)SJS Officers/Members v. Hon. Executive
Secretary, et al, petitioners contending that the proclamation is a circumvention of the report
requirement under the same Section 18, Article VII, commanding the President to submit a report to
Congress within 48 hours from the proclamation of martial law. Finally, they contend that the
presidential issuances cannot be construed as an exercise of emergency powers as Congress has
not delegated any such power to the President. (3) Rep. Suplico et al. v. President Macapagal-
Arroyo and Executive Secretary Romulo, petitioners contending that there was usurpation of the
power of Congress granted by Section 23 (2), Article VI of the Constitution. (4) Pimentel v. Romulo,
et al, petitioner fears that the declaration of a state of rebellion "opens the door to the
unconstitutional implementation of warrantless arrests" for the crime of rebellion.

Issue:

Whether or Not Proclamation No. 427 and General Order No. 4 are constitutional?

Whether or Not the petitioners have a legal standing or locus standi to bring suit?

Held:

The Court rendered that the both the Proclamation No. 427 and General Order No. 4 are
constitutional. Section 18, Article VII does not expressly prohibit declaring state or rebellion. The
President in addition to its Commander-in-Chief Powers is conferred by the Constitution executive
powers. It is not disputed that the President has full discretionary power to call out the armed forces
and to determine the necessity for the exercise of such power. While the Court may examine
whether the power was exercised within constitutional limits or in a manner constituting grave abuse
of discretion, none of the petitioners here have, by way of proof, supported their assertion that the
President acted without factual basis. The issue of the circumvention of the report is of no merit as
there was no indication that military tribunals have replaced civil courts or that military authorities
have taken over the functions of Civil Courts. The issue of usurpation of the legislative power of the
Congress is of no moment since the President, in declaring a state of rebellion and in calling out the
armed forces, was merely exercising a wedding of her Chief Executive and Commander-in-Chief
powers. These are purely executive powers, vested on the President by Sections 1 and 18, Article
VII, as opposed to the delegated legislative powers contemplated by Section 23 (2), Article VI. The
fear on warrantless arrest is unreasonable, since any person may be subject to this whether there
is rebellion or not as this is a crime punishable under the Revised Penal Code, and as long as a
valid warrantless arrest is present.

Legal standing or locus standi has been defined as a personal and substantial interest in the case
such that the party has sustained or will sustain direct injury as a result of the governmental act that
is being challenged. The gist of the question of standing is whether a party alleges "such personal
stake in the outcome of the controversy as to assure that concrete adverseness which sharpens the
presentation of Issue upon which the court depends for illumination of difficult constitutional
questions. Based on the foregoing, petitioners Sanlakas and PM, and SJS Officers/Members have
no legal standing to sue. Only petitioners Rep. Suplico et al. and Sen. Pimentel, as Members of
Congress, have standing to challenge the subject issuances. It sustained its decision in Philippine
Page 14
Constitution Association v. Enriquez, that the extent the powers of Congress are impaired, so is the
power of each member thereof, since his office confers a right to participate in the exercise of the
powers of that institution.

JOYA VS. PCGG


[225 SCRA 568; G.R. No. 96541; 24 Aug 1993]

Facts:

On 9 August 1990, Mateo A.T. Caparas, then Chairman of PCGG, wrote then President Corazon C.
Aquino, requesting her for authority to sign the proposed Consignment Agreement between the
Republic of the Philippines through PCGG and Christie, Manson and Woods International, Inc
concerning the scheduled sale on 11 January 1991 of eighty-two) Old Masters Paintings and
antique silverware seized from Malacañang and the Metropolitan Museum of Manila alleged to be
part of the ill-gotten wealth of the late President Marcos, his relatives and cronies. On 14 August
1990, then President Aquino, through former Executive Secretary Catalino Macaraig, Jr., authorized
Chairman Caparas to sign the Consignment Agreement allowing Christie's of New York to auction
off the subject art pieces for and in behalf of the Republic of the Philippines. On 15 August 1990,
PCGG, through Chairman Caparas, representing the Government of the Republic of the Philippines,
signed the Consignment Agreement with Christie's of New York. According to the agreement,
PCGG shall consign to CHRISTIE'S for sale at public auction the eighty-two Old Masters Paintings
then found at the Metropolitan Museum of Manila as well as the silverware contained in seventy-one
cartons in the custody of the Central Bank of the Philippines, and such other property as may
subsequently be identified by PCGG and accepted by CHRISTIE'S to be subject to the provisions of
the agreement.

On 26 October 1990, the Commission on Audit through then Chairman Eufemio C. Domingo
submitted to President Aquino the audit findings and observations of COA on the Consignment
Agreement of 15 August 1990 to the effect that: the authority of former PCGG Chairman Caparas to
enter into the Consignment Agreement was of doubtful legality; the contract was highly
disadvantageous to the government; PCGG had a poor track record in asset disposal by auction in
the U.S.; and, the assets subject of auction were historical relics and had cultural significance,
hence, their disposal was prohibited by law.

After the oral arguments of the parties on 9 January 1991, we issued immediately our resolution
denying the application for preliminary injunction to restrain the scheduled sale of the artworks on
the ground that petitioners had not presented a clear legal right to a restraining order and that
proper parties had not been impleaded.

On 11 January 1991, the sale at public auction proceeded as scheduled and the proceeds of
$13,302,604.86 were turned over to the Bureau of Treasury.

Issue:

Whether or not petitioners have legal standing.

Whether or not the Old Masters Paintings and antique silverware are embraced in the phrase
"cultural treasure of the nation".

Whether or not the paintings and silverware are properties of public dominion on which can be
disposed of through the joint concurrence of the President and Congress.

Whether or not PCGG has complied with the due process clause and other statutory requirements
for the exportation and sale of the subject items.

Page 15
Whether or not the petition has become moot and academic, and if so, whether the above Issue
warrant resolution from this Court.

Held:

This is premised on Sec. 2, Rule 3, of the Rules of Court which provides that every action must be
prosecuted and defended in the name of the real party-in-interest, and that all persons having
interest in the subject of the action and in obtaining the relief demanded shall be joined as plaintiffs.
The Court will exercise its power of judicial review only if the case is brought before it by a party who
has the legal standing to raise the constitutional or legal question. "Legal standing" means a
personal and substantial interest in the case such that the party has sustained or will sustain direct
injury as a result of the governmental act that is being challenged. The term "interest" is material
interest, an interest in issue and to be affected by the decree, as distinguished from mere interest in
the question involved, or a mere incidental interest. Moreover, the interest of the party plaintiff must
be personal and not one based on a desire to vindicate the constitutional right of some third and
related party.

There are certain instances however when this Court has allowed exceptions to the rule on legal
standing, as when a citizen brings a case for mandamus to procure the enforcement of a public duty
for the fulfillment of a public right recognized by the Constitution, and when a taxpayer questions the
validity of a governmental act authorizing the disbursement of public funds.

Petitioners' arguments are devoid of merit. They lack basis in fact and in law. The ownership of
these paintings legally belongs to the foundation or corporation or the members thereof, although
the public has been given the opportunity to view and appreciate these paintings when they were
placed on exhibit.

The confiscation of these properties by the Aquino administration however should not be
understood to mean that the ownership of these paintings has automatically passed on the
government without complying with constitutional and statutory requirements of due process and
just compensation. If these properties were already acquired by the government, any constitutional
or statutory defect in their acquisition and their subsequent disposition must be raised only by the
proper parties the true owners thereof whose authority to recover emanates from their proprietary
rights which are protected by statutes and the Constitution. Having failed to show that they are the
legal owners of the artworks or that the valued pieces have become publicly owned, petitioners do
not possess any clear legal right whatsoever to question their alleged unauthorized disposition.

Neither can this petition be allowed as a taxpayer's suit. Obviously, petitioners are not challenging
any expenditure involving public funds but the disposition of what they allege to be public properties.
It is worthy to note that petitioners admit that the paintings and antique silverware were acquired
from private sources and not with public money.
Anent the second requisite of actual controversy, petitioners argue that this case should be resolved
by this Court as an exception to the rule on moot and academic cases; that although the sale of the
paintings and silver has long been consummated and the possibility of retrieving the treasure trove
is nil, yet the novelty and importance of the Issue raised by the petition deserve this Court's
attention. They submit that the resolution by the Court of the Issue in this case will establish future
guiding principles and doctrines on the preservation of the nation's priceless artistic and cultural
possessions for the benefit of the public as a whole.

For a court to exercise its power of adjudication, there must be an actual case of controversy — one
which involves a conflict of legal rights, an assertion of opposite legal claims susceptible of judicial
resolution; the case must not be moot or academic or based on extra-legal or other similar
considerations not cognizable by a court of justice. A case becomes moot and academic when its
purpose has become stale, such as the case before us. Since the purpose of this petition for
prohibition is to enjoin respondent public officials from holding the auction sale of the artworks on a

Page 16
particular date — 11 January 1991 — which is long past, the Issue raised in the petition have
become moot and academic.

The cultural properties of the nation which shall be under the protection of the state are classified as
the "important cultural properties" and the "national cultural treasures." On the other hand, a
"national cultural treasures" is a unique object found locally, possessing outstanding historical,
cultural, artistic and/or scientific value which is highly significant and important to this country and
nation. This Court takes note of the certification issued by the Director of the Museum that the
Italian paintings and silverware subject of this petition do not constitute protected cultural properties
and are not among those listed in the Cultural Properties Register of the National Museum.

WHEREFORE, for lack of merit, the petition for prohibition and mandamus is DISMISSED.

OPOSA VS. FACTORAN, JR.


[224 SCRA 792; G.R. No. 101083; 30 Jul 1993]

Facts:

Principal petitioners, are all minors duly represented and joined by their respective parents.
Impleaded as an additional plaintiff is the Philippine Ecological Network, Inc. (PENI), a domestic,
non-stock and non-profit corporation organized for the purpose of, inter alia, engaging in concerted
action geared for the protection of our environment and natural resources. The original defendant
was the Honorable Fulgencio S. Factoran, Jr., then Secretary of the Department of Environment
and Natural Resources (DENR). His substitution in this petition by the new Secretary, the Honorable
Angel C. Alcala, was subsequently ordered upon proper motion by the petitioners. The complaint
was instituted as a taxpayers' class suit and alleges that the plaintiffs "are all citizens of the Republic
of the Philippines, taxpayers, and entitled to the full benefit, use and enjoyment of the natural
resource treasure that is the country's virgin tropical forests." The same was filed for themselves
and others who are equally concerned about the preservation of said resource but are "so
numerous that it is impracticable to bring them all before the Court."

On 22 June 1990, the original defendant, Secretary Factoran, Jr., filed a Motion to Dismiss the
complaint based on two grounds, namely: the plaintiffs have no cause of action against him and,
the issue raised by the plaintiffs is a political question which properly pertains to the legislative or
executive branches of Government. In their 12 July 1990 Opposition to the Motion, the petitioners
maintain that, the complaint shows a clear and unmistakable cause of action, the motion is dilatory
and the action presents a justiciable question as it involves the defendant's abuse of discretion.

On 18 July 1991, respondent Judge issued an order granting the aforementioned motion to dismiss.
In the said order, not only was the defendant's claim that the complaint states no cause of action
against him and that it raises a political question sustained, the respondent Judge further ruled that
the granting of the relief prayed for would result in the impairment of contracts which is prohibited by
the fundamental law of the land.
Plaintiffs thus filed the instant special civil action for certiorari under Rule 65 of the Revised Rules of
Court and ask this Court to rescind and set aside the dismissal order on the ground that the
respondent Judge gravely abused his discretion in dismissing the action. Again, the parents of the
plaintiffs-minors not only represent their children, but have also joined the latter in this case.

Petitioners contend that the complaint clearly and unmistakably states a cause of action as it
contains sufficient allegations concerning their right to a sound environment based on Articles 19,
20 and 21 of the Civil Code (Human Relations), Section 4 of Executive Order (E.O.) No. 192
creating the DENR, Section 3 of Presidential Decree (P.D.) No. 1151 (Philippine Environmental
Policy), Section 16, Article II of the 1987 Constitution recognizing the right of the people to a
balanced and healthful ecology, the concept of generational genocide in Criminal Law and the
concept of man's inalienable right to self-preservation and self-perpetuation embodied in natural
Page 17
law. Petitioners likewise rely on the respondent's correlative obligation per Section 4 of E.O. No.
192, to safeguard the people's right to a healthful environment.

It is further claimed that the issue of the respondent Secretary's alleged grave abuse of discretion in
granting Timber License Agreements (TLAs) to cover more areas for logging than what is available
involves a judicial question.

Anent the invocation by the respondent Judge of the Constitution's non-impairment clause,
petitioners maintain that the same does not apply in this case because TLAs are not contracts. They
likewise submit that even if TLAs may be considered protected by the said clause, it is well settled
that they may still be revoked by the State when the public interest so requires.

Issue:

Whether or not the petitioners have locus standi.

Whether or not the petiton is in a form of a class suit.

Whether or not the TLA’s can be out rightly cancelled.

Whether or not the petition should be dismissed.

Held:

As to the matter of the cancellation of the TLAs, respondents submit that the same cannot be done
by the State without due process of law. Once issued, a TLA remains effective for a certain period
of time — usually for twenty-five (25) years. During its effectivity, the same can neither be revised
nor cancelled unless the holder has been found, after due notice and hearing, to have violated the
terms of the agreement or other forestry laws and regulations. Petitioners' proposition to have all the
TLAs indiscriminately cancelled without the requisite hearing would be violative of the requirements
of due process.

The subject matter of the complaint is of common and general interest not just to several, but to all
citizens of the Philippines. Consequently, since the parties are so numerous, it, becomes
impracticable, if not totally impossible, to bring all of them before the court. The plaintiffs therein are
numerous and representative enough to ensure the full protection of all concerned interests. Hence,
all the requisites for the filing of a valid class suit under Section 12, Rule 3 of the Revised Rules of
Court are present both in the said civil case and in the instant petition, the latter being but an
incident to the former.
Petitioners minors assert that they represent their generation as well as generations yet unborn.
Their personality to sue in behalf of the succeeding generations can only be based on the concept
of intergenerational responsibility insofar as the right to a balanced and healthful ecology is
concerned. Nature means the created world in its entirety. Every generation has a responsibility to
the next to preserve that rhythm and harmony for the full enjoyment of a balanced and healthful
ecology. The minors' assertion of their right to a sound environment constitutes, at the same time,
the performance of their obligation to ensure the protection of that right for the generations to come.

The complaint focuses on one specific fundamental legal right the right to a balanced and healthful
ecology which, for the first time in our nation's constitutional history, is solemnly incorporated in the
fundamental law. Section 16, Article II of the 1987 Constitution.

While the right to a balanced and healthful ecology is to be found under the Declaration of Principles
and State Policies and not under the Bill of Rights, it does not follow that it is less important than
any of the civil and political rights enumerated in the latter. Such a right belongs to a different
category of rights altogether for it concerns nothing less than self-preservation and self-perpetuation
— aptly and fittingly stressed by the petitioners the advancement of which may even be said to
predate all governments and constitutions. As a matter of fact, these basic rights need not even be
Page 18
written in the Constitution for they are assumed to exist from the inception of humankind. If they are
now explicitly mentioned in the fundamental charter, it is because of the well-founded fear of its
framers that unless the rights to a balanced and healthful ecology and to health are mandated as
state policies by the Constitution itself, thereby highlighting their continuing importance and
imposing upon the state a solemn obligation to preserve the first and protect and advance the
second, the day would not be too far when all else would be lost not only for the present generation,
but also for those to come generations which stand to inherit nothing but parched earth incapable of
sustaining life.

Conformably with the enunciated right to a balanced and healthful ecology and the right to health,
as well as the other related provisions of the Constitution concerning the conservation, development
and utilization of the country's natural resources, then President Corazon C. Aquino promulgated on
10 June 1987 E.O. No. 192, Section 4 of which expressly mandates that the Department of
Environment and Natural Resources "shall be the primary government agency responsible for the
conservation, management, development and proper use of the country's environment and natural
resources, specifically forest and grazing lands, mineral, resources, including those in reservation
and watershed areas, and lands of the public domain, as well as the licensing and regulation of all
natural resources as may be provided for by law in order to ensure equitable sharing of the benefits
derived therefrom for the welfare of the present and future generations of Filipinos." Section 3
thereof makes the following statement of policy:

The above provision stresses "the necessity of maintaining a sound ecological balance and
protecting and enhancing the quality of the environment." Section 2 of the same Title, on the other
hand, specifically speaks of the mandate of the DENR; however, it makes particular reference to the
fact of the agency's being subject to law and higher authority.

It may, however, be recalled that even before the ratification of the 1987 Constitution, specific
statutes already paid special attention to the "environmental right" of the present and future
generations. On 6 June 1977, P.D. No. 1151 and P.D. No. 1152 were issued. Thus, the right of the
petitioners to a balanced and healthful ecology is as clear as the DENR's duty under its mandate
and by virtue of its powers and functions under E.O. No. 192 and the Administrative Code of 1987
to protect and advance the said right.

A denial or violation of that right by the other who has the correlative duty or obligation to respect or
protect the same gives rise to a cause of action. Petitioners maintain that the granting of the TLAs,
which they claim was done with grave abuse of discretion, violated their right to a balanced and
healthful ecology; hence, the full protection thereof requires that no further TLAs should be renewed
or granted.

It is settled in this jurisdiction that in a motion to dismiss based on the ground that the complaint fails
to state a cause of action; the question submitted to the court for resolution involves the sufficiency
of the facts alleged in the complaint itself. No other matter should be considered; furthermore, the
truth of falsity of the said allegations is beside the point for the truth thereof is deemed hypothetically
admitted. Policy formulation or determination by the executive or legislative branches of
Government is not squarely put in issue. What is principally involved is the enforcement of a right
vis-a-vis policies already formulated and expressed in legislation. It must, nonetheless, be
emphasized that the political question doctrine is no longer, the insurmountable obstacle to the
exercise of judicial power or the impenetrable shield that protects executive and legislative actions
from judicial inquiry or review.

In the second place, even if it is to be assumed that the same are contracts, the instant case does
not involve a law or even an executive issuance declaring the cancellation or modification of existing
timber licenses. Hence, the non-impairment clause cannot as yet be invoked. Nevertheless,
granting further that a law has actually been passed mandating cancellations or modifications, the
same cannot still be stigmatized as a violation of the non-impairment clause. This is because by its
very nature and purpose, such as law could have only been passed in the exercise of the police

Page 19
power of the state for the purpose of advancing the right of the people to a balanced and healthful
ecology, promoting their health and enhancing the general welfare.

Finally, it is difficult to imagine, as the trial court did, how the non-impairment clause could apply
with respect to the prayer to enjoin the respondent Secretary from receiving, accepting, processing,
renewing or approving new timber licenses for, save in cases of renewal, no contract would have as
of yet existed in the other instances. Moreover, with respect to renewal, the holder is not entitled to
it as a matter of right.

Petition is hereby GRANTED, and the challenged Order of respondent Judge of 18 July 1991
dismissing Civil Case No. 90-777 is hereby set aside. The petitioners may therefore amend their
complaint to implead as defendants the holders or grantees of the questioned timber license
agreements.

AGAN JR. VS. PIATCO


[402 SCRA 612; G.R. No. 155001; 5 May 2003]

Facts:

Some time in 1993, six business leaders, explored the possibility of investing in the new NAIA
airport terminal, so they formed Asians Emerging Dragon Corp. They submitted proposals to the
government for the development of NAIA Intl. Passenger Terminal III (NAIA IPT III). The NEDA
approved the NAIA IPT III project. Bidders were invited, and among the proposal Peoples Air Cargo
(Paircargo) was chosen. AEDC protested alleging that preference was given to Paircargo, but still
the project was awarded to Paircargo. Because of that, it incorporated into, Phil. Intl. Airport
Terminals Co. (PIATCO). The DOTC and PIATCO entered into a concession agreement in 1997 to
franchise and operate the said terminal for 21years. In Nov. 1998 it was amended in the matters of
pertaining to the definition of the obligations given to the concessionaire, development of facilities
and proceeds, fees and charges, and the termination of contract. Since MIAA is charged with the
maintenance and operations of NAIA terminals I and II, it has a contract with several service
providers. The workers filed the petition for prohibition claiming that they would lose their job, and
the service providers joined them, filed a motion for intervention. Likewise several employees of the
MIAA filed a petition assailing the legality of arrangements. A group of congressmen filed similar
petitions. Pres. Arroyo declared in her speech that she will not honor PIATCO contracts which the
Exec. Branch's legal office concluded null and void.

Issue:

Whether or Not the 1997 concession agreement is void, together with its amendments for being
contrary to the constitution.

Held:

The 1997 concession agreement is void for being contrary to public policy. The amendments have
the effect of changing it into and entirely different agreement from the contract bidded upon. The
amendments present new terms and conditions which provide financial benefit to PIATCO which
may have the altered the technical and financial parameters of other bidders had they know that
such terms were available. The 1997 concession agreement, the amendments and supplements
thereto are set aside for being null and void.

Page 20
The petitioners have local standi. They are prejudiced by the concession agreement as their
livelihood is to be taken away from them.

UMALI VS. GUINGONA


[305 SCRA 533; G.R. No. 131124; 21 Mar 1999]

Facts:

Osmundo Umali the petitioner was appointed Regional Director of the Bureau of Internal Revenue
by Pres Fidel V. Ramos. He assigned him in Manila, November 29, 1993 to March 15, 1994 and
Makati, March 16, 1994 to August 4, 1994. On August 1, 1994, President Ramos received a
confidential memorandum against the petitioner for alleged violations of internal revenue laws, rules
and regulations during his incumbency as Regional Director, more particularly the following
malfeasance, misfeasance and nonfeasance. upon receipt of the said confidential memorandum,
former President authorized the issuance of an Order for the preventive suspension of the petitioner
and immediately referred the Complaint against the latter to the Presidential Commission on Anti-
Graft and Corruption (PCAGC), for investigation. Petitioner was duly informed of the charges
against him. And was directed him to send in his answer, copies of his Statement of Assets, and
Liabilities for the past three years (3), and Personal Data Sheet. Initial hearing was set on August
25, 1994, at 2:00 p.m., at the PCAGC Office. On August 23, the petitioner filed his required answer.
After evaluating the evidence on record, the PCAGC issued its Resolution of September 23, 1994,
finding a prima facie evidence to support six (6) of the twelve (12) charges against petitioner. On
October 6, 1994, acting upon the recommendation of the PCAGC, then President Ramos issued
Administrative Order No. 152 dismissing petitioner from the service, with forfeiture of retirement and
all benefits under the law.

Issue:

Whether or Not AO No. 152 violated petitioner's Right to Security of Tenure.

Whether or Not Petitioner was denied due process of law

Whether or Not the PCAGC is a validly Constituted government agency and whether the petitioner
can raise the issue of constitutionality belatedly in its motion for reconsideration of the trial courts
decision.

Whether or Not the ombudsman's resolution dismissing the charges against the petitioner is still
basis for the petitioner's dismissal with forfeiture of benefits as ruled in AO No. 152

Held:

Petitioner maintains that as a career executive service officer, he can only be removed for cause
and under the Administrative Code of 1987, 6 loss of confidence is not one of the legal causes or
grounds for removal. Consequently, his dismissal from office on the ground of loss confidence
violated his right to security of tenure, petitioner theorized. After a careful study, we are of the
irresistible conclusion that the Court of Appeals ruled correctly on the first three Issue. To be sure,
petitioner was not denied the right to due process before the PCAGC. Records show that the
petitioner filed his answer and other pleadings with respect to his alleged violation of internal
revenue laws and regulations, and he attended the hearings before the investigatory body. It is thus
decisively clear that his protestation of non-observance of due process is devoid of any factual or
legal basis. Neither can it be said that there was a violation of what petitioner asserts as his security
of tenure. According to petitioner, as a Regional Director of Bureau of Internal Revenue, he is
CESO eligible entitled to security of tenure. However, petitioner's claim of CESO eligibility is anemic
of evidentiary support. It was incumbent upon him to prove that he is a CESO eligible but
unfortunately, he failed to adduce sufficient evidence on the matter. His failure to do so is fatal. As
regards the issue of constitutionality of the PCAGC, it was only posed by petitioner in his motion for
Page 21
reconsideration before the Regional Trial Court of Makati. It was certainly too late to raise for the
first time at such late stage of the proceedings. As to last issue, It is worthy to note that in the case
under consideration, the administrative action against the petitioner was taken prior to the institution
of the criminal case. The charges included in Administrative Order No. 152 were based on the
results of investigation conducted by the PCAGC and not on the criminal charges before the
Ombudsman. In sum, the petition is dismissable on the ground that the Issue posited by the
petitioner do not constitute a valid legal basis for overturning the finding and conclusion arrived at by
the Court of Appeals. However, taking into account the antecedent facts and circumstances
aforementioned, the Court, in the exercise of its equity powers, has decided to consider the
dismissal of the charges against petitioner before the Ombudsman, the succinct and unmistakable
manifestation by the Commissioner of the Bureau of Internal Revenue that his office is no longer
interested in pursuing the case, and the position taken by the Solicitor General, that there is no
more basis for Administrative Order No. 152, as effective and substantive supervening events that
cannot be overlooked.

IN RE CUNANAN
[94 Phil 534; Resolution; 18 Mar 1954]

Facts:

Congress passed Republic Act Number 972, commonly known as the “Bar Flunkers’ Act of 1953.” In
accordance with the said law, the Supreme Court then passed and admitted to the bar those
candidates who had obtained an average of 72 per cent by raising it to 75 percent.

After its approval, many of the unsuccessful postwar candidates filed petitions for admission to the
bar invoking its provisions, while other motions for the revision of their examination papers were still
pending also invoked the aforesaid law as an additional ground for admission. There are also others
who have sought simply the reconsideration of their grades without, however, invoking the law in
question. To avoid injustice to individual petitioners, the court first reviewed the motions for
reconsideration, irrespective of whether or not they had invoked Republic Act No. 972.

Issue:

Whether or Not RA No. 972 is constitutional and valid.

Held:

RA No. 972 has for its object, according to its author, to admit to the Bar, those candidates who
suffered from insufficiency of reading materials and inadequate preparation.

In the judicial system from which ours has been evolved, the admission, suspension, disbarment
and reinstatement of attorneys at law in the practice of the profession and their supervision have
been indisputably a judicial function and responsibility. We have said that in the judicial system from
which ours has been derived, the admission, suspension, disbarment or reinstatement of attorneys
at law in the practice of the profession is concededly judicial.

On this matter, there is certainly a clear distinction between the functions of the judicial and
legislative departments of the government.

It is obvious, therefore, that the ultimate power to grant license for the practice of law belongs
exclusively to this Court, and the law passed by Congress on the matter is of permissive character,
or as other authorities may say, merely to fix the minimum conditions for the license.

Republic Act Number 972 is held to be unconstitutional.

Page 22
REPUBLIC ACT 6735, INITIATIVE AND REFERENDUM ACT

R.A. No. 6735 was, as its history reveals, intended to cover initiative to propose amendments to the
Constitution. The Act is a consolidation of House Bill No. 21505 and Senate Bill No. 17. The former
was prepared by the committee on Suffrage and Electoral Reforms of Representatives on the basis
of two House Bills referred to it, viz., (a) House Bill No. 497, which dealt with the initiative and
referendum mentioned in Sections 1 and 32 of Article VI of the Constitution; and (b) House Bill No.
988, which dealt with the subject matter of House Bill No. 497, as well as with initiative and
referendum under Section 3 of Article XVII of the Constitution. Senate Bill No. 17 solely, dealt with
initiative and referendum concerning ordinances or resolutions of local government units. The
Bicameral Conference Committee consolidated Senate Bill No. 17 and House Bill No. 21505 into a
draft bill, which was subsequently approved on 8 June 1989 by the Senate and by the House of
Representatives. This approved bill is now R.A. No. 6735.

Page 23
THE FUNDAMENTAL POWERS OF
THE STATE

THE POLICE POWER

AGUSTIN VS. EDU


[88 SCRA 195; G.R. No. L-49112; 2 Feb 1979]

Facts:

President Marcos issued the Letter of Instruction No. 229 which states that all owners, users or
drivers shall have at all times one pair of early warning devise (EWD) in their cars acquire from any
source depending on the owner’s choice. The Letter of Instruction was assailed by petitioner
Leovillo Agustin to have violated the constitution guarantee of due process against Hon Edu, Land
Transportation Commissioner, Hon. Juan Ponce Enrile, Minister of national Defense, Hon. Juinio,
Minister of Public Works, Transportation and Communication and Hon. Aquino, Minister of Public
Highways. Because of such contentions, the Implementing Rules and Regulation was ordered to be
suspended for a period of 6 months. Petitioner alleges that EWD are not necessary because
vehicles already have hazard lights (blinking lights) that can be use as a warning device. Also
petitioner contest that the letter of instruction violates the delegation of police power because it is
deemed harsh, oppressive and unreasonable for the motorists and those dealers of EWD will
become instant millionaires because of such law.

Issue:

Whether or not Petitioner’s contentions possess merit.

Held:

Petitioner’s contentions are without merit because the exercise of police power may interfere with
personal liberty or property to ensure and promote the safety, health and prosperity of the State.
Also, such letter of instruction is intended to promote public safety and it is indeed a rare occurrence
that such contention was alleged in a instruction with such noble purpose. Petitioner also failed to
present the factual foundation that is necessary to invalidate the said letter of instruction. In cases
where there is absence in the factual foundation, it should be presumed that constitutionality shall
prevail. Pres. Marcos on the other hand possesses vital statistics that will justify the need for the
implementation of this instruction. As signatory to the 1968 Vienna Conventions on Road Signs and
Signals, our country must abide with the standards given as stated in our Constitution that “the
Philippines adopts the generally accepted principles of International Law as part of the law of the
land. In the case at bar, the Vienna Convention also requires the use of EWD. Vehicle owners are
not obliged to buy an EDW. They can personally create a EWD provided that it is in accordance to
the specifications provided by law. Petitioner’s allegation against the manufacturers of EDW being
millionaires is deemed to be an unfounded speculation. Wherefore, the petition is dismissed. The
restraining order regarding the implementation of the Reflector Law is lifted making the said law
immediately executory.

ICHONG VS. HERNANDEZ


[101 Phil 1117; G.R. No. L-7995; 31 May 1957]

Facts:

Page 24
Republic Act 1180 or commonly known as “An Act to Regulate the Retail Business” was passed.
The said law provides for a prohibition against foreigners as well as corporations owned by
foreigners from engaging from retail trade in our country. This was protested by the petitioner in this
case. According to him, the said law violates the international and treaty of the Philippines therefore
it is unconstitutional. Specifically, the Treaty of Amity between the Philippines and China was
violated according to him.

Issue:

Whether or Not Republic Act 1180 is a valid exercise of police power.

Held:

According to the Court, RA 1180 is a valid exercise of police power. It was also then provided that
police power can not be bargained away through the medium of a treaty or a contract. The Court
also provided that RA 1180 was enacted to remedy a real and actual danger to national economy
posed by alien dominance and control. If ever the law infringes upon the said treaty, the latter is
always subject to qualification or amendment by a subsequent law and the same may never curtain
or restrict the scope of the police power of the state.

LUTZ VS. ARANETA


[98 Phil 148; G.R. No. L-7859; 22 Dec 1955]

Facts:

Walter Lutz, as the Judicial Administrator of the Intestate Estate of Antonio Jayme Ledesma, seeks
to recover from J. Antonio Araneta, the Collector of Internal Revenue, the sum of money paid by the
estate as taxes, pursuant to the Sugar Adjustment Act. Under Section 3 of said Act, taxes are
levied on the owners or persons in control of the lands devoted to the cultivation of sugar cane.
Furthermore, Section 6 states all the collections made under said Act shall be for aid and support of
the sugar industry exclusively. Lutz contends that such purpose is not a matter of public concern
hence making the tax levied for that cause unconstitutional and void. The Court of First Instance
dismissed his petition, thus this appeal before the Supreme Court.

Issue:

Whether or Not the tax levied under the Sugar Adjustment Act ( Commonwealth Act 567) is
unconstitutional.

Held:

The tax levied under the Sugar Adjustment Act is constitutional. The tax under said Act is levied with
a regulatory purpose, to provide means for the rehabilitation and stabilization of the threatened
sugar industry. Since sugar production is one of the great industries of our nation, its promotion,
protection, and advancement, therefore redounds greatly to the general welfare. Hence, said
objectives of the Act is a public concern and is therefore constitutional. It follows that the
Legislature may determine within reasonable bounds what is necessary for its protection and
expedient for its promotion. If objectives and methods are alike constitutionally valid, no reason is
seen why the state may not levy taxes to raise funds for their prosecution and attainment. Taxation
may be made with the implement of the state’s police power. In addition, it is only rational that the
taxes be obtained from those that will directly benefit from it. Therefore, the tax levied under the
Sugar Adjustment Act is held to be constitutional.

Page 25
TIO VS. VIDEOGRAM REGULATORY BOARD
[151 SCRA 208; G.R. No. L-75697; 18 Jun 1987]

Facts:

The case is a petition filed by petitioner on behalf of videogram operators adversely affected by
Presidential Decree No. 1987, “An Act Creating the Videogram Regulatory Board" with broad
powers to regulate and supervise the videogram industry.

A month after the promulgation of the said Presidential Decree, the amended the National Internal
Revenue Code provided that:

"SEC. 134. Video Tapes. — There shall be collected on each processed video-
tape cassette, ready for playback, regardless of length, an annual tax of five pesos;
Provided, That locally manufactured or imported blank video tapes shall be subject to
sales tax."

"Section 10. Tax on Sale, Lease or Disposition of Videograms. — Notwithstanding


any provision of law to the contrary, the province shall collect a tax of thirty percent
(30%) of the purchase price or rental rate, as the case may be, for every sale, lease
or disposition of a videogram containing a reproduction of any motion picture or
audiovisual program.”

“Fifty percent (50%) of the proceeds of the tax collected shall accrue to the province,
and the other fifty percent (50%) shall accrue to the municipality where the tax is
collected; PROVIDED, That in Metropolitan Manila, the tax shall be shared equally by
the City/Municipality and the Metropolitan Manila Commission.”

The rationale behind the tax provision is to curb the proliferation and unregulated circulation of
videograms including, among others, videotapes, discs, cassettes or any technical improvement or
variation thereof, have greatly prejudiced the operations of movie houses and theaters. Such
unregulated circulation have caused a sharp decline in theatrical attendance by at least forty
percent (40%) and a tremendous drop in the collection of sales, contractor's specific, amusement
and other taxes, thereby resulting in substantial losses estimated at P450 Million annually in
government revenues.

Videogram(s) establishments collectively earn around P600 Million per annum from rentals, sales
and disposition of videograms, and these earnings have not been subjected to tax, thereby
depriving the Government of approximately P180 Million in taxes each year.

The unregulated activities of videogram establishments have also affected the viability of the movie
industry.

Issue:

Whether or not tax imposed by the DECREE is a valid exercise of police power.

Whether or nor the DECREE is constitutional .

Held:

Taxation has been made the implement of the state's police power. The levy of the 30% tax is for a
public purpose. It was imposed primarily to answer the need for regulating the video industry,
particularly because of the rampant film piracy, the flagrant violation of intellectual property rights,
and the proliferation of pornographic video tapes. And while it was also an objective of the DECREE
to protect the movie industry, the tax remains a valid imposition.

Page 26
We find no clear violation of the Constitution which would justify us in pronouncing Presidential
Decree No. 1987 as unconstitutional and void. While the underlying objective of the DECREE is to
protect the moribund movie industry, there is no question that public welfare is at bottom of its
enactment, considering "the unfair competition posed by rampant film piracy; the erosion of the
moral fiber of the viewing public brought about by the availability of unclassified and unreviewed
video tapes containing pornographic films and films with brutally violent sequences; and losses in
government revenues due to the drop in theatrical attendance, not to mention the fact that the
activities of video establishments are virtually untaxed since mere payment of Mayor's permit and
municipal license fees are required to engage in business."

WHEREFORE, the instant Petition is hereby dismissed. No costs.

ASSO. OF SMALL LANDOWNERS VS. SEC. OF DAR


[175 SCRA 343; G.R. NO. L-78742; 14 JUL 1989]

Facts:

Several petitions are the root of the case:

a. A petition alleging the constitutionality of PD No. 27, EO 228 and 229 and RA 6657.
Subjects of the petition are a 9-hectare and 5 hectare Riceland worked by four tenants.
Tenants were declared full owners by EO 228 as qualified farmers under PD 27. The
petitioners now contend that President Aquino usurped the legislature’s power.
b. A petition by landowners and sugarplanters in Victoria’s Mill Negros Occidental against
Proclamation 131 and EO 229. Proclamation 131 is the creation of Agrarian Reform
Fund with initial fund of P50Billion.
c. A petition by owners of land which was placed by the DAR under the coverage of
Operation Land Transfer.
d. A petition invoking the right of retention under PD 27 to owners of rice and corn lands not
exceeding seven hectares.

Issue:

Whether or Not the aforementioned EO’s, PD, and RA were constitutional.

Held:

The promulgation of PD 27 by President Marcos was valid in exercise of Police power and eminent
domain.

The power of President Aquino to promulgate Proc. 131 and EO 228 and 229 was authorized under
Sec. 6 of the Transitory Provisions of the 1987 Constitution. Therefore it is a valid exercise of Police
Power and Eminent Domain.

RA 6657 is likewise valid. The carrying out of the regulation under CARP becomes necessary to
deprive owners of whatever lands they may own in excess of the maximum area allowed, there is
definitely a taking under the power of eminent domain for which payment of just compensation is
imperative. The taking contemplated is not a mere limitation of the use of the land. What is required
is the surrender of the title and the physical possession of said excess and all beneficial rights
accruing to the owner in favour of the farmer.

A statute may be sustained under the police power only if there is concurrence of the lawful subject
and the method.

Subject and purpose of the Agrarian Reform Law is valid, however what is to be determined is the
method employed to achieve it.
Page 27
LOZANO VS. MARTINEZ
[146 SCRA 323; G.R. No. L-63419; 18 Dec 1986]

Facts:

A motion to quash the charge against the petitioners for violation of the BP 22 was made,
contending that no offense was committed, as the statute is unconstitutional. Such motion was
denied by the RTC. The petitioners thus elevate the case to the Supreme Court for relief. The
Solicitor General, commented that it was premature for the accused to elevate to the Supreme
Court the orders denying their motions to quash. However, the Supreme Court finds it justifiable to
intervene for the review of lower court's denial of a motion to quash.

Issue:

Whether or not BP 22 is constitutional as it is a proper exercise of police power of the State.

Held:

The enactment of BP 22 a valid exercise of the police power and is not repugnant to the
constitutional inhibition against imprisonment for debt.

The offense punished by BP 22 is the act of making and issuing a worthless check or a check that is
dishonored upon its presentation for payment. It is not the non-payment of an obligation which the
law punishes. The law is not intended or designed to coerce a debtor to pay his debt.

The law punishes the act not as an offense against property, but an offense against public order.
The thrust of the law is to prohibit, under pain of penal sanctions, the making of worthless checks
and putting them in circulation. An act may not be considered by society as inherently wrong,
hence, not malum in se but because of the harm that it inflicts on the community, it can be outlawed
and criminally punished as malum prohibitum. The state can do this in the exercise of its police
power.

KWONG SING VS. CITY OF MANILA


[41 Phil 103; G.R. No. 15972; 11 Oct 1920]

Facts:

Kwong Sing, in his own behalf and of other Chinese laundrymen who has general and the same
interest, filed a complaint for a preliminary injunction. The Plaintiffs also questioned the validity of
enforcing Ordinance No. 532 by the city of Manila. Ordinance No. 532 requires that the receipt be
in duplicate in English and Spanish duly signed showing the kind and number of articles delivered
by laundries and dyeing and cleaning establishments. The permanent injunction was denied by the
trial court. The appellants claim is that Ordinance No. 532 savors of class legislation; putting in
mind that they are Chinese nationals. It unjustly discriminates between persons in similar
circumstances; and that it constitutes an arbitrary infringement of property rights. They also contest
that the enforcement of the legislation is an act beyond the scope of their police power. In view of
the foregoing, this is an appeal with the Supreme Court.

Issue:

Whether or Not the enforcement of Ordinance no, 532 is an act beyond the scope of police power

Whether or Not the enforcement of the same is a class legislation that infringes property rights.

Page 28
Held:

Reasonable restraints of a lawful business for such purposes are permissible under the police
power. The police power of the City of Manila to enact Ordinance No. 532 is based on Section
2444, paragraphs (l) and (ee) of the Administrative Code, as amended by Act No. 2744, authorizes
the municipal board of the city of Manila, with the approval of the mayor of the city:

(l) To regulate and fix the amount of the license fees for the following: xxxx
xxxxxlaundries xxxx.

(ee) To enact all ordinances it may deem necessary and proper for the sanitation and
safety, the furtherance of the prosperity, and the promotion of the morality, peace,
good order, comfort, convenience, and general welfare of the city and its inhabitants.

The court held that the obvious purpose of Ordinance No. 532 was to avoid disputes between
laundrymen and their patrons and to protect customers of laundries who are not able to decipher
Chinese characters from being defrauded. (Considering that in the year 1920s, people of Manila
are more familiar with Spanish and maybe English.)

In whether the ordinance is class legislation, the court held that the ordinance invades no
fundamental right, and impairs no personal privilege. Under the guise of police regulation, an
attempt is not made to violate personal property rights. The ordinance is neither discriminatory nor
unreasonable in its operation. It applies to all public laundries without distinction, whether they
belong to Americans, Filipinos, Chinese, or any other nationality. All, without exception, and each
every one of them without distinction, must comply with the ordinance. The obvious objection for
the implementation of the ordinance is based in sec2444 (ee) of the Administrative Code. Although,
an additional burden will be imposed on the business and occupation affected by the ordinance
such as that of the appellant by learning even a few words in Spanish or English, but mostly Arabic
numbers in order to properly issue a receipt, it seems that the same burdens are cast upon the
them. Yet, even if private rights of person or property are subjected to restraint, and even if loss will
result to individuals from the enforcement of the ordinance, this is not sufficient ground for failing to
uphold the power of the legislative body. The very foundation of the police power is the control of
private interests for the public welfare.

Finding that the ordinance is valid, judgment is affirmed, and the petition for a preliminary injunction
is denied, with costs against the appellants.

TABLARIN VS. GUTIERREZ


[152 SCRA 730; G.R. No. 78164; 31 July 1987]

Facts:

The petitioners sought to enjoin the Secretary of Education, Culture and Sports, the Board of
Medical Education and the Center for Educational Measurement from enforcing Section 5 (a) and (f)
of Republic Act No. 2382, as amended, and MECS Order No. 52, series of 1985, dated 23 August
1985 and from requiring the taking and passing of the NMAT as a condition for securing certificates
of eligibility for admission, from proceeding with accepting applications for taking the NMAT and
from administering the NMAT as scheduled on 26 April 1987 and in the future. The trial court
denied said petition on 20 April 1987. The NMAT was conducted and administered as previously
scheduled.

Republic Act 2382, as amended by Republic Acts Nos. 4224 and 5946, known as the "Medical Act
of 1959" defines its basic objectives in the following manner:

Page 29
"SECTION 1. Objectives. — This Act provides for and shall govern (a) the
standardization and regulation of medical education; (b) the examination for
registration of physicians; and (c) the supervision, control and regulation of the
practice of medicine in the Philippines."

The statute, among other things, created a Board of Medical Education. Its functions as specified in
Section 5 of the statute include the following:

"(a) To determine and prescribe requirements for admission into a recognized


college of medicine;
x x x
(f) To accept applications for certification for admission to a medical school and
keep a register of those issued said certificate; and to collect from said applicants the
amount of twenty-five pesos each which shall accrue to the operating fund of the
Board of Medical Education;”

Section 7 prescribes certain minimum requirements for applicants to medical schools:

"Admission requirements. — The medical college may admit any student who has
not been convicted by any court of competent jurisdiction of any offense involving
moral turpitude and who presents (a) a record of completion of a bachelor's degree in
science or arts; (b) a certificate of eligibility for entrance to a medical school from the
Board of Medical Education; (c) a certificate of good moral character issued by two
former professors in the college of liberal arts; and (d) birth certificate. Nothing in this
act shall be construed to inhibit any college of medicine from establishing, in addition
to the preceding, other entrance requirements that may be deemed admissible.”

MECS Order No. 52, s. 1985, issued by the then Minister of Education, Culture and Sports and
dated 23 August 1985, established a uniform admission test called the National Medical Admission
Test (NMAT) as an additional requirement for issuance of a certificate of eligibility for admission into
medical schools of the Philippines, beginning with the school year 1986-1987. This Order goes on
to state that: "2. The NMAT, an aptitude test, is considered as an instrument toward upgrading
the selection of applicants for admission into the medical schools and its calculated to improve the
quality of medical education in the country. The cutoff score for the successful applicants, based on
the scores on the NMAT, shall be determined every year by the Board of Medical Education after
consultation with the Association of Philippine Medical Colleges. The NMAT rating of each
applicant, together with the other admission requirements as presently called for under existing
rules, shall serve as a basis for the issuance of the prescribed certificate of eligibility for admission
into the medical colleges.

Issue:

Whether or not Section 5 (a) and (f) of Republic Act No. 2382, as amended, and MECS Order No.
52, s. 1985 are constitutional.

Held:

Yes. We conclude that prescribing the NMAT and requiring certain minimum scores therein as a
condition for admission to medical schools in the Philippines, do not constitute an unconstitutional
imposition.

The police power, it is commonplace learning, is the pervasive and non-waivable power and
authority of the sovereign to secure and promote all the important interests and needs — in a word,
the public order — of the general community. An important component of that public order is the
health and physical safety and well being of the population, the securing of which no one can deny
is a legitimate objective of governmental effort and regulation. Perhaps the only issue that needs
some consideration is whether there is some reasonable relation between the prescribing of
Page 30
passing the NMAT as a condition for admission to medical school on the one hand, and the
securing of the health and safety of the general community, on the other hand. This question is
perhaps most usefully approached by recalling that the regulation of the practice of medicine in all
its branches has long been recognized as a reasonable method of protecting the health and safety
of the public.

MECS Order No. 52, s. 1985 articulates the rationale of regulation of this type: the improvement of
the professional and technical quality of the graduates of medical schools, by upgrading the quality
of those admitted to the student body of the medical schools. That upgrading is sought by selectivity
in the process of admission, selectivity consisting, among other things, of limiting admission to those
who exhibit in the required degree the aptitude for medical studies and eventually for medical
practice. The need to maintain, and the difficulties of maintaining, high standards in our professional
schools in general, and medical schools in particular, in the current stage of our social and
economic development, are widely known. We believe that the government is entitled to prescribe
an admission test like the NMAT as a means for achieving its stated objective of "upgrading the
selection of applicants into [our] medical schools" and of "improv[ing] the quality of medical
education in the country. We are entitled to hold that the NMAT is reasonably related to the
securing of the ultimate end of legislation and regulation in this area. That end, it is useful to recall,
is the protection of the public from the potentially deadly effects of incompetence and ignorance in
those who would undertake to treat our bodies and minds for disease or trauma.

WHEREFORE, the Petition for Certiorari is DISMISSED and the Order of the respondent trial court
denying the petition for a writ of preliminary injunction is AFFIRMED. Costs against petitioners.

CITY GOVERNMENT OF QUEZON CITY VS. ERICTA


[122 SCRA 759; G.R. No. L-34915; 24 Jun 1983]

Facts:

Section 9 of Ordinance No. 6118, S-64, entitled "Ordinance Regulating The Establishment,
Maintenance And Operation Of Private Memorial Type Cemetery Or Burial Ground Within The
Jurisdiction Of Quezon City And Providing Penalties For The Violation Thereof" provides:

Sec. 9. At least six (6) percent of the total area of the memorial park cemetery shall
be set aside for charity burial of deceased persons who are paupers and have been
residents of Quezon City for at least 5 years prior to their death, to be determined by
competent City Authorities. The area so designated shall immediately be developed
and should be open for operation not later than six months from the date of approval
of the application.

For several years, the aforequoted section of the Ordinance was not enforced but seven years after
the enactment of the ordinance, the Quezon City Council passed a resolution to request the City
Engineer, Quezon City, to stop any further selling and/or transaction of memorial park lots in
Quezon City where the owners thereof have failed to donate the required 6% space intended for
paupers burial.

The Quezon City Engineer then notified respondent Himlayang Pilipino, Inc. in writing that Section 9
of the ordinance would be enforced.

Respondent Himlayang Pilipino reacted by filing a petition for declaratory relief, prohibition and
mandamus with preliminary injunction seeking to annul Section 9 of the Ordinance in question.
Respondent alleged that the same is contrary to the Constitution, the Quezon City Charter, the
Local Autonomy Act, and the Revised Administrative Code.

Issue:
Page 31
Whether or Not Section 9 of the ordinance in question is a valid exercise of police power.

Held:

Section 9 of the City ordinance in question is not a valid exercise of police power. Section 9 cannot
be justified under the power granted to Quezon City to tax, fix the license fee, and regulate such
other business, trades, and occupation as may be established or practiced in the City.

Bill of rights states that 'no person shall be deprived of life, liberty or property without due process of
law' (Art. Ill, Section 1 subparagraph 1, Constitution). On the other hand, there are three inherent
powers of government by which the state interferes with the property rights, namely-. (1) police
power, (2) eminent domain, (3) taxation.

The police power of Quezon City is defined in sub-section 00, Sec. 12, Rep. Act 537 that reads as
follows:

“To make such further ordinance and regulations not repugnant to law as may be
necessary to carry into effect and discharge the powers and duties conferred by this
act and such as it shall deem necessary and proper to provide for the health and
safety, …, and for the protection of property therein; and enforce obedience thereto
with such lawful fines or penalties as the City Council may prescribe under the
provisions of subsection (jj) of this section.”

The power to regulate does not include the power to prohibit. The power to regulate does not
include the power to confiscate. The ordinance in question not only confiscates but also prohibits
the operation of a memorial park cemetery, because under Section 13 of said ordinance, 'Violation
of the provision thereof is punishable with a fine and/or imprisonment and that upon conviction
thereof the permit to operate and maintain a private cemetery shall be revoked or cancelled’. The
confiscatory clause and the penal provision in effect deter one from operating a memorial park
cemetery.

Moreover, police power is defined by Freund as 'the power of promoting the public welfare by
restraining and regulating the use of liberty and property'. It is usually exerted in order to merely
regulate the use and enjoyment of property of the owner. If he is deprived of his property outright, it
is not taken for public use but rather to destroy in order to promote the general welfare.

It seems to the court that Section 9 of Ordinance No. 6118, Series of 1964 of Quezon City is not a
mere police regulation but an outright confiscation. It deprives a person of his private property
without due process of law, nay, even without compensation.

MMDA Vs. Bel-Air Village


[328 SCRA 836; G.R. No. 135962; 27 Mar 2000]

Facts:

Metropolitan Manila Development Authority (MMDA), petitioner herein, is a Government Agency


tasked with the delivery of basic services in Metro Manila. Bel-Air Village Association (BAVA),
respondent herein, received a letter of request from the petitioner to open Neptune Street of Bel-Air
Village for the use of the public. The said opening of Neptune Street will be for the safe and
convenient movement of persons and to regulate the flow of traffic in Makati City. This was pursuant
to MMDA law or Republic Act No. 7924. On the same day, the respondent was appraised that the
perimeter wall separating the subdivision and Kalayaan Avenue would be demolished.

The respondent, to stop the opening of the said street and demolition of the wall, filed a preliminary
injunction and a temporary restraining order. Respondent claimed that the MMDA had no authority
Page 32
to do so and the lower court decided in favor of the Respondent. Petitioner appealed the decision of
the lower courts and claimed that it has the authority to open Neptune Street to public traffic
because it is an agent of the State that can practice police power in the delivery of basic services in
Metro Manila.

Issue:

Whether or not the MMDA has the mandate to open Neptune Street to public traffic pursuant to its
regulatory and police powers.

Held:

The Court held that the MMDA does not have the capacity to exercise police power. Police power is
primarily lodged in the National Legislature. However, police power may be delegated to
government units. Petitioner herein is a development authority and not a political government unit.
Therefore, the MMDA cannot exercise police power because it cannot be delegated to them. It is
not a legislative unit of the government. Republic Act No. 7924 does not empower the MMDA to
enact ordinances, approve resolutions and appropriate funds for the general welfare of the
inhabitants of Manila. There is no syllable in the said act that grants MMDA police power.

It is an agency created for the purpose of laying down policies and coordinating with various
national government agencies, people’s organizations, non-governmental organizations and the
private sector for the efficient and expeditious delivery of basic services in the vast metropolitan
area.

TATEL VS. MUNICIPALITY OF VIRAC


[207 SCRA 157; G.R. No. 40243; 11 Mar 1992]

Facts:

Petitioner Celestino Tatel owns a warehouse in barrio Sta. Elena, Municipality of Virac. Complaints
were received by the municipality concerning the disturbance caused by the operation of the abaca
bailing machine inside petitioner’s warehouse. A committee was then appointed by the municipal
council, and it noted from its investigation on the matter that an accidental fire within the warehouse
of the petitioner created a danger to the lives and properties of the people in the neighborhood.
Resolution No. 29 was then passed by the Municipal council declaring said warehouse as a public
nuisance within a purview of Article 694 of the New Civil Code. According to respondent municipal
officials, petitioner’s warehouse was constructed in violation of Ordinance No. 13, series of 1952,
prohibiting the construction of warehouses near a block of houses either in the poblacion or barrios
without maintaining the necessary distance of 200 meters from said block of houses to avoid loss of
lives and properties by accidental fire. On the other hand, petitioner contends that Ordinance No. 13
is unconstitutional.

Issue:

Whether or not petitioner’s warehouse is a nuisance within the meaning Article 694 of the Civil Code

Whether or not Ordinance No. 13, series of 1952 of the Municipality of Virac is unconstitutional and
void.

Held:

Page 33
The storage of abaca and copra in petitioner’s warehouse is a nuisance under the provisions of
Article 694 of the Civil Code. At the same time, Ordinance No. 13 was passed by the Municipal
Council of Virac in the exercise of its police power. It is valid because it meets the criteria for a valid
municipal ordinance: 1) must not contravene the Constitution or any statute, 2) must not be unfair or
oppressive, 3) must not be partial or discriminatory, 4) must not prohibit but may regulate trade, 5)
must be general and consistent with public policy, and 6) must not be unreasonable. The purpose of
the said ordinance is to avoid the loss of property and life in case of fire which is one of the
primordial obligation of government. The lower court did not err in its decision.

Page 34
THE POWER OF EMINENT DOMAIN

REPUBLIC VS. TAGLE


[299 SCRA 549; G.R. No. 129079; 2 Dec 1998]

Facts:

Private respondent Helena Z. Benitez is the registered owner of two (2) parcels of land located in
Barangay Salawag, Dasmariñas, Cavite containing an area of 483,331 square meters more or less.

The Philippine Government, through the Philippine Human Resources Development Center
(PHRDC), negotiated with the Japanese International Cooperation Agency (JICA) Survey Team on
the technicalities of the establishment of the ASEAN Human Resources Development Project in the
Philippines. Among the five (5) main programs of the proposed project was Program III
(Construction Manpower Development) which involved the establishment of a Construction
Manpower Development Center (CMDC). PHRDC and private respondent Helena Z. Benitez,
signed a Memorandum of Agreement which provides, among others, that Benitez undertakes to
lease within the period of twenty (20) years and/or sell a portion of that property (which is no less
than ten-hectares) in favor of PHRDC which likewise agrees to lease within a period of twenty (20)
years and/or buy said property site.

The Philippine Women’s University (PWU) and Benitez granted a permit to PHRDC to occupy and
use the land in question and to undertake land development, electrical and road network
installations and other related works necessary to attain its objectives. Pursuant thereto, the CMDC
took possession of the property and erected buildings and other related facilities necessary for its
operations. A deposit made by the plaintiff with the Philippine National Bank (PNB) in the amount of
P708,490.00 which is equivalent to the assessed value of the property subject matter hereof based
on defendant’s 1990 tax declaration, was made.

In view of the agreement on the sale of the land in question, PHRDC prepared a Deed of Absolute
Sale with Benitez, as vendor, and PHRDC and CMDC, as vendees, duly represented by then
Undersecretary Gloria M. Arroyo, for the signature of Benitez. Benitez in her own capacity did not
sign the deed of absolute sale.
Failing to acquire the property involved through negotiated sale, petitioner, through the Department
of Trade and Industry, to which CMDC is attached, instituted a complaint for Eminent Domain,
pursuant to the provisions of Executive Order No. 1035, dated June 25, 1985.

A Motion for Issuance of Writ of Possession was granted by the court but quashed it subsequently.

Issue:

Whether or Not the respondent judge may quash a writ of possession on the ground that the
expropriating government agency is already occupying the property sought to be expropriated.

Held:

No. Under Section 7 of EO 1035, when the government or its authorized agent makes the required
deposit, the trial court has a ministerial duty to issue a writ of possession. The expropriation of real
property does not include mere physical entry or occupation of land. Although eminent domain
usually involves a taking of title, there may also be compensable taking of only some, not all, of the
property interests in the bundle of rights that constitute ownership.

In the instant case, it is manifest that the petitioner, in pursuit of an objective beneficial to public
interest, seeks to realize the same through its power of eminent domain. In exercising this power,
petitioner intended to acquire not only physical possession but also the legal right to possess and
ultimately to own the subject property. Hence, its mere physical entry and occupation of the
Page 35
property fall short of the taking of title, which includes all the rights that may be exercised by an
owner over the subject property.

CITY OF MANILA VS. CHINESE COMMUNITY


[40 Phil 349; No. 14355; 31 Oct 1919]

Facts: The City of Manila, plaintiff herein, prayed for the expropriation of a portion private cemetery
for the conversion into an extension of Rizal Avenue. Plaintiff claims that it is necessary that such
public improvement be made in the said portion of the private cemetery and that the said lands are
within their jurisdiction.

Defendants herein answered that the said expropriation was not necessary because other routes
were available. They further claimed that the expropriation of the cemetery would create irreparable
loss and injury to them and to all those persons owing and interested in the graves and monuments
that would have to be destroyed.

The lower court ruled that the said public improvement was not necessary on the particular-strip of
land in question. Plaintiff herein assailed that they have the right to exercise the power of eminent
domain and that the courts have no right to inquire and determine the necessity of the expropriation.
Thus, the same filed an appeal.

Issue:

Whether or not the courts may inquire into, and hear proof of the necessity of the expropriation.

Held:

The courts have the power of restricting the exercise of eminent domain to the actual reasonable
necessities of the case and for the purposes designated by the law. The moment the municipal
corporation or entity attempts to exercise the authority conferred, it must comply with the conditions
accompanying the authority. The necessity for conferring the authority upon a municipal corporation
to exercise the right of eminent domain is admittedly within the power of the legislature. But whether
or not the municipal corporation or entity is exercising the right in a particular case under the
conditions imposed by the general authority, is a question that the courts have the right to inquire to.

REPUBLIC VS. PLDT


[26 SCRA 320; G.R. No. L-18841; 27 Jan 1969]

Facts:

The plaintiff Republic of the Philippines is a political entity exercising government powers through
one of its branches, the Bureau of Telecommunication. Herein defendant, PLDT is a public service
corporation holding a franchise to install operates and maintains a telephone system. After its
creation, the BOT set up its own government telephone system by utilizing its own appropriations
and other equipment and by renting trunk lines of the PLDT to enable the govt offices to call
privately. BOT entered into an agreement with the RCA communications for joint overseas
telephone service whereby BOT would convey overseas calls received by RCA to local residents.
PLDT complained to the BOT that it was a violation of the condition of their agreement since the
BOT had used trunk lines only for the use of government offices but even to serve private persons
or the general public in competition with the business of PLDT. Subsequently, the plaintiff
commenced suit against PLDT asking the court judgment be rendered ordering the PLDT to
Page 36
execute a contract with the plaintiff, through the BOT for the use of the facilities of PLDT's telephone
system throughout the country under such conditions as the court may consider reasonable. The
CFI rendered judgment stating that it could not compel PLDT to enter into such agreement. Hence
this petition.

Issue:

Whether or Not PLDT may be compelled to enter into such agreement.

Held:

Yes, the state, may, in the interest of national welfare transfer utilities to public ownership upon
payment of just compensation, there is no reason why the state ma not require a public utility to
render services in the general interest provided just compensation is paid.

PEOPLE VS. FAJARDO


[104 Phil 443; G.R. No. L-12172; 29 Aug 1958]

Facts:

The municipal council of baao, camarines sur stating among others that construction of a building,
which will destroy the view of the plaza, shall not be allowed and therefore be destroyed at the
expense of the owner, enacted an ordinance. Herein appellant filed a written request with the
incumbent municipal mayor for a permit to construct a building adjacent to their gasoline station on
a parcel of land registered in Fajardo's name, located along the national highway and separated
from the public plaza by a creek. The request was denied, for the reason among others that the
proposed building would destroy the view or beauty of the public plaza. Defendants reiterated their
request for a building permit, but again the mayor turned down the request. Whereupon, appellants
proceeded with the construction of the building without a permit, because they needed a place of
residence very badly, their former house having been destroyed by a typhoon and hitherto they had
been living on leased property. Thereafter, defendants were charged in violation of the ordinance
and subsequently convicted. Hence this appeal.

Issue:

Whether or Not the ordinance is a valid exercise of police power.

Held:

No. It is not a valid exercise of police power. The ordinance is unreasonable and oppressive, in that
it operates to permanently deprive appellants of the right to use their own property; hence, it
oversteps the bounds of police power, and amounts to a taking of appellant’s property without just
compensation. We do not overlook that the modern tendency is to regard the beautification of
neighborhoods as conducive to the comfort and happiness of residents.

As the case now stands, every structure that may be erected on appellants' land, regardless of its
own beauty, stands condemned under the ordinance in question, because it would interfere with the
view of the public plaza from the highway. The appellants would, in effect, be constrained to let their
land remain idle and unused for the obvious purpose for which it is best suited, being urban in
character. To legally achieve that result, the municipality must give appellants just compensation
and an opportunity to be heard.

CITY OF BAGUIO V. NAWASA


[106 Phil; G.R. No. L-12032; 31 Aug 1959]
Page 37
Facts:

Plaintiff a municipal corporation filed a complaint against defendant a public corporation, created
under Act.1383. It contends that the said act does not include within its purview the Baguio Water
Works system, assuming that it does, is unconstitutional because it deprives the plaintiff ownership,
control and operation of said water works without just compensation and due process of law. The
defendant filed a motion to dismiss ion the ground that it is not a proper exercise of police power
and eminent domain. The court denied the motion and ordered the defendants to file an answer.
The court holds that the water works system of Baguio belongs to private property and cannot be
expropriated without just compensation. Sec. 8 of R.A.1383 provides for the exchange of the
NAWASA assets for the value of the water works system of Baguio is unconstitutional for this is not
just compensation. Defendants motion for reconsideration was denied hence this appeal.

Issue:

Whether or Not there is a valid exercise of police power of eminent domain.

Held:

R.A. 1383 does not constitute a valid exercise of police power. The act does not confiscate, destroy
or appropriate property belonging to a municipal corporation. It merely directs that all water works
belonging to cities, municipalities and municipal districts in the Philippines to be transferred to the
NAWASA. The purpose is placing them under the control and supervision of an agency with a view
to promoting their efficient management, but in so doing does not confiscate them because it directs
that they be paid with equal value of the assets of NAWASA.

The Baguio water works system is not like a public road, the park, street other public property held
in trust by a municipal corporation for the benefit of the public. But it is a property of a municipal
corporation, water works cannot be taken away except for public use and upon payment of just
compensation. Judgment affirmed.

NATIONAL POWER CORP. VS. GUTIERREZ


[193 SCRA 1; G.R. No. 60077; 18 Jan 1991]

Facts:

Petitioner filed an action to acquire a right of way over the land of Respondents for the construction
of transmission lines. Petitioner was adjudged to pay the full market value of land traversed by the
transmission lines. Petitioner argued that it was only asking for a right of way.

Issue:

Whether or Not the acquisition of the right of way constitutes "taking" and such the case will be
entitled just compensation.

Held:

The acquisition of the right of way constitutes taking. It perpetually deprives Respondents of their
proprietary rights. No plant higher than three meters is allowed below the transmission lines.
Because of high tension current conveyed through the transmission lines, danger to life and limbs
cannot be discounted. The owner of the property is entitled to just compensation.

REPUBLIC VS. CASTELVI


[58 SCRA 336; G.R. No. L-20620; 15 Aug 1974]
Page 38
Facts:

In 1947, the republic, through the Armed Forces of the Philippines (AFP), entered into a lease
agreement with Castelvi on a year-to-year basis. When Castelvi gave notice to terminate the lease
in 1956, the AFP refused. She then instituted an ejectment proceeding against the AFP. In 1959,
however, the republic commenced the expropriation proceedings for the land in question.

Issue:

Whether or Not the compensation should be determined as of 1947 or 1959.

Held:

The Supreme Court ruled that the “taking” should not be reckoned as of 1947, and that just
compensation should not be determined on the basis of the value of the property as of that year.

The requisites for taking are: 1) the expropriator must enter a private property, 2) the entry must be
for more than a momentary period, 3) it must be under warrant or color of authorities, 4) the
property must be devoted for public use or otherwise informally appropriated or injuriously affected,
and 5) the utilization of the property for public use must be such a way as to oust the owner and
deprive him of beneficial enjoyment of the property. Under Sec. 4 Rule 67 of the Rules of Court,
“just compensation” is to be determined as of the date of the filing of the complaint. The Supreme
Court has ruled that when the taking of the property sought to be expropriated coincides with the
commencement of the expropriation proceedings, or takes place subsequent to the filing of the
complaint for eminent domain, the just compensation should be determined as of the date of the
filing of the complaint. In the instant case, it is undisputed that the Republic was placed in
possession of the Castelvi property, by authority of court, on August 10, 1959. The “taking” of the
Castelvi property for the purposes of determining the just compensation to be paid must, therefore,
be reckoned as of June 26, 1959 when the complaint for eminent domain was filed. There is no
basis to the contention of the Republic that a lease on a year-to-year basis can give rise to
permanent right to occupy since by express provision a lease made for a determinate time, as was
the lease of Castelvi land in the instant case, ceases upon the day fixed, without need of a demand
(Art. 1669, New Civil Code). The Supreme Court, however, did not apply Art. 1250 of the New Civil
Code for the adjustment of the peso rate in times of extraordinary inflation or deflation because in
eminent domain cases the obligation to pay arises from law independent of contract.

EPZA VS. DULAY


[148 SCRA 305; G.R. No. L-59603; 29 Apr 1987]

Facts:

The four parcels of land which are the subject of this case is where the Mactan Export Processing
Zone Authority in Cebu (EPZA) is to be constructed. Private respondent San Antonio Development
Corporation (San Antonio, for brevity), in which these lands are registered under, claimed that the
lands were expropriated to the government without them reaching the agreement as to the
compensation. Respondent Judge Dulay then issued an order for the appointment of the
commissioners to determine the just compensation. It was later found out that the payment of the
government to San Antonio would be P15 per square meter, which was objected to by the latter
contending that under PD 1533, the basis of just compensation shall be fair and according to the fair
market value declared by the owner of the property sought to be expropriated, or by the assessor,
whichever is lower. Such objection and the subsequent Motion for Reconsideration were denied and
hearing was set for the reception of the commissioner’s report. EPZA then filed this petition for
certiorari and mandamus enjoining the respondent from further hearing the case.

Issue:
Page 39
Whether or Not the exclusive and mandatory mode of determining just compensation in PD 1533 is
unconstitutional.

Held:

The Supreme Court ruled that the mode of determination of just compensation in PD 1533 is
unconstitutional.

The method of ascertaining just compensation constitutes impermissible encroachment to judicial


prerogatives. It tends to render the courts inutile in a matter in which under the Constitution is
reserved to it for financial determination. The valuation in the decree may only serve as guiding
principle or one of the factors in determining just compensation, but it may not substitute the court’s
own judgment as to what amount should be awarded and how to arrive at such amount. The
determination of just compensation is a judicial function. The executive department or the legislature
may make the initial determination but when a party claims a violation of the guarantee in the Bill of
Rights that the private party may not be taken for public use without just compensation, no statute,
decree, or executive order can mandate that its own determination shall prevail over the court’s
findings. Much less can the courts be precluded from looking into the justness of the decreed
compensation.

AMIGABLE VS. CUENCA


[43 SCRA 360; G.R. No. L-26400; 29 Feb. 1972]

Facts:

Victoria Amigable is the registered owner of a particular lot. At the back of her Transfer Certificate of
Title (1924), there was no annotation in favor of the government of any right or interest in the
property. Without prior expropriation or negotiated sale, the government used a portion of the lot for
the construction of the Mango and Gorordo Avenues. On 1958, Amigable’s counsel wrote the
President of the Philippines, requesting payment of the portion of the said lot. It was disallowed by
the Auditor General in his 9th Endorsement. Petitioner then filed in the court a quo a complaint
against the Republic of the Philippines and Nicolas Cuenca, in his capacity as Commissioner of
Public Highways for the recovery of ownership and possession of the lot. According to the
defendants, the action was premature because it was not filed first at the Office of the Auditor
General. According to them, the right of action for the recovery of any amount had already
prescribed, that the Government had not given its consent to be sued, and that plaintiff had no
cause of action against the defendants.

Issue:

Whether or Not, under the facts of the case, appellant may properly sue the government.

Held:

In the case of Ministerio v. Court of First Instance of Cebu, it was held that when the government
takes away property from a private landowner for public use without going through the legal process
of expropriation or negotiated sale, the aggrieved party may properly maintain a suit against the
government without violating the doctrine of governmental immunity from suit without its consent. In
the case at bar, since no annotation in favor of the government appears at the back of the certificate
of title and plaintiff has not executed any deed of conveyance of any portion of the lot to the
government, then she remains the owner of the lot. She could then bring an action to recover
possession of the land anytime, because possession is one of the attributes of ownership. However,
since such action is not feasible at this time since the lot has been used for other purposes, the only

Page 40
relief left is for the government to make due compensation—price or value of the lot at the time of
the taking.

PHILIPPINE PRESS INSTITUTE VS. COMELEC


[244 SCRA 272; G.R. No. 119694; 22 May 1995]

Facts:

Respondent Comelec promulgated Resolution No. 2772 directing newspapers to provide free
Comelec space of not less than one-half page for the common use of political parties and
candidates. The Comelec space shall be allocated by the Commission, free of charge, among all
candidates to enable them to make known their qualifications, their stand on public Issue and their
platforms of government. The Comelec space shall also be used by the Commission for
dissemination of vital election information.

Petitioner Philippine Press Institute, Inc. (PPI), a non-profit organization of newspaper and
magazine publishers, asks the Supreme Court to declare Comelec Resolution No. 2772
unconstitutional and void on the ground that it violates the prohibition imposed by the Constitution
upon the government against the taking of private property for public use without just compensation.
On behalf of the respondent Comelec, the Solicitor General claimed that the Resolution is a
permissible exercise of the power of supervision (police power) of the Comelec over the information
operations of print media enterprises during the election period to safeguard and ensure a fair,
impartial and credible election.

Issue:

Whether or not Comelec Resolution No. 2772 is unconstitutional.

Held:

The Supreme Court declared the Resolution as unconstitutional. It held that to compel print media
companies to donate “Comelec space” amounts to “taking” of private personal property without
payment of the just compensation required in expropriation cases. Moreover, the element of
necessity for the taking has not been established by respondent Comelec, considering that the
newspapers were not unwilling to sell advertising space. The taking of private property for public
use is authorized by the constitution, but not without payment of just compensation. Also Resolution
No. 2772 does not constitute a valid exercise of the police power of the state. In the case at bench,
there is no showing of existence of a national emergency to take private property of newspaper or
magazine publishers.

REYES VS. NATIONAL HOUSING AUTHORITY


[395 SCRA 494; GR NO. 147511; 20 JAN 2003]

Facts:

Respondent National Housing Authority (NHA) filed complaints for the expropriation of sugarcane
lands belonging to the petitioners. The stated public purpose of the expropriation was the expansion
of the Dasmariñas Resettlement Project to accommodate the squatters who were relocated from
the Metropolitan Manila area. The trial court rendered judgment ordering the expropriation of these
lots and the payment of just compensation. The Supreme Court affirmed the judgment of the lower
court.
Page 41
A few years later, petitioners contended that respondent NHA violated the stated public purpose for
the expansion of the Dasmariñas Resettlement Project when it failed to relocate the squatters from
the Metro Manila area, as borne out by the ocular inspection conducted by the trial court which
showed that most of the expropriated properties remain unoccupied. Petitioners likewise question
the public nature of the use by respondent NHA when it entered into a contract for the construction
of low cost housing units, which is allegedly different from the stated public purpose in the
expropriation proceedings. Hence, it is claimed that respondent NHA has forfeited its rights and
interests by virtue of the expropriation judgment and the expropriated properties should now be
returned to herein petitioners.

Issue:

Whether or not the judgment of expropriation was forfeited in the light of the failure of respondent
NHA to use the expropriated property for the intended purpose but for a totally different purpose.

Held:

The Supreme Court held in favor of the respondent NHA. Accordingly, petitioners cannot insist on a
restrictive view of the eminent domain provision of the Constitution by contending that the contract
for low cost housing is a deviation from the stated public use. It is now settled doctrine that the
concept of public use is no longer limited to traditional purposes. The term "public use" has now
been held to be synonymous with "public interest," "public benefit," "public welfare," and "public
convenience." Thus, whatever may be beneficially employed for the general welfare satisfies the
requirement of public use."

In addition, the expropriation of private land for slum clearance and urban development is for a
public purpose even if the developed area is later sold to private homeowners, commercials firms,
entertainment and service companies, and other private concerns. Moreover, the Constitution itself
allows the State to undertake, for the common good and in cooperation with the private sector, a
continuing program of urban land reform and housing which will make at affordable cost decent
housing and basic services to underprivileged and homeless citizens in urban centers and
resettlement areas. The expropriation of private property for the purpose of socialized housing for
the marginalized sector is in furtherance of social justice.

MUNICIPALITY OF PARAÑAQUE VS. VM REALTY CORPORATION


[292 SCRA 676; G. R. NO. 127820; 20 JUL 1998]

Facts:

Petitioner sought to exercise its power of eminent domain based on a resolution by the municipal
council. Petitioner cites a previous case wherein a resolution gave authority to exercise eminent
domain. Petitioner also relies on the Implementing Rules, which provides that a resolution
authorizes a Local Government Unit to exercise eminent domain.

Issue:

Whether or Not an LGU can exercise its power of eminent domain pursuant to a resolution by its
law-making body.

Held:

Under Section 19, of the present Local Government Code (RA 7160), it is stated as the first
requisite that LGUs can exercise its power of eminent domain if there is an ordinance enacted by its
legislative body enabling the municipal chief executive. A resolution is not an ordinance, the former
is only an opinion of a law-making body, the latter is a law. The case cited by Petitioner involves BP
Page 42
337, which was the previous Local Government Code, which is obviously no longer in effect. RA
7160 prevails over the Implementing Rules, the former being the law itself and the latter only an
administrative rule which cannot amend the former.

ASLP VS. SEC. OF AGRARIAN REFORM


[175 SCRA 343; G.R. NO. 78742; 14 JUL 1989]

Facts:

Several petitions are the root of the case:

e. A petition alleging the constitutionality of PD No. 27, EO 228 and 229 and RA 6657.
Subjects of the petition are a 9-hectare and 5 hectare Riceland worked by four tenants.
Tenants were declared full owners by EO 228 as qualified farmers under PD 27. The
petitioners now contend that President Aquino usurped the legislature’s power.
f. A petition by landowners and sugarplanters in Victoria’s Mill Negros Occidental against
Proclamation 131 and EO 229. Proclamation 131 is the creation of Agrarian Reform
Fund with initial fund of P50Billion.
g. A petition by owners of land which was placed by the DAR under the coverage of
Operation Land Transfer.
h. A petition invoking the right of retention under PD 27 to owners of rice and corn lands not
exceeding seven hectares.

Issue:

Whether or Not the aforementioned EO’s, PD, and RA were constitutional.

Held:

The promulgation of PD 27 by President Marcos was valid in exercise of Police power and eminent
domain.

The power of President Aquino to promulgate Proc. 131 and EO 228 and 229 was authorized under
Sec. 6 of the Transitory Provisions of the 1987 Constitution. Therefore it is a valid exercise of Police
Power and Eminent Domain.

RA 6657 is likewise valid. The carrying out of the regulation under CARP becomes necessary to
deprive owners of whatever lands they may own in excess of the maximum area allowed, there is
definitely a taking under the power of eminent domain for which payment of just compensation is
imperative. The taking contemplated is not a mere limitation of the use of the land. What is required
is the surrender of the title and the physical possession of said excess and all beneficial rights
accruing to the owner in favour of the farmer.

A statute may be sustained under the police power only if there is concurrence of the lawful subject
and the method.

Subject and purpose of the Agrarian Reform Law is valid, however what is to be determined is the
method employed to achieve it.

ESLABAN VS. ONORIO


[360 SCRA 230; G.R. NO. 146062; 28 JUN 2001]

Page 43
Facts:

Clarita Vda. De Onorio is the owner of the land in Barangay M. Roxas, Sto. Nino, South Cotabato.
Such land is the subject for the construction of an irrigation canal of the National Irrigation
Administration (NIA). Mr. Santiago Eslaban Jr. is the project manager of NIA. The parties agreed to
the construction of the canal provided that the government will pay for the area that has been taken.
A right-of-way agreement was entered into by the parties in which respondent was paid the amount
of P4, 180.00 as right of way damages. Subsequently, respondent executed an Affidavit of Waiver
of Rights and Fees which waives her rights for the damage to the crops due to construction of the
right of way. After which, respondent demands that petitioner pay P111, 299.55 for taking her
property but the petitioner refused. Petitioner states that the government had not consented to be
sued and that the respondent is not entitled for compensation by virtue of the homestead patent
under CA no. 141. The RTC held that the NIA should pay respondent the amount of P107, 517.60
as just compensation for the 24,660 sq meters that have been used for the construction of the
canal. The Court of Appeals also affirmed the decision of the RTC.

Issue:

Whether or Not the CA erred in affirming the decision of the RTC.

Held:

The CA is correct in affirming the decision of the RTC but modifications shall be made regarding the
value of the just compensation. The following are the points to be considered in arriving in this
decision.

First, Rule 7 par 5 of the Rule of Civil Procedure provides that the certification against forum
shopping should only be executed by the plaintiff or the principal. The petition for review was filed
by Mr. Eslaban jr. while the verification or certification were signed by Mr. Cesar Gonzales, an
administrator of the agency. Neither of the two has the authority to sign such certificate for they are
not the plaintiff or principal. Such case is a sufficient ground for dismissing this petition.

Second, PD NO. 1529 provides that the owner is required to recognize in favor of the government
the easement of a “public highway, way, private way established by law, or any government canal
where the certificate of title does not state that the boundaries thereof have been pre-determined. In
the case at bar, the irrigation canal was constructed on Oct 1981 after the property had been
registered in May of 1976. In this case, prior expropriation proceedings must be filed and just
compensation shall be paid to the owner before the land could be taken for public use.

Third, In this case, just compensation is defined as not only the correct amount to be paid but the
reasonable time for the Government to pay the owner. The CA erred in this point by stating that the
market value (just compensation) of the land is determined in the filing of the complaint in 1991.The
determination of such value should be from the time of its taking by the NIA in 1981.

Lastly, the petitioner cannot argue that the Affidavit of waiver of rights and fees executed by the
respondent pertains to the payment of the value of the land therefore exempting NIA to pay the
value of the land taken. Such waiver pertains only to the crops and improvements that were
damage due to the construction of the right-of-way not the value of the land.

Wherefore, decision of CA affirmed with modification regarding the just compensation in the amount
of P16, 047.61 per hectare.

KNECHT VS. COURT OF APPEALS


[290 SCRA 223; G.R. NO. 108015, 20 MAY 1998]

Facts:
Page 44
The instant case is an unending sequel to several suits commenced almost twenty years ago
involving a parcel of land located at the corner of the south end of EDSA and F.B. Harrison in
Pasay City. The land was owned by petitioners Cristina de Knecht and her son, Rene Knecht. On
the land, the Knechts constructed eight houses, leased out the seven and occupied one of them as
their residence. In 1979, the government filed for the expropriation of Knechts’ property. The
government wanted to use the land for the completion of the Manila Flood Control and Drainage
Project and the extension of the EDSA towards Roxas Boulevard. In 1982, the City Treasurer of
Pasay discovered that the Knechts failed to pay real estate taxes on the property from 1980 to
1982. As a consequence of this deficiency, the City Treasurer sold the property at public auction for
the same amount of their deficiency taxes. The highest bidders were respondent Spouses
Anastacio and Felisa Babiera (the Babieras) and respondent Spouses Alejandro and Flor
Sangalang (the Sangalangs). Subsequently, Sangalang and Babiera sold the land to respondent
Salem Investment Corporation. On February 17, 1983, the Batasang Pambansa passed B.P. Blg.
340 authorizing the national government to expropriate certain properties in Pasay City for the
EDSA Extension. The property of the Knechts was part of those expropriated under B.P. Blg. 340.
The government gave out just compensation for the lands expropriated under B.P. Blg. 340. Salem
was included and received partial payment. Seven of the eight houses of the Knechts were
demolished and the government took possession of the portion of land on which the houses stood.
Since the Knechts refused to vacate their one remaining house, Salem filed a case against them for
unlawful detainer. As defense, the Knechts claimed ownership of the land and building. The
Municipal Trial Court however ordered the Knechts' ejectment thus their residence was demolished.

The Knechts continuously claimed ownership of the property and allege that they must be given just
compensation.

Issue:

Whether or not Knechts are the lawful owners of the land at subject.

Held:

The Supreme Court held that the Knechts were not the owners anymore of the said land. The
Knechts' right to the land had been foreclosed after they failed to redeem it one year after the sale
at public auction. Since the petitions questioning the order of dismissal were likewise dismissed by
the Court of Appeals and this Court, the order of dismissal became final and res judicata on the
issue of ownership of the land. Petitioners contended that they did not receive notice of their tax
delinquency. Neither did they receive notice of the auction sale. However, this question has been
previously raised in the cases which have been already set aside. The court is not a trier of facts.
Res judicata has already set it. The Knechts therefore are not the lawful owners of the land and are
not any longer accountable for just compensation given by the government.

Note: Res judicata is a ground for dismissal of an action. It is a rule that precludes parties from
relitigating Issue actually litigated and determined by a prior and final judgment. It pervades every
well-regulated system of jurisprudence, and is based upon two grounds embodied in various
maxims of the common law — one, public policy and necessity, that there should be a limit to
litigation; and another, the individual should not be vexed twice for the same cause. When a right of
fact has been judicially tried and determined by a court of competent jurisdiction, or an opportunity
for such trial has been given, the judgment of the court, so long as it remains unreversed, should be
conclusive upon the parties and those in privity with them in law or estate. To follow a contrary
doctrine would subject the public peace and quiet to the will and neglect of individuals and prefer
the gratification of the litigious disposition of the parties to the preservation of the public tranquility.

Res judicata applies when: (1) the former judgment or order is final; (2) the judgment or order is one
on the merits; (3) it was rendered by a court having jurisdiction over the subject matter and the
parties; (4) there is between the first and second actions, identity of parties, of subject matter and of
cause of action.
Page 45
REPUBLIC VS. KER
[383 SCRA 584; G.R. NO. 136171, 2 JULY 2002]

Facts:

Petitioner filed before the Regional Trial Court of Davao City a petition for expropriation of portions
of two parcels of land owned by respondent. Petitioner needed the parcels of land for the widening
of the road component of J.P. Laurel-Buhangin Interchange in Davao City. The Regional trial court
rendered decision of a fair just compensation for defendant Ker Corporation. However, it was
challenged by Petitioner Republic of the Philippines, represented by the Department of Public
Works and Highways alleging that just compensation for site must be reduced. Petitioner alleged
that when the petition for expropriation was filed, the tax declaration of the property indicated its
assessed value at a lower price.

Issue:

Whether or not respondent Ker Company was given a decision for fair just compensation.

Held:

The Supreme Court held that the valuation for the lot Sites are excessive and unreasonable. Just
compensation cannot be measured by the assessed value of the property as stated in the tax
declaration and schedule of market values. For the purpose of appraisal, the fair market value of the
property is taken into account and such value refers to the highest price in terms of money which a
property will bring if exposed for sale in the public market.

In computing just compensation for expropriation proceedings, it is the value of the land at the time
of the taking or at the time of the filing of the complaint not at the time of the rendition of judgment
which should be taken into consideration. 4 Section 4, Rule 67 of the 1997 Rules of Civil Procedure
provides that just compensation is to be determined as of the date of the taking or the filing of the
complaint whichever came first. On this matter, the appellate court is correct in disregarding
petitioner's claim.

MANOSCA VS. COURT OF APPEALS


[252 SCRA 412; G.R. NO. 106440, 29 JAN. 1996]

Facts:

The National Historical Institute declared the parcel of land owned by Petitioners as a national
historical landmark, because it was the site of the birth of Felix Manalo, the founder of Iglesia ni
Cristo. The Republic of the Philippines filed an action to appropriate the land. Petitioners argued
that the expropriation was not for a public purpose.

Issue:

Whether or Not the taking or exercise of eminent domain may be granted.

Held:

Public use should not be restricted to the traditional uses. The taking is for a public use because of
the contribution of Felix Manalo to the culture and history of the Philippines.

Page 46
THE POWER OF TAXATION

PASCUAL VS. SEC. OF PUBLIC WORKS


[110 PHIL 331; G.R. NO.L-10405; 29 DEC 1960]

Facts:

Petitioner, the governor of the Province of Rizal, filed an action for declaratory relief with injunction
on the ground that RA 920, Act appropriating funds for public works, providing P85,000 for the
construction, reconstruction, repair, extension and improvement of Pasig feeder road terminals,
were nothing but projected and planned subdivision roads within Antonio Subdivision. Antonio
Subdivision is owned by the respondent, Jose Zulueta, a member of the Senate of the Philippines.
Respondent offered to donate the said feeder roads to the municipality of Pasig and the offer was
accepted by the council, subject to a condition that the donor would submit plan of the roads and an
agreement to change the names of two of the street. However, the donation was not executed,
which prompted Zuleta to write a letter to the district engineer calling attention the approval of RA
920. The district engineer, on the other hand, did not endorse the letter that inasmuch the feeder
roads in question were private property at the time of passage and approval of RA 920, the
appropriation for the construction was illegal and therefore, void ab initio. Petitioner, prayed for RA
920 be declared null and void and the alleged deed of donation be declared unconstitutional. Lower
court dismissed the case and dissolved the writ of preliminary injunction.

Issue:

Whether or Not the deed of donation and the appropriation of funds stipulated in RA 920 are
constitutional.

Held:

The ruling case law rules that the legislature is without power to appropriate public revenue for
anything but public purpose. The taxing power must be exercised for public purposes only and the
money raised by taxation can be expended only for public purposes and not for the advantage of
private individuals.

In the case at bar, the legality of the appropriation of the feeder roads depend upon whether the
said roads were public or private property when the bill was passed by congress or when it became
effective. The land which was owned by Zulueta, the appropriation sought a private purpose and
hence, null and void. The donation did not cure the nullity of the appropriation; therefore a judicial
nullification of a said donation need not precede the declaration of unconstitutionality of the said
appropriation.

The decision appealed from is reversed.

PUNSALAN VS. MUNICIPAL BOARD OF MANILA


[95 PHIL 46; NO.L-4817; 26 MAY 1954]

Facts:

Petitioners, who are professionals in the city, assail Ordinance No. 3398 together with the law
authorizing it (Section 18 of the Revised Charter of the City of Manila). The ordinance imposes a
municipal occupation tax on persons exercising various professions in the city and penalizes non-
Page 47
payment of the same. The law authorizing said ordinance empowers the Municipal Board of the city
to impose a municipal occupation tax on persons engaged in various professions. Petitioners,
having already paid their occupation tax under section 201 of the National Internal Revenue Code,
paid the tax under protest as imposed by Ordinance No. 3398. The lower court declared the
ordinance invalid and affirmed the validity of the law authorizing it.

Issue:

Whether or Not the ordinance and law authorizing it constitute class legislation, and authorize what
amounts to double taxation.

Held:

The Legislature may, in its discretion, select what occupations shall be taxed, and in its discretion
may tax all, or select classes of occupation for taxation, and leave others untaxed. It is not for the
courts to judge which cities or municipalities should be empowered to impose occupation taxes
aside from that imposed by the National Government. That matter is within the domain of political
departments. The argument against double taxation may not be invoked if one tax is imposed by
the state and the other is imposed by the city. It is widely recognized that there is nothing inherently
terrible in the requirement that taxes be exacted with respect to the same occupation by both the
state and the political subdivisions thereof. Judgment of the lower court is reversed with regards to
the ordinance and affirmed as to the law authorizing it.

OSMEÑA VS. ORBOS


[220 SCRA 703; G.R. NO. 99886; 31 MAR 1993]

Facts:

On October 10, 1984, Pres. Marcos issued P.D. 1956 creating a Special Account in the General
Fund, designated as the Oil Price Stabilization Fund (OPSF). The OPSF was designed to reimburse
oil companies for cost increases in crude oil and imported petroleum products resulting from
exchange rate adjustments and from increases in the world market prices of crude oil.

Subsequently, the OPSF was reclassified into a "trust liability account," in virtue of E.O. 1024, and
ordered released from the National Treasury to the Ministry of Energy.

Pres. Aquino, amended P.D. 1956. She promulgated Executive Order No. 137 on February 27,
1987, expanding the grounds for reimbursement to oil companies for possible cost underrecovery
incurred as a result of the reduction of domestic prices of petroleum products, the amount of the
underrecovery being left for determination by the Ministry of Finance.
The petition avers that the creation of the trust fund violates
29(3), Article VI of the Constitution, reading as follows:

(3) All money collected on any tax levied for a special purpose shall be treated as a
special fund and paid out for such purposes only. If the purpose for which a special
fund was created has been fulfilled or abandoned, the balance, if any, shall be
transferred to the general funds of the Government.

The petitioner argues that "the monies collected pursuant to . . P.D. 1956, as amended, must be
treated as a 'SPECIAL FUND,' not as a 'trust account' or a 'trust fund,' and that "if a special tax is
collected for a specific purpose, the revenue generated therefrom shall 'be treated as a special fund'
to be used only for the purpose indicated, and not channeled to another government objective."
Petitioner further points out that since "a 'special fund' consists of monies collected through the
taxing power of a State, such amounts belong to the State, although the use thereof is limited to the
special purpose/objective for which it was created."

Page 48
He also contends that the "delegation of legislative authority" to the ERB violates 28 (2). Article VI of
the Constitution, viz.:

(2) The Congress may, by law, authorize the President to fix, within specified limits,
and subject to such limitations and restrictions as it may impose, tariff rates, import
and export quotas, tonnage and wharfage dues, and other duties or imposts within
the framework of the national development program of the Government;

and, inasmuch as the delegation relates to the exercise of the power of taxation, "the limits,
limitations and restrictions must be quantitative, that is, the law must not only specify how to tax,
who (shall) be taxed (and) what the tax is for, but also impose a specific limit on how much to tax." 12
Issue:

Whether or Not the invalidity of the "TRUST ACCOUNT" in the books of account of the Ministry of
Energy (now, the Office of Energy Affairs), created pursuant to § 8, paragraph 1, of P.D. No. 1956,
as amended, "said creation of a trust fund being contrary to Section 29 (3), Article VI of the
Constitution.

Whether or Not the unconstitutionality of 8, paragraph 1 (c) of P.D. No. 1956, as amended by
Executive Order No. 137, for "being an undue and invalid delegation of legislative power to the
Energy Regulatory Board.

Held:

The OPSF is a "Trust Account" which was established "for the purpose of minimizing the frequent
price changes brought about by exchange rate adjustment and/or changes in world market prices of
crude oil and imported petroleum products." Under P.D. No. 1956, as amended by Executive Order
No. 137 dated 27 February 1987, this Trust Account may be funded from any of the following
sources:

a) Any increase in the tax collection from ad valorem tax or customs duty imposed on
petroleum products subject to tax under this Decree arising from exchange rate adjustment,
as may be determined by the Minister of Finance in consultation with the Board of Energy;

b) Any increase in the tax collection as a result of the lifting of tax exemptions of government
corporations, as may be determined by the Minister of Finance in consultation with the Board
of Energy;

c) Any additional amount to be imposed on petroleum products to augment the resources of


the Fund through an appropriate Order that may be issued by the Board of Energy requiring
payment of persons or companies engaged in the business of importing, manufacturing
and/or marketing petroleum products;

d) Any resulting peso cost differentials in case the actual peso costs paid by oil companies in
the importation of crude oil and petroleum products is less than the peso costs computed
using the reference foreign exchange rate as fixed by the Board of Energy.

Hence, it seems clear that while the funds collected may be referred to as taxes, they are exacted in
the exercise of the police power of the State. Moreover, that the OPSF is a special fund is plain from
the special treatment given it by E.O. 137. It is segregated from the general fund; and while it is
placed in what the law refers to as a "trust liability account," the fund nonetheless remains subject to
the scrutiny and review of the COA. The Court is satisfied that these measures comply with the
constitutional description of a "special fund." Indeed, the practice is not without precedent.

Page 49
With regard to the alleged undue delegation of legislative power, the Court finds that the provision
conferring the authority upon the ERB to impose additional amounts on petroleum products
provides a sufficient standard by which the authority must be exercised. In addition to the general
policy of the law to protect the local consumer by stabilizing and subsidizing domestic pump rates, §
8(c) of P.D. 1956 expressly authorizes the ERB to impose additional amounts to augment the
resources of the Fund.

What petitioner would wish is the fixing of some definite, quantitative restriction, or "a specific limit
on how much to tax." The Court is cited to this requirement by the petitioner on the premise that
what is involved here is the power of taxation; but as already discussed, this is not the case. What is
here involved is not so much the power of taxation as police power. Although the provision
authorizing the ERB to impose additional amounts could be construed to refer to the power of
taxation, it cannot be overlooked that the overriding consideration is to enable the delegate to act
with expediency in carrying out the objectives of the law which are embraced by the police power of
the State.

The interplay and constant fluctuation of the various factors involved in the determination of the
price of oil and petroleum products, and the frequently shifting need to either augment or exhaust
the Fund, do not conveniently permit the setting of fixed or rigid parameters in the law as proposed
by the petitioner. To do so would render the ERB unable to respond effectively so as to mitigate or
avoid the undesirable consequences of such fluidity. As such, the standard as it is expressed
suffices to guide the delegate in the exercise of the delegated power, taking account of the
circumstances under which it is to be exercised.

LLADOC VS. COMMISSIONER OF INTERNAL REVENUE


[14 SCRA 292; NO.L-19201; 16 JUN 1965]

Facts:

Sometime in 1957, M.B. Estate Inc., of Bacolod City, donated 10,000.00 pesos in cash to Fr. Crispin
Ruiz, the parish priest of Victorias, Negros Occidental, and predecessor of Fr. Lladoc, for the
construction of a new Catholic church in the locality. The donated amount was spent for such
purpose.

On March 3, 1958, the donor M.B. Estate filed the donor's gift tax return. Under date of April 29,
1960. Commissioner of Internal Revenue issued an assessment for the donee's gift tax against the
Catholic Parish of Victorias of which petitioner was the parish priest.

Issue:

Whether or not the imposition of gift tax despite the fact the Fr. Lladoc was not the Parish priest at
the time of donation, Catholic Parish priest of Victorias did not have juridical personality as the
constitutional exemption for religious purpose is valid.

Held:

Yes, imposition of the gift tax was valid, under Section 22(3) Article VI of the Constitution
contemplates exemption only from payment of taxes assessed on such properties as Property taxes
contra distinguished from Excise taxes The imposition of the gift tax on the property used for
religious purpose is not a violation of the Constitution. A gift tax is not a property by way of gift inter
vivos.

The head of the Diocese and not the parish priest is the real party in interest in the imposition of the
donee's tax on the property donated to the church for religious purpose.
Page 50
CASSANOVAS VS. HORD
[8 Phil 125; No. 3473; 22 Mar 1907]

Facts:

The Spanish Govt. by virtue of a royal decree granted the plaintiff certain mines. The plaintiff is now
the owner of those mines. The Collector of Internal Revenue imposed tax on the properties,
contending that they were valid perfected mine concessions and it falls within the provisions of
sec.134 of Act No. 1189 known as Internal Revenue Act. The plaintiff paid under protest. He
brought an action against the defendant Collector of Internal Revenue to recover the sum of Php. 9,
600 paid by him as taxes. Judgment was rendered in favor of the defendant, so the plaintiff
appealed.

Issue:

Whether or Not Sec. 164 is void or valid.

Held:

The deed constituted a contract between the Spanish Government and the plaintiff. The obligation
of which contract was impaired by the enactment of sec. 134 of the Internal Revenue Law infringing
sec. 5 of the Act of Congress which provides that “no law impairing the obligation of contracts shall
be enacted”. Sec. 134 of the Internal Revenue Law of 1904 is void because it impairs the obligation
of contracts contained in the concessions of mine made by the Spanish Government. Judgment
reversed.

Page 51
THE BILL
OF RIGHTS

Page 52
DUE PROCESS OF LAW

Art 3, Sec. 1. “No person shall be deprived of life, liberty, or property without due process of law…”

ERMITA-MALATE HOTEL AND MOTEL OPERATORS ASSO. VS. MAYOR OF MANILA


[20 SCRA 849; G.R. NO.L-24693; 31 JULY 1967]

Facts:

Petitioners Ermita-Malate Hotel and Motel Operators Association with one of its members, Hotel del
Mar Inc., and Go Chiu, the president and general manager of the second petitioner, filed a petition
for prohibition against Ordinance No. 4760 against the respondent Mayor of the City of Manila who
was sued in his capacity as such charged with the general power and duty to enforce ordinances of
the City of Manila and to give the necessary orders for the execution and enforcement of such
ordinances. It was alleged that the petitioner non-stock corporation is dedicated to the promotion
and protection of the interest of its eighteen members operating hotels and motels, characterized as
legitimate businesses duly licensed by both national and city authorities and regularly paying taxes.
It was alleged that on June 13, 1963, the Municipal Board of the City of Manila enacted Ordinance
No. 4760, approved on June 14, 1963 by the then acting City Mayor, Vice-Mayor Herminio Astorga.
After which the alleged grievances against the ordinance were set forth in detail. There was the
assertion of its being beyond the powers of the Municipal Board of the City of Manila to enact
insofar as it regulate motels, on the ground that in the revised charter of the City of Manila or in any
other law, no reference is made to motels. it also being provided that the premises and facilities of
such hotels, motels and lodging houses would be open for inspection either by the City Mayor, or
the Chief of Police, or their duly authorized representatives. The lower court on July 6, 1963 issued
a writ of preliminary injunction ordering respondent Mayor to refrain from enforcing said Ordinance
No. 4760 from and after July 8, 1963.

Issue:

Whether or Not Ordinance No. 4760 of the City of Manila is unconstitutional, therefore, null and
void.

Held:

A decent regard for constitutional doctrines of a fundamental character ought to have admonished
the lower court against such a sweeping condemnation of the challenged ordinance. Its decision
cannot be allowed to stand, consistently with what has been the accepted standards of
constitutional adjudication, in both procedural and substantive aspects.

Primarily what calls for a reversal of such a decision is the absence of any evidence to offset the
presumption of validity that attaches to a challenged statute or ordinance. As was expressed
categorically by Justice Malcolm: "The presumption is all in favor of validity x x x . The action of the
elected representatives of the people cannot be lightly set aside. The councilors must, in the very
nature of things, be familiar with the necessities of their particular municipality and with all the facts
and circumstances which surround the subject and necessitate action. The local legislative body, by
enacting the ordinance, has in effect given notice that the regulations are essential to the well being
of the people x x x . The Judiciary should not lightly set aside legislative action when there is not a
clear invasion of personal or property rights under the guise of police regulation.

It admits of no doubt therefore that there being a presumption of validity, the necessity for evidence
to rebut it is unavoidable, unless the statute or ordinance is void on its face which is not the case
here. The principle has been nowhere better expressed than in the leading case of O'Gorman &
Young v. Hartford Fire Insurance Co. where the American Supreme Court through Justice Brandeis
tersely and succinctly summed up the matter thus: The statute here questioned deals with a subject
Page 53
clearly within the scope of the police power. We are asked to declare it void on the ground that the
specific method of regulation prescribed is unreasonable and hence deprives the plaintiff of due
process of law. As underlying questions of fact may condition the constitutionality of legislation of
this character, the resumption of constitutionality must prevail in the absence of some factual
foundation of record for overthrowing the statute." No such factual foundation being laid in the
present case, the lower court deciding the matter on the pleadings and the stipulation of facts, the
presumption of validity must prevail and the judgment against the ordinance set aside.

VILLEGAS VS. HIU CHIONG


[86 SCRA 270; NO.L-29646; 10 NOV 1978]

Facts:

The controverted Ordinance no. 6537 was passed by the Municipal Board of Manila on February
22, 1968 and signed by Mayor Villegas. It is an ordinance making it unlawful for any person not a
citizen of the Philippines to be employed in any place of employment or to be engaged in any
kind of trade business or occupation within the city of Manila without securing an employment
permit from the Mayor of Manila and for other purposes.

Hiu Chiong Tsai Pao Ho, who was employed in Manila filed a petition praying for the writ of
preliminary injunction and restraining order to stop the enforcement of said ordinance.

Issue:

Whether or Not Ordinance no.6537 violates the due process and equal protection clauses of the
Constitution.

Held:

It is a revenue measure. The city ordinance which imposes a fee of 50.00 pesos to enable aliens
generally to be employed in the city of Manila is not only for the purpose of regulation.

While it is true that the first part which requires the alien to secure an employment permit from the
Mayor involves the exercise of discretion and judgment in processing and approval or disapproval of
application is regulatory in character, the second part which requires the payment of a sum of
50.00 pesos is not a regulatory but a revenue measure.

Ordinance no. 6537 is void and unconstitutional. This is tantamount to denial of the basic human
right of the people in the Philippines to engaged in a means of livelihood. While it is true that the
Philippines as a state is not obliged to admit aliens within it's territory, once an alien is admitted he
cannot be deprived of life without due process of law. This guarantee includes the means of
livelihood. Also it does not lay down any standard to guide the City Mayor in the issuance or
denial of an alien employment permit fee.

NAMIL VS. COMELEC


[414 SCRA 553; G.R. NO. 150540; 28 OCT 2003]

Facts:

On May 20, 2001, the Municipal Board of Canvassers of Palimbang, Sultan Kudarat proclaimed the
petitioners as winning candidates for their Sangguniang Bayan. The following day, herein private
respondents were proclaimed winners as well. Private respondents claimed that they should be
recognized as the winners, and not the petitioners. Upon receipt of such letter, the Commissioner-
in-charge for Region XII asked the Law Department, the Regional Election Registrar and the
Provincial Elections Supervisor to submit their reports on the matter. All of them found the second
Page 54
proclamation valid. Hence, the COMELEC issued a Resolution ordering the immediate installation
of the private respondents as the newly elected members of the Sangguniang Bayan, even though
petitioners herein have already taken their oath and have assumed office. Petitioners contend that
such Resolution is null and void because they were not accorded due notice and hearing, hence
constituting a violation of the due process principle.

Issue:

Whether or Not due the COMELEC has the power to suspend a proclamation or the effects thereof
without notice and hearing.

Held:

No. The COMELEC is without power to partially or totally annul a proclamation or suspend the
effects of a proclamation without notice and hearing. The proclamation on May 20, 2001 enjoys the
presumption of regularity and validity since no contest or protest was even filed assailing the same.
The petitioners cannot be removed from office without due process of law. Due process in quasi-
judicial proceedings before the COMELEC requires due notice and hearing. Furthermore, the
proclamation of a winning candidate cannot be annulled if he has not been notified of any motion to
set aside his proclamation. Hence, as ruled in Fariñas vs. COMELEC, Reyes vs. COMELEC and
Gallardo vs. COMELEC, the COMELEC is without power to partially or totally annul a proclamation
or suspend the effects of a proclamation without notice and hearing.
ICHONG VS. HERNANDEZ
[101 PHIL 1155; L-7995; 31 MAY 1957]

Facts:

Republic Act 1180 or commonly known as “An Act to Regulate the Retail Business” was passed.
The said law provides for a prohibition against foreigners as well as corporations owned by
foreigners from engaging from retail trade in our country. This was protested by the petitioner in this
case. According to him, the said law violates the international and treaty of the Philippines therefore
it is unconstitutional. Specifically, the Treaty of Amity between the Philippines and China was
violated according to him.

Issue:

Whether or Not Republic Act 1180 is a valid exercise of police power.

Held:

According to the Court, RA 1180 is a valid exercise of police power. It was also then provided that
police power can not be bargained away through the medium of a treaty or a contract. The Court
also provided that RA 1180 was enacted to remedy a real and actual danger to national economy
posed by alien dominance and control. If ever the law infringes upon the said treaty, the latter is
always subject to qualification or amendment by a subsequent law and the same may never curtain
or restrict the scope of the police power of the state.

PHIL. PHOSPHATE FERTILIZER CORP. VS. TORRES


[231 SCRA 335; G.R. NO.98050; 17 MAR 1994]

Facts:

Philphos Movement for Progress, Inc. (PMPI for brevity), filed with the Department of Labor and
Employment a petition for certification election among the supervisory employees of petitioner,
alleging that as a supervisory union duly registered with the Department of Labor and Employment it
was seeking to represent the supervisory employees of Philippine Phosphate Fertilizer Corporation.
Page 55
Mediator-Arbiter Rodolfo S. Milado issued an order directing the holding of a certification election
among the supervisory employees of petitioner, excluding therefrom the superintendents and the
professional and technical employees. However, the PMPI filed an amended petition with the
Mediator-Arbiter wherein it sought to represent not only the supervisory employees of petitioner but
also its professional/technical and confidential employees. The parties therein agreed to submit their
respective position papers and to consider the amended petition submitted for decision on the basis
thereof and related documents. Mediator-Arbiter Milado issued an order granting the petition and
directing the holding of a certification election among the "supervisory, professional (engineers,
analysts, mechanics, accountants, nurses, midwives, etc.), technical, and confidential employees.
PHILPHOS appealed the order to the Secretary of Labor and Employment who rendered a decision
through Undersecretary Bienvenido Laguesma dismissing the appeal. PHILPHOS moved for
reconsideration but the same was denied; hence, the instant petition alleging denial of due process
on the part of the DOLE to which the mediator-arbiter was under.

Issue:

Whether or Not there was denial of due process.

Held:

There was no denial of due process. The essence of due process is simply an opportunity to be
heard or, as applied to administrative proceedings, an opportunity to explain one's side or an
opportunity to seek a reconsideration of the action or ruling complained of petitioner PHILPHOS
agreed to file its position paper with the Mediator-Arbiter and to consider the case submitted for
decision on the basis of the position papers filed by the parties, there was sufficient compliance with
the requirement of due process, as petitioner was afforded reasonable opportunity to present its
side. Moreover, petitioner could have, if it so desired, insisted on a hearing to confront and examine
the witnesses of the other party. But it did not; instead it opted to submit its position paper with the
Mediator-Arbiter. Besides, petitioner had all the opportunity to ventilate its arguments in its appeal to
the Secretary of Labor.

RUBI VS. PROVINCIAL BOARD OF MINDORO


[39 PHIL 660; G.R. NO. 14078; 7 MAR 1919]

Facts:

This is an application for habeas corpus in favor of Rubi and other Manguianes of the Province of
Mindoro.

The provincial board of Mindoro adopted resolution No. 25 which states that “provincial governor of
any province in which non-Christian inhabitants (uncivilized tribes) are found is authorized, when
such a course is deemed necessary in the interest of law and order, to direct such inhabitants to
take up their habitation on sites on unoccupied public lands to be selected by him and approved by
the provincial board”. It is resolved that under section 2077 of the Administrative Code, 800
hectares of public land in the sitio of Tigbao on Naujan Lake be selected as a site for the permanent
settlement of Mangyanes in Mindoro. Further, Mangyans may only solicit homesteads on this
reservation providing that said homestead applications are previously recommended by the
provincial governor.

Thereafter, the provincial governor of Mindoro issued executive order No. 2, which says that the
provincial governor has selected a site in the sitio of Tigbao on Naujan Lake for the permanent
settlement of Mangyanes in Mindoro. In that case, pursuant to Section 2145 of the Revised
Administrative Code, all the Mangyans in the townships of Naujan and Pola and the Mangyans east
of the Baco River including those in the districts of Dulangan and Rubi's place in Calapan, were
ordered to take up their habitation on the site of Tigbao, Naujan Lake. Also, that any Mangyan who

Page 56
shall refuse to comply with this order shall upon conviction be imprisoned not exceed in sixty days,
in accordance with section 2759 of the revised Administrative Code.

Said resolution of the provincial board of Mindoro were claimed as necessary measures for the
protection of the Mangyanes of Mindoro as well as the protection of public forests in which they
roam, and to introduce civilized customs among them.

It appeared that Rubi and those living in his rancheria have not fixed their dwelling within the
reservation of Tigbao and are liable to be punished.

It is alleged that the Manguianes are being illegally deprived of their liberty by the provincial officials
of that province. Rubi and his companions are said to be held on the reservation established at
Tigbao, Mindoro, against their will, and one Dabalos is said to be held under the custody of the
provincial sheriff in the prison at Calapan for having run away form the reservation.

Issue:

Whether or Not Section 2145 of the Administrative Code deprive a person of his liberty without due
process of law.

Whether or Not Section 2145 of the Administrative Code of 1917 is constitutional.

Held:

The Court held that section 2145 of the Administrative Code does not deprive a person of his liberty
without due process of law and does not deny to him the equal protection of the laws, and that
confinement in reservations in accordance with said section does not constitute slavery and
involuntary servitude. The Court is further of the opinion that section 2145 of the Administrative
Code is a legitimate exertion of the police power, somewhat analogous to the Indian policy of the
United States. Section 2145 of the Administrative Code of 1917 is constitutional.

The preamble of the resolution of the provincial board of Mindoro which set apart the Tigbao
reservation, it will be read, assigned as reasons fort the action, the following: (1) The failure of
former attempts for the advancement of the non-Christian people of the province; and (2) the only
successfully method for educating the Manguianes was to oblige them to live in a permanent
settlement. The Solicitor-General adds the following; (3) The protection of the Manguianes; (4) the
protection of the public forests in which they roam; (5) the necessity of introducing civilized customs
among the Manguianes.

Considered purely as an exercise of the police power, the courts cannot fairly say that the
Legislature has exceeded its rightful authority. It is, indeed, an unusual exercise of that power. But a
great malady requires an equally drastic remedy. One cannot hold that the liberty of the citizen is
unduly interfered without when the degree of civilization of the Manguianes is considered. They are
restrained for their own good and the general good of the Philippines. Nor can one say that due
process of law has not been followed.

None of the rights of the citizen can be taken away except by due process of law. To constitute "due
process of law," as has been often held, a judicial proceeding is not always necessary. In some
instances, even a hearing and notice are not requisite a rule which is especially true where much
must be left to the discretion of the administrative officers in applying a law to particular cases.

The idea of the provision in question is to unify the people of the Philippines so that they may
approach the highest conception of nationality. The public policy of the Government of the
Philippine Islands is shaped with a view to benefit the Filipino people as a whole. The Manguianes,
in order to fulfill this governmental policy, must be confined for a time, as we have said, for their own
good and the good of the country.

Page 57
Therefore, petitioners are not unlawfully imprisoned or restrained of their liberty. Habeas corpus
can, therefore, not issue.

KWONG SING VS. CITY OF MANILA


[41 PHIL 103; G.R. NO. 15972; 11 OCT 1920]

Facts:

Kwong Sing, in his own behalf and of other Chinese laundrymen who has general and the same
interest, filed a complaint for a preliminary injunction. The Plaintiffs also questioned the validity of
enforcing Ordinance No. 532 by the city of Manila. Ordinance No. 532 requires that the receipt be
in duplicate in English and Spanish duly signed showing the kind and number of articles delivered
by laundries and dyeing and cleaning establishments. The permanent injunction was denied by the
trial court. The appellants claim is that Ordinance No. 532 savors of class legislation; putting in
mind that they are Chinese nationals. It unjustly discriminates between persons in similar
circumstances; and that it constitutes an arbitrary infringement of property rights. They also contest
that the enforcement of the legislation is an act beyond the scope of their police power. In view of
the foregoing, this is an appeal with the Supreme Court.

Issue:

Whether or Not the enforcement of Ordinance no, 532 is an act beyond the scope of police power

Whether or not the enforcement of the same is a class legislation that infringes property rights.

Held:

Reasonable restraints of a lawful business for such purposes are permissible under the police
power. The police power of the City of Manila to enact Ordinance No. 532 is based on Section
2444, paragraphs (l) and (ee) of the Administrative Code, as amended by Act No. 2744, authorizes
the municipal board of the city of Manila, with the approval of the mayor of the city:

(l) To regulate and fix the amount of the license fees for the following: xxxx xxxxxlaundries
xxxx.
(ee) To enact all ordinances it may deem necessary and proper for the sanitation and
safety, the furtherance of the prosperity, and the promotion of the morality, peace, good
order, comfort, convenience, and general welfare of the city and its inhabitants.

The court held that the obvious purpose of Ordinance No. 532 was to avoid disputes between
laundrymen and their patrons and to protect customers of laundries who are not able to decipher
Chinese characters from being defrauded. (Considering that in the year 1920s, people of Manila
are more familiar with Spanish and maybe English.)

In whether the ordinance is class legislation, the court held that the ordinance invades no
fundamental right, and impairs no personal privilege. Under the guise of police regulation, an
attempt is not made to violate personal property rights. The ordinance is neither discriminatory nor
unreasonable in its operation. It applies to all public laundries without distinction, whether they
belong to Americans, Filipinos, Chinese, or any other nationality. All, without exception, and each
every one of them without distinction, must comply with the ordinance. The obvious objection for
the implementation of the ordinance is based in sec2444 (ee) of the Administrative Code. Although,
an additional burden will be imposed on the business and occupation affected by the ordinance
such as that of the appellant by learning even a few words in Spanish or English, but mostly Arabic
numbers in order to properly issue a receipt, it seems that the same burdens are cast upon the
them. Yet, even if private rights of person or property are subjected to restraint, and even if loss will
result to individuals from the enforcement of the ordinance, this is not sufficient ground for failing to

Page 58
uphold the power of the legislative body. The very foundation of the police power is the control of
private interests for the public welfare.

Finding that the ordinance is valid, judgment is affirmed, and the petition for a preliminary injunction
is denied, with costs against the appellants.

YU CONG ENG VS. TRINIDAD


[47 PHIL 385; G.R. NO. 20479; 6 FEB 1925]

Facts:

The petitioner, Yu Cong Eng, was charged by information in the court of first instance of Manila,
with a violation of Act 2972, which provides that (Section 1) it shall be unlawful for any person,
company, or partnership or corporation engaged in commerce, industry or any other activity for the
purpose of profit in the Philippine Islands, in accordance with existing law, to keep its account books
in any language other than English, Spanish or any local dialect. He was arrested, his books were
seized, and the trial was about to proceed, when he and the other petitioner, Co Liam, on their own
behalf, and on behalf of all the other Chinese merchants in the Philippines, filed the petition against
the fiscal, or prosecuting attorney of Manila, and the collector of internal revenue engaged in the
prosecution, and against the judge presiding.

Issue:

Whether or Not Act 2972 is unconstitutional.

Held:

Yes. The Philippine government may make every reasonable requirement of its taxpayers to keep
proper records of their business transactions in English or Spanish or Filipino dialect by which an
adequate measure of what is due from them in meeting the cost of government can be had. But we
are clearly of opinion that it is not within the police power of the Philippine Legislature, because it
would be oppressive and arbitrary, to prohibit all Chinese merchants from maintaining a set of
books in the Chinese language, and in the Chinese characters, and thus prevent them from keeping
advised of the status of their business and directing its conduct.

ANIAG VS. COMELEC


[237 SCRA 194; G.R. NO. 104961; 7 OCT 1994]

Facts:

In preparation for the synchronized national and local elections, the COMELEC issued Resolution
No. 2323, “Gun Ban”, promulgating rules and regulations on bearing, carrying and transporting of
firearm or other deadly weapons on security personnel or bodyguards, on bearing arms by
members of security agencies or police organizations, and organization or maintenance of reaction
forces during the election period. COMELEC also issued Resolution No. 2327 providing for the
summary disqualification of candidates engaged in gunrunning, using and transporting of firearms,
organizing special strike forces, and establishing spot checkpoints. Pursuant to the “Gun Ban”, Mr.
Serrapio Taccad, Sergeant at Arms of the House of Representatives, wrote petitioner for the return
of the two firearms issued to him by the House of Representatives. Petitioner then instructed his
driver, Arellano, to pick up the firearms from petitioner’s house and return them to Congress. The
PNP set up a checkpoint. When the car driven by Arellano approached the checkpoint, the PNP
searched the car and found the firearms. Arellano was apprehended and detained. He then
explained the order of petitioner. Petitioner also explained that Arellano was only complying with the
firearms ban, and that he was not a security officer or a bodyguard. Later, COMELEC issued
Page 59
Resolution No.92-0829 directing the filing of information against petitioner and Arellano for violation
of the Omnibus Election Code, and for petitioner to show cause why he should not be disqualified
from running for an elective position. Petitioner then questions the constitutionality of Resolution No.
2327. He argues that “gunrunning, using or transporting firearms or similar weapons” and other acts
mentioned in the resolution are not within the provisions of the Omnibus Election Code. Thus,
according to petitioner, Resolution No. 2327 is unconstitutional. The issue on the disqualification of
petitioner from running in the elections was rendered moot when he lost his bid for a seat in
Congress in the elections.

Issue:

Whether or Not petitioner can be validly prosecuted for instructing his driver to return the firearms
issued to him on the basis of the evidence gathered from the warrant less search of his car

Held:

A valid search must be authorized by a search warrant issued by an appropriate authority. However,
a warrantless search is not violative of the Constitution for as long as the vehicle is neither searched
nor its occupants subjected to a body search, and the inspection of the vehicle is merely limited to a
visual search. In the case at bar, the guns were not tucked in Arellano’s waist nor placed within his
reach, as they were neatly packed in gun cases and placed inside a bag at the back of the car.
Given these circumstances, the PNP could not have thoroughly searched the car lawfully as well as
the package without violating the constitutional injunction. Absent any justifying circumstance
specifically pointing to the culpability of petitioner and Arellano, the search could not have been
valid. Consequently, the firearms obtained from the warrantless search cannot be admitted for any
purpose in any proceeding. It was also shown in the facts that the PNP had not informed the public
of the purpose of setting up the checkpoint. Petitioner was also not among those charged by the
PNP with violation of the Omnibus Election Code. He was not informed by the City Prosecutor that
he was a respondent in the preliminary investigation. Such constituted a violation of his right to due
process. Hence, it cannot be contended that petitioner was fully given the opportunity to meet the
accusation against him as he was not informed that he was himself a respondent in the case. Thus,
the warrantless search conducted by the PNP is declared illegal and the firearms seized during the
search cannot be used as evidence in any proceeding against the petitioner. Resolution No. 92-
0829 is unconstitutional, and therefore, set aside.

JAVIER VS. COMELEC


[144 SCRA 194; G.R. NOS. L-68379-81; 22 SEPT 1986]

Facts:

The petitioner and the private respondent were candidates in Antique for the Batasang Pambansa
in the May 1984 elections. The former appeared to enjoy more popular support but the latter had
the advantage of being the nominee of the KBL with all its perquisites of power. On May 13, 1984,
the eve of the elections, the bitter contest between the two came to a head when several followers
of the petitioner were ambushed and killed, allegedly by the latter's men. Seven suspects, including
respondent Pacificador, are now facing trial for these murders. Owing to what he claimed were
attempts to railroad the private respondent's proclamation, the petitioner went to the Commission on
Elections to question the canvass of the election returns. His complaints were dismissed and the
private respondent was proclaimed winner by the Second Division of the said body. The petitioner
thereupon came to this Court, arguing that the proclamation was void because made only by a
division and not by the Commission on Elections en banc as required by the Constitution.
Meanwhile, on the strength of his proclamation, the private respondent took his oath as a member
of the Batasang Pambansa.
Page 60
Issue:

Whether or Not the Second Division of the Commission on Elections authorized to promulgate its
decision of July 23, 1984, proclaiming the private respondent the winner in the election.

Held:

This Court has repeatedly and consistently demanded "the cold neutrality of an impartial judge" as
the indispensable imperative of due process. To bolster that requirement, we have held that the
judge must not only be impartial but must also appear to be impartial as an added assurance to the
parties that his decision will be just. The litigants are entitled to no less than that. They should be
sure that when their rights are violated they can go to a judge who shall give them justice. They
must trust the judge, otherwise they will not go to him at all. They must believe in his sense of
fairness, otherwise they will not seek his judgment. Without such confidence, there would be no
point in invoking his action for the justice they expect.

Due process is intended to insure that confidence by requiring compliance with what Justice
Frankfurter calls the rudiments of fair play. Fair play cans for equal justice. There cannot be equal
justice where a suitor approaches a court already committed to the other party and with a judgment
already made and waiting only to be formalized after the litigants shall have undergone the charade
of a formal hearing. Judicial (and also extra-judicial) proceedings are not orchestrated plays in
which the parties are supposed to make the motions and reach the denouement according to a
prepared script. There is no writer to foreordain the ending. The judge will reach his conclusions
only after all the evidence is in and all the arguments are filed, on the basis of the established facts
and the pertinent law.

YNOT VS. IAC


[148 SCRA 659; G.R. NO. 74457; 20 MAR 1987]

Facts:

Executive Order No. 626-A prohibited the transportation of carabaos and carabeef from one
province to another. The carabaos of petitioner were confiscated for violation of Executive Order No
626-A while he was transporting them from Masbate to Iloilo. Petitioner challenged the
constitutionality of Executive Order No. 626-A. The government argued that Executive Order No.
626-A was issued in the exercise of police power to conserve the carabaos that were still fit for farm
work or breeding.

Issue:

Whether or Not EO No. 626-A is a violation of Substantive Due Process.

Held:

The challenged measure is an invalid exercise of police power, because it is not reasonably
necessary for the purpose of the law and is unduly oppressive. It is difficult to see how prohibiting
the transfer of carabaos from one province to another can prevent their indiscriminate killing.
Retaining the carabaos in one province will not prevent their slaughter there. Prohibiting the transfer
of carabeef, after the slaughter of the carabaos, will not prevent the slaughter either.

PHILCOMSAT VS. ALCUAZ


[180 SCRA 218; G.R. NO.84818; 18 DEC 1989]
Page 61
Facts:

Herein petitioner is engaged in providing for services involving telecommunications. Charging rates
for certain specified lines that were reduced by order of herein respondent Jose Alcuaz
Commissioner of the National Telecommunications Commission. The rates were ordered to be
reduced by fifteen percent (15%) due to Executive Order No. 546 which granted the NTC the power
to fix rates. Said order was issued without prior notice and hearing.

Issue:

Whether or Not E.O. 546 is unconstitutional.

Held:

Yes. Respondents admitted that the application of a policy like the fixing of rates as exercised by
administrative bodies is quasi-judicial rather than quasi-legislative. But respondent’s contention that
notice and hearing are not required since the assailed order is merely incidental to the entire
proceedings and temporary in nature is erroneous. Section 16(c) of the Public Service Act,
providing for the proceedings of the Commission, upon notice and hearing, dictates that a
Commission has power to fix rates, upon proper notice and hearing, and, if not subject to the
exceptions, limitations or saving provisions.

It is thus clear that with regard to rate-fixing, respondent has no authority to make such order
without first giving petitioner a hearing, whether the order be temporary or permanent, and it is
immaterial whether the same is made upon a complaint, a summary investigation, or upon the
commission's own motion as in the present case.

WHEREFORE, the writ prayed for is GRANTED and the order of respondents is hereby SET
ASIDE.

EASTERN BROADCASTING CORP (DYRE) V. DANS JR.


[137 SCRA 628; L-59329; 19 JUL 1985]

Facts:

A petition was filed to reopen the Radio Station DYRE. DYRE was “summarily closed” on grounds
of national security. The radio station was allegedly used to incite people to sedition. Petitioner,
DYRE contends that they were denied due process. There was no hearing to establish factual
evidence for the closure. Furthermore, the closure of the radio station violates freedom of
expression. Before the court could even promulgate a decision upon the Issue raised, Petitioner,
through its president Mr. Rene Espina, filed a motion to withdraw the petition. The rights of the
station were sold to a new owner, Manuel Pastrana; who is no longer interested in pursuing the
case. Despite the case becoming moot and academic, (because there are no longer interested
parties, thus the dismissal of the case) the Supreme Court still finds that there is need to pass a
“RESOLUTION” for the guidance of inferior courts and administrative tribunals in matters as this
case.

Issue:

Whether or not due process was exercised in the case of DYRE.

Whether or not the closure of DYRE is a violation of the Constitutional Right of Freedom of
Expression.

Held:
Page 62
The court finds that the closure of the Radio Station in 1980 as null and void. The absence of a
hearing is a violation of Constitutional Rights. The primary requirements in administrative
proceedings are laid down in the case of Ang Tibay v. Court of Industrial Relation (69 Phil.635).
The Ang Tibay Doctrine should be followed before any broadcast station may be closed. The Ang
Tibay Doctrine provides the following requirements:

(1) The right to hearing, includes the right to present one’s case and submit evidence
presented.
(2) The tribunal must consider the evidence presented
(3) The decision must have something to support itself.
(4) Evidence must be substantial (reasonable evidence that is adequate to support
conclusion)
(5) Decision must be based on the evidence presented at hearing
(6) The tribunal body must act on its own independent consideration of law and facts
and not simply accept subordinate’s views
(7) Court must render decision in such a manner that the proceeding can know the
various issued involved and reasons for decisions rendered.

The court stresses that while there is no controlling and precise definition of Due Process, it gives
an unavoidable standard that government actions must conform in order that deprivation of life,
liberty and property is valid.

The closure of the radio station is like wise a violation of the constitutional right of freedom of
speech and expression. The court stresses that all forms of media, whether print or broadcast are
entitled to this constitutional right. Although the government still has the right to be protected
against broadcasts which incite the listeners to violently overthrow it. The test for the limitation of
freedom of expression is the “clear and present danger” rule. If in the circumstances that the media
is used in such nature as to create this danger that will bring in such evils, then the law has the right
to prevent it. However, Radio and television may not be used to organize a rebellion or signal a
start of widespread uprising. The freedom to comment on public affairs is essential to the vitality of
a representative democracy. The people continues to have the right to be informed on public affairs
and broadcast media continues to have the pervasive influence to the people being the most
accessible form of media. Therefore, broadcast stations deserve the the special protection given to
all forms of media by the due process and freedom of expression clauses of the Constitution.

ANG TIBAY VS. COURT OF INDUSTRIAL RELATIONS (CIR)


[69 PHIL 635; G.R. NO. 46496; 27 FEB 1940]

Facts:

There was agreement between Ang Tibay and the National Labor Union, Inc (NLU). The NLU
alleged that the supposed lack of leather material claimed by Toribio Teodoro was but a scheme
adopted to systematically discharge all the members of the NLU, from work. And this averment is
desired to be proved by the petitioner with the records of the Bureau of Customs and Books of
Accounts of native dealers in leather. That National Worker's Brotherhood Union of Ang Tibay is a
company or employer union dominated by Toribio Teodoro, which was alleged by the NLU as an
illegal one. The CIR, decided the case and elevated it to the Supreme Court, but a motion for new
trial was raised by the NLU. But the Ang Tibay filed a motion for opposing the said motion.

Issue:

Whether or Not, the motion for new trial is meritorious to be granted.

Held:

Page 63
To begin with the issue before us is to realize the functions of the CIR. The CIR is a special court
whose functions are specifically stated in the law of its creation which is the Commonwealth Act No.
103). It is more an administrative board than a part of the integrated judicial system of the nation. It
is not intended to be a mere receptive organ of the government. Unlike a court of justice which is
essentially passive, acting only when its jurisdiction is invoked and deciding only cases that are
presented to it by the parties litigant, the function of the CIR, as will appear from perusal of its
organic law is more active, affirmative and dynamic. It not only exercises judicial or quasi-judicial
functions in the determination of disputes between employers and employees but its functions are
far more comprehensive and extensive. It has jurisdiction over the entire Philippines, to consider,
investigate, decide, and settle any question, matter controversy or disputes arising between, and/ or
affecting employers and employees or laborers, and landlords and tenants or farm-laborers, and
regulates the relations between them, subject to, and in accordance with, the provisions of CA 103.

As laid down in the case of Goseco v. CIR, the SC had the occasion to point out that the CIR is not
narrowly constrained by technical rules of procedure, and equity and substantial merits of the case,
without regard to technicalities or legal forms and shall not be bound by any technical rules of legal
evidence but may inform its mind in such manner as it may deem just and equitable.

The fact, however, that the CIR may be said to be free from rigidity of certain procedural
requirements does not mean that it can in justiciable cases coming before it, entirely ignore or
disregard the fundamental and essential requirements of due process in trials and investigations of
an administrative character. There cardinal primary rights which must be respected even in
proceedings of this character:

(1) the right to a hearing, which includes the right to present one's cause and submit
evidence in support thereof;
(2) The tribunal must consider the evidence presented;
(3) The decision must have something to support itself;
(4) The evidence must be substantial;
(5) The decision must be based on the evidence presented at the hearing; or at least
contained in the record and disclosed to the parties affected;
(6) The tribunal or body or any of its judges must act on its own independent
consideration of the law and facts of the controversy, and not simply accept the
views of a subordinate;
(7) The Board or body should, in all controversial questions, render its decision in
such manner that the parties to the proceeding can know the various Issue
involved, and the reason for the decision rendered.

The failure to grasp the fundamental issue involved is not entirely attributable to the parties
adversely affected by the result. Accordingly, the motion for a new trial should be, and the same is
hereby granted, and the entire record of this case shall be remanded to the CIR, with instruction that
it reopen the case receive all such evidence as may be relevant, and otherwise proceed in
accordance with the requirements set forth. So ordered.

ATENEO DE MANILA UNIVERSITY VS. HON. JUDGE IGNACIO CAPULONG


[222 SCRA 644; G.R. 99327; 27 MAY 1993]

Facts:

Leonardo H. Villa, a first year law student of Petitioner University, died of serious physical injuries at
Chinese General Hospital after the initiation rites of Aquila Legis. Bienvenido Marquez was also
hospitalized at the Capitol Medical Center for acute renal failure occasioned by the serious physical
injuries inflicted upon him on the same occasion. Petitioner Dean Cynthia del Castillo created a
Joint Administration-Faculty-Student Investigating Committee which was tasked to investigate and
Page 64
submit a report within 72 hours on the circumstances surrounding the death of Lennie Villa. Said
notice also required respondent students to submit their written statements within twenty-four (24)
hours from receipt. Although respondent students received a copy of the written notice, they failed
to file a reply. In the meantime, they were placed on preventive suspension. The Joint
Administration-Faculty-Student Investigating Committee, after receiving the written statements and
hearing the testimonies of several witness, found a prima facie case against respondent students
for violation of Rule 3 of the Law School Catalogue entitled "Discipline." Respondent students were
then required to file their written answers to the formal charge. Petitioner Dean created a
Disciplinary Board to hear the charges against respondent students. The Board found respondent
students guilty of violating Rule No. 3 of the Ateneo Law School Rules on Discipline which prohibits
participation in hazing activities. However, in view of the lack of unanimity among the members of
the Board on the penalty of dismissal, the Board left the imposition of the penalty to the University
Administration. Accordingly, Fr. Bernas imposed the penalty of dismissal on all respondent students.
Respondent students filed with RTC Makati a TRO since they are currently enrolled. This was
granted. A TRO was also issued enjoining petitioners from dismissing the respondents. A day after
the expiration of the temporary restraining order, Dean del Castillo created a Special Board to
investigate the charges of hazing against respondent students Abas and Mendoza. This was
requested to be stricken out by the respondents and argued that the creation of the Special Board
was totally unrelated to the original petition which alleged lack of due process. This was granted and
reinstatement of the students was ordered.

Issue:

Was there denial of due process against the respondent students.

Held:

There was no denial of due process, more particularly procedural due process. Dean of the Ateneo
Law School, notified and required respondent students to submit their written statement on the
incident. Instead of filing a reply, respondent students requested through their counsel, copies of the
charges. The nature and cause of the accusation were adequately spelled out in petitioners' notices.
Present is the twin elements of notice and hearing.

Respondent students argue that petitioners are not in a position to file the instant petition under
Rule 65 considering that they failed to file a motion for reconsideration first before the trial court,
thereby by passing the latter and the Court of Appeals. It is accepted legal doctrine that an
exception to the doctrine of exhaustion of remedies is when the case involves a question of law, as
in this case, where the issue is whether or not respondent students have been afforded procedural
due process prior to their dismissal from Petitioner University.

Minimum standards to be satisfied in the imposition of disciplinary sanctions in academic


institutions, such as petitioner university herein, thus:

(1) the students must be informed in writing of the nature and cause of any
accusation against them;
(2) that they shall have the right to answer the charges against them with the
assistance of counsel, if desired:
(3) they shall be informed of the evidence against them
(4) they shall have the right to adduce evidence in their own behalf; and
(5) the evidence must be duly considered by the investigating committee or official
designated by the school authorities to hear and decide the case.

US GOVERNMENT VS. JUDGE PURUNGAN


[389 SCRA 623; G.R. NO. 148571, 24 SEPT 2002]

Page 65
Facts:

The United States of America, pursuant to the existing RP-US extradition treaty, requested the
extradition of Mark B. Jimenez. Upon receipt of the request, the secretary of foreign affairs (SFA)
transmitted them to the secretary of justice (SOJ) for appropriate action. In such event, the RTC
held that Jimenez shell be deprived of the right to notice and hearing during the evaluation stage of
the extradition process. Thereafter the US government, through DOJ, filed Petition for Extradition
and Jimenez’s immediate arrest, to avoid flight. Before the RTC could render its decision, Jimenez
filed an "Urgent Manifestation/Ex-Parte Motion," praying that his application for an arrest warrant be
set for hearing, which was granted. During which, the lower court issued its questioned July 3, 2001
Order, directing the issuance of a warrant for his arrest and fixing bail for his temporary liberty at
one million pesos in cash. After Jimenez had surrendered his passport and posted the required
cash bond, he was granted provisional liberty via the challenged Order dated July 4, 2001. Thus,
Petition prays for the lifting of the bail Order, the cancellation of the bond, and the taking of Jimenez
into legal custody.

Issue:

Whether or not Jimenez is entitled to notice and hearing before a warrant for his arrest can be
issued.

Whether or not he is entitled to bail and to provisional liberty while the extradition proceedings are
pending.

Held:

By nature, extradition proceedings are not equivalent to a criminal case in which guilt or innocence
is determined. Consequently, an extradition case is not one in which the constitutional rights of the
accused are necessarily available. Having once escaped the jurisdiction of the requesting state, the
reasonable prima facie presumption is that the person would escape again if given the opportunity.
Hence, if the judge is convinced that a prima facie case exists, he immediately Issue a warrant for
the arrest of the potential extraditee and summons him or her to answer and to appear at scheduled
hearings on the petition. Potential extraditees are entitled to the rights to due process and to
fundamental fairness. Due process does not always call for a prior opportunity to be heard. A
subsequent opportunity is sufficient due to the flight risk involved. Indeed, available during the
hearings on the petition and the answer is the full chance to be heard and to enjoy fundamental
fairness that is compatible with the summary nature of extradition.

After being taken into custody, potential extraditees may apply for bail. Since the applicants have a
history of absconding, they have the burden of showing that (a) there is no flight risk and no danger
to the community; and (b) there exist special, humanitarian or compelling circumstances. In
extradition cases, bail is not a matter of right; it is subject to judicial discretion in the context of the
peculiar facts of each case.

Page 66
EQUAL PROTECTION

Art 3, Sec. 1. “…nor shall any person be denied the equal protection of the laws.”

PEOPLE VS. CAYAT


[68 PHIL 12; G.R. NO. 45987; 5 MAY 1939]

Facts:

“Law prohibits any member of a non-Christian tribe to buy, receive, have in his possession, or drink,
any intoxicating liquors of any kind.” The law, Act No. 1639, exempts only the so-called native wines
or liquors which the members of such tribes have been accustomed to take.

Issue:

Whether or Not the law denies equal protection to one prosecuted and sentenced for violation of
said law.

Held:

No. It satisfies the requirements of a valid classification, one of which is that the classification under
the law must rest on real or substantial distinctions.

The distinction is reasonable. The classification between the members of the non- Christian and the
members of the Christian tribes is not based upon accident of birth or parentage but upon the
degree of civilization and culture. The term ‘non-Christian tribes’ refers to a geographical area and
more directly to natives of the Philippines of a low grade civilization usually living in tribal
relationship apart from settled communities. The distinction is reasonable for the Act was intended
to meet the peculiar conditions existing in the non- Christian tribes”

The prohibition is germane to the purposes of the law. It is designed to insure peace and order in
and among the non- Christian tribes has often resulted in lawlessness and crime thereby hampering
the efforts of the government to raise their standards of life and civilization. This law is not limited in
its application to conditions existing at the time of the enactment. It is intended to apply for all times
as long as those conditions exists. The Act applies equally to all members of the class. That it may
be unfair in its operation against a certain number of non- Christians by reason of their degree of
culture is not an argument against the equality of its operation nor affect the reasonableness of the
classification thus established.

PASEI VS. DRILON


[163 SCRA 386; L-81958; 30 JUN 1988]

Facts:

Page 67
Petitioner, Phil association of Service Exporters, Inc., is engaged principally in the recruitment of
Filipino workers, male and female of overseas employment. It challenges the constitutional validity
of Dept. Order No. 1 (1998) of DOLE entitled “Guidelines Governing the Temporary Suspension of
Deployment of Filipino Domestic and Household Workers.” It claims that such order is a
discrimination against males and females. The Order does not apply to all Filipino workers but only
to domestic helpers and females with similar skills, and that it is in violation of the right to travel, it
also being an invalid exercise of the lawmaking power. Further, PASEI invokes Sec 3 of Art 13 of
the Constitution, providing for worker participation in policy and decision-making processes affecting
their rights and benefits as may be provided by law. Thereafter the Solicitor General on behalf of
DOLE submitting to the validity of the challenged guidelines involving the police power of the State
and informed the court that the respondent have lifted the deployment ban in some states where
there exists bilateral agreement with the Philippines and existing mechanism providing for sufficient
safeguards to ensure the welfare and protection of the Filipino workers.

Issue:

Whether or not there has been a valid classification in the challenged Department Order No. 1.

Held:

SC in dismissing the petition ruled that there has been valid classification, the Filipino female
domestics working abroad were in a class by themselves, because of the special risk to which their
class was exposed. There is no question that Order No.1 applies only to female contract workers
but it does not thereby make an undue discrimination between sexes. It is well settled hat equality
before the law under the constitution does not import a perfect identity of rights among all men and
women. It admits of classification, provided that:

1. Such classification rests on substantial distinctions


2. That they are germane to the purpose of the law
3. They are not confined to existing conditions
4. They apply equally to al members of the same class

In the case at bar, the classifications made, rest on substantial distinctions.

Dept. Order No. 1 does not impair the right to travel. The consequence of the deployment ban has
on the right to travel does not impair the right, as the right to travel is subjects among other things,
to the requirements of “public safety” as may be provided by law. Deployment ban of female
domestic helper is a valid exercise of police power. Police power as been defined as the state
authority to enact legislation that may interfere with personal liberty or property in order to promote
general welfare. Neither is there merit in the contention that Department Order No. 1 constitutes an
invalid exercise of legislative power as the labor code vest the DOLE with rule making powers.

DUMLAO VS. COMELEC


[95 SCRA 392; L-52245; 22 JAN 1980]

Facts:

Petitioner questions the constitutionality of section 4 of Batas Pambansa Blg. 52 as discriminatory


and contrary to the equal protection and due process guarantees of the Constitution.

Section 4 provided that any retired municipal or provincial city official that already received
retirement benefits and is 65 years of age shall not be qualified to run for the same local elective
office from which he has retired.

Page 68
Issue:

Whether or Not Sec. 4 of BP.52 is unconstitutional being contrary to the equal protection and due
process rights.

Held:

No. The guarantee of equal protection is subject to rational classification based on reasonable and
real differentiations. In the present case, employees 65 years of age have been classified differently
from younger employees. The former are subject to compulsory retirement while the latter are not.

Retirement is not a reasonable disqualification for elective local officials because there can be
retirees who are even younger and a 65 year old retiree could be as good as a 65 year old official
who is not a retiree. But there is reason to disqualify a 65 year old elective official who is trying to
run for office because there is the “need for new blood to assume relevance”. When an official has
retired he has already declared himself tired and unavailable for the same government work.

WHEREFORE, the first paragraph of section 4 of Batas pambansa Bilang 52 is hereby declared
valid.

TELECOMMUNICATIONS AND BROADCAST ATTORNEYS OF THE PHILS. VS. COMELEC


[289 SCRA 337; G.R. NO. 132922; 21 APR 1998]

Facts:

Petitioner Telecommunications and Broadcast Attorneys of the Philippines, Inc. (TELEBAP) is an


organization of lawyers of radio and television broadcasting companies. It was declared to be
without legal standing to sue in this case as, among other reasons, it was not able to show that it
was to suffer from actual or threatened injury as a result of the subject law. Petitioner GMA
Network, on the other hand, had the requisite standing to bring the constitutional challenge.
Petitioner operates radio and television broadcast stations in the Philippines affected by the
enforcement of Section 92, B.P. No. 881.

Petitioners challenge the validity of Section 92, B.P. No. 881 which provides:
“Comelec Time- The Commission shall procure radio and television time to be
known as the “Comelec Time” which shall be allocated equally and impartially among
the candidates within the area of coverage of all radio and television stations. For
this purpose, the franchise of all radio broadcasting and television stations are
hereby amended so as to provide radio or television time, free of charge, during the
period of campaign.”

Petitioner contends that while Section 90 of the same law requires COMELEC to procure print
space in newspapers and magazines with payment, Section 92 provides that air time shall be
procured by COMELEC free of charge. Thus it contends that Section 92 singles out radio and
television stations to provide free air time.

Petitioner claims that it suffered losses running to several million pesos in providing COMELEC
Time in connection with the 1992 presidential election and 1995 senatorial election and that it
stands to suffer even more should it be required to do so again this year. Petitioners claim that the
primary source of revenue of the radio and television stations is the sale of air time to advertisers
and to require these stations to provide free air time is to authorize unjust taking of private property.
According to petitioners, in 1992 it lost P22,498,560.00 in providing free air time for one hour each
day and, in this year’s elections, it stands to lost P58,980,850.00 in view of COMELEC’s
requirement that it provide at least 30 minutes of prime time daily for such.

Issue:
Page 69
Whether of not Section 92 of B.P. No. 881 denies radio and television broadcast companies the
equal protection of the laws.

Whether or not Section 92 of B.P. No. 881 constitutes taking of property without due process of law
and without just compensation.

Held:

Petitioner’s argument is without merit. All broadcasting, whether radio or by television stations, is
licensed by the government. Airwave frequencies have to be allocated as there are more
individuals who want to broadcast that there are frequencies to assign. Radio and television
broadcasting companies, which are given franchises, do not own the airwaves and frequencies
through which they transmit broadcast signals and images. They are merely given the temporary
privilege to use them. Thus, such exercise of the privilege may reasonably be burdened with the
performance by the grantee of some form of public service. In granting the privilege to operate
broadcast stations and supervising radio and television stations, the state spends considerable
public funds in licensing and supervising them.

The argument that the subject law singles out radio and television stations to provide free air time
as against newspapers and magazines which require payment of just compensation for the print
space they may provide is likewise without merit. Regulation of the broadcast industry requires
spending of public funds which it does not do in the case of print media. To require the broadcast
industry to provide free air time for COMELEC is a fair exchange for what the industry gets.

As radio and television broadcast stations do not own the airwaves, no private property is taken by
the requirement that they provide air time to the COMELEC.

LACSON VS. EXECUTIVE SECRETARY


[301 SCRA 298; G.R. NO. 128096; 20 JAN 1999]

Facts:

Eleven persons believed to be members of the Kuratong Baleleng gang, an organized crime
syndicate involved in bank robberies, were slain by elements of the Anti-Bank Robbery and
Intelligence Task Group (ABRITG). Among those included in the ABRITG were petitioners and
petitioner-intervenors.

Acting on a media expose of SPO2 Eduardo delos Reyes, a member of the Criminal Investigation
Command, that what actually transpired was a summary execution and not a shoot-out between the
Kuratong Baleleng gang members and the ABRITG, Ombudsman Aniano Desierto formed a panel
of investigators to investigate the said incident. Said panel found the incident as a legitimate police
operation. However, a review board modified the panel’s finding and recommended the indictment
for multiple murder against twenty-six respondents including herein petitioner, charged as principal,
and herein petitioner-intervenors, charged as accessories. After a reinvestigation, the Ombudsman
filed amended informations before the Sandiganbayan, where petitioner was charged only as an
accessory.

The accused filed separate motions questioning the jurisdiction of the Sandiganbayan, asserting
that under the amended informations, the cases fall within the jurisdiction of the Regional Trial Court
pursuant to Section 2 of R.A. 7975. They contend that the said law limited the jurisdiction of the
Sandiganbayan to cases where one or ore of the “principal accused” are government officals with
Salary Grade 27 or higher, or PNP officials with rank of Chief Superintendent or higher. Thus, they
did not qualify under said requisites. However, pending resolution of their motions, R.A. 8249 was

Page 70
approved amending the jurisdiction of the Sandiganbayan by deleting the word “principal” from the
phrase “principal accused” in Section 2 of R.A. 7975.

Petitioner questions the constitutionality of Section 4 of R.A. 8249, including Section 7 which
provides that the said law shall apply to all cases pending in any court over which trial has not
begun as of the approval hereof.

Issue:

Whether or not Sections 4 and 7 of R.A. 8249 violate the petitioners’ right to due process and the
equal protection clause of the Constitution as the provisions seemed to have been introduced for
the Sandiganbayan to continue to acquire jurisdiction over the Kuratong Baleleng case.

Whether or not said statute may be considered as an ex-post facto statute.

Whether or not the multiple murder of the alleged members of the Kuratong Baleleng was
committed in relation to the office of the accused PNP officers which is essential to the
determination whether the case falls within the Sandiganbayan’s or Regional Trial Court’s
jurisdiction.

Held:

Petitioner and intervenors’ posture that Sections 4 and 7 of R.A. 8249 violate their right to equal
protection of the law is too shallow to deserve merit. No concrete evidence and convincing
argument were presented to warrant such a declaration. Every classification made by the law is
presumed reasonable and the party who challenges the law must present proof of arbitrariness.
The classification is reasonable and not arbitrary when the following concur: (1) it must rest on
substantial distinction; (2) it must be germane to the purpose of the law; (3) must not be limited to
existing conditions only, and (4) must apply equally to all members of the same class; all of which
are present in this case.

Paragraph a of Section 4 provides that it shall apply “to all cases involving” certain public officials
and under the transitory provision in Section 7, to “all cases pending in any court.” Contrary to
petitioner and intervenors’ argument, the law is not particularly directed only to the Kuratong
Baleleng cases. The transitory provision does not only cover cases which are in the
Sandiganbayan but also in “any court.”

There is nothing ex post facto in R.A. 8249. Ex post facto law, generally, provides retroactive effect
of penal laws. R.A. 8249 is not a penal law. It is a substantive law on jurisdiction which is not penal
in character. Penal laws are those acts of the Legislature which prohibit certain acts and establish
penalties for their violations or those that define crimes and provide for their punishment. R.A. 7975,
as regards the Sandiganbayan’s jurisdiction, its mode of appeal and other procedural matters, has
been declared by the Court as not a penal law, but clearly a procedural statute, one which
prescribes rules of procedure by which courts applying laws of all kinds can properly administer
justice. Not being a penal law, the retroactive application of R.A. 8249 cannot be challenged as
unconstitutional.

In People vs. Montejo, it was held that an offense is said to have been committed in relation to the
office if it is intimately connected with the office of the offender and perpetrated while he was in the
performance of his official functions. Such intimate relation must be alleged in the information which
is essential in determining the jurisdiction of the Sandiganbayan. However, upon examination of the
amended information, there was no specific allegation of facts that the shooting of the victim by the
said principal accused was intimately related to the discharge of their official duties as police
officers. Likewise, the amended information does not indicate that the said accused arrested and
investigated the victim and then killed the latter while in their custody. The stringent requirement
that the charge set forth with such particularity as will reasonably indicate the exact offense which
the accused is alleged to have committed in relation to his office was not established.
Page 71
Consequently, for failure to show in the amended informations that the charge of murder was
intimately connected with the discharge of official functions of the accused PNP officers, the offense
charged in the subject criminal cases is plain murder and, therefore, within the exclusive original
jurisdiction of the Regional Trial Court and not the Sandiganbayan.

INT'L. SCHOOL ALLIANCE VS. QUISUMBING


[333 SCRA 13; G.R. NO. 128845; 1 JUN 2000]

Facts:

Receiving salaries less than their counterparts hired abroad, the local-hires of private respondent
School, mostly Filipinos, cry discrimination. We agree. That the local-hires are paid more than their
colleagues in other schools is, of course, beside the point. The point is that employees should be
given equal pay for work of equal value.

Private respondent International School, Inc. (the School, for short), pursuant to Presidential Decree
732, is a domestic educational institution established primarily for dependents of foreign diplomatic
personnel and other temporary residents. To enable the School to continue carrying out its
educational program and improve its standard of instruction, Section 2(c) of the same decree
authorizes the School to
employ its own teaching and management personnel selected by it either locally or abroad, from
Philippine or other nationalities, such personnel being exempt from otherwise applicable laws and
regulations attending their employment, except laws that have been or will be enacted for the
protection of employees.
Accordingly, the School hires both foreign and local teachers as members of its faculty, classifying
the same into two: (1) foreign-hires and (2) local-hires.

The School grants foreign-hires certain benefits not accorded local-hires. These include housing,
transportation, shipping costs, taxes, and home leave travel allowance. Foreign-hires are also paid
a salary rate twenty-five percent (25%) more than local-hires. The School justifies the difference on
two "significant economic disadvantages" foreign-hires have to endure, namely: (a) the "dislocation
factor" and (b) limited tenure.

Issue:

Whether or Not the grants provided by the school to foreign hires and not to local hires
discriminative of their constitutional right to the equal protection clause.

Held:

The foregoing provisions impregnably institutionalize in this jurisdiction the long honored legal truism
of "equal pay for equal work." Persons who work with substantially equal qualifications, skill, effort
and responsibility, under similar conditions, should be paid similar salaries. This rule applies to the
School, its "international character" notwithstanding.

The School contends that petitioner has not adduced evidence that local-hires perform work equal
to that of foreign-hires. The Court finds this argument a little cavalier. If an employer accords
employees the same position and rank, the presumption is that these employees perform equal
work. This presumption is borne by logic and human experience. If the employer pays one
employee less than the rest, it is not for that employee to explain why he receives less or why the
others receive more. That would be adding insult to injury. The employer has discriminated against
that employee; it is for the employer to explain why the employee is treated unfairly.

While we recognize the need of the School to attract foreign-hires, salaries should not be used as
an enticement to the prejudice of local-hires. The local-hires perform the same services as foreign-
Page 72
hires and they ought to be paid the same salaries as the latter. For the same reason, the
"dislocation factor" and the foreign-hires' limited tenure also cannot serve as valid bases for the
distinction in salary rates.

The Constitution enjoins the State to "protect the rights of workers and promote their welfare," "to
afford labor full protection." The State, therefore, has the right and duty to regulate the relations
between labor and capital. These relations are not merely contractual but are so impressed with
public interest that labor contracts, collective bargaining agreements included, must yield to the
common good. Should such contracts contain stipulations that are contrary to public policy, courts
will not hesitate to strike down these stipulations.

In this case, we find the point-of-hire classification employed by respondent School to justify the
distinction in the salary rates of foreign-hires and local hires to be an invalid classification. There is
no reasonable distinction between the services rendered by foreign-hires and local-hires.

Wherefore, the petition is given due course. The petition is hereby granted in part. The orders of the
secretary of labor and employment dated June 10, 1996 and march 19, 1997, are hereby reversed
and set aside insofar as they uphold the practice of respondent school of according foreign-hires
higher salaries than local-hires.

ORMOC SUGAR COMPANY VS. TREASURER OF ORMOC CITY


[22 SCRA 603; L-23794; 17 FEB 1968]

Facts:

On January 29, 1964, the Municipal Board of Ormoc City passed Ordinance No. 4, Series of 1964,
imposing "on any and all productions of centrifugal sugar milled at the Ormoc Sugar Company, Inc.,
in Ormoc City a municipal tax equivalent to one per centum (1%) per export sale to the United
States of America and other foreign countries." Payments for said tax were made, under protest, by
Ormoc Sugar Company, Inc. on March 20, 1964 for P7, 087.50 and on April 20, 1964 for P5, 000, or
a total of P12, 087.50.

On June 1, 1964, Ormoc Sugar Company, Inc. filed before the Court of First Instance of Leyte, with
service of a copy upon the Solicitor General, a complaint against the City of Ormoc as well as its
Treasurer, Municipal Board and Mayor, alleging that the afore-stated ordinance is unconstitutional
for being violative of the equal protection clause (Sec. 1[1], Art. III, Constitution) and the rule of
uniformity of taxation (Sec. 22[1]), Art. VI, Constitution).

Answering, the defendants asserted that the tax ordinance was within defendant city's power to
enact under the Local Autonomy Act and that the same did not violate the afore-cited constitutional
limitations. After pre-trial and submission of the case on memoranda, the Court of First Instance, on
August 6, 1964, rendered a decision that upheld the constitutionality of the ordinance and declared
the taxing power of defendant chartered city broadened by the Local Autonomy Act to include all
other forms of taxes, licenses or fees not excluded in its charter.

Issue:

Whether or Not the ordinance is unconstitutional for being violative of the equal protection clause
under Sec. 1[1], Art. III, Constitution.

Whether or not it was violative of the rule of uniformity of taxation under the Bill of Rights, Sec.
22[1], Art. VI, Constitution.

Held:

Page 73
The Constitution in the bill of rights provides: ". . . nor shall any person be denied the equal
protection of the laws." (Sec. 1 [1], Art. III) In Felwa vs. Salas, We ruled that the equal protection
clause applies only to persons or things identically situated and does not bar a reasonable
classification of the subject of legislation, and a classification is reasonable where (1) it is based on
substantial distinctions which make real differences; (2) these are germane to the purpose of the
law; (3) the classification applies not only to present conditions but also to future conditions which
are substantially identical to those of the present; (4) the classification applies only to those who
belong to the same class.

A perusal of the requisites instantly shows that the questioned ordinance does not meet them, for it
taxes only centrifugal sugar produced and exported by the Ormoc Sugar Company, Inc. and none
other. At the time of the taxing ordinance's enactment, Ormoc Sugar Company, Inc., it is true, was
the only sugar central in the city of Ormoc. Still, the classification, to be reasonable, should be in
terms applicable to future conditions as well. The taxing ordinance should not be singular and
exclusive as to exclude any subsequently established sugar central, of the same class as plaintiff,
for the coverage of the tax. As it is now, even if later a similar company is set up, it cannot be
subject to the tax because the ordinance expressly points only to Ormoc City Sugar Company, Inc.
as the entity to be levied upon.

Appellant, however, is not entitled to interest; on the refund because the taxes were not arbitrarily
collected (Collector of Internal Revenue v. Binalbagan). 6 At the time of collection, the ordinance
provided a sufficient basis to preclude arbitrariness, the same being then presumed constitutional
until declared otherwise.

Wherefore, the decision appealed from is hereby reversed, the challenged ordinance is declared
unconstitutional and the defendants-appellees are hereby ordered to refund the P12,087.50 plaintiff-
appellant paid under protest. No costs. So ordered.
PHILIPPINE JUDGES ASSO. VS. PRADO
[227 SCRA 703; G.R. NO. 105371; 11 NOV 1993]

Facts:

The Philippine Postal Corporation issued circular No. 92-28 to implement Section 35 of RA 7354
withdrawing the franking privilege from the SC, CA, RTCs, MeTCs, MTCs and Land Registration
Commission and with certain other government offices. It is alleged that RA 7354 is discriminatory
becasue while withdrawing the franking privilege from judiciary, it retains the same for the President
& Vice-President of the Philippines, Senator & members of the House of Representatives,
COMELEC, National Census & Statistics Office and the general public. The respondents counter
that there is no discrimination because the law is based on a valid classification in accordance with
the equal protection clause.

Issue:

Whether or Not Section 35 of RA 7354 is constitutional.

Held:

The equal protection of the laws is embraced in the concept of due process, as every unfair
discrimination offends the requirements of justice and fair play. It has nonetheless been embodied
in a separate clause in Article III Section 1 of the Constitution to provide for amore specific
guarantee against any form of undue favoritism or hostility from the government. Arbitrariness in
general may be challenged on the basis of the due process clause. But if the particular act assailed
partakes of an unwarranted partiality or prejudice, the sharper weapon to cut it down is the equal
protection clause. Equal protection simply requires that all persons or things similarly situated
should be treated alike, both as to rights conferred and responsibilities imposed. What the clause
requires is equality among equals as determined according to a valid classification. Section 35 of
RA 7354 is declared unconstitutional. Circular No. 92-28 is set aside insofar
Page 74
Page 75
SEARCHES AND SEIZURES

Art 3, Sec. 2. “The right of the people to be secure in their persons, houses, papers, and effects
against unreasonable searches and seizures of whatever nature and for any purpose shall be
inviolable, and no search warrant or warrant of arrest shall issue except upon probable cause to be
determined personally by the judge after examination under oath or affirmation of the complainant
and the witnesses he may produce, and particularly describing the place to be searched and the
persons or things to be seized.”

Art 3, Sec. 3. “(1) The privacy of communication and correspondence shall be inviolable except
upon lawful order of the court, or when public safety or order requires otherwise as prescribed by
law.
(2) Any evidence obtained in violation of this or the preceding section shall be inadmissible for
any purpose in any proceeding.”

PEOPLE VS. MARTI


[193 SCRA 57; G.R. NO. 81561; 18 JAN 1991]

Facts:

Accused-appellant went to a forwarding agency to send four packages to a friend in Zurich. Initially,
the accused was asked by the proprietress if the packages can be examined. However, he refused.
Before delivering said packages to the Bureau of Customs and the Bureau of Posts, the husband of
the proprietress opened said boxes for final inspection. From that inspection, included in the
standard operating procedure and out of curiosity, he took several grams of its contents.

He brought a letter and the said sample to the National Bureau of Investigation. When the NBI was
informed that the rest of the shipment was still in his office, three agents went back with him. In their
presence, the husband totally opened the packages. Afterwards, the NBI took custody of said
packages. The contents , after examination by forensic chemists, were found to be marijuana
flowering tops.

The appellant, while claiming his mail at the Central Post Office, was invited by the agents for
questioning. Later on, the trial court found him guilty of violation of the Dangerous Drugs Act.

Issue:

Whether or Not the items admitted in the searched illegally searched and seized.

Whether or Not custodial investigation properly applied.

Whether or Not the trial court not give credence to the explanation of the appellant on how said
packages came to his possession.

Held:

No. “The case at bar assumes a peculiar character since the evidence sought to be excluded was
primarily discovered and obtained by a private person, acting in a private capacity and without the
intervention and participation of State authorities. Under the circumstances, can accused/appellant
validly claim that his constitutional right against unreasonable searches and seizure has been
violated. Stated otherwise, may an act of a private individual, allegedly in violation of appellant's
constitutional rights, be invoked against the State. In the absence of governmental interference, the
liberties guaranteed by the Constitution cannot be invoked against the State. It was Mr. Job Reyes,
the proprietor of the forwarding agency, who made search/inspection of the packages. Said
Page 76
inspection was reasonable and a standard operating procedure on the part of Mr. Reyes as a
precautionary measure before delivery of packages to the Bureau of Customs or the Bureau of
Posts. Second, the mere presence of the NBI agents did not convert the reasonable search effected
by Reyes into a warrantless search and seizure proscribed by the Constitution. Merely to observe
and look at that which is in plain sight is not a search. Having observed that which is open, where
no trespass has been committed in aid thereof, is not search.”
No. “The law enforcers testified that accused/appellant was informed of his constitutional rights. It is
presumed that they have regularly performed their duties (See. 5(m), Rule 131) and their
testimonies should be given full faith and credence, there being no evidence to the contrary.”
No. “Appellant signed the contract as the owner and shipper thereof giving more weight to the
presumption that things which a person possesses, or exercises acts of ownership over, are owned
by him (Sec. 5 [j], Rule 131). At this point, appellant is therefore estopped to claim otherwise.”

WATEROUS DRUG VS. NLRC


[280 SCRA 735 ; G.R.NO. 113271; 16 OCT 1997]

Facts:

Catolico was hired as a pharmacist by petitioner Waterous Drug Corporation on 15 August 1988.
On 31 July 1989, Catolico received a memorandum from WATEROUS Vice President-General
Manager Emma R. Co warning her not to dispense medicine to employees chargeable to the latter's
accounts because the same was a prohibited practice. On the same date, Co issued another
memorandum to Catolico warning her not to negotiate with suppliers of medicine without consulting
the Purchasing Department, as this would impair the company's control of purchases and, besides
she was not authorized to deal directly with the suppliers.

As regards the first memorandum, Catolico did not deny her responsibility but explained that her act
was "due to negligence," since fellow employee Irene Soliven "obtained the medicines in bad faith
and through misrepresentation when she claimed that she was given a charge slip by the Admitting
Dept." Catolico then asked the company to look into the fraudulent activities of Soliven.

In a memorandum dated 21 November 1989, WATEROUS Supervisor Luzviminda E. Bautro


warned Catolico against the "rush delivery of medicines without the proper documents." On 29
January 1990, WATEROUS Control Clerk Eugenio Valdez informed Co that he noticed an
irregularity involving Catolico and Yung Shin Pharmaceuticals, Inc.
Forthwith, in her memorandum dated 37 January 1990, Co asked Catolico to explain, within twenty-
four hours, her side of the reported irregularity. Catolico asked for additional time to give her
explanation, and she was granted a 48-hour extension from 1 to 3 February 1990. However, on 2
February 1990, she was informed that effective 6 February 1990 to 7 March 1990, she would be
placed on preventive suspension to protect the interests of the company.

In a letter dated 2 February 1990, Catolico requested access to the file containing Sales Invoice No.
266 for her to be able to make a satisfactory explanation. In said letter she protested Saldaña's
invasion of her privacy when Saldaña opened an envelope addressed to Catolico.

In a letter to Co dated 10 February 1990, Catolico, through her counsel, explained that the check
she received from YSP was a Christmas gift and not a "refund of overprice." She also averred that
the preventive suspension was ill-motivated, as it sprang from an earlier incident between her and
Co's secretary, Irene Soliven.

On 5 March 1990, WATEROUS Supervisor Luzviminda Bautro, issued a memorandum notifying


Catolico of her termination. On 5 May 1990, Catolico filed before the Office of the Labor Arbiter a
complaint for unfair labor practice, illegal dismissal, and illegal suspension. In his decision of 10 May
1993, Labor Arbiter Alex Arcadio Lopez found no proof of unfair labor practice against petitioners.
Nevertheless, he decided in favor of Catolico because petitioners failed to "prove what alleged as
complainant's dishonesty," and to show that any investigation was conducted. Hence, the dismissal
Page 77
was without just cause and due process. He thus declared the dismissal and suspension illegal but
disallowed reinstatement.

Petitioners seasonably appealed from the decision and urged the NLRC to set it aside because the
Labor Arbiter erred in finding that Catolico was denied due process and that there was no just cause
to terminate her services.
In its decision of 30 September 1993, the NLRC affirmed the findings of the Labor Arbiter on the
ground that petitioners were not able to prove a just cause for Catolico's dismissal from her
employment. It found that petitioner's evidence consisted only of the check of P640.00 drawn by
YSP in favor of complainant, which her co-employee saw when the latter opened the envelope. But,
it declared that the check was inadmissible in evidence pursuant to Sections 2 and 3(1 and 2) of
Article III of the Constitution. It concluded:

With the smoking gun evidence of respondents being rendered inadmissible, by


virtue of the constitutional right invoked by complainants, respondents' case falls
apart as it is bereft of evidence which cannot be used as a legal basis for
complainant's dismissal.

The NLRC then dismissed the appeal for lack of merit, but modified the dispositive portion of the
appealed decision by deleting the award for illegal suspension as the same was already included in
the computation of the aggregate of the awards in the amount of P35,401.86.

Issue:

Whether or Not the dismissal of the private respondent is in violation of the Constitution, under the
Bill of Rights.

Held:

As to the first and second grounds, petitioners insist that Catolico had been receiving "commissions"
from YSP, or probably from other suppliers, and that the check issued to her on 9 November 1989
was not the first or the last. They also maintained that Catolico occupied a confidential position and
that Catolico's receipt of YSP's check, aggravated by her "propensity to violate company rules,"
constituted breach of confidence. And contrary to the findings of NLRC, Catolico was given ample
opportunity to explain her side of the controversy.

In her Comment, Catolico asserts that petitioners' evidence is too "flimsy" to justify her dismissal.
The check in issue was given to her, and she had no duty to turn it over to her employer. Company
rules do not prohibit an employee from accepting gifts from clients, and there is no indication in the
contentious check that it was meant as a refund for overpriced medicines. Besides, the check was
discovered in violation of the constitutional provision on the right to privacy and communication;
hence, as correctly held by the NLRC, it was inadmissible in evidence.

Catolico was denied due process. Procedural due process requires that an employee be apprised of
the charge against him, given reasonable time to answer the charge, allowed ample opportunity to
be heard and defend himself, and assisted by a representative if the employee so desires. Ample
opportunity connotes every kind of assistance that management must accord the employee to
enable him to prepare adequately for his defense, including legal representation. In the case at bar,
although Catolico was given an opportunity to explain her side, she was dismissed from the service
in the memorandum of 5 March 1990 issued by her Supervisor after receipt of her letter and that of
her counsel. No hearing was ever conducted after the issues were joined through said letters.

Catolico was also unjustly dismissed. It is settled that the burden is on the employer to prove just
and valid cause for dismissing an employee, and its failure to discharge that burden would result in
a finding that the dismissal is unjustified. It clearly appears then that Catolico's dismissal was based
on hearsay information. Catolico's dismissal then was obviously grounded on mere suspicion, which
Page 78
in no case can justify an employee's dismissal. Suspicion is not among the valid causes provided by
the Labor Code for the termination of employment; and even the dismissal of an employee for loss
of trust and confidence must rest on substantial grounds and not on the employer's arbitrariness,
whims, caprices, or suspicion. Besides, Catolico was not shown to be a managerial employee, to
which class of employees the term "trust and confidence" is restricted.
As regards the constitutional violation upon which the NLRC anchored its decision, that the Bill of
Rights does not protect citizens from unreasonable searches and seizures perpetrated by private
individuals. It is not true, as counsel for Catolico claims, that the citizens have no recourse against
such assaults. On the contrary, and as said counsel admits, such an invasion gives rise to both
criminal and civil liabilities.

Finally, since it has been determined by the Labor Arbiter that Catolico's reinstatement would not be
to the best interest of the parties, he correctly awarded separation pay to Catolico. Separation pay
in lieu of reinstatement is computed at one month's salary for every year of service. In this case,
however, Labor Arbiter Lopez computed the separation pay at one-half month's salary for every
year of service. Catolico did not oppose or raise an objection. As such, we will uphold the award of
separation pay as fixed by the Labor Arbiter.

WHEREFORE, the instant petition is hereby DISMISSED and the challenged decision and
resolution of the National Labor Relations Commission dated 30 September 1993 and 2 December
1993, respectively, in NLRC-NCR CA No. 005160-93 are AFFIRMED, except as to its reason for
upholding the Labor Arbiter's decision, viz., that the evidence against private respondent was
inadmissible for having been obtained in violation of her constitutional rights of privacy of
communication and against unreasonable searches and seizures which is hereby set aside.

STONEHILL VS. DIOKNO


[20 SCRA 383; L-19550; 19 JUN 1967]

Facts:

Upon application of the officers of the government named on the margin 1 — hereinafter referred to
as Respondents-Prosecutors — several judges2 — hereinafter referred to as Respondents-Judges
— issued, on different dates,3 a total of 42 search warrants against petitioners herein 4 and/or the
corporations of which they were officers,5 directed to the any peace officer, to search the persons
above-named and/or the premises of their offices, warehouses and/or residences, and to seize and
take possession of the following personal property to wit:
Books of accounts, financial records, vouchers, correspondence, receipts, ledgers, journals,
portfolios, credit journals, typewriters, and other documents and/or papers showing all
business transactions including disbursements receipts, balance sheets and profit and loss
statements and Bobbins (cigarette wrappers).

as "the subject of the offense; stolen or embezzled and proceeds or fruits of the offense," or "used
or intended to be used as the means of committing the offense," which is described in the
applications adverted to above as "violation of Central Bank Laws, Tariff and Customs Laws,
Internal Revenue (Code) and the Revised Penal Code."

Petitioners contentions are:


(1) they do not describe with particularity the documents, books and things to be seized;
(2) cash money, not mentioned in the warrants, were actually seized;
(3) the warrants were issued to fish evidence against the aforementioned petitioners in deportation
cases filed against them;
(4) the searches and seizures were made in an illegal manner; and
(5) the documents, papers and cash money seized were not delivered to the courts that issued the
warrants, to be disposed of in accordance with law —

Respondents-prosecutors contentions
Page 79
(1) that the contested search warrants are valid and have been issued in accordance with law;
(2) that the defects of said warrants, if any, were cured by petitioners' consent; and
(3) that, in any event, the effects seized are admissible in evidence against herein petitioners,
regardless of the alleged illegality of the aforementioned searches and seizures.

The documents, papers, and things seized under the alleged authority of the warrants in question
may be split into two (2) major groups, namely: (a) those found and seized in the offices of the
aforementioned corporations, and (b) those found and seized in the residences of petitioners herein.

Issue:

Whether or not those found and seized in the offices of the aforementioned corporations are
obtained legally.

Whether or not those found and seized in the residences of petitioners herein are obtained legally.

Held:

The petitioners have no cause of action to assail the legality of the contested warrants and of the
seizures made in pursuance thereof, for the simple reason that said corporations have their
respective personalities, separate and distinct from the personality of herein petitioners, regardless
of the amount of shares of stock or of the interest of each of them in said corporations, and
whatever the offices they hold therein may be. Indeed, it is well settled that the legality of a seizure
can be contested only by the party whose rights have been impaired thereby, and that the objection
to an unlawful search and seizure is purely personal and cannot be availed of by third parties.

With respect to the documents, papers and things seized in the residences of petitioners herein, the
aforementioned resolution of June 29, 1962, lifted the writ of preliminary injunction previously issued
by this Court, thereby, in effect, restraining herein Respondents-Prosecutors from using them in
evidence against petitioners herein.
Two points must be stressed in connection with this constitutional mandate, namely: (1) that no
warrant shall issue but upon probable cause, to be determined by the judge in the manner set forth
in said provision; and (2) that the warrant shall particularly describe the things to be seized.

None of these requirements has been complied with in the contested warrants. Indeed, the same
were issued upon applications stating that the natural and juridical person therein named had
committed a "violation of Central Ban Laws, Tariff and Customs Laws, Internal Revenue (Code) and
Revised Penal Code." In other words, no specific offense had been alleged in said applications. The
averments thereof with respect to the offense committed were abstract. As a consequence, it was
impossible for the judges who issued the warrants to have found the existence of probable cause,
for the same presupposes the introduction of competent proof that the party against whom it is
sought has performed particular acts, or committed specific omissions, violating a given provision of
our criminal laws. As a matter of fact, the applications involved in this case do not allege any
specific acts performed by herein petitioners. It would be the legal heresy, of the highest order, to
convict anybody of a "violation of Central Bank Laws, Tariff and Customs Laws, Internal Revenue
(Code) and Revised Penal Code," — as alleged in the aforementioned applications — without
reference to any determinate provision of said laws or
__________________________
1
Hon. Jose W. Diokno, in his capacity as Secretary of Justice, Jose Lukban, in his capacity as
Acting Director, National Bureau of Investigation, Special Prosecutors Pedro D. Cenzon, Efren I.
Plana and Manuel Villareal, Jr. and Assistant Fiscal Maneses G. Reyes, City of Manila.
2
Hon. Amado Roan, Judge of the Municipal (now City) Court of Manila, Hon. Roman Cansino,
Judge of the Municipal (now City) Court of Manila, Hon. Hermogenes Caluag, Judge of the Court of
First Instance of Rizal, Quezon City Branch, Hon. Eulogio Mencias, Judge of the Court of First
Instance of Rizal, Pasig Branch, and Hon. Damian Jimenez, Judge of the Municipal (now City) Court
of Quezon City.
3
Covering the period from March 3 to March 9, 1962.
Page 80
4
Harry S. Stonehill, Robert P. Brooks, John J. Brooks and Karl Beck.
5
U.S. Tobacco Corporation, Atlas Cement Corporation, Atlas Development Corporation, Far East
Publishing Corporation (Evening News), Investment Inc., Industrial Business Management
Corporation, General Agricultural Corporation, American Asiatic Oil Corporation, Investment
Management Corporation, Holiday Hills, Inc., Republic Glass Corporation, Industrial and Business
Management Corporation, United Housing Corporation, The Philippine Tobacco-Flue-Curing and
Redrying Corporation, Republic Real Estate Corporation and Merconsel Corporation.

BURGOS, SR. V. CHIEF OF STAFF, AFP


[133 SCRA 800; G.R. NO. 64261; 26 DEC 1984]

Facts:

Petitioners assail the validity of 2 search warrants issued on December 7, 1982 by respondent
Judge Cruz-Pano of the then Court of First Instance of Rizal, under which the premises known as
No. 19, Road 3, Project 6, Quezon City, and 784 Units C & D, RMS Building, Quezon Avenue,
Quezon City, business addresses of the "Metropolitan Mail" and "We Forum" newspapers,
respectively, were searched, and office and printing machines, equipment, paraphernalia, motor
vehicles and other articles used in the printing, publication and distribution of the said newspapers,
as well as numerous papers, documents, books and other written literature alleged to be in the
possession and control of petitioner Jose Burgos, Jr. publisher-editor of the "We Forum" newspaper,
were seized. As a consequence of the search and seizure, these premises were padlocked and
sealed, with the further result that the printing and publication of said newspapers were
discontinued. Respondents contend that petitioners should have filed a motion to quash said
warrants in the court that issued them before impugning the validity of the same before this Court.
Respondents also assail the petition on ground of laches (Failure or negligence for an unreasonable
and unexplained length of time to do that which, by exercising due diligence, could or should have
been done earlier. It is negligence or omission to assert a right within a reasonable time, warranting
a presumption that the party entitled to assert it either has abandoned it or declined to assert it).
Respondents further state that since petitioner had already used as evidence some of the
documents seized in a prior criminal case, he is stopped from challenging the validity of the search
warrants.

Petitioners submit the following reasons to nullify the questioned warrants:


1. Respondent Judge failed to conduct an examination under oath or affirmation of the
applicant and his witnesses, as mandated by the above-quoted constitutional provision as
well as Sec. 4, Rule 126 of the Rules of Court.
2. The search warrants pinpointed only one address which would be the former
abovementioned address.
3. Articles belonging to his co-petitioners were also seized although the warrants were only
directed against Jose Burgos, Jr.
4. Real properties were seized.
5. The application along with a joint affidavit, upon which the warrants were issued, from the
Metrocom Intelligence and Security Group could not have provided sufficient basis for the
finding of a probable cause upon which a warrant may be validly issued in accordance with
Section 3, Article IV of the 1973 Constitution.

Respondents justify the continued sealing of the printing machines on the ground that they have
been sequestered under Section 8 of Presidential Decree No. 885, as amended, which authorizes
sequestration of the property of any person engaged in subversive activities against the government
in accordance with implementing rules and regulations as may be issued by the Secretary of
National Defense.

Issue:

Whether or Not the 2 search warrants were validly issued and executed.
Page 81
Held:

In regard to the quashal of warrants that petitioners should have initially filed to the lower court, this
Court takes cognizance of this petition in view of the seriousness and urgency of the constitutional
Issue raised, not to mention the public interest generated by the search of the "We Forum" offices
which was televised in Channel 7 and widely publicized in all metropolitan dailies. The existence of
this special circumstance justifies this Court to exercise its inherent power to suspend its rules. With
the contention pertaining to laches, the petitioners gave an explanation evidencing that they have
exhausted other extra-judicial efforts to remedy the situation, negating the presumption that they
have abandoned their right to the possession of the seized property.

On the enumerated reasons:


1. This objection may properly be considered moot and academic, as petitioners themselves
conceded during the hearing on August 9, 1983, that an examination had indeed been
conducted by respondent judge of Col. Abadilla and his witnesses.
2. The defect pointed out is obviously a typographical error. Precisely, two search warrants
were applied for and issued because the purpose and intent were to search two distinct
premises. It would be quite absurd and illogical for respondent judge to have issued two
warrants intended for one and the same place.
3. Section 2, Rule 126, of the Rules of Court, does not require that the property to be seized
should be owned by the person against whom the search warrant is directed. It may or may
not be owned by him.
4. Petitioners do not claim to be the owners of the land and/or building on which the
machineries were placed. This being the case, the machineries in question, while in fact
bolted to the ground, remain movable property susceptible to seizure under a search
warrant.
5. The broad statements in the application and joint affidavit are mere conclusions of law and
does not satisfy the requirements of probable cause. Deficient of such particulars as would
justify a finding of the existence of probable cause, said allegation cannot serve as basis for
the issuance of a search warrant and it was a grave error for respondent judge to have done
so. In Alvarez v. Court of First Instance, this Court ruled that "the oath required must refer to
the truth of the facts within the personal knowledge of the petitioner or his witnesses,
because the purpose thereof is to convince the committing magistrate, not the individual
making the affidavit and seeking the issuance of the warrant, of the existence of probable
cause." Another factor which makes the search warrants under consideration
constitutionally objectionable is that they are in the nature of general warrants. The
description of the articles sought to be seized under the search warrants in question are too
general.

With regard to the respondents invoking PD 885, there is an absence of any implementing rules and
regulations promulgated by the Minister of National Defense. Furthermore, President Marcos
himself denies the request of military authorities to sequester the property seized from petitioners.
The closure of the premises subjected to search and seizure is contrary to the freedom of the press
as guaranteed in our fundamental law. The search warrants are declared null and void.

TAMBASEN VS. PEOPLE


[246 SCRA 184; G.R. NO. 89103; 14 JUL 1995]

Facts:

Page 82
In August 1988, P/Sgt. Natuel applied for issuance of search warrant alleging that he received
information that Petitioner had in his possession at his house “M-16 Armalite rifles, hand
grenades, .45 Cal. pistols, dynamite sticks and subversive documents”, which were “used or
intended to be used” for illegal purposes. The application was granted.

In September, a police team, searched the house of petitioner and seized “2 envelopes containing
P14000, handset with antennae, transceiver with antennae, regulator supply, academy notebook
and assorted papers and handset battery pack”. In October, petitioner moved that the search and
seizure be declared illegal and that the seized articles be returned to him. In December, MTCC, in
its order, directed Lt. Col. Torres to return the money seized to petitioner ruling that any seizure
should be limited to the specified items covered thereby. SolGen petitioned with the RTC for the
annulment of the order of MTCC citing that pending the determination of legality of seizure of the
articles, they should remain in custogia legis. RTC granted the petition.

Issue:

Whether or Not the seizure of the articles which were not mentioned in the search warrant was
legal.

Held:

Section 2 Article III of the 1987 Constitution requires that a search warrant should particularly
describe the things to be seized. The police acts beyond the parameters of their authority if they
seize articles not described in the search warrants. The evident purpose and intent of the
requirement is to limit the things to be seized, to leave the officers of the law with no discretion; that
unreasonable search and seizure may not be made and that abuses may not be committed.

Petition granted. People of the Philippines is ordered to return the money seized.

PLACER VS. JUDGE VILLANUEVA


[126 SCRA 463; G.R. NOS. L-60349-62; 29 DEC 1983]

Facts:

Petitioners filed informations in the city court and they certified that Preliminary Investigation and
Examination had been conducted and that prima facie cases have been found. Upon receipt of said
informations, respondent judge set the hearing of the criminal cases to determine propriety of
issuance of warrants of arrest. After the hearing, respondent issued an order requiring petitioners to
submit to the court affidavits of prosecution witnesses and other documentary evidence in support
of the informations to aid him in the exercise of his power of judicial review of the findings of
probable cause by petitioners. Petitioners petitioned for certiorari and mandamus to compel
respondent to issue warrants of arrest. They contended that the fiscal’s certification in the
informations of the existence of probable cause constitutes sufficient justification for the judge to
issue warrants of arrest.

Issue:

Whether or Not respondent city judge may, for the purpose of issuing warrants of arrest, compel the
fiscal to submit to the court the supporting affidavits and other documentary evidence presented
during the preliminary investigation.

Held:

Judge may rely upon the fiscal’s certification for the existence of probable cause and on the basis
thereof, issue a warrant of arrest. But, such certification does not bind the judge to come out with
the warrant. The issuance of a warrant is not a mere ministerial function; it calls for the exercise of
Page 83
judicial discretion on the part of issuing magistrate. Under Section 6 Rule 112 of the Rules of Court,
the judge must satisfy himself of the existence of probable cause before issuing a warrant of arrest.
If on the face of the information, the judge finds no probable cause, he may disregard the fiscal’s
certification and require submission of the affidavits of witnesses to aid him in arriving at the
conclusion as to existence of probable cause.

Petition dismissed.

SOLIVEN VS. MAKASIAR


[167 SCRA 393; G.R. NO. 82585; 14 NOV 1988]

Facts:

In these consolidated cases, three principal issues were raised: (1) whether or not petitioners were
denied due process when informations for libel were filed against them although the finding of the
existence of a prima facie case was still under review by the Secretary of Justice and, subsequently,
by the President; and (2) whether or not the constitutional rights of Beltran were violated when
respondent RTC judge issued a warrant for his arrest without personally examining the complainant
and the witnesses, if any, to determine probable cause. Subsequent events have rendered the first
issue moot and academic. On March 30, 1988, the Secretary of Justice denied petitioners' motion
for reconsideration and upheld the resolution of the Undersecretary of Justice sustaining the City
Fiscal's finding of a prima facie case against petitioners. A second motion for reconsideration filed
by petitioner Beltran was denied by the Secretary of Justice on April 7, 1988. On appeal, the
President, through the Executive Secretary, affirmed the resolution of the Secretary of Justice on
May 2, 1988. The motion for reconsideration was denied by the Executive Secretary on May 16,
1988. With these developments, petitioners' contention that they have been denied the
administrative remedies available under the law has lost factual support.

Issue:

Whether or Not petitioners were denied due process when informations for libel were filed against
them although the finding of the existence of a prima facie case was still under review by the
Secretary of Justice and, subsequently, by the President.

Whether or Not the constitutional rights of Beltran were violated when respondent RTC judge issued
a warrant for his arrest without personally examining the complainant and the witnesses, if any, to
determine probable cause

Held:

With respect to petitioner Beltran, the allegation of denial of due process of law in the preliminary
investigation is negated by the fact that instead of submitting his counter- affidavits, he filed a
"Motion to Declare Proceedings Closed," in effect waiving his right to refute the complaint by filing
counter-affidavits. Due process of law does not require that the respondent in a criminal case
actually file his counter-affidavits before the preliminary investigation is deemed completed. All that
is required is that the respondent be given the opportunity to submit counter-affidavits if he is so
minded.

The second issue, raised by petitioner Beltran, calls for an interpretation of the constitutional
provision on the issuance of warrants of arrest. The pertinent provision reads:

Art. III, Sec. 2. The right of the people to be secure in their persons, houses, papers
and effects against unreasonable searches and seizures of whatever nature and for
any purpose shall be inviolable, and no search warrant or warrant of arrest shall
Page 84
issue except upon probable cause to be determined personally by the judge after
examination nder oath or affirmation of the complainant and the witnesses he may
produce, and particularly describing the place to be searched and the persons or
things to be seized.

The addition of the word "personally" after the word "determined" and the deletion of the grant of
authority by the 1973 Constitution to issue warrants to "other responsible officers as may be
authorized by law," has apparently convinced petitioner Beltran that the Constitution now requires
the judge to personally examine the complainant and his witnesses in his determination of probable
cause for the issuance of warrants of arrest. This is not an accurate interpretation.

What the Constitution underscores is the exclusive and personal responsibility of the issuing judge
to satisfy himself of the existence of probable cause. In satisfying himself of the existence of
probable cause for the issuance of a warrant of arrest, the judge is not required to personally
examine the complainant and his witnesses. Following established doctrine and procedure, he shall:
(1) personally evaluate the report and the supporting documents submitted by the fiscal regarding
the existence of probable cause and, on the basis thereof, issue a warrant of arrest; or (2) if on the
basis thereof he finds no probable cause, he may disregard the fiscal's report and require the
submission of supporting affidavits of witnesses to aid him in arriving at a conclusion as to the
existence of probable cause.

Sound policy dictates this procedure, otherwise judges would be unduly laden with the preliminary
examination and investigation of criminal complaints instead of concentrating on hearing and
deciding cases filed before their courts. It has not been shown that respondent judge has deviated
from the prescribed procedure. Thus, with regard to the issuance of the warrants of arrest, a finding
of grave abuse of discretion amounting to lack or excess of jurisdiction cannot be sustained. The
petitions fail to establish that public respondents, through their separate acts, gravely abused their
discretion as to amount to lack of jurisdiction. Hence, the writs of certiorari and prohibition prayed for
cannot issue.

WHEREFORE, finding no grave abuse of discretion amounting to excess or lack of jurisdiction on


the part of the public respondents, the Court Resolved to DISMISS the petitions in G. R. Nos.
82585, 82827 and 83979. The Order to maintain the status quo contained in the Resolution of the
Court en banc dated April 7, 1988 and reiterated in the Resolution dated April 26, 1988 is LIFTED.

SALAZAR VS. ACHACOSO


[183 SCRA 145; G.R. NO. 81510; 14 MAR 1990]

Facts:

Rosalie Tesoro of Pasay City in a sworn statement filed with the POEA, charged petitioner with
illegal recruitment. Public respondent Atty. Ferdinand Marquez sent petitioner a telegram directing
him to appear to the POEA regarding the complaint against him. On the same day, after knowing
that petitioner had no license to operate a recruitment agency, public respondent Administrator
Tomas Achacoso issued a Closure and Seizure Order No. 1205 to petitioner. It stated that there will
a seizure of the documents and paraphernalia being used or intended to be used as the means of
committing illegal recruitment, it having verified that petitioner has— (1) No valid license or authority
from the Department of Labor and Employment to recruit and deploy workers for overseas
employment; (2) Committed/are committing acts prohibited under Article 34 of the New Labor Code
in relation to Article 38 of the same code. A team was then tasked to implement the said Order. The
group, accompanied by mediamen and Mandaluyong policemen, went to petitioner’s residence.
They served the order to a certain Mrs. For a Salazar, who let them in. The team confiscated
assorted costumes. Petitioner filed with POEA a letter requesting for the return of the seized
properties, because she was not given prior notice and hearing. The said Order violated due
process. She also alleged that it violated sec 2 of the Bill of Rights, and the properties were
confiscated against her will and were done with unreasonable force and intimidation.

Page 85
Issue:

Whether or Not the Philippine Overseas Employment Administration (or the Secretary of Labor) can
validly issue warrants of search and seizure (or arrest) under Article 38 of the Labor Code

Held:

Under the new Constitution, “. . . no search warrant or warrant of arrest shall issue except upon
probable cause to be determined personally by the judge after examination under oath or
affirmation of the complainant and the witnesses he may produce, and particularly describing the
place to be searched and the persons or things to be seized”. Mayors and prosecuting officers
cannot issue warrants of seizure or arrest. The Closure and Seizure Order was based on Article 38
of the Labor Code. The Supreme Court held, “We reiterate that the Secretary of Labor, not being a
judge, may no longer issue search or arrest warrants. Hence, the authorities must go through the
judicial process. To that extent, we declare Article 38, paragraph (c), of the Labor Code,
unconstitutional and of no force and effect… The power of the President to order the arrest of aliens
for deportation is, obviously, exceptional. It (the power to order arrests) cannot be made to extend to
other cases, like the one at bar. Under the Constitution, it is the sole domain of the courts.”
Furthermore, the search and seizure order was in the nature of a general warrant. The court held
that the warrant is null and void, because it must identify specifically the things to be seized.

WHEREFORE, the petition is GRANTED. Article 38, paragraph (c) of the Labor Code is declared
UNCONSTITUTIONAL and null and void. The respondents are ORDERED to return all materials
seized as a result of the implementation of Search and Seizure Order No. 1205.

MORANO VS. VIVO


[20 SCRA 562; G.R. L-22196; 30 JUN 1967]

Facts:

Chan Sau Wah, a Chinese citizen born in Fukien, China arrived in the Philippines on November
1961 to visit her cousin, Samuel Lee Malaps. She left China and her children by a first marriage: Fu
Tse Haw and Fu Yan Kai both minors, in the care of neighbors in Fukien, China. Chan Sau wah
arrived in the Philippines with Fu Yan Fun, her minor son also by the first marriage. Chan Sau Wah
and her minor son Fu Yan Fun were permitted only into the Philippines under a temporary visitor's
visa for two months and after they posted a cash bond of 4,000 pesos. On January 1962, Chan Sau
Wah married Esteban Morano, a native-born Filipino citizen. Born to this union on September 1962
was Esteban Morano, Jr. To prolong their stay in the Philippines, Chan Sau Wah and Fu Yan Fun
obtained several extensions. The last extension expired on September 10, 1962. In a letter dated
August 31, 1962, the Commissioner of Immigration ordered Chan Sau Wah and her son, Fu Yan
Fun, to leave the country on or before September 10, 1962 with a warning that upon failure so to do,
he will issue a warrant for their arrest and will cause the confiscation of their bond.

Issue:

Whether or Not the issuance of the warrant of arrest is unconstitutional.

Held:

Chan Sau Wah entered the Philippines on a tourist-temporary visitor's visa. She is a non-immigrant.
Under Section 13 just quoted, she may therefore be admitted if she were a qualified and desirable
alien and subject to the provisions of the last paragraph of Section 9. Therefore, first, she must
depart voluntarily to some foreign country; second, she must procure from the appropriate consul
the proper visa; and third, she must thereafter undergo examination by the officials of the Bureau of
Page 86
Immigration at the port of entry for determination of her admissibility in accordance with the
requirements of the immigration Act. This Court in a number of cases has ruled, and consistently
too, that an alien admitted as a temporary visitor cannot change his or her status without first
departing from the country and complying with the requirements of Section 9 of the Immigration Act.
The gravamen of petitioners' argument is that Chan Sau Wah has, since her entry, married in
Manila a native-born Filipino, Esteban Morano. It will not particularly help analysis for petitioners to
appeal to family solidarity in an effort to thwart her deportation. Chan Sau Wah, seemingly is not
one who has a high regard for such solidarity. Proof: She left two of her children by the first
marriage, both minors, in the care of neighbors in Fukien, China.Then, the wording of the statute
heretofore adverted to is a forbidding obstacle which will prevent this Court from writing into the law
an additional provision that marriage of a temporary alien visitor to a Filipino would ipso facto make
her a permanent resident in his country. This is a field closed to judicial action. No breadth of
discretion is allowed. We cannot insulate her from the State's power of deportation. it would be an
easy matter for an alien woman to enter the Philippines as a temporary visitor, go through a mock
marriage, but actually live with another man as husband and wife, and thereby skirt the provisions of
our immigration law. Also, a woman of undesirable character may enter this country, ply a
pernicious trade, marry a Filipino, and again throw overboard Sections 9 and 13 of the Act. Such a
flanking movement, we are confident, is impermissible.Recently we confirmed the rule that an alien
wife of a Filipino may not stay permanently without first departing from the Philippines. Reason:
Discourage entry under false pretenses.

HARVEY V. DEFENSOR-SANTIAGO
[162 SCRA 840; G.R. NO. 82544; 28 JUN 1988]

Facts:

This is a petition for Habeas Corpus. Petitioners are the following: American nationals Andrew
Harvey, 52 and Jonh Sherman 72. Dutch Citizen Adriaan Van Den Elshout, 58. All reside at
Pagsanjan Laguna respondent Commissioner Miriam Defensor Santiago issued Mission Orders to
the Commission of Immigration and Deportation (CID) to apprehended petitioners at their
residences. The “Operation Report” read that Andrew Harvey was found together with two young
boys. Richard Sherman was found with two naked boys inside his room. While Van Den Elshout in
the “after Mission Report” read that two children of ages 14 and 16 has been under his care and
subjects confirmed being live-in for sometime now.

Seized during the petitioner’s apprehension were rolls of photo negatives and photos of suspected
child prostitutes shown in scandalous poses as well as boys and girls engaged in sex. Posters and
other literature advertising the child prostitutes were also found.

Petitioners were among the 22 suspected alien pedophiles. They were apprehended 17
February1988 after close surveillance for 3 month of the CID in Pagsanjan, Laguna. 17 of the
arrested aliens opted for self-deportation. One released for lack of evidence, another charged not
for pedophile but working with NO VISA, the 3 petitioners chose to face deportation proceedings.
On 4 March1988, deportation proceedings were instituted against aliens for being undesirable
aliens under Sec.69 of Revised Administrative Code.

Warrants of Arrest were issued 7March1988 against petitioners for violation of Sec37, 45 and 46 of
Immigration Act and sec69 of Revised Administrative Code. Trial by the Board of Special Inquiry III
commenced the same date. Petition for bail was filed 11March 1988 but was not granted by the
Commissioner of Immigration. 4 April1988 Petitioners filed a petition for Writ of Habeas Corpus.
The court heard the case on oral argument on 20 April 1988.

Issue:
Page 87
Whether or Not the Commissioner has the power to arrest and detain petitioners pending
determination of existence of probable cause.

Whether or Not there was unreasonable searches and seizures by CID agents.

Whether or Not the writ of Habeas Corpus may be granted to petitioners.

Held:

While pedophilia is not a crime under the Revised Penal Code, it violates the declared policy of the
state to promote and protect the physical, moral, spiritual and social well being of the youth. The
arrest of petitioners was based on the probable cause determined after close surveillance of 3
months. The existence of probable cause justified the arrest and seizure of articles linked to the
offense. The articles were seized as an incident to a lawful arrest; therefore the articles are
admissible evidences (Rule 126, Section12 of Rules on Criminal Procedure).

The rule that search and seizures must be supported by a valid warrant of arrest is not an absolute
rule. There are at least three exceptions to this rule. 1.) Search is incidental to the arrest. 2.)
Search in a moving vehicle. 3.) Seizure of evidence in plain view. In view of the foregoing, the
search done was incidental to the arrest.

The filing of the petitioners for bail is considered as a waiver of any irregularity attending their arrest
and estops them from questioning its validity. Furthermore, the deportation charges and the
hearing presently conducted by the Board of Special Inquiry made their detention legal. It is a
fundamental rule that habeas corpus will not be granted when confinement is or has become legal,
although such confinement was illegal at the beginning.

The deportation charges instituted by the Commissioner of Immigration are in accordance with
Sec37 (a) of the Philippine Immigration Act of 1940 in relation to sec69 of the Revised
Administrative code. Section 37 (a) provides that aliens shall be arrested and deported upon
warrant of the Commissioner of Immigration and Deportation after a determination by the Board of
Commissioners of the existence of a ground for deportation against them. Deportation proceedings
are administrative in character and never construed as a punishment but a preventive measure.
Therefore, it need not be conducted strictly in accordance with ordinary Court proceedings. What is
essential is that there should be a specific charge against the alien intended to be arrested and
deported. A fair hearing must also be conducted with assistance of a counsel if desired.

Lastly, the power to deport aliens is an act of the State and done under the authority of the
sovereign power. It a police measure against the undesirable aliens whose continued presence in
the country is found to be injurious to the public good and tranquility of the people.

SALES VS. SANDIGANBAYAN


[369 SCRA 293 G.R. NO. 143802; 16 NOV 2001]

Facts:

The petitioner, the incumbent mayor of Pagudpud Ilocos Norte, shot the former mayor and his
political rival Atty. Benemerito. After the shooting, he surrendered himself and hence the police
inspector and wife of the victim filed a criminal complaint for murder against him. The judge after
conducting the preliminary examination (p.e. for brevity) found probable cause and issued a warrant
of arrest. Also after conducting the preliminary investigation (p.i. for brevity), he issued a resolution
forwarding the case to the prosecutor for appropriate action. Petitioner received a subpoena
directing him to file his counter affidavit, affidavit of witnesses and other supporting documents. He
did it the following day. While proceedings are ongoing, he filed a petition for habeas corpus with
Page 88
the C.A alleging that: the warrant was null and void because the judge who issued it was a relative
by affinity of the private respondent and the p.e. and the p.i. were illegal and irregular as the judge
doesn’t have jurisdiction on the case. The C.A. granted the petition holding that the judge was a
relative by affinity by 3rd degree to the private respondent and the p.i. he conducted has 2 stages,
the p.e. and the p.i. proper. The proceeding now consists only of one stage. He conducted the
requisite investigation prior to the issuance of warrant of arrest. Moreover he did not complete it. He
only examined the witness of the complainant. But the prosecution instead of conducting p.i. of his
own forwarded the records to the Ombudsman (OMB for brevity) for the latter to conduct the same.
The OMB directed the petitioner to submit his counter affidavit, but he did not comply with it finding
the same superfluous. The graft investigator recommended the filing of information for murder which
the OMB approved. Petitioner received a copy of the resolution but prevented seeking
reconsideration thereof he filed a motion to defer issuance of warrant of arrest pending the
determination of probable cause. The Sandiganbayan denied the motion. This is now a petition for
review on the decision of the Sandiganbayan,

Issue:

Whether or Not the OMB followed the procedure in conducting preliminary investigation.

Whether or Not petitioner was afforded an opportunity to be heard and to submit controverting
evidence.

Held:

The proper procedure in the conduct of preliminary investigation was not followed because of the
following reasons. Firstly, the preliminary investigation was conducted by 3 different investigators,
none of whom completed the preliminary investigation There was not one continuous proceeding
but rather, cases of passing the buck, the last one being the OMB throwing the buck to the
Sandiganbayan. Secondly, the charge of murder is a non bailable offense. The gravity of the
offense alone should have merited a deeper and more thorough preliminary investigation. The OMB
did nothing of the sort but wallowed the resolution of the graft investigator. He did a worse job than
the judge, by actually adopting the resolution of the graft investigator without doing anything and
threw everything to the Sandiganbayan for evaluation. Thirdly, a person under preliminary
investigation by the OMB is entitled to a motion for reconsideration, as maintained by the Rules of
Procedure by the OMB. The filing of the motion for reconsideration is an integral part of the
preliminary investigation proper. The denial thereof is tantamount to the denial of the right itself to a
preliminary investigation. This fact alone renders preliminary investigation conducted in this case
incomplete. And lastly, it was patent error for the Sandiganbayan to have relied purely on the OMB’s
certification of probable cause given the prevailing facts of the case much more so in the face of the
latter’s flawed report and one side factual findings.

The court cannot accept the Sandiganbayan’s assertion of having found probable cause on its own,
considering the OMB’s defective report and findings, which merely rekied on the testimonies of the
witnesses for the prosecution and disregarded the evidence for the defense.

Judgment is rendered setting aside the resolution of the Sandiganbayan, ordering the
Sandiganbayan to quash the warrant of arrest and remanding the OMB for completion of the
preliminary investigation.

SILVA VS. PRESIDING JUDGE


[203 SCRA 140; G.R. No. 81756; 21 Oct 1991]

Facts:

Sgt. Villamor, chief of the PC Narcom Detachment in Dumaguete City filed an "application for
search warrant" and "Deposition of witness" against petitioner Nicomedes Silva and Martin Silva.
Page 89
Judge Nickarter Ontal, then the presiding judge of RTC of Dumaguete issued Search Warrant No.1
pursuant to the said applications for violation of RA 6425 Dangerous Drugs ACT of 1972. Such
warrant states that there is a probable cause to believe that Mr. Tama Silva has the possession and
control of marijuana dried leaves, cigarette and joint. The warrant authorizes Sgt. Villamor to make
an immediate search at any time of the room of Mr. Tama Silva at the residence of his father
Comedes Silva and to open aparadors, lockers, cabinets, cartons and containers to look for said
illegal drugs. In the course of the search, the officers seized money belonging to Antonieta Silva in
the amount of P1,231.40. Petitioner filed a motion to quash Search Warrant No.1 on the ground that
1) it was issued on the sole basis of mimeographed 2) the judge failed to personally examine the
complainant and witness by searching questions and answers.

Issue:

Whether or Not Search Warrant No.1 is invalid. WON the officers abused their authority in seizing
the money of Antonieta Silva.

Held:

Search Warrant No. 1 is invalid due to the failure of the judge to examine the witness in the form of
searching questions and answers. The questions asked were leading as they are answerable by
mere yes or no. Such questions are not sufficiently searching to establish probable cause. The
questions were already mimeographed and all the witness had to do was fill in their answers on the
blanks provided. Judge Ontal is guilty of grave abuse of discretion when he rejected the motion of
Antonieta Silva seeking the return of her money.

The officers who implemented the search warrant clearly abused their authority when they seized
the money of Antonieta Silva. The warrant did not indicate the seizure of money but only for
marijuana leaves, cigarettes..etc. Search Warrant No. 1 is declared null and void.

*** Sec 4 Rule 126 Rules of Court


Examination of the complainant, record -the judge before issuing the warrant, personally examine in
the form of searching questions and answers, in writing and under oath the complainant and any
witness he may produce the facts personally known to them and attach to the record their sworn
statements together with their affidavits.

VEROY VS. LAYAGUE


[210 SCRA 97; G.R. No. 95630; 18 Jun 1992]

Facts:

Petitioners are husband and wife who owned and formerly resided at No. 13 Isidro St., Skyline
Village. Catalunan Grande, Davao City. When petitioner Leopoldo Veroy was promoted to the
position of Assistant Administrator of the Social Security System sometime in June, 1988, he and
his family transferred to 130 K-8th St., East Kamias, Quezon City, where they are presently
residing. The care and upkeep of their residence in Davao City was left to two (2) houseboys,
Jimmy Favia and Eric Burgos, who had their assigned quarters at a portion of the premises. The
Veroys would occasionally send money to Edna Soguilon for the salary of the said houseboys and
other expenses for the upkeep of their house. While the Veroys had the keys to the interior of the
house, only the key to the kitchen, where the circuit breakers were located, was entrusted to Edna
Soguilon to give her access in case of an emergency. Hence, since 1988, the key to the master's
Page 90
bedroom as well as the keys to the children's rooms were retained by herein Petitioners so that
neither Edna Soguilon nor the caretakers could enter the house.

Police Officers had an information that the petitioner’s residence was being used as a safehouse of
rebel soldiers. They were able to enter the yard with the help of the caretakers but did not enter the
house since the owner was not present and they did not have a search warrant. Petitioner Ma.
Luisa was contacted by telephone in her Quezon City residence by Capt. Obrero to ask permission
to search the house in Davao City as it was reportedly being used as a hideout and recruitment
center of rebel soldiers. Petitioner Ma. Luisa Veroy responded that she is flying to Davao City to
witness the search but relented if the search would not be conducted in the presence of Major
Ernesto Macasaet, an officer of the PC/INP, Davao City and a long time family friend of the Veroys.

The following day, Capt. Obrero and Major Macasaet met at the house of herein petitioners in
Skyline Village to conduct the search pursuant to the authority granted by petitioner Ma. Luisa
Veroy. The caretakers facilitated their entry into the yard, and using the key entrusted to Edna
Soguilon, they were able to gain entrance into the kitchen. However, a locksmith by the name of
George Badiang had to be employed to open the padlock of the door leading to the children's room.
Capt. Obrero and Major Macasaet then entered the children's room and conducted the search.
Capt. Obrero recovered a .45 cal. handgun with a magazine containing seven (7) live bullets in a
black clutch bag inside an unlocked drawer. Three (3) half-full jute sacks containing printed
materials of RAM-SFP were also found in the children's room. A search of the children's recreation
and study area revealed a big travelling bag containing assorted polo shirts, men's brief, two (2)
pieces polo barong and short sleeve striped gray polo. sweat shirt, two (2) pairs men's socks, a
towel made in U.S.A., one blanket, a small black bag, Gandhi brand, containing a book entitled
"Islamic Revolution Future Path of the Nation", a road map of the Philippines, a telescope, a plastic
bag containing assorted medicines and religious pamphlets was found in the master's bedroom.
Sgt. Leo Justalero was instructed by Capt. Obrero to make an inventory and receipt of the articles
seized, in the house.

The case was referred for preliminary investigation to Quezon City Assistant Prosecutor Rodolfo
Ponferrada who was designated Acting Provincial Prosecutor for Davao City. In a resolution dated
August 6, 1990, Fiscal Ponferrada recommended the filing of an information against herein
petitioners for Violation of Presidential Decree No. 1866 (Illegal Possession of Firearms and
Ammunitions in Furtherance of Rebellion). No bail was recommended.

Issue:

Whether or Not Presidential Decree No. 1866, or at least the third paragraph of Section 1 thereof, is
unconstitutional for being violative of the due process and equal protection clauses of the
Constitution.

Held:

The issue of constitutionality of Presidential Decree No. 1866 has been laid to rest in the case of
Misolas v. Panga, G.R. No. 83341, January 30, 1990 (181 SCRA 648), where this Court held that
the declaration of unconstitutionality of the third paragraph of Section 1 of Presidential Decree No.
1866 is wanting in legal basis since it is neither a bill of attainder nor does it provide a possibility of a
double jeopardy.

Petitioners' contention that Republic Act 6968 has repealed Presidential Decree No. 1866 is bereft
of merit. It is a cardinal rule of statutory construction that where the words and phrases of a statute
are not obscure or ambiguous. its meaning and the intention of the legislature must be determined
from the language employed, and where there is no ambiguity in the words, there is no room for
construction. Petitioners contend that Section 1 of Presidential Decree No. 1866 is couched in
general or vague terms. The terms "deal in", "acquire", "dispose" or "possess" are capable of
various interpretations such that there is no definiteness as to whether or not the definition includes
"constructive possession" or how the concept of constructive possession should be applied.
Page 91
Petitioners were not found in actual possession of the firearm and ammunitions. They were in
Quezon City while the prohibited articles were found in Davao City. Yet they were being charged
under Presidential Decree No. 1866 upon the sole circumstance that the house wherein the items
were found belongs to them.

Petitioners question the admissibility in evidence of the articles seized in violation of their
constitutional right against unreasonable search and seizure. Petitioners aver that while they
concede that Capt. Obrero had permission from Ma. Luisa Veroy to break open the door of their
residence, it was merely for the purpose of ascertaining thereat the presence of the alleged "rebel"
soldiers. The permission did not include any authority to conduct a room to room search once inside
the house. The items taken were, therefore, products of an illegal search, violative of their
constitutional rights As such, they are inadmissible in evidence against them.

The Constitution guarantees the right of the people to be secure in their persons, houses, papers
and effects against unreasonable searches and seizures (Article III, Section 2 of the 1987
Constitution). However, the rule that searches and seizures must be supported by a valid warrant is
not an absolute one. Among the recognized exceptions thereto are: (1) a search incidental to an
arrest; (2) a search of a moving vehicle; and (3) seizure of evidence in plain view (People v. Lo Ho
Wing, G.R. No. 88017, January 21, 1991 [193 SCRA 122]).

None of these exceptions pertains to the case at bar. The reason for searching the house of herein
petitioners is that it was reportedly being used as a hideout and recruitment center for rebel soldiers.
While Capt. Obrero was able to enter the compound, he did not enter the house because he did not
have a search warrant and the owners were not present. This shows that he himself recognized the
need for a search warrant, hence, he did not persist in entering the house but rather contacted the
Veroys to seek permission to enter the same. Permission was indeed granted by Ma. Luisa Veroy to
enter the house but only to ascertain the presence of rebel soldiers. Under the circumstances it is
undeniable that the police officers had ample time to procure a search warrant but did not.

Undeniably, the offense of illegal possession of firearms is malum prohibitum but it does not follow
that the subject thereof is necessarily illegal per se. Motive is immaterial in mala prohibita but the
subjects of this kind of offense may not be summarily seized simply because they are prohibited. A
search warrant is still necessary. Hence, the rule having been violated and no exception being
applicable, the articles seized were confiscated illegally and are therefore protected by the
exclusionary principle. They cannot be used as evidence against the petitioners in the criminal
action against them for illegal possession of firearms. (Roan v. Gonzales, 145 SCRA 689-690
[1986]). Besides, assuming that there was indeed a search warrant, still in mala prohibita, while
there is no need of criminal intent, there must be knowledge that the same existed. Without the
knowledge or voluntariness there is no crime.

PREMISES CONSIDERED, the petition as granted and the criminal case against the petitioners for
illegal possession of firearms is DISMISSED.

PEOPLE VS. DEL ROSARIO


[234 SCRA 246; G.R. NO. 109633; 20 JUL 1994]

Facts:

Accused was charged and convicted by the trial court of illegal possession of firearms and illegal
possession and sale of drugs, particularly methamphetamine or shabu. After the issuance of the
search warrant, which authorized the search and seizure of an undetermined quantity of
methamphetamine and its paraphernalia’s, an entrapment was planned that led to the arrest of del

Page 92
Rosario and to the seizure of the shabu, its paraphernalia’s and of a .22 caliber pistol with 3 live
ammunition.

Issue:

Whether or Not the seizure of the firearms was proper.

Held:

No. Sec 2 art. III of the constitution specifically provides that a search warrant must particularly
describe the things to be seized. In herein case, the only objects to be seized that the warrant
determined was the methamphetamine and the paraphernalia’s therein. The seizure of the firearms
was unconstitutional.

Wherefore the decision is reversed and the accused is acquitted.

PEOPLE VS. GESMUNDO


[219 SCRA 743; G.R. NO. 89373; 19 MAR 1993]

Facts:

According to the prosecution, in the morning of Nov. 17, 1986, PO Jose Luciano gave money and
instructed his civilian informer to buy marijuana from the accused at the Cocoland Hotel. He actually
saw the accused selling marijuana to his civilian informer and that same day Luciano applied for a
search warrant.

About 2pm that day, a police raiding team armed with a search warrant went to the Brgy captain for
them to be accompanied in serving the said warrant at the residence of the accused. The police
was allowed to enter the house upon the strength of the warrant shown to the accused. The
accused begged the police not to search and to leave the house. The police still searched the
house and was led to the kitchen. She pointed a metal basin on top of a table as the hiding place of
died marijuana flowering tops contained in a plastic bag marked ISETANN. The police also
recovered from a native “uway” cabinet dried marijuana flowering tops wrapped in 3 pieces of
komiks paper.

According to the accused, when the police arrived at her house, she saw Sgt. Yte and PFC Jose
Luciano. She invited Sgt. Yte to enter her house while Luciano was left in the jeep that was parked
near the house. While inside the house Yte showed the accused something he claimed as a search
warrant, when someone coming from the kitchen uttered “eto na” They proceeded to the kitchen
and saw Luciano holding a plastic bag with four other companions. They confronted the accused
and insisted that the bags belonged to her. Accused denied the accusation and told them that she
doesn’t know anything about it. She was made to sign a prepared document. She was brought to
the police station and was detained.

The court renders judgment finding the accused guilty.

Issue:

Whether or Not the evidence was properly obtained by the police.

Held:

Page 93
In the investigation report prepared by Luciano stated that during the search they discovered a hole
at the backyard of the house of the suspect, there was a big biscuit can inside the hole and on top
of the cover a flower pot was placed wherein the marijuana was kept. However, there was no
mention of any marijuana obtained from a flower pot in any of their testimonies. There were
inconsistencies insofar the prosecution is concerned, as to what was recovered and where, the trial
court concluded that these inconsistencies are trivial. There must sufficient evidence that the
marijuana was actually surrendered by the accused. As held in PP vs. Remorosa, Irreconcilable
and unexplained contradictions in the testimonies of the prosecution witnesses cast doubt on the
guilt of appellant and his culpability to the crime charged.

The claim that the marijuana was planted was strengthen as the police violated sec 7, rule 126
rules of the court provides no search of a house, room or any other premise shall be made except in
the presence of the lawful occupant thereof or any member of his family or in the absence of the
latter, in the presence of two (2) witnesses of sufficient age and discretion residing in the same
locality. This requirement is mandatory to ensure regularity in the execution of the search warrant.
Violation of said rule is in fact punishable under Article 130 of the Revised Penal Code.

The document (PAGPAPATUNAY) was inadmissible to the court as the accused was not informed
of her right not to sign the document neither was she informed that she has the right to the
assistance of a counsel and the fact that it may be used as evidence against her. It was not proved
that the marijuana belonged to her. Not only does the law require the presence of witnesses when
the search is conducted, but it also imposes upon the person making the search the duty to issue a
detailed receipt for the property seized. He is likewise required to deliver the property seized to the
judge who issued the warrant, together with a true and accurate inventory thereof duly verified
under oath. Again, these duties are mandatory and are required to preclude substitution of the items
seized by interested parties.

The guilt of the accused was has not been established. Judgment is reversed.

UMIL VS. RAMOS


[187 SCRA 311; G.R. NO. 81567; 3 OCT 1991]

Facts:

On 1 February 1988, military agents were dispatched to the St. Agnes Hospital, Roosevelt Avenue,
Quezon City, to verify a confidential information which was received by their office, about a "sparrow
man" (NPA member) who had been admitted to the said hospital with a gunshot wound. That the
wounded man in the said hospital was among the five (5) male "sparrows" who murdered two (2)
Capcom mobile patrols the day before, or on 31 January 1988 at about 12:00 o'clock noon, before a
road hump along Macanining St., Bagong Barrio, Caloocan City. The wounded man's name was
listed by the hospital management as "Ronnie Javellon," twenty-two (22) years old of Block 10, Lot
4, South City Homes, Biñan, Laguna however it was disclosed later that the true name of the
wounded man was Rolando Dural. In view of this verification, Rolando Dural was transferred to the
Regional Medical Servicesof the CAPCOM, for security reasons. While confined thereat, he was
positively identified by the eyewitnesses as the one who murdered the 2 CAPCOM mobile patrols.

Issue:

Whether or Not Rolando was lawfully arrested.

Held:

Rolando Dural was arrested for being a member of the NPA, an outlawed subversive organization.
Subversion being a continuing offense, the arrest without warrant is justified as it can be said that
he was committing as offense when arrested. The crimes rebellion, subversion, conspiracy or
Page 94
proposal to commit such crimes, and crimes or offenses committed in furtherance therefore in
connection therewith constitute direct assaults against the state and are in the nature of continuing
crimes.

PEOPLE VS. SUCRO


[195 SCRA 388; G.R. No. 93239; 18 Mar 1991]

Facts:

Pat. Fulgencio went to Arlie Regalado’s house at C. Quimpo to monitor activities of Edison SUCRO
(accused). Sucro was reported to be selling marijuana at a chapel 2 meters away from Regalado’s
house. Sucro was monitored to have talked and exchanged things three times. These activities are
reported through radio to P/Lt. Seraspi. A third buyer was transacting with appellant and was
reported and later identified as Ronnie Macabante. From that moment, P/Lt.Seraspi proceeded to
the area. While the police officers were at the Youth Hostel in Maagama St. Fulgencio told Lt.
Seraspi to intercept. Macabante was intercepted at Mabini and Maagama crossing in front of Aklan
Medical center. Macabante saw the police and threw a tea bag of marijuana on the ground.
Macabante admitted buying the marijuana from Sucro in front of the chapel.

The police team intercepted and arrested SUCRO at the corner of C. Quimpo and Veterans.
Recovered were 19 sticks and 4 teabags of marijuana from a cart inside the chapel and another
teabag from Macabante.

Issue:

Whether or Not arrest without warrant is lawful.

Whether or Not evidence from such arrest is admissible.

Held:

Search and seizures supported by a valid warrant of arrest is not an absolute rule. Rule 126, Sec
12 of Rules of Criminal Procedure provides that a person lawfully arrested may be searched for
dangerous weapons or anything, which may be used as proff of the commission of an offense,
without a search warrant.(People v. Castiller) The failure of the police officers to secure a warrant
stems from the fact that their knowledge required from the surveillance was insufficient to fulfill
requirements for its issuance. However, warantless search and seizures are legal as long as
PROBABLE CAUSE existed. The police officers have personal knowledge of the actual
commission of the crime from the surveillance of the activities of the accused. As police officers
were the ones conducting the surveillance, it is presumed that they are regularly in performance of
their duties.

PEOPLE V. RODRIGUEZA
[205 SCRA 791; G.R. No. 95902; 4 Feb 1992]

Facts:

NARCOM agents staged a buy-bust operation, after gaining information that there was an ongoing
illegal traffic of prohibited drugs in Tagas, Albay. The participating agents were given money
treated with ultraviolet powder. One of the agents went to said location, asked for a certain Don.
Thereafter, the Don, herein accused, met with him and “a certain object wrapped in a plastic” later
identified as marijuana was given in exchange for P200. The agent went back to headquarters and
made a report, based on which, a team was subsequently organized and a raid was conducted in
the house of the father of the accused. During the raid, the NARCOM agents were able to
confiscate dried marijuana leaves and a plastic syringe among others. There was no authorization
Page 95
by any search warrant. The accused was found positive of ultraviolet powder. The lower court,
considering the evidences obtained and testimonies from the prosecution, found him guilty of
violating the Dangerous Drugs Act of 1972 and sentenced him to reclusion perpetua.

Issue:

Whether or Not the lower court was correct in its judgment.

Held:

The NARCOM agents’ procedure in the entrapment of the accused failed to meet the qualification
that the suspected drug dealer must be caught red-handed in the act of selling marijuana to a
person posing as a buyer, since the operation was conducted after the actual exchange. Said raid
also violated accused’ right against unreasonable search and seizure, as the situation did not fall in
the circumstances wherein a search may be validly made even without a search warrant, i.e. when
the search is incidental to a lawful arrest; when it involves prohibited articles in plain view. The
NARCOM agents could not have justified their act by invoking the urgency and necessity of the
situation because the testimonies of the prosecution witnesses reveal that the place had already
been put under surveillance for quite some time. Had it been their intention to conduct the raid, then
they should, because they easily could, have first secured a search warrant during that time. The
Court further notes the confusion and ambiguity in the identification of the confiscated marijuana
leaves and other prohibited drug paraphernalia presented as evidence against appellant:

CIC Taduran, who acted as the poseur buyer, testified that appellant sold him 100 grams of dried
marijuana leaves wrapped in a plastic bag. Surprisingly, and no plausible explanation has been
advanced therefor, what were submitted to and examined by the PCCL and thereafter utilized as
evidence against the appellant were the following items:

One (1) red and white colored plastic bag containing the following:

Exh. "A"—Thirty (30) grams of suspected dried marijuana fruiting tops contained
inside a transparent plastic bag.
Exh. "B"— Fifty (50) grams of suspected dried marijuana leaves and seeds contained
inside a white colored plastic labelled "Robertson".
Exh. "C"— Four (4) aluminum foils each containing suspected dried marijuana
fruiting tops having a total weight of seven grams then further wrapped
with a piece of aluminum foil.
Exh. "D"— Five (5) small transparent plastic bags each containing suspected dried
marijuana fruiting tops having a total weight of seventeen grams.
Exh. "E"— One plastic syringe.

Evidently, these prohibited articles were among those confiscated during the so-called follow-up raid
in the house of Rodrigueza’s father. The unanswered question then arises as to the identity of the
marijuana leaves that became the basis of appellant's conviction. In People vs. Rubio, this Court
had the occasion to rule that the plastic bag and the dried marijuana leaves contained therein
constitute the corpus delicti of the crime. As such, the existence thereof must be proved with
certainty and conclusiveness. Failure to do so would be fatal to the cause of the prosecution.
Conviction is reversed and set aside and accused is acquitted.

PEOPLE VS. SY CHUA


[396 SCRA 657; G.R. No.136066-67; 4 Feb 2003]

Facts:
Page 96
Accused-appellant Binad Sy Chua was charged with violation of Section 16, Article III of R.A. 6425,
as amended by R.A. 7659, and for Illegal Possession of Ammunitions and Illegal Possession of
Drugs in two separate Informations.

SPO2 Nulud and PO2 Nunag received a report from their confidential informant that accused-
appellant was about to deliver drugs that night at the Thunder Inn Hotel in Balibago, Angeles City.
So, the PNP Chief formed a team of operatives. The group positioned themselves across McArthur
Highway near Bali Hai Restaurant, fronting the hotel. The other group acted as their back up.

Afterwards, their informer pointed to a car driven by accused-appellant which just arrived and
parked near the entrance of the hotel. After accused-appellant alighted from the car carrying a
sealed Zest-O juice box, SPO2 Nulud and PO2 Nunag hurriedly accosted him and introduced
themselves as police officers. As accused-appellant pulled out his wallet, a small transparent plastic
bag with a crystalline substance protruded from his right back pocket. Forthwith, SPO2 Nulud
subjected him to a body search which yielded twenty (20) pieces of live .22 caliber firearm bullets
from his left back pocket. When SPO2 Nunag peeked into the contents of the Zest-O box, he saw
that it contained a crystalline substance. SPO2 Nulud instantly confiscated the small transparent
plastic bag, the Zest-O juice box, the twenty (20) pieces of .22 caliber firearm bullets and the car
used by accused-appellant. SPO2 Nulud and the other police operatives who arrived at the scene
brought the confiscated items to the office of Col. Guttierez at the PNP Headquarters in Camp
Pepito, Angeles City.

Accused-appellant vehemently denied the accusation against him and narrated a different version
of the incident.

Accused-appellant alleged that he was driving the car of his wife to follow her and his son to Manila.
He felt sleepy, so he decided to take the old route along McArthur Highway. He stopped in front of a
small store near Thunder Inn Hotel to buy cigarettes and candies. While at the store, he noticed a
man approaches and examines the inside of his car. When he called the attention of the onlooker,
the man immediately pulled out a .45 caliber gun and made him face his car with raised hands. The
man later on identified himself as a policeman. During the course of the arrest, the policeman took
out his wallet and instructed him to open his car. He refused, so the policeman took his car keys
and proceeded to search his car. At this time, the police officer’s companions arrived at the scene in
two cars. PO2 Nulud, who just arrived at the scene, pulled him away from his car in a nearby bank,
while the others searched his car.

Thereafter, he was brought to a police station and was held inside a bathroom for about fifteen
minutes until Col. Guttierez arrived, who ordered his men to call the media. In the presence of
reporters, Col. Guttierez opened the box and accused-appellant was made to hold the box while
pictures were being taken.

The lower court acquitted Sy Chua for the Illegal Possession of Ammunitions, yet convicted him for
Illegal Possession of 1,955.815 grams of shabu. Hence, this appeal to the Court.

Issue:

Whether or Not the arrest of accused-appellant was lawful; and (2) WON the search of his person
and the subsequent confiscation of shabu allegedly found on him were conducted in a lawful and
valid manner.

Held:

The lower court believed that since the police received information that the accused will distribute
illegal drugs that evening at the Thunder Inn Hotel and its vicinities. The police officer had to act
quickly and there was no more time to secure a search warrant. The search is valid being akin to a
“stop and frisk”.
Page 97
The trial court confused the concepts of a “stop-and-frisk” and of a search incidental to a lawful
arrest. These two types of warrantless searches differ in terms of the requisite quantum of proof
before they may be validly effected and in their allowable scope.

In a search incidental to a lawful arrest, as the precedent arrest determines the validity of the
incidental search, the legality of the arrest is questioned, e.g., whether an arrest was merely used
as a pretext for conducting a search. In this instance, the law requires that there first be arrest
before a search can be made—the process cannot be reversed. Accordingly, for this exception to
apply, two elements must concur: (1) the person to be arrested must execute an overt act
indicating that he has just committed, is actually committing, or is attempting to commit a crime; and
(2) such overt act is done in the presence or within the view of the arresting officer.

We find the two aforementioned elements lacking in the case at bar. Accused-appellant did not act
in a suspicious manner. For all intents and purposes, there was no overt manifestation that
accused-appellant has just committed, is actually committing, or is attempting to commit a crime.
“Reliable information” alone, absent any overt act indicative of a felonious enterprise in the
presence and within the view of the arresting officers, is not sufficient to constitute probable cause
that would justify an in flagrante delicto arrest.

With regard to the concept of “stop-and frisk”: mere suspicion or a hunch will not validate a “stop-
and-frisk”. A genuine reason must exist, in light of the police officer’s experience and surrounding
conditions, to warrant the belief that the person detained has weapons concealed about him.
Finally, a “stop-and-frisk” serves a two-fold interest: (1) the general interest of effective crime
prevention and detection for purposes of investigating possible criminal behavior even without
probable cause; and (2) the interest of safety and self-preservation which permit the police officer to
take steps to assure himself that the person with whom he deals is not armed with a deadly weapon
that could unexpectedly and fatally be used against the police officer.

A stop-and-frisk was defined as the act of a police officer to stop a citizen on the street, interrogate
him, and pat him for weapon(s) or contraband. It should also be emphasized that a search and
seizure should precede the arrest for this principle to apply. The foregoing circumstances do not
obtain in the case at bar. To reiterate, accused-appellant was first arrested before the search and
seizure of the alleged illegal items found in his possession. The apprehending police operative
failed to make any initial inquiry into accused-appellant’s business in the vicinity or the contents of
the Zest-O juice box he was carrying. The apprehending police officers only introduced themselves
when they already had custody of accused-appellant.

In the case at bar, neither the in flagrante delicto nor the “stop and frisk” principles is applicable to
justify the warrantless arrest and consequent search and seizure made by the police operatives on
accused-appellant.

Wherefore, accused-appellant Binad Sy Chua is hereby Acquitted.

GO VS. COURT OF APPEALS


[206 SCRA 138; G.R. NO. 101837; 11 FEB 1992]

Facts:

Petitioner, while traveling in the wrong direction on a one-way street, almost had a collision with
another vehicle. Petitioner thereafter got out of his car, shot the driver of the other vehicle, and
drove off. An eyewitness of the incident was able to take down petitioner’s plate number and
reported the same to the police, who subsequently ordered a manhunt for petitioner. 6 days after
the shooting, petitioner presented himself in the police station, accompanied by 2 lawyers, the
police detained him. Subsequently a criminal charge was brought against him. Petitioner posted
bail, the prosecutor filed the case to the lower court, setting and commencing trial without
preliminary investigation. Prosecutor reasons that the petitioner has waived his right to preliminary
Page 98
investigation as bail has been posted and that such situation, that petitioner has been arrested
without a warrant lawfully, falls under Section 5, Rule 113 and Section 7, Rule 112 of The 1985
Rules of Criminal Procedure which provides for the rules and procedure pertaining to situations of
lawful warrantless arrests. Petitioner in his petition for certiorari assails such procedure and actions
undertaken and files for a preliminary investigation.

Issue:

Whether or Not warrantless arrest of petitioner was lawful.

Whether or Not petitioner effectively waived his right to preliminary investigation.

Held:

Petitioner and prosecutor err in relying on Umil v. Ramos, wherein the Court upheld the warrantless
arrest as valid effected 1 to 14 days from actual commission of the offenses, which however
constituted “continuing crimes,” i.e. subversion, membership in an outlawed organization, etc.
There was no lawful warrantless arrest under Section 5, Rule 113. This is because the arresting
officers were not actually there during the incident, thus they had no personal knowledge and their
information regarding petitioner were derived from other sources. Further, Section 7, Rule 112,
does not apply.

Petitioner was not arrested at all, as when he walked in the police station, he neither expressed
surrender nor any statement that he was or was not guilty of any crime. When a complaint was filed
to the prosecutor, preliminary investigation should have been scheduled to determine probable
cause. Prosecutor made a substantive error, petitioner is entitled to preliminary investigation,
necessarily in a criminal charge, where the same is required appear thereat. Petition granted,
prosecutor is ordered to conduct preliminary investigation, trial for the criminal case is suspended
pending result from preliminary investigation, petitioner is ordered released upon posting a bail
bond.

CALLANTA VS. VILLANUEVA


[77 SCRA 377; G.R. NOS. 24646 & L-24674; 20 JUN 1977]

Facts:

Two complaints for grave oral defamation were filed against Faustina Callanta. The City Judge of
Dagupan City, Felipe Villanueva, denied the motions to quash the complaints. Thus, petitioner
Callanta brought the suits for certiorari in the Supreme Court. Petitioner questions the validity of the
issuance of warrant of arrest by respondent, arguing that the City Fiscal should have conducted the
preliminary investigation. According to petitioner’s counsel, there was jurisdictional infirmity. After
the issuance of the warrants of arrest and the bail fixed at P600, petitioner posted the bail bond,
thus obtaining her provisional liberty. The City Fiscal in this case did not disagree with the judge’s
investigation, and agreed with the complaints filed.

Issue:

Whether or Not petitioner’s contentions are to be given merit.

Held:

Based on many precedent cases of the Supreme Court, “where the accused has filed bail and
waived the preliminary investigation proper, he has waived whatever defect, if any, in the
preliminary examination conducted prior to the issuance of the warrant of arrest”. In the case at bar,
it is futile for the petitioner to question the validity of the issuance of the warrant of arrest, because
Page 99
she posted the bail bond. Petitioner also erred in arguing that only the City Fiscal can conduct a
preliminary investigation. According to the Charter of the City of Dagupan, “the City Court of
Dagupan City may also conduct preliminary investigation for any offense, without regard to the limits
of punishment, and may release, or commit and bind over any person charged with such offense to
secure his appearance before the proper court”. Petition for certiorari is denied. Restraining order
issued by the Court is lifted and set aside.

POSADAS VS. COURT OF APPEALS


[188 SCRA 288; G.R. NO. 89139; 2 AUG 1990]

Facts:

Members of the Integrated National Police (INP) of the Davao Metrodiscom assigned with the
Intelligence Task Force, Pat. Ursicio Ungab and Pat. Umbra Umpar conducted surveillance along
Magallanes Street, Davao City. While in the vicinity of Rizal Memorial Colleges they spotted
petitioner carrying a "buri" bag and they noticed him to be acting suspiciously. They approached the
petitioner and identified themselves as members of the INP. Petitioner attempted to flee but his
attempt to get away was unsuccessful. They then checked the "buri" bag of the petitioner where
they found one (1) caliber .38 Smith & Wesson revolver with Serial No. 770196, two (2) rounds of
live ammunition for a .38 caliber gun, a smoke (tear gas) grenade, and two (2) live ammunitions for
a .22 caliber gun. They brought the petitioner to the police station for further investigation. In the
course of the same, the petitioner was asked to show the necessary license or authority to possess
firearms and ammunitions found in his possession but he failed to do so. He was then taken to the
Davao Metrodiscom office and the prohibited articles recovered from him were indorsed to M/Sgt.
Didoy the officer then on duty. He was prosecuted for illegal possession of firearms and
ammunitions in the Regional Trial Court of Davao City.

Issue:

Whether or Not the warantless search is valid.

Held:

In justifying the warrantless search of the buri bag then carried by the petitioner, argues that under
Section 12, Rule 136 of the Rules of Court a person lawfully arrested may be searched for
dangerous weapons or anything used as proof of a commission of an offense without a search
warrant. It is further alleged that the arrest without a warrant of the petitioner was lawful under the
circumstances.

in the case at bar, there is no question that, indeed, it is reasonable considering that it was effected
on the basis of a probable cause. The probable cause is that when the petitioner acted suspiciously
and attempted to flee with the buri bag there was a probable cause that he was concealing
something illegal in the bag and it was the right and duty of the police officers to inspect the same.

It is too much indeed to require the police officers to search the bag in the possession of the
petitioner only after they shall have obtained a search warrant for the purpose. Such an exercise
may prove to be useless, futile and much too late.

Page 100
Clearly, the search in the case at bar can be sustained under the exceptions heretofore discussed,
and hence, the constitutional guarantee against unreasonable searches and seizures has not been
violated.

PEOPLE V. MENGOTE
[210 SCRA 174; G.R. NO. 87059; 22 JUN 1992]

Facts:

The Western Police District received a telephone call from an informer that there were three
suspicious looking persons at the corner of Juan Luna and North Bay Boulevard in Tondo, Manila. A
surveillance team of plainclothesmen was forthwith dispatched to the place. The patrolmen saw two
men looking from side to side, one of whom holding his abdomen. They approached the persons
and identified themselves as policemen, whereupon the two tried to run but unable to escape
because the other lawmen surrounded them. The suspects were then searched. One of them the
accused-appellant was found with a .38 caliber with live ammunitions in it, while his companion had
a fan knife. The weapons were taken from them and they were turned over to the police
headquarters for investigation. An information was filed before the RTC convicting the accused of
illegal possession of firearm arm. A witness testified that the weapon was among the articles stolen
at his shop, which he reported to the police including the revolver. For his part, Mengote made no
effort to prove that he owned the fire arm or that he was licensed to possess it but instead, he
claimed that the weapon was planted on him at the time of his arrest. He was convicted for violation
of P.D.1866 and was sentenced to reclusion perpetua. In his appeal he pleads that the weapon was
not admissible as evidence against him because it had been illegally seized and therefore the fruit
of a poisonous tree.

Issue:

Whether or not the warrantless search and arrest was illegal.

Held:

An evidence obtained as a result of an illegal search and seizure inadmissible in any proceeding for
any purpose as provided by Art. III sec 32 of the Constitution. Rule 113 sec.5 of the Rules of Court,
provides arrest without warrant lawful when: (a) the person to be arrested has committed, is actually
committing, or is attempting to commit an offense, (b) when the offense in fact has just been
committed, and he has personal knowledge of the facts indicating the person arrested has
committed it and (c) the person to be arrested has escaped from a penal establishment or a place
where he is serving final judgment or temporarily confined while his case is pending, or has
escaped while being transferred from one confinement to another.

These requirements have not been established in the case at bar. At the time of the arrest in
question, the accused appellant was merely looking from side to side and holding his abdomen,
according to the arresting officers themselves. There was apparently no offense that has just been
committed or was being actually committed or at least being attempt by Mengote in their presence.
Moreover a person may not be stopped and frisked in a broad daylight or on a busy street on
unexplained suspicion.

Judgment is reversed and set aside. Accused-appellant is acquitted.

Page 101
PEOPLE VS. TANGLIBEN
[184 SCRA 220; G.R. No.L-63630; 6 Apr 1990]

Facts:

Patrolmen Silverio and Romeo Punzalan were conducting surveillance at the San Fernando Victory
Liner Terminal. At around 9:30pm they noticed a person, Medel Tangliben, carrying a traveling bag
who acted suspiciously. They confronted him, inspected his bag, and there they found marijuana
leaves. The accused was then taken to the Police Headquarters for further investigations. The TC
found Tangliben guilty of violating sec.4 art. 2 of the RA 6425 or the Dangerous Drugs Act of 1972.

Issue:

Whether or Not there was an unlawful search due to lack of search warrant.

Held;

No. Rule 113 sec. 5 provides the a peace officer or a private person may w/o a warrant arrest a
person when in his presence the person to be arrested has committed, is committing, or is
attempting to commit an offense.

In the present case, the accused was found to have been committing possession of marijuana and
can be therefore searched lawfully even without a search warrant. Another reason is that this case
poses urgency on the part of the arresting police officers. It was found out that an informer pointed
to the accused telling the policemen that the accused was carrying marijuana. The police officers
had to act quickly and there was not enough time to secure a search warrant.

PEOPLE VS. MALMSTEDT


[198 SCRA 401; G.R. No. 91107; 19 Jun 1991]

Facts:

In an information filed against the accused- appellant Mikael Malmstead was charged before the
RTC of La Trinidad, Benguet, for violation of Section 4, Art. II of Republic Act 6425, as amended,
otherwise known as the Dangerous Drugs Act of 1972, as amended.
Accused Mikael Malmstedt, a Swedish national, entered the Philippines for the third time in
December 1988 as a tourist. He had visited the country sometime in 1982 and 1985.

In the evening of 7 May 1989, accused left for Baguio City. Upon his arrival thereat in the morning of
the following day, he took a bus to Sagada and stayed in that place for two (2) days. Then in the 7
in the morning of May 11, 1989, the accused went to Nangonogan bus stop in Sagada.

At about 8: 00 o'clock in the morning of that same day (11 May 1989), Captain Alen Vasco, the
Commanding Officer of the First Regional Command (NARCOM) stationed at Camp Dangwa,
ordered his men to set up a temporary checkpoint at Kilometer 14, Acop, Tublay, Mountain
Province, for the purpose of checking all vehicles coming from the Cordillera Region. The order to
establish a checkpoint in the said area was prompted by persistent reports that vehicles coming
from Sagada were transporting marijuana and other prohibited drugs. Moreover, information was
received by the Commanding Officer of NARCOM, that same morning that a Caucasian coming
from Sagada had in his possession prohibited drugs. The group composed of seven (7) NARCOM
officers, in coordination with Tublay Police Station, set up a checkpoint at the designated area at
about 10:00 o'clock in the morning and inspected all vehicles coming from the Cordillera Region.

The two (2) NARCOM officers started their inspection from the front going towards the rear of the
bus. Accused who was the sole foreigner riding the bus was seated at the rear thereof.
Page 102
During the inspection, CIC Galutan noticed a bulge on accused's waist. Suspecting the bulge on
accused's waist to be a gun, the officer asked for accused's passport and other identification
papers. When accused failed to comply, the officer required him to bring out whatever it was that
was bulging on his waist. The bulging object turned out to be a pouch bag and when accused
opened the same bag, as ordered, the officer noticed four (4) suspicious-looking objects wrapped in
brown packing tape, prompting the officer to open one of the wrapped objects. The wrapped objects
turned out to contain hashish, a derivative of marijuana.

Thereafter, accused was invited outside the bus for questioning. But before he alighted from the
bus, accused stopped to get two (2) travelling bags from the luggage carrier. Upon stepping out of
the bus, the officers got the bags and opened them. A teddy bear was found in each bag. Feeling
the teddy bears, the officer noticed that there were bulges inside the same which did not feel like
foam stuffing. It was only after the officers had opened the bags that accused finally presented his
passport.

Accused was then brought to the headquarters of the NARCOM at Camp Dangwa, La Trinidad,
Benguet for further investigation. At the investigation room, the officers opened the teddy bears and
they were found to also contain hashish. Representative samples were taken from the hashish
found among the personal effects of accused and the same were brought to the PC Crime
Laboratory for chemical analysis.
In the chemistry report, it was established that the objects examined were hashish. a prohibited
drug which is a derivative of marijuana. Thus, an information was filed against accused for violation
of the Dangerous Drugs Act.
ACCUSED’S DEFENSE
During the arraignment, accused entered a plea of "not guilty." For his defense, he raised the issue
of illegal search of his personal effects. He also claimed that the hashish was planted by the
NARCOM officers in his pouch bag and that the two (2) travelling bags were not owned by him, but
were merely entrusted to him by an Australian couple whom he met in Sagada. He further claimed
that the Australian couple intended to take the same bus with him but because there were no more
seats available in said bus, they decided to take the next ride and asked accused to take charge of
the bags, and that they would meet each other at the Dangwa Station.

The trial court found the guilt of the accused Mikael Malmstedt established beyond reasonable
doubt.

Seeking the reversal of the decision of the trial court finding him guilty of the crime charged,
accused argues that the search of his personal effects was illegal because it was made without a
search warrant and, therefore, the prohibited drugs which were discovered during the illegal search
are not admissible as evidence against him.

Issue:

Whether or Not the contention of the accused is valid, and therefore the RTC ruling be reversed.

Held:

The Constitution guarantees the right of the people to be secure in their persons, houses, papers
and effects against unreasonable searches and seizures. However, where the search is made
pursuant to a lawful arrest, there is no need to obtain a search warrant. A lawful arrest without a
warrant may be made by a peace officer or a private person under the following circumstances.

Sec. 5 Arrest without warrant; when lawful. –– A peace officer or a private person
may, without a warrant, arrest a person:

Page 103
(a) When, in his presence, the person to be arrested has committed is actually
committing, or is attempting to commit an offense;
(b) When an offense has in fact just been committed, and he has personal
knowledge of facts indicating that the person to be arrested has committed it;
and
(c) When the person to be arrested is a prisoner who has escaped from a penal
establishment or place where he is serving final judgment or temporarily
confined while his case is pending, or has escaped while being transferred
from one confinement to another.

Accused was searched and arrested while transporting prohibited drugs (hashish). A crime was
actually being committed by the accused and he was caught in flagrante delicto. Thus, the search
made upon his personal effects falls squarely under paragraph (1) of the foregoing provisions of
law, which allow a warrantless search incident to a lawful arrest. While it is true that the NARCOM
officers were not armed with a search warrant when the search was made over the personal effects
of accused, however, under the circumstances of the case, there was sufficient probable cause for
said officers to believe that accused was then and there committing a crime.

Probable cause has been defined as such facts and circumstances which could lead a reasonable,
discreet and prudent man to believe that an offense has been committed, and that the objects
sought in connection with the offense are in the place sought to be searched. Warrantless search of
the personal effects of an accused has been declared by this Court as valid, because of existence
of probable cause, where the smell of marijuana emanated from a plastic bag owned by the
accused, 10 or where the accused was acting suspiciously, 11 and attempted to flee.

The appealed judgment of conviction by the trial court is hereby affirmed. Costs against the
accused-appellant.

PEOPLE VS. AMMINUDIN


[163 SCRA 402; G.R. L-74869; 6 Jul 1988]

Facts:

Idel Aminnudin, accused-appellant was arrested on June 25, 1984, shortly after disembarking from
the M/V Wilcon 9 at about 8:30 in the evening, in Iloilo City. The PC officers who were in fact waiting
for him because of a tip from one their informers simply accosted him, inspected his bag and finding
what looked liked marijuana leaves took him to their headquarters for investigation. The two bundles
of suspect articles were confiscated from him and later taken to the NBI laboratory for examination.
It was found to contain three kilos of what were later analyzed as marijuana leaves by an NBI
forensic examiner. An information for violation of the Dangerous Drugs Act was filed against him.
Later, the information was amended to include Farida Ali y Hassen, who had also been arrested
with him that same evening and likewise investigated. Both were arraigned and pleaded not guilty.
Subsequently, the fiscal filed a motion to dismiss the charge against Ali on the basis of a sworn
statement of the arresting officers absolving her after a 'thorough investigation." The motion was
granted, and trial proceeded only against the accused-appellant, who was eventually convicted . In
his defense, Aminnudin disclaimed the marijuana, averring that all he had in his bag was his
clothing consisting of a jacket, two shirts and two pairs of pants. He alleged that he was arbitrarily
arrested and immediately handcuffed. His bag was confiscated without a search warrant. At the PC
headquarters, he was manhandled to force him to admit he was carrying the marijuana, the
investigator hitting him with a piece of wood in the chest and arms even as he parried the blows
while he was still handcuffed. He insisted he did not even know what marijuana looked like and that
his business was selling watches and sometimes cigarettes. However the RTC rejected his
allegations. Saying that he only has two watches during that time and that he did not sufficiently
proved the injuries allegedly sustained.

Issue:
Page 104
Whether or not search of defendant’s bag is legal.

Held:

The search was illegal. Defendant was not caught in flagrante delicto, which could allow warrantless
arrest or search. At the moment of his arrest, he was not committing a crime. Nor was he about to
do so or had just done so. To all appearances, he was like any of the other passengers innocently
disembarking from the vessel. The said marijuana therefore could not be appreciated as evidence
against the defendant, and furthermore he is acquitted of the crime as charged.

PEOPLE VS. SAYCON


[236 SCRA 325; G.R. NO. 110995; 5 SEPT 1994]

Facts:

On or about 8 July 1992, at about 6:00 in the morning, the Coastguard personnel received
information from NARCOM agent Ruben Laddaran that a suspected "shabu" courier by the name of
Alvaro Saycon was on board the MV Doña Virginia, which was arriving at that moment in
Dumaguete City. Upon receipt of the information, the Coastguard chief officer CPO Tolin, instructed
them to intercept the suspect. A combined team of NARCOM agents and Philippine Coastguard
personnel consisting of CPO Tolin, a certain Miagme, and Senior Police Officers Ruben Laddaran
and Winifredo Noble of NARCOM posted themselves at the gate of Pier 1. The MV Doña Virginia
docked at 6:00 a.m. that same morning at Pier 1 in Dumaguete City. Alvaro Saycon alighted from
the boat carrying a black bag and went through the checkpoint manned by the Philippine
Coastguard where he was identified by police officer Winifredo Noble of NARCOM. Saycon was
then invited to the Coastguard Headquarters at the Pier area. He willingly went with them. At the
headquarters, the coastguard asked Saycon to open his bag, and the latter willingly obliged. In it
were personal belongings and a maong wallet. Inside that maong wallet, there was a Marlboro pack
containing the suspected "shabu". When police officer Winifredo Noble asked Saycon whether the
Marlboro pack containing the suspected "shabu" was his, Saycon merely bowed his head. Then
Saycon, his bag and the suspected "shabu" were brought to the NARCOM office for booking. When
Alvaro Saycon was arrested, the NARCOM agents did not have a warrant of arrest. The PNP's
Forensic Analyst declared in court that she had conducted an examination of the specimens and
found out that the specimens weighed 4.2 grams in total, consisted of methamphetamine
hydrochloride, more widely known as "shabu."

Issue:

Whether or Not the warrantless search was valid.

Held:

The warrantless search was valid, as the accused was a passenger of a motor vehicle. There was
probable cause to believe that the accused was carrying prohibited drugs. Three weeks earlier,
agents of the Narcotics Command bought methamine hydrochloride from him. An agent of the
Narcotics Command reported that the accused would be arriving on board the vessel and carrying
methamphetamine hydrochloride with him. Drug couriers do not go about their trade with some
external sign indicating that they are transporting prohibited drugs. This must be taken into account
in determining probable cause.

PEOPLE VS. MUSA


[217 SCRA 597; G.,R. NO. 96177; 27 JAN 1993]

Facts:
Page 105
A civilian informer gave the information that Mari Musa was engaged in selling marijuana in
Suterville, Zamboanga City. Sgt. Ani was ordered by NARCOM leader T/Sgt. Belarga, to conduct a
surveillance and test buy on Musa. The civilian informer guided Ani to Musa’s house and gave the
description of Musa. Ani was able to buy one newspaper-wrapped dried marijuana for P10.00.

The next day, a buy-bust was planned. Ani was to raise his right hand if he successfully buys
marijuana from Musa. As Ani proceeded to the house, the NARCOM team positioned themselves
about 90 to 100 meters away. From his position, Belarga could see what was going on. Musa came
out of the house and asked Ani what he wanted. Ani said he wanted more marijuana and gave
Musa the P20.00 marked money. Musa went into the house and came back, giving Ani two
newspaper wrappers containing dried marijuana. Ani opened and inspected it. He raised his right
hand as a signal to the other NARCOM agents, and the latter moved in and arrested Musa inside
the house. Belarga frisked Musa in the living room but did not find the marked money (gave it to his
wife who slipped away). T/Sgt. Belarga and Sgt. Lego went to the kitchen and found a ‘cellophane
colored white and stripe hanging at the corner of the kitchen.’ They asked Musa about its contents
but failed to get a response. So they opened it and found dried marijuana leaves inside. Musa was
then placed under arrest.

Issue:

Whether or Not the seizure of the plastic bag and the marijuana inside it is unreasonable, hence,
inadmissible as evidence.

Held:

Yes. It constituted unreasonable search and seizure thus it may not be admitted as evidence. The
warrantless search and seizure, as an incident to a suspect’s lawful arrest, may extend beyond the
person of the one arrested to include the premises or surroundings under his immediate control.
Objects in the ‘plain view’ of an officer who has the right to be in the position to have that view are
subject to seizure and may be presented as evidence. The ‘plain view’ doctrine is usually applied
where a police officer is not searching for evidence against the accused, but nonetheless
inadvertently comes across an incriminating object. It will not justify the seizure of the object where
the incriminating nature of the object is not apparent from the ‘plain view’ of the object.

In the case at bar, the plastic bag was not in the ‘plain view’ of the police. They arrested the
accused in the living room and moved into the kitchen in search for other evidences where they
found the plastic bag. Furthermore, the marijuana inside the plastic bag was not immediately
apparent from the ‘plain view’ of said object.

Therefore, the ‘plain view’ does not apply. The plastic bag was seized illegally and cannot be
presented in evidence pursuant to Article III Section 3 (2) of the Constitution.

PITA VS. COURT OF APPEALS


[178 SCRA 362; G.R. NO. 80806; 5 OCT 1989]

Facts:

On December 1 and 3, 1983, pursuing an Anti-Smut Campaign initiated by the Mayor of the City of
Manila, Ramon D. Bagatsing, elements of the Special Anti-Narcotics Group, Auxilliary Services
Bureau, Western Police District, INP of the Metropolitan Police Force of Manila, seized and
confiscated from dealers, distributors, newsstand owners and peddlers along Manila sidewalks,
magazines, publications and other reading materials believed to be obscene, pornographic and
indecent and later burned the seized materials in public at the University belt along C.M. Recto
Avenue, Manila, in the presence of Mayor Bagatsing and several officers and members of various
student organizations.

Page 106
Among the publications seized, and later burned, was "Pinoy Playboy" magazines published and
co-edited by plaintiff Leo Pita.

Plaintiff filed a case for injunction with prayer for issuance of the writ of preliminary injunction against
Mayor Bagatsing and Narcisco Cabrera, as superintendent of Western Police District of the City of
Manila, seeking to enjoin said defendants and their agents from confiscating plaintiff’s magazines or
from preventing the sale or circulation thereof claiming that the magazine is a decent, artistic and
educational magazine which is not per se obscene, and that the publication is protected by the
Constitutional guarantees of freedom of speech and of the press. Plaintiff also filed an Urgent
Motion for issuance of a temporary restraining order against indiscriminate seizure, confiscation and
burning of plaintiff's "Pinoy Playboy" Magazines, pending hearing on the petition for preliminary
injunction. The Court granted the temporary restraining order. The case was set for trial upon the
lapse of the TRO. RTC ruled that the seizure was valid. This was affirmed by the CA.

Issue:

Whether or Not the seizure was illegal.

Held:

The Court ruled that the government authorities have not shown the required proof to justify a ban
and to warrant confiscation of the literature. First of all, they were not possessed of a lawful court
order: (1) finding the said materials to be pornography, and (2) authorizing them to carry out a
search and seizure, by way of a search warrant. The court provides the following guidelines to be
observed:

1. The authorities must apply for the issuance of a search warrant from a judge, if in their
opinion an obscenity seizure is in order;
2. The authorities must convince the court that the materials sought to be seized are obscene
and pose a clear and present danger of an evil substantive enough to warrant State
interference and action;
3. The judge must determine whether or not the same are indeed obscene. The question is to
be resolved on a case-to-case basis and on the judge’s sound discretion;
4. If in the opinion of the court, probable cause exists, it shall issue the search warrant prayed
for;
5. The proper suit is then brought in the court under Article 201 of the RPC (Obscene
publications).
6. Any conviction is subject to appeal. The appellate court may assess whether or not the
properties seized are indeed obscene.

GUANZON VS. DE VILLA


[181 SCRA 623; G.R. 80508; 30 JAN 1990]

Facts:

The 41 petitioners alleged that the "saturation drive" or "aerial target zoning" that were conducted in
their place (Tondo Manila) were unconstitutional. They alleged that there is no specific target house
to be search and that there is no search warrant or warrant of arrest served. Most of the policemen
are in their civilian clothes and without nameplates or identification cards. The residents were rudely
rouse from their sleep by banging on the walls and windows of their houses. The residents were at
the point of high-powered guns and herded like cows. Men were ordered to strip down to their briefs
for the police to examine their tattoo marks. The residents complained that they're homes were
ransacked, tossing their belongings and destroying their valuables. Some of their money and
valuables had disappeared after the operation. The residents also reported incidents of maulings,
spot-beatings and maltreatment. Those who were detained also suffered mental and physical
torture to extract confessions and tactical informations. The respondents said that such accusations
Page 107
were all lies. Respondents contends that the Constitution grants to government the power to seek
and cripple subversive movements for the maintenance of peace in the state. The aerial target
zoning were intended to flush out subversives and criminal elements coddled by the communities
were the said drives were conducted. They said that they have intelligently and carefully planned
months ahead for the actual operation and that local and foreign media joined the operation to
witness and record such event.

Issue:

Whether or Not the saturation drive committed consisted of violation of human rights.

Held:

It is not the police action per se which should be prohibited rather it is the procedure used or the
methods which "offend even hardened sensibilities" .Based on the facts stated by the parties, it
appears to have been no impediment to securing search warrants or warrants of arrest before any
houses were searched or individuals roused from sleep were arrested. There is no showing that the
objectives sought to be attained by the "aerial zoning" could not be achieved even as th rights of the
squatters and low income families are fully protected. However, the remedy should not be brought
by a tazpaer suit where not one victim complaints and not one violator is properly charged. In the
circumstances of this taxpayers' suit, there is no erring soldier or policeman whom the court can
order prosecuted. In the absence of clear facts no permanent relief can be given.

In the meantime where there is showing that some abuses were committed, the court temporary
restraint the alleged violations which are shocking to the senses. Petition is remanded to the RTC of
Manila.

PEOPLE VS. ARUTA


[288 SCRA 626; G.R. NO. 120515; 13 APR 1998]

Facts:

On Dec. 13, 1988, P/Lt. Abello was tipped off by his informant that a certain “Aling Rosa” will be
arriving from Baguio City with a large volume of marijuana and assembled a team. The next day, at
the Victory Liner Bus terminal they waited for the bus coming from Baguio, when the informer
pointed out who “Aling Rosa” was, the team approached her and introduced themselves as
NARCOM agents. When Abello asked “aling Rosa” about the contents of her bag, the latter handed
it out to the police. They found dried marijuana leaves packed in a plastic bag marked “cash
katutak”.

Instead of presenting its evidence, the defense filed a demurrer to evidence alleging the illegality of
the search and seizure of the items. In her testimony, the accused claimed that she had just come
from Choice theatre where she watched a movie “Balweg”. While about to cross the road an old
woman asked her for help in carrying a shoulder bag, when she was later on arrested by the police.
She has no knowledge of the identity of the old woman and the woman was nowhere to be found.
Also, no search warrant was presented.

The trial court convicted the accused in violation of the dangerous drugs of 1972

Issue:

Whether or Not the police correctly searched and seized the drugs from the accused.

Held:

The following cases are specifically provided or allowed by law:


Page 108
1. Warrantless search incidental to a lawful arrest recognized under Section 12, Rule 126
of the Rules of Court 8 and by prevailing jurisprudence
2. Seizure of evidence in "plain view," the elements of which are: (a) a prior valid intrusion
based on the valid warrantless arrest in which the police are legally present in the pursuit
of their official duties; (b) the evidence was inadvertently discovered by the police who
had the right to be where they are; (c) the evidence must be immediately apparent, and
(d) "plain view" justified mere seizure of evidence without further search;
3. Search of a moving vehicle. Highly regulated by the government, the vehicle's inherent
mobility reduces expectation of privacy especially when its transit in public thoroughfares
furnishes a highly reasonable suspicion amounting to probable cause that the occupant
committed a criminal activity;
4. Consented warrantless search;
5. Customs search;
6. Stop and Frisk;
7. Exigent and Emergency Circumstances.

The essential requisite of probable cause must still be satisfied before a warrantless search and
seizure can be lawfully conducted.

The accused cannot be said to be committing a crime, she was merely crossing the street and was
not acting suspiciously for the Narcom agents to conclude that she was committing a crime. There
was no legal basis to effect a warrantless arrest of the accused’s bag, there was no probable cause
and the accused was not lawfully arrested.

The police had more than 24 hours to procure a search warrant and they did not do so. The seized
marijuana was illegal and inadmissible evidence.

RULE 113, RULES OF COURT

Section 5. Arrest without warrant; when lawful. — A peace officer or a private person may, without a
warrant, arrest a person:

(a) When, in his presence, the person to be arrested has committed, is actually committing,
or is attempting to commit an offense;
(b) When an offense has just been committed, and he has probable cause to believe based
on personal knowledge of facts or circumstances that the person to be arrested has
committed it; and
(c) When the person to be arrested is a prisoner who has escaped from a penal
establishment or place where he is serving final judgment or is temporarily confined
while his case is pending, or has escaped while being transferred from one confinement
to another.

In cases falling under paragraph (a) and (b) above, the person arrested without a warrant shall be
forthwith delivered to the nearest police station or jail and shall be proceeded against in accordance
with section 7 of Rule 112.

RULE 126, RULES OF COURT

Section 2. Court where application for search warrant shall be filed. — An application for search
warrant shall be filed with the following:

a) Any court within whose territorial jurisdiction a crime was committed.

Page 109
b) For compelling reasons stated in the application, any court within the judicial region where the
crime was committed if the place of the commission of the crime is known, or any court within
the judicial region where the warrant shall be enforced.

However, if the criminal action has already been filed, the application shall only be made in the court
where the criminal action is pending.

Section 7. Right to break door or window to effect search. — The officer, if refused admittance to
the place of directed search after giving notice of his purpose and authority, may break open any
outer or inner door or window of a house or any part of a house or anything therein to execute the
warrant or liberate himself or any person lawfully aiding him when unlawfully detained therein.

Section 12. Delivery of property and inventory thereof to court; return and proceedings thereon.

(a) The officer must forthwith deliver the property seized to the judge who issued the warrant,
together with a true inventory thereof duly verified under oath.
(b) Ten (10) days after issuance of the search warrant, the issuing judge shall ascertain if the
return has been made, and if none, shall summon the person to whom the warrant was
issued and require him to explain why no return was made. If the return has been made, the
judge shall ascertain whether section 11 of this Rule has been complained with and shall
require that the property seized be delivered to him. The judge shall see to it that subsection
(a) hereof has been complied with.
(c) The return on the search warrant shall be filed and kept by the custodian of the log book on
search warrants who shall enter therein the date of the return, the result, and other actions of
the judge.

A violation of this section shall constitute contempt of court.

Page 110
LIBERTY OF ABODE AND OF TRAVEL

Art 3, Sec. 6. “The liberty of abode and of changing the same within the limits prescribed by law
shall not be impaired except upon lawful order of the court. Neither shall the right to travel be
impaired except in the interest of national security, public safety, or public health, as may be
provided by law.”

CAUNCA VS. SALAZAR


[82 PHIL 851; NO.L-2690; 1 JAN 1949]

Facts:

This is an action for habeas corpus brought by Bartolome Caunca in behalf of his cousin Estelita
Flores who was employed by the Far Eastern Employment Bureau, owned by Julia Salazar,
respondent herein. An advanced payment has already been given to Estelita by the employment
agency, for her to work as a maid. However, Estelita wanted to transfer to another residence, which
was disallowed by the employment agency. Further she was detained and her liberty was
restrained. The employment agency wanted that the advance payment, which was applied to her
transportation expense from the province should be paid by Estelita before she could be allowed to
leave.

Issue:

Whether or Not an employment agency has the right to restrain and detain a maid without returning
the advance payment it gave?

Held:

An employment agency, regardless of the amount it may advance to a prospective employee or


maid, has absolutely no power to curtail her freedom of movement. The fact that no physical force
has been exerted to keep her in the house of the respondent does not make less real the
deprivation of her personal freedom of movement, freedom to transfer from one place to another,
freedom to choose one’s residence. Freedom may be lost due to external moral compulsion, to
founded or groundless fear, to erroneous belief in the existence of an imaginary power of an
impostor to cause harm if not blindly obeyed, to any other psychological element that may curtail the
mental faculty of choice or the unhampered exercise of the will. If the actual effect of such
psychological spell is to place a person at the mercy of another, the victim is entitled to the
protection of courts of justice as much as the individual who is illegally deprived of liberty by duress
or physical coercion.

MANOTOC VS. COURT OF APPEALS


[142 SCRA 149; G.R. NO. L-62100; 30 MAY 1986]

Facts:

Page 111
Petitioner was charged with estafa. He posted bail. Petitioner filed before each of the trial courts a
motion entitled, "motion for permission to leave the country," stating as ground therefor his desire to
go to the United States, "relative to his business transactions and opportunities." The prosecution
opposed said motion and after due hearing, both trial judges denied the same. Petitioner thus filed a
petition for certiorari and mandamus before the then Court of Appeals seeking to annul the orders
dated March 9 and 26, 1982, of Judges Camilon and Pronove, respectively, as well as the
communication-request of the Securities and Exchange Commission, denying his leave to travel
abroad. He likewise prayed for the issuance of the appropriate writ commanding the Immigration
Commissioner and the Chief of the Aviation Security Command (AVSECOM) to clear him for
departure. The Court of Appeals denied the petition.

Petitioner contends that having been admitted to bail as a matter of right, neither the courts which
granted him bail nor the Securities and Exchange Commission which has no jurisdiction over his
liberty could prevent him from exercising his constitutional right to travel.

Issue:

Whether or Not his constitutional right to travel has been violated.

Held:

A court has the power to prohibit a person admitted to bail from leaving the Philippines. This is a
necessary consequence of the nature and function of a bail bond. The condition imposed upon
petitioner to make himself available at all times whenever the court requires his presence operates
as a valid restriction on his right to travel. Indeed, if the accused were allowed to leave the
Philippines without sufficient reason, he may be placed beyond the reach of the courts. Petitioner
has not shown the necessity for his travel abroad. There is no indication that the business
transactions cannot be undertaken by any other person in his behalf.

MARCOS VS. MANGLAPUS


[177 SCRA 668; G.R. NO. 88211; 15 SEPT 1989]

Facts:

This case involves a petition of mandamus and prohibition asking the court to order the respondents
Secretary of Foreign Affairs, etc. To issue a travel documents to former Pres. Marcos and the
immediate members of his family and to enjoin the implementation of the President's decision to bar
their return to the Philippines. Petitioners assert that the right of the Marcoses to return in the
Philippines is guaranteed by the Bill of Rights, specifically Sections 1 and 6. They contended that
Pres. Aquino is without power to impair the liberty of abode of the Marcoses because only a court
may do so within the limits prescribed by law. Nor the President impair their right to travel because
no law has authorized her to do so.

They further assert that under international law, their right to return to the Philippines is guaranteed
particularly by the Universal Declaration of Human Rights and the International Covenant on Civil
and Political Rights, which has been ratified by the Philippines.

Issue:

Whether or not, in the exercise of the powers granted by the constitution, the President (Aquino)
may prohibit the Marcoses from returning to the Philippines.

Held:

"It must be emphasized that the individual right involved is not the right to travel from the Philippines
to other countries or within the Philippines. These are what the right to travel would normally
Page 112
connote. Essentially, the right involved in this case at bar is the right to return to one's country, a
distinct right under international law, independent from although related to the right to travel. Thus,
the Universal Declaration of Human Rights and the International Covenant on Civil and Political
Rights treat the right to freedom of movement and abode within the territory of a state, the right to
leave the country, and the right to enter one's country as separate and distinct rights. What the
Declaration speaks of is the "right to freedom of movement and residence within the borders of each
state". On the other hand, the Covenant guarantees the right to liberty of movement and freedom to
choose his residence and the right to be free to leave any country, including his own. Such rights
may only be restricted by laws protecting the national security, public order, public health or morals
or the separate rights of others. However, right to enter one's country cannot be arbitrarily deprived.
It would be therefore inappropriate to construe the limitations to the right to return to ones country in
the same context as those pertaining to the liberty of abode and the right to travel.

The Bill of rights treats only the liberty of abode and the right to travel, but it is a well considered
view that the right to return may be considered, as a generally accepted principle of International
Law and under our Constitution as part of the law of the land.

The court held that President did not act arbitrarily or with grave abuse of discretion in determining
that the return of the Former Pres. Marcos and his family poses a serious threat to national interest
and welfare. President Aquino has determined that the destabilization caused by the return of the
Marcoses would wipe away the gains achieved during the past few years after the Marcos regime.

The return of the Marcoses poses a serious threat and therefore prohibiting their return to the
Philippines, the instant petition is hereby DISMISSED.

SILVERIO VS. COURT OF APPEALS


[195 SCRA 760 ; G.R. 94284; 8 APR 1991]

Facts:

Petitioner was charged with violation of Section 2 (4) of the revised securities act. Respondent filed
to cancel the passport of the petitioner and to issue a hold departure order. The RTC ordered the
DFA to cancel petitioner’s passport, based on the finding that the petitioner has not been arraigned
and there was evidence to show that the accused has left the country with out the knowledge and
the permission of the court.

Issue:

Whether or Not the right to travel may be impaired by order of the court.

Held:

The bail bond posted by petitioner has been cancelled and warrant of arrest has been issued by
reason that he failed to appear at his arraignments. There is a valid restriction on the right to travel,
it is imposed that the accused must make himself available whenever the court requires his
presence. A person facing criminal charges may be restrained by the Court from leaving the country
or, if abroad, compelled to return (Constitutional Law, Cruz, Isagani A., 1987 Edition, p. 138). So it is
also that "An accused released on bail may be re-arrested without the necessity of a warrant if he
attempts to depart from the Philippines without prior permission of the Court where the case is
pending (ibid., Sec. 20 [2nd
par. ]).

Article III, Section 6 of the 1987 Constitution should be interpreted to mean that while the liberty of
travel may be impaired even without Court Order, the appropriate executive officers or
Page 113
administrative authorities are not armed with arbitrary discretion to impose limitations. They can
impose limits only on the basis of "national security, public safety, or public health" and "as may be
provided by law," a limitive phrase which did not appear in the 1973 text (The Constitution, Bernas,
Joaquin G.,S.J., Vol. I, First Edition, 1987, p. 263). Apparently, the phraseology in the 1987
Constitution was a reaction to the ban on international travel imposed under the previous regime
when there was a Travel Processing Center, which issued certificates of eligibility to travel upon
application of an interested party (See Salonga vs. Hermoso & Travel Processing Center, No.
53622, 25 April 1980, 97 SCRA 121).

Holding an accused in a criminal case within the reach of the Courts by preventing his departure
from the Philippines must be considered as a valid restriction on his right to travel so that he may be
dealt with in accordance with law. The offended party in any criminal proceeding is the People of the
Philippines. It is to their best interest that criminal prosecutions should run their course and proceed
to finality without undue delay, with an accused holding himself amenable at all times to Court
Orders and processes

DEFENSOR-SANTIAGO VS. VASQUEZ


[217 SCRA 633; G.R. NOS. 99289-90; 27 JAN 1993]

Facts:

An information was filed against petitioner with the Sandiganbayan for violation of the Anti Graft and
Corrupt Practices Act. The order of arrest was issued with bail for release fixed at Php. 15,000 so
she filed a motion for acceptance of cash bail bond. On the same day the Sandiganbayan issued a
resolution authorizing the petitioner to post cash bond which the later filed in the amount of Php.15,
000. Her arraignment was set, but petitioner asked for the cancellation of her bail bond and that she
be allowed provisional release on recognizance. The Sandiganbayan deferred it. The
Sandiganbayan issued a hold departure order against petitioner, by reason of the announcement
she made that she would be leaving for the U.S. to accept a fellowship a Harvard. In the instant
motion she submitted before the S.C. she argues that her right to travel is impaired.

Issue:

Whether or Not the petitioner’s right to travel is impaired.

Held:

The petitioner does not deny and as a matter of fact even made a public statement, that she he
every intension of leaving the country to pursue higher studies abroad. The court upholds the
course of action of the Sandiganbayan in taking judicial notice of such fact of petitioners pal to go
abroad and in thereafter issuing a sua sponte the hold departure order is but an exercise of
respondent court’s inherent power to preserve and to maintain effectiveness of its jurisdiction over
the case and the person of the accused.

Also, the petitioner assumed obligations, when she posted bail bond. She holds herself amenable at
all times to the orders and process of eth court. She may legally be prohibited from leaving the
country during the pendency of the case. (Manotoc v. C.A.)

MARCOS VS. SANDIGANBAYAN


[247 SCRA 127; G.R. NO. 115132-34; 9 AUG 1995]

Facts:

Page 114
This is a petition for certiorari to set aside as arbitrary and in grave abuse of discretion resolutions of
the Sandiganbayan's First Division denying petitioner's motion for leave to travel abroad for medical
treatment.

The former first lady Imelda Marcos was found guilty by the First Division of the Sandiganbayan of
violating 3 of the Anti Graft and Corrupt Practices Act. After conviction she filed a "Motion for Leave
to Travel Abroad" to seek diagnostic tests and treatment by practitioners of oriental medicine in
China allegedly because of "a serious and life threatening medical condition" requiring facilities not
available in the Philippines that was denied. Then she again filed an "Urgent Ex-Parte Motion for
Permission to Travel Abroad" to undergo diagnosis and treatment in China. This was supported by
several medical reports that were prepared by her doctor Roberto Anastacio.

Again another Motion to leave was filed by Mrs. Marcos to US and Europe for treatment of several
Heart diseases alleging that the tests were not available here.

The presiding justice, Garchitorena, contacted Dr. Gregorio B. Patacsil, Officer-in-Charge of the
Philippine Heart Center, and later wrote him a letter, asking for "expert opinion on coronary
medicine". The court still found no merit to allow the petitioners motion to leave and denied all of the
motions.

Petitioner filed a motion for reconsideration and a "Motion to Admit Clinical Summary and to
Resolve Motion for Reconsideration." Attached was a recent medical report and letters of Vice
President Joseph E. Estrada offering to be guarantor for the return of petitioner and those of twenty
four members of the House of Representatives requesting the court to allow petitioner to travel
abroad. This was also denied by the Court also stating their express disapproval of the involvement
of the VP and the Cabinet members so as to influence the resolutions, decisions or orders or any
judicial action of respondent court.

Issue:

Whether or Not the Sandiganbayan erred in disallowing the Motion for Leave to Travel Abroad
because it (1) disregarded the medical findings (2) it motu propio contacted a third party asking the
latter to give an opinion on petitioner's motion and medical findings (3) said that there was no
necessity to get medical treatment abroad.

Held:

No. The contention of the petitioner that was invalid to contact a third party asking the latter to give
an opinion on petitioner's motion and medical findings was erroneous. Respondent court had to
seek expert opinion because petitioner's motion was based on the advice of her physician. The
court could not be expected to just accept the opinion of petitioner's physician in resolving her
request for permission to travel. What would be objectionable would be if respondent court obtained
information without disclosing its source to the parties and used it in deciding a case against them.

In disregarding the medical reports, the petitioner failed to prove the necessity for a trip abroad. It
should be emphasized that considering the fact that she is facing charges before the courts in
several cases, in two of which she was convicted although the decision is still pending
reconsideration, petitioner did not have an absolute right to leave the country and the burden was
on her to prove that because of danger to health if not to her life there was necessity to seek
medical treatment in foreign countries.

On the third issue, the Court ordered petitioner to undergo several tests which summarily states that
the required medical treatment was available here in the Philippines and that the expertise and
facilities here were more than adequate to cater to her medical treatment. The heart ailments of the
petitioner were not as severe as that was reported by Dr. Anastacio.

Page 115
Wherefore, the petitioner is Dismissed without prejudice to the filling of another motion for leave to
travel abroad, should petitioner still desire, based on her heart condition. In such an event the
determination of her medical condition should be made by joint panel of medical specialists
recommended by both the accused and the prosecution.

RUBI VS. PROVINCIAL BOARD OF MINDORO


[39 PHIL 660; NO. 14078; 7 MAR 1919]

Facts:

The provincial board of Mindoro adopted resolution No. 25 wherein non-Christian inhabitants
(uncivilized tribes) will be directed to take up their habitation on sites on unoccupied public lands. It
is resolved that under section 2077 of the Administrative Code, 800 hectares of public land in the
sitio of Tigbao on Naujan Lake be selected as a site for the permanent settlement of Mangyanes in
Mindoro. Further, Mangyans may only solicit homesteads on this reservation providing that said
homestead applications are previously recommended by the provincial governor.

In that case, pursuant to Section 2145 of the Revised Administrative Code, all the Mangyans in the
townships of Naujan and Pola and the Mangyans east of the Baco River including those in the
districts of Dulangan and Rubi's place in Calapan, were ordered to take up their habitation on the
site of Tigbao, Naujan Lake. Also, that any Mangyan who shall refuse to comply with this order shall
upon conviction be imprisoned not exceed in sixty days, in accordance with section 2759 of the
revised Administrative Code.

Said resolution of the provincial board of Mindoro were claimed as necessary measures for the
protection of the Mangyanes of Mindoro as well as the protection of public forests in which they
roam, and to introduce civilized customs among them.

It appeared that Rubi and those living in his rancheria have not fixed their dwelling within the
reservation of Tigbao and are liable to be punished.

It is alleged that the Manguianes are being illegally deprived of their liberty by the provincial officials
of that province. Rubi and his companions are said to be held on the reservation established at
Tigbao, Mindoro, against their will, and one Dabalos is said to be held under the custody of the
provincial sheriff in the prison at Calapan for having run away form the reservation.

Issue:

Whether or Not Section 2145 of the Administrative Code deprive a person of his liberty pf abode.
Thus, WON Section 2145 of the Administrative Code of 1917 is constitutional.

Held:

The Court held that section 2145 of the Administrative Code does not deprive a person of his liberty
of abode and does not deny to him the equal protection of the laws, and that confinement in
reservations in accordance with said section does not constitute slavery and involuntary servitude.
The Court is further of the opinion that section 2145 of the Administrative Code is a legitimate
exertion of the police power. Section 2145 of the Administrative Code of 1917 is constitutional.
Assigned as reasons for the action: (1) attempts for the advancement of the non-Christian people of
the province; and (2) the only successfully method for educating the Manguianes was to oblige
them to live in a permanent settlement. The Solicitor-General adds the following; (3) The protection
of the Manguianes; (4) the protection of the public forests in which they roam; (5) the necessity of
introducing civilized customs among the Manguianes.

Page 116
One cannot hold that the liberty of the citizen is unduly interfered without when the degree of
civilization of the Manguianes is considered. They are restrained for their own good and the general
good of the Philippines.

“Liberty regulated by law": Implied in the term is restraint by law for the good of the individual and for
the greater good of the peace and order of society and the general well-being. No man can do
exactly as he pleases.

None of the rights of the citizen can be taken away except by due process of law.

Therefore, petitioners are not unlawfully imprisoned or restrained of their liberty. Habeas corpus
can, therefore, not issue.

Page 117
FREEDOM OF RELIGION

Art 3, Sec. 5. “No law shall be made respecting an establishment of religion, or prohibiting the free
exercise thereof. The free exercise and enjoyment of religious profession and worship, without
discrimination or preference, shall forever be allowed. No religious test shall be required for the
exercise of civil or political rights.”

AGLIPAY VS. RUIZ


[64 PHIL 201; G.R. NO. 45459; 13 MAR 1937]

Facts:

Petitioner seeks the issuance of a writ of prohibition against respondent Director of Posts from
issuing and selling postage stamps commemorative of the 33 rd International Eucharistic Congress.
Petitioner contends that such act is a violation of the Constitutional provision stating that no public
funds shall be appropriated or used in the benefit of any church, system of religion, etc. This
provision is a result of the principle of the separation of church and state, for the purpose of avoiding
the occasion wherein the state will use the church, or vice versa, as a weapon to further their ends
and aims. Respondent contends that such issuance is in accordance to Act No. 4052, providing for
the appropriation funds to respondent for the production and issuance of postage stamps as would
be advantageous to the government.

Issue:

Whether or Not there was a violation of the freedom to religion.

Held:

What is guaranteed by our Constitution is religious freedom and not mere religious toleration. It is
however not an inhibition of profound reverence for religion and is not a denial of its influence in
human affairs. Religion as a profession of faith to an active power that binds and elevates man to
his Creator is recognized. And in so far as it instills into the minds the purest principles of morality,
its influence is deeply felt and highly appreciated. The phrase in Act No. 4052 “advantageous to the
government” does not authorize violation of the Constitution. The issuance of the stamps was not
inspired by any feeling to favor a particular church or religious denomination. They were not sold for
the benefit of the Roman Catholic Church. The postage stamps, instead of showing a Catholic
chalice as originally planned, contains a map of the Philippines and the location of Manila, with the
words “Seat XXXIII International Eucharistic Congress.” The focus of the stamps was not the
Eucharistic Congress but the city of Manila, being the seat of that congress. This was to “to
advertise the Philippines and attract more tourists,” the officials merely took advantage of an event
considered of international importance. Although such issuance and sale may be inseparably linked
with the Roman Catholic Church, any benefit and propaganda incidentally resulting from it was no
the aim or purpose of the Government.

GARCES VS. ESTENZO


[104 SCRA 510; G.R. L-53487; 25 MAY 1981]

Facts:

Two resolutions of the Barangay Council of Valencia, Ormoc City were passed:

a. Resolution No. 5- Reviving the traditional socio-religious celebration every fifth of April.
This provided for the acquisition of the image of San Vicente Ferrer and the construction

Page 118
of a waiting shed. Funds for the said projects will be obtained through the selling of
tickets and cash donations.
b. Resolution No. 6- The chairman or hermano mayor of the fiesta would be the caretaker
of the image of San Vicente Ferrer and that the image would remain in his residence for
one year and until the election of his successor. The image would be made available to
the Catholic Church during the celebration of the saint’s feast day.

These resolutions have been ratified by 272 voters, and said projects were implemented. The image
was temporarily placed in the altar of the Catholic Church of the barangay. However, after a mass,
Father Sergio Marilao Osmeña refused to return the image to the barangay council, as it was the
church’s property since church funds were used in its acquisition.

Resolution No. 10 was passed for the authorization of hiring a lawyer for the replevin case against
the priest for the recovery of the image. Resolution No. 12 appointed Brgy. Captain Veloso as a
representative to the case. The priest, in his answer assailed the constitutionality of the said
resolutions. The priest with Andres Garces, a member of the Aglipayan Church, contends that Sec.
8 Article IV1 and Sec 18(2) Article VIII) 2 of the constitution was violated.

Issue:

Whether or Not any freedom of religion clause in the Constitution violated.

Held:

No. As said by the Court this case is a petty quarrel over the custody of the image. The image was
purchased in connection with the celebration of the barrio fiesta and not for the purpose of favoring
any religion nor interfering with religious matters or beliefs of the barrio residents. Any activity
intended to facilitate the worship of the patron saint(such as the acquisition) is not illegal. Practically,
the image was placed in a layman’s custody so that it could easily be made available to any family
desiring to borrow the image in connection with prayers and novena. It was the council’s funds that
were used to buy the image, therefore it is their property. Right of the determination of custody is
their right, and even if they decided to give it to the Church, there is no violation of the Constitution,
since private funds were used. Not every government activity which involves the expenditure of
public funds and which has some religious tint is violative of the constitutional provisions regarding
separation of church and state, freedom of worship and banning the use of public money or
property.

AMERICAN BIBLE SOCIETY VS. CITY OF MANILA


[101PHIL 386; G.R. NO. 9637; 30 APR 1957]

Facts:

New York’s Education Law requires local public school authorities to lend textbooks free of charge
to all students in grade 7 to 12, including those in private schools. The Board of Education
contended that said statute was invalid and violative of the State and Federal Constitutions. An
order barring the Commissioner of Education (Allen) from removing appellant’s members from office
for failure to comply with the requirement and an order preventing the use of state funds for the
purchase of textbooks to be lent to parochial schools were sought for. The trial court held the statute
unconstitutional. The Appellate Division reversed the decision and dismissed the complaint since
the appellant have no standing. The New York Court of Appeals, ruled that the appellants have
standing but the law is not unconstitutional.

Issue:

Whether or Not the said ordinances are constitutional and valid (contention: it restrains the free
exercise and enjoyment of the religious profession and worship of appellant).

Page 119
Held:

Section 1, subsection (7) of Article III of the Constitution, provides that:

(7) No law shall be made respecting an establishment of religion, or prohibiting the free
exercise thereof, and the free exercise and enjoyment of religious profession and worship,
without discrimination or preference, shall forever be allowed. No religion test shall be required
for the exercise of civil or political rights.

The provision aforequoted is a constitutional guaranty of the free exercise and enjoyment of
religious profession and worship, which carries with it the right to disseminate religious information.

It may be true that in the case at bar the price asked for the bibles and other religious pamphlets
was in some instances a little bit higher than the actual cost of the same but this cannot mean that
appellant was engaged in the business or occupation of selling said "merchandise" for profit. For
this reason. The Court believe that the provisions of City of Manila Ordinance No. 2529, as
amended, cannot be applied to appellant, for in doing so it would impair its free exercise and
enjoyment of its religious profession and worship as well as its rights of dissemination of religious
beliefs.

With respect to Ordinance No. 3000, as amended, the Court do not find that it imposes any charge
upon the enjoyment of a right granted by the Constitution, nor tax the exercise of religious practices.

It seems clear, therefore, that Ordinance No. 3000 cannot be considered unconstitutional, however
inapplicable to said business, trade or occupation of the plaintiff. As to Ordinance No. 2529 of the
City of Manila, as amended, is also not applicable, so defendant is powerless to license or tax the
business of plaintiff Society.
WHEREFORE, defendant shall return to plaintiff the sum of P5,891.45 unduly collected from it.

GERMAN VS. BARANGAN


[135 SCRA 514; G.R. NO. 68828; 27 MAR 1985]

Facts:

Petitioners converged at J.P. Laurel Street to hear Mass at the St. Jude Chapel, which adjoined
Malacañang. Respondent barred them for security reasons. Petitioners filed a petition for
mandamus.

Issue:

Whether or Not there was a violation of the constitutional freedom.

Held:

Petitioners' intention was not really to perform an act of religious worship but to conduct an anti-
government demonstration since they wore yellow T-shirts, raised their clenched fists and shouted
anti- government slogans. While every citizen has the right to religious freedom, the exercise must
be done in good faith. Besides, the restriction was reasonable as it was designed to protect the lives
of the President and his family, government officials and diplomatic and foreign guests transacting
business with Malacanang. The restriction was also intended to secure the executive offices within
the Malacanang grounds from possible external attacks and disturbances. (Minority opinion) The
sole justification for a prior restraint or limitation on the exercise of the freedom of religion is the
existence of a grave and imminent, of a serious evil to public safety, public morals, public health or
any other legitimate public interest that the State has a right to prevent. The burden to show the
existence of grave and imminent danger lies on the officials who would restrain petitioners.
Respondents were in full control and had the capability to stop any untoward move. There was no
Page 120
clear and present danger of any serious evil to public safety or the security of Malacanang.

EBRALINAG VS. DIVISION SUPERINTENDENT OF CEBU


[219 SCRA 256 ; G.R. NO. 95770; 1 MAR 1993]

Facts:

Two special civil actions for certiorari, Mandamus and Prohibition were filed and consolidated for
raising same issue. Petitioners allege that the public respondents acted without or in excess of their
jurisdiction and with grave abuse of discretion. Respondents ordered expulsion of 68 HS and GS
students of Bantayan, Pinamungajan, Caracar, Taburan and Asturias in Cebu. Public school
authorities expelled these students for refusing to salute the flag, sing the national anthem and
recite the “Panatang Makabayan” required by RA1265. They are Jehovah’s Witnesses believing
that by doing these is religious worship/devotion akin to idolatry against their teachings. They
contend that to compel transcends constitutional limits and invades protection against official control
and religious freedom. The respondents relied on the precedence of Gerona et al v. Secretary of
Education. Gerona doctrine provides that we are a system of separation of the church and state
and the flag is devoid of religious significance and it doesn’t involve any religious ceremony. The
freedom of religious belief guaranteed by the Constitution does not mean exception from non-
discriminatory laws like the saluting of flag and singing national anthem. This exemption disrupts
school discipline and demoralizes the teachings of civic consciousness and duties of citizenship.

Issue:

Whether or Not religious freedom has been violated.

Held:

Religious freedom is a fundamental right of highest priority. The 2 fold aspect of right to religious
worship is: 1.) Freedom to believe which is an absolute act within the realm of thought. 2.) Freedom
to act on one’s belief regulated and translated to external acts. The only limitation to religious
freedom is the existence of grave and present danger to public safety, morals, health and interests
where State has right to prevent. The expulsion of the petitioners from the school is not justified.

The 30 yr old previous GERONA decision of expelling and dismissing students and teachers who
refuse to obey RA1265 is violates exercise of freedom of speech and religious profession and
worship. Jehovah’s Witnesses may be exempted from observing the flag ceremony but this right
does not give them the right to disrupt such ceremonies. In the case at bar, the Students expelled
were only standing quietly during ceremonies. By observing the ceremonies quietly, it doesn’t
present any danger so evil and imminent to justify their expulsion. What the petitioner’s request is
exemption from flag ceremonies and not exclusion from public schools. The expulsion of the
students by reason of their religious beliefs is also a violation of a citizen’s right to free education.
The non-observance of the flag ceremony does not totally constitute ignorance of patriotism and
civic consciousness. Love for country and admiration for national heroes, civic consciousness and
form of government are part of the school curricula. Therefore, expulsion due to religious beliefs is
unjustified.

Petition for Certiorari and Prohibition is GRANTED. Expulsion is ANNULLED.

FONACIER VS. COURT OF APPEALS


[96 PHIL 417; G.R. L-5917; 28 JAN 1955]

Facts:

Page 121
Case was filed by Iglesia Filipina Independiente (IFI), represented by its supreme bishop Gerardo
Bayaca, against Bishop Fonacier seeking to render an accounting of his administration of all the
temporal properties and to recover the same on the ground that he ceased to be the supreme
bishop of IFI. Isabelo De los Reyes Jr. had been elected as the Supreme Bishop.

Petitioner claims that he was not properly removed as Supreme Bishop and his legal successor was
Juan Jamias. He claims that the there was an accounting of his administration and was turned over
to bishop Jamias. Also, that Isabelo De los Reyes and Bayaca have abandoned their faith and
formally joined the Prostestant Episcopal Church of America.

CFI rendered judgment declaring Isabelo De Los Reyes, Jr. as the sole and legitimate Supreme
Bishop of IFI and ordered Fonacier to render an accounting of his admistration
CA affirmed the decision of the CFI

Issue:

Whether or not the petitioner should still be regarded as the legitimate supreme bishop of IFI.

Held:

Supreme Court affirmed CA’s decision. The legitimate Supreme Bishop of IFI is Isabelo De los
Reyes, Jr. The Supreme Court affirms the validity of the election of Bishop Delos Reyes as the
Supreme Bishop based on their internal laws

To finally dispose of the property issue, the Court, citing Watson v. Jones,368 declared that the rule
in property controversies within religious congregations strictly independent of any other superior
ecclesiastical association (such as the Philippine Independent Church) is that the rules for resolving
such controversies should be those of any voluntary association. If the congregation adopts the
majority rule then the majority should prevail; if it adopts adherence to duly constituted authorities
within the congregation, then that should be followed.

PAMIL VS. TELECOM


[86 SCRA 413; G.R. 34854; 20 NOV 1978]

Facts:

Fr. Margarito Gonzaga was elected as Municipal Mayor in Alburquerque, Bohol. Petitioner, also an
aspirant for said office, then filed a suit for quo warranto for Gonzaga’s disqualification based on the
Administrative Code provision: “In no case shall there be elected or appointed to a municipal office
ecclesiastics, soldiers in active service, persons receiving salaries or compensation from provincial
or national funds, or contractors for public works of the municipality." The respondent Judge, in
sustaiing Fr. Gonzaga’s right to the office, ruled that the provision had already been impliedly
repealed by the Election Code of 1971. Petitioner on the other hand argues that there was no
implied repeal.

Issue:

Whether or Not Fr. Gonzaga is eligible for the position of municipal mayor, according to law.

Whether or Not the prohibition regarding elected or appointed ecclesiastics is constitutional.

Held:

The court was divided. Five voted that the prohibition was not unconstitutional. Seven others voted
that the provision was impliedly repealed. However, the minority vote overruled the seven.
Page 122
According to the dissenting seven, there are three reasons for the said provision to be inoperative.
First, the 1935 Constitution stated, “No religious test shall be required for the exercise of civil or
political rights.” Second, said section 2175 is superseded by the Constitution. Third, section 2175
has been repealed by Sec. 23 of the Election Code (1971): “Appointive public office holders and
active members of the Armed Forces are no longer disqualified from running for an elective office”.
Ecclesiastics were no longer included in the enumeration of persons ineligible under the said
Election Code. On the other hand, the controlling five argued: Section 2175 of the Administrative
Code deals with a matter different from that of section 23 of the Election Code. Also, section 2175 of
the Administrative Code did not violate the right to freedom of religion because it did not give any
requirement for a religious test.

The view of the dissenting seven failed to obtain a vote of eight members, so it was not controlling.
The provision of the Administrative Code remained operative.

ESTRADA VS. ESCRITOR


[492 SCRA 1 ; AM NO P-02-1651; 22 JUN 2006]

Facts:

Escritor is a court interpreter since 1999 in the RTC of Las Pinas City. She has been living with
Quilapio, a man who is not her husband, for more than twenty five years and had a son with him as
well. Respondent’s husband died a year before she entered into the judiciary while Quilapio is still
legally married to another woman.

Complainant Estrada requested the Judge of said RTC to investigate respondent. According to
complainant, respondent should not be allowed to remain employed therein for it will appear as if
the court allows such act.

Respondent claims that their conjugal arrangement is permitted by her religion—the Jehovah’s
Witnesses and the Watch Tower and the Bible Trace Society. They allegedly have a ‘Declaration of
Pledging Faithfulness’ under the approval of their congregation. Such a declaration is effective
when legal impediments render it impossible for a couple to legalize their union.

Issue:

Whether or Not the State could penalize respondent for such conjugal arrangement.

Held:

No. The State could not penalize respondent for she is exercising her right to freedom of religion.
The free exercise of religion is specifically articulated as one of the fundamental rights in our
Constitution. As Jefferson put it, it is the most inalienable and sacred of human rights. The State’s
interest in enforcing its prohibition cannot be merely abstract or symbolic in order to be sufficiently
compelling to outweigh a free exercise claim. In the case at bar, the State has not evinced any
concrete interest in enforcing the concubinage or bigamy charges against respondent or her
partner. Thus the State’s interest only amounts to the symbolic preservation of an unenforced
prohibition.

Furthermore, a distinction between public and secular morality and religious morality should be kept
in mind. The jurisdiction of the Court extends only to public and secular morality.

The Court further states that our Constitution adheres the benevolent neutrality approach that gives
room for accommodation of religious exercises as required by the Free Exercise Clause. This
benevolent neutrality could allow for accommodation of morality based on religion, provided it does
Page 123
not offend compelling state interests. Assuming arguendo that the OSG has proved a compelling
state interest, it has to further demonstrate that the state has used the least intrusive means
possible so that the free exercise is not infringed any more than necessary to achieve the legitimate
goal of the state. Thus the conjugal arrangement cannot be penalized for it constitutes an
exemption to the law based on her right to freedom of religion.

ISLAMIC DA'WAH COUNCIL OF THE PHILIPPINES VS. EXECUTIVE SECRETARY


[405 SCRA 497;GR 153888; 9 JUL 2003]

Facts:

Petitioner Islamic Da'wah Council of the Philippines, Inc. (IDCP) is a corporation that operates under
Department of Social Welfare and Development, a non-governmental organization that extends
voluntary services to the Filipino people, especially to Muslim communities. It claims to be a
federation of national Islamic organizations and an active member of international organizations
such as the Regional Islamic Da'wah Council of Southeast Asia and the Pacific (RISEAP) and The
World Assembly of Muslim Youth. The RISEAP accredited petitioner to issue halal certifications in
the Philippines. Thus, among the functions petitioner carries out is to conduct seminars, orient
manufacturers on halal food and issue halal certifications to qualified products and manufacturers.

Petitioner alleges that, the actual need to certify food products as halal and also due to halal food
producers' request, petitioner formulated in 1995 internal rules and procedures based on the Qur'an
and the Sunnah for the analysis of food, inspection thereof and issuance of halal certifications. In
that same year, petitioner began to issue, for a fee, certifications to qualified products and food
manufacturers. Petitioner even adopted for use on its halal certificates a distinct sign or logo
registered in the Philippine Patent Office.

On 2001, respondent Office of the Executive Secretary issued EO 465 creating the Philippine Halal
Certification Scheme and designating respondent OMA to oversee its implementation. Under the
EO, respondent OMA has the exclusive authority to issue halal certificates and perform other
related regulatory activities.

Issue:

Whether or Not EO violates the constitutional provision on the separation of Church and State.

Held:

It is unconstitutional for the government to formulate policies and guidelines on the halal certification
scheme because said scheme is a function only religious organizations, entity or scholars can
lawfully and validly perform for the Muslims. According to petitioner, a food product becomes halal
only after the performance of Islamic religious ritual and prayer. Thus, only practicing Muslims are
qualified to slaughter animals for food. A government agency like herein respondent OMA cannot
therefore perform a religious function like certifying qualified food products as halal. Without doubt,
classifying a food product as halal is a religious function because the standards used are drawn
from the Qur'an and Islamic beliefs. By giving OMA the exclusive power to classify food products as
halal, EO 46 encroached on the religious freedom of Muslim organizations like herein petitioner to
interpret for Filipino Muslims what food products are fit for Muslim consumption. Also, by arrogating
to itself the task of issuing halal certifications, the State has in effect forced Muslims to accept its
own interpretation of the Qur'an and Sunnah on halal food.

In the case at bar, we find no compelling justification for the government to deprive Muslim
organizations, like herein petitioner, of their religious right to classify a product as halal, even on the
premise that the health of Muslim Filipinos can be effectively protected by assigning to OMA the
exclusive power to issue halal certifications. The protection and promotion of the muslim Filipinos'
right to health are already provided for in existing laws and ministered to by government agencies
Page 124
charged with ensuring that food products released in the market are fit for human consumption,
properly labeled and safe. Unlike EO 46, these laws do not encroach on the religious freedom of
Muslims.

Page 125
FREEDOM OF EXPRESSION

Art 3, Sec. 4. “No law shall be passed abridging the freedom of speech, of expression, or of the
press, or the right of the people peaceably to assemble and petition the government for redress of
grievances.”

Art 3, Sec. 7. “The right of the people to information on matters of public concern shall be
recognized. Access to official records, and to documents, and papers pertaining to official acts,
transactions, or decisions, as well as to government research data used as basis for policy
development, shall be afforded the citizen, subject to such limitations as may be provided by law.”

Art 3, Sec. 8. “The right of the people, including those employed in the public and private sectors,
to form unions, associations, or societies for purposes not contrary to law shall not be abridged.”

Art 3, Sec. 18. “(1) No person shall be detained solely by reason of his political beliefs and
aspirations.”

NEAR VS. MINNESOTA


[283 US 697]

Facts:

A complaint alleged that the defendants, on September 24, 1927, and on eight subsequent dates in
October and November, 1927, published and circulated editions of “The Saturday Press”(published
in Minneapolis) which were 'largely devoted to malicious, scandalous and defamatory
articles'(based on Session Laws of Minnesota). The articles charged, in substance, provides that a
Jewish gangster was in control of gambling, bootlegging, and racketeering in Minneapolis, and that
law enforcing officers and agencies were not energetically performing their duties. Most of the
charges were directed against the chief of police; he was charged with gross neglect of duty, illicit
relations with gangsters, and with participation in graft. The county attorney was charged with
knowing the existing conditions and with failure to take adequate measures to remedy them. The
mayor was accused of inefficiency and dereliction. One member of the grand jury was stated to be
in sympathy with the gangsters. A special grand jury and a special prosecutor were demanded to
deal with the situation in general, and, in particular, to investigate an attempt to assassinate one
Guilford, one of the original defendants, who, it appears from the articles, was shot by gangsters
after the first issue of the periodical had been published. Now defendants challenged the Minnesota
statute which provides for the abatement, as a public nuisance, of a malicious, scandalous and
defamatory news paper, magazine or other periodical. The District Court ruled against defendants.
Hence the appeal.

Issue:

Whether or Not the proceeding authorized by the statute herein constitutes an infringement of the
freedom of the press.

Held:

Yes. The insistence that the statute is designed to prevent the circulation of scandal which tends to
disturb the public peace and to provoke assaults and the commission of crime is unavailing.

The reason for the enactment, as the state court has said, is that prosecutions to enforce penal
statutes for libel do not result in 'efficient repression or suppression of the evils of scandal.' In the
present instance, the proof was that nine editions of the newspaper or periodical in question were
published on successive dates, and that they were chiefly devoted to charges against public officers
and in relation to the prevalence and protection of crime. In such a case, these officers are not left
Page 126
to their ordinary remedy in a suit for libel, or the authorities to a prosecution for criminal libel. The
statute not only operates to suppress the offending newspaper or periodical, but to put the publisher
under an effective censorship.

Every freeman has an undoubted right to lay what sentiments he pleases before the public; to forbid
this, is to destroy the freedom of the press; but if he publishes what is improper, mischievous or
illegal, he must take the consequence of his own temerity.

The liberty of the press was to be unrestrained, but he who used it was to be responsible in case of
its abuse.' Public officers, whose character and conduct remain open to debate and free discussion
in the press, find their remedies for false accusations in actions under libel laws providing for
redress and punishment, and not in proceedings to restrain the publication of newspapers and
periodicals.

Characterizing the publication as a business, and the business as a nuisance, does not permit an
invasion of the constitutional immunity against restraint. Nor can it be said that the constitutional
freedom from previous restraint is lost because charges are made of derelictions which constitute
crimes.

The preliminary freedom, by virtue of the very reason for its existence, does not depend, as this
court has said, on proof of truth.

GROSJEAN VS. AMERICAN PRESS CO.


[297 US 233]

Facts:

The nine publishers(corporations) who brought the suit publish thirteen newspapers and these
thirteen publications are the only ones within the state of Louisiana having each a circulation of
more than 20,000 copies per week. The suit assailed Act No. 23 1 of the Louisiana Legislature, as
their freedom of the press was abridged in contravention to the due process clause.

Issue:

Whether or Not Act 23 unconstitutional.

Held:

Yes. Freedom of speech and of the press are rights of the same fundamental character,
safeguarded by the due process of law clause. The word 'liberty' contained in that amendment
embraces not only the right of a person to be free from physical restraint, but the right to be free in
the enjoyment of all his faculties as well.
The Act operates as a restraint in a double sense. First, its effect is to curtail the amount of revenue
realized from advertising; and, second, its direct tendency is to restrict circulation. This is plain
enough when we consider that, if it were increased to a high degree, as it could be if valid it well
might result in destroying both advertising and circulation.

Judge Cooley has laid down the test to be applied: The evils to be prevented were not the
censorship of the press merely, but any action of the government by means of which it might
prevent such free and general discussion of public matters as seems absolutely essential to prepare
the people for an intelligent exercise of their rights as citizens.

The tax here involved is bad not because it takes money from the pockets of the appellees. It is bad
because, it is seen to be a deliberate and calculated device in the guise of a tax to limit the
circulation of information to which the public is entitled in virtue of the constitutional guaranties. A
free press stands as one of the great interpreters between the government and the people.
Page 127
The form in which the tax is imposed is in itself suspicious. It is not measured or limited by the
volume of advertisements. It is measured alone by the extent of the circulation of the publication in
which the advertisements are carried, with the plain purpose of penalizing the publishers and
curtailing the circulation of a selected group of newspapers.

NEW YORK TIMES VS. UNITED STATES


[403 US 713]

Facts:

The court granted certiorari in the cases in which the United States seeks to enjoin the New York
Times and the Washington Post from publishing the contents of a classified study entitled "History
of U.S. Decision-Making Process on Viet Nam Policy." Said articles reveal the workings of
government that led to the Vietnam war. The Government argues that "the authority of the
Executive Department to protect the nation against publication of information whose disclosure
would endanger the national security stems from two interrelated sources: the constitutional power
of the President over the conduct of foreign affairs and his authority as Commander-in-Chief. In
such case the Executive Branch seeks judicial aid in preventing publication. The court ruled in favor
of the newspaper companies hence the appeal.

Issue:

Whether or not the freedom of the press was abridged.

Held:

Yes. To find that the President has "inherent power" to halt the publication of news by resort to the
courts would wipe out the First Amendment (Bill of Rights) and destroy the fundamental liberty and
security of the very people the Government hopes to make "secure."

No branch of government could abridge the people's rights granted by the Constitution including the
freedom of the press. The language of the First Amendment support the view that the press must be
left free to publish news, whatever the source, without censorship, injunctions, or prior restraints.
The press was protected so that it could bare the secrets of government and inform the people.
Only a free and unrestrained press can effectively expose deception in government. And paramount
among the responsibilities of a free press is the duty to prevent any part of the government from
deceiving the people and sending them off to distant lands to die of foreign fevers and foreign shot
and shell.

GONZALES VS. COMELEC


[27 SCRA 835; G.R. L-27833; 18 APR 1969]

Facts:

RA 4880 which took effect on June 17, 1967, prohibiting the too early nomination of candidates and
limiting the period of election campaign or partisan political activity was challenged on constitutional
grounds. More precisely, the basic liberties of free speech and free press, freedom of assembly and
freedom of association are invoked to nullify the act. Petitioner Cabigao was, at the time of the filing
the petition, an incumbent councilor in the 4th District of Manila and the Nacionalista Party official
candidate for Vice-Mayor of Manila to which he was subsequently elected on November 11, 1967;
petitioner Gonzales, on the other hand, is a private individual, a registered voter in the City of Manila
and a political leader of his co-petitioner. There was the further allegation that the nomination of a
candidate and the fixing of period of election campaign are matters of political expediency and
Page 128
convenience which only political parties can regulate or curtail by and among themselves through
self-restraint or mutual understanding or agreement and that the regulation and limitation of these
political matters invoking the police power, in the absence of clear and present danger to the state,
would render the constitutional rights of petitioners meaningless and without effect. Senator Lorenzo
M. Tañada was asked to appear as amicus curiae, and elucidated that Act No. 4880 could indeed
be looked upon as a limitation on the preferred rights of speech and press, of assembly and of
association. He did justify its enactment however under the clear and present danger doctrine, there
being the substantive evil of elections, whether for national or local officials, being debased and
degraded by unrestricted campaigning, excess of partisanship and undue concentration in politics
with the loss not only of efficiency in government but of lives as well. The Philippine Bar Association,
the Civil Liberties Union, the U.P. Law Center and the U.P. Women Lawyers' Circle were requested
to give their opinions. Respondents contend that the act was based on the police power of the state.

Issue:

Whether or Not RA 4880 unconstitutional.

Held:

Yes. As held in Cabansag v. Fernandez there are two tests that may supply an acceptable criterion
for permissible restriction on freedom of speech. These are the “clear and present danger” rule and
the 'dangerous tendency' rule. The first, means that the evil consequence of the comment or
utterance must be extremely serious and the degree of imminence extremely high before the
utterance can be punished. The danger to be guarded against is the 'substantive evil' sought to be
prevented. It has the advantage of establishing according to the above decision a definite rule in
constitutional law. It provides the criterion as to what words may be publicly established. The
"dangerous tendency rule" is such that “If the words uttered create a dangerous tendency which the
state has a right to prevent, then such words are punishable.” It is not necessary that some definite
or immediate acts of force, violence, or unlawfulness be advocated. It is sufficient that such acts be
advocated in general terms. Nor is it necessary that the language used be reasonably calculated to
incite persons to acts of force, violence, or unlawfulness. It is sufficient if the natural tendency and
probable effect of the utterance be to bring about the substantive evil which the legislative body
seeks to prevent.

The challenged statute could have been more narrowly drawn and the practices prohibited more
precisely delineated to satisfy the constitutional requirements as to a valid limitation under the clear
and present danger doctrine. As the author Tañada clearly explained, such provisions were deemed
by the legislative body to be part and parcel of the necessary and appropriate response not merely
to a clear and present danger but to the actual existence of a grave and substantive evil of
excessive partisanship, dishonesty and corruption as well as violence that of late has invariably
marred election campaigns and partisan political activities in this country.

The very idea of a government, republican in form, implies a right on the part of its citizens to meet
peaceably for consultation in respect to public affairs and to petition for redress of grievances. As in
the case of freedom of expression, this right is not to be limited, much less denied, except on a
showing of a clear and present danger of a substantive evil that Congress has a right to prevent.

The prohibition of any speeches, announcements or commentaries, or the holding of interviews for
or against the election of any party or candidate for public office and the prohibition of the
publication or distribution of campaign literature or materials, against the solicitation of votes
whether directly or indirectly, or the undertaking of any campaign literature or propaganda for or
against any candidate or party is repugnant to a constitutional command.

Page 129
IGLESIA NI CRISTO VS. COURT OF APPEALS
[259 SCRA 529; G.R. NO. 119673; 26 JUL 1996]

Facts:

Petitioner has a television program entitled "Ang Iglesia ni Cristo" aired on Channel 2 every
Saturday and on Channel 13 every Sunday. The program presents and propagates petitioner's
religious beliefs, doctrines and practices often times in comparative studies with other religions.
Petitioner submitted to the respondent Board of Review for Moving Pictures and Television the VTR
tapes of its TV program Series Nos. 116, 119, 121 and 128. The Board classified the series as "X"
or not for public viewing on the ground that they "offend and constitute an attack against other
religions which is expressly prohibited by law." On November 28, 1992, it appealed to the Office of
the President the classification of its TV Series No. 128 which allowed it through a letter of former
Executive Secretary Edelmiro A. Amante, Sr., addressed for Henrietta S. Mendez reversing the
decision of the respondent Board. According to the letter the episode in is protected by the
constitutional guarantee of free speech and expression and no indication that the episode poses
any clear and present danger. Petitioner also filed Civil Case. Petitioner alleged that the respondent
Board acted without jurisdiction or with grave abuse of discretion in requiring petitioner to submit the
VTR tapes of its TV program and in x-rating them. It cited its TV Program Series Nos. 115, 119, 121
and 128. In their Answer, respondent Board invoked its power under PD No. 1986 1 in relation to
Article 201 of the Revised Penal Code. The Iglesia ni Cristo insists on the literal translation of the
bible and says that our (Catholic) veneration of the Virgin Mary is not to be condoned because
nowhere it is found in the bible. The board contended that it outrages Catholic and Protestant's
beliefs. RTC ruled in favor of petitioners. CA however reversed it hence this petition.

Issue:

Whether or Not the "ang iglesia ni cristo" program is not constitutionally protected as a form of
religious exercise and expression.

Held:

Yes. Any act that restrains speech is accompanied with presumption of invalidity. It is the burden of
the respondent Board to overthrow this presumption. If it fails to discharge this burden, its act of
censorship will be struck down. This is true in this case. So-called "attacks" are mere criticisms of
some of the deeply held dogmas and tenets of other religions. RTC’s ruling clearly suppresses
petitioner's freedom of speech and interferes with its right to free exercise of religion. “attack” is
different from “offend” any race or religion. The respondent Board may disagree with the criticisms
of other religions by petitioner but that gives it no excuse to interdict such criticisms, however,
unclean they may be. Under our constitutional scheme, it is not the task of the State to favor any
religion by protecting it against an attack by another religion. Religious dogmas and beliefs are often
at war and to preserve peace among their followers, especially the fanatics, the establishment
clause of freedom of religion prohibits the State from leaning towards any religion. Respondent
board cannot censor the speech of petitioner Iglesia ni Cristo simply because it attacks other
religions, even if said religion happens to be the most numerous church in our country. The basis of
freedom of religion is freedom of thought and it is best served by encouraging the marketplace of
dueling ideas. It is only where it is unavoidably necessary to prevent an immediate and grave
danger to the security and welfare of the community that infringement of religious freedom may be
justified, and only to the smallest extent necessary to avoid the danger. There is no showing
whatsoever of the type of harm the tapes will bring about especially the gravity and imminence of
the threatened harm. Prior restraint on speech, including religious speech, cannot be justified by
hypothetical fears but only by the showing of a substantive and imminent evil. It is inappropriate to
apply the clear and present danger test to the case at bar because the issue involves the content of
speech and not the time, place or manner of speech. Allegedly, unless the speech is first allowed,
its impact cannot be measured, and the causal connection between the speech and the evil
apprehended cannot be established. The determination of the question as to whether or not such
vilification, exaggeration or fabrication falls within or lies outside the boundaries of protected speech
Page 130
or expression is a judicial function which cannot be arrogated by an administrative body such as a
Board of Censors." A system of prior restraint may only be validly administered by judges and not
left to administrative agencies.

ADIONG VS. COMELEC


[207 SCRA 712; G.R. NO. 103956; 31 MAR 1992]

Facts:

COMELEC promulgated Resolution No. 2347 which provides that decals and stickers may be
posted only in any of the authorized posting areas, prohibiting posting in "mobile" places, public or
private. Petitioner Blo Umpar Adiong, a senatorial candidate in the May 11, 1992 elections now
assails the Resolution. In addition, the petitioner believes that with the ban on radio, television and
print political advertisements, he, being a neophyte in the field of politics stands to suffer grave and
irreparable injury with this prohibition.

Issue:

Whether or Not the COMELEC’s prohibition unconstitutional.

Held:

The prohibition unduly infringes on the citizen's fundamental right of free speech. The preferred
freedom of expression calls all the more for the utmost respect when what may be curtailed is the
dissemination of information to make more meaningful the equally vital right of suffrage. The so-
called balancing of interests — individual freedom on one hand and substantial public interests on
the other — is made even more difficult in election campaign cases because the Constitution also
gives specific authority to the Commission on Elections to supervise the conduct of free, honest,
and orderly elections. When faced with border line situations where freedom to speak by a
candidate or party and freedom to know on the part of the electorate are invoked against actions
intended for maintaining clean and free elections, the police, local officials and COMELEC, should
lean in favor of freedom. The regulation of election campaign activity may not pass the test of
validity if it is too general in its terms or not limited in time and scope in its application, if it restricts
one's expression of belief in a candidate or one's opinion of his or her qualifications, if it cuts off the
flow of media reporting, and if the regulatory measure bears no clear and reasonable nexus with the
constitutionally sanctioned objective.

The posting of decals and stickers in mobile places like cars and other moving vehicles does not
endanger any substantial government interest. There is no clear public interest threatened by such
activity so as to justify the curtailment of the cherished citizen's right of free speech and expression.
Under the clear and present danger rule not only must the danger be patently clear and pressingly
present but the evil sought to be avoided must be so substantive as to justify a clamp over one's
mouth or a writing instrument to be stilled. The regulation strikes at the freedom of an individual to
express his preference and, by displaying it on his car, to convince others to agree with him. A
sticker may be furnished by a candidate but once the car owner agrees to have it placed on his
private vehicle, the expression becomes a statement by the owner, primarily his own and not of
anybody else. The restriction as to where the decals and stickers should be posted is so broad that
it encompasses even the citizen's private property, which in this case is a privately-owned vehicle.
In consequence of this prohibition, another cardinal rule prescribed by the Constitution would be
violated. Section 1, Article III of the Bill of Rights provides that no person shall be deprived of his
property without due process of law.

The prohibition on posting of decals and stickers on "mobile" places whether public or private except
in the authorized areas designated by the COMELEC becomes censorship.

Page 131
NATIONAL PRESS CLUB VS. COMELEC
[201 SCRA 1; G.R. NO. 1026653; 5 MAR 1992]

Facts:

Petitioners in these cases consist of representatives of the mass media which are prevented from
selling or donating space and time for political advertisements; two (2) individuals who are
candidates for office (one for national and the other for provincial office) in the coming May 1992
elections; and taxpayers and voters who claim that their right to be informed of election Issue and of
credentials of the candidates is being curtailed. It is principally argued by petitioners that Section 11
(b) of Republic Act No. 66461 invades and violates the constitutional guarantees comprising
freedom of expression. Petitioners maintain that the prohibition imposed by Section 11 (b) amounts
to censorship, because it selects and singles out for suppression and repression with criminal
sanctions, only publications of a particular content, namely, media-based election or political
propaganda during the election period of 1992. It is asserted that the prohibition is in derogation of
media's role, function and duty to provide adequate channels of public information and public
opinion relevant to election Issue. Further, petitioners contend that Section 11 (b) abridges the
freedom of speech of candidates, and that the suppression of media-based campaign or political
propaganda except those appearing in the Comelec space of the newspapers and on Comelec time
of radio and television broadcasts, would bring about a substantial reduction in the quantity or
volume of information concerning candidates and Issue in the election thereby curtailing and limiting
the right of voters to information and opinion.

Issue:

Whether or Not Section 11 (b) of Republic Act No. 6646 constitutional.

Held:

Yes. It seems a modest proposition that the provision of the Bill of Rights which enshrines freedom
of speech, freedom of expression and freedom of the press has to be taken in conjunction with
Article IX (C) (4) which may be seen to be a special provision applicable during a specific limited
period — i.e., "during the election period." In our own society, equality of opportunity to proffer
oneself for public office, without regard to the level of financial resources that one may have at one's
disposal, is clearly an important value. One of the basic state policies given constitutional rank by
Article II, Section 26 of the Constitution is the egalitarian demand that "the State shall guarantee
equal access to opportunities for public service and prohibit political dynasties as may be defined by
law." The essential question is whether or not the assailed legislative or administrative provisions
constitute a permissible exercise of the power of supervision or regulation of the operations of
communication and information enterprises during an election period, or whether such act has gone
beyond permissible supervision or regulation of media operations so as to constitute
unconstitutional repression of freedom of speech and freedom of the press. The Court considers
that Section 11 (b) has not gone outside the permissible bounds of supervision or regulation of
media operations during election periods.

Section 11 (b) is limited in the duration of its applicability and enforceability. By virtue of the
operation of Article IX (C) (4) of the Constitution, Section 11 (b) is limited in its applicability in time to
election periods. Section 11 (b) does not purport in any way to restrict the reporting by newspapers
or radio or television stations of news or news-worthy events relating to candidates, their
qualifications, political parties and programs of government. Moreover, Section 11 (b) does not
reach commentaries and expressions of belief or opinion by reporters or broadcasters or editors or
commentators or columnists in respect of candidates, their qualifications, and programs and so
forth, so long at least as such comments, opinions and beliefs are not in fact advertisements for
particular candidates covertly paid for. In sum, Section 11 (b) is not to be read as reaching any
report or commentary other coverage that, in responsible media, is not paid for by candidates for
political office. Section 11 (b) as designed to cover only paid political advertisements of particular
candidates.
Page 132
The limiting impact of Section 11 (b) upon the right to free speech of the candidates themselves is
not unduly repressive or unreasonable.

US VS. BUSTOS
[37 PHIL. 731; G.R. L-12592; 8 MAR 1918]

Facts:

In the latter part of 1915, numerous citizens of the Province of Pampanga assembled, and prepared
and signed a petition to the Executive Secretary(privileged communication) through the law office of
Crossfield and O'Brien, and five individuals signed affidavits, charging Roman Punsalan, justice of
the peace of Macabebe and Masantol, Pampanga, with malfeasance in office and asking for his
removal. The specific charges against the justice of the peace include the solicitation of money from
persons who have pending cases before the judge. Now, Punsalan alleged that accused published
a writing which was false, scandalous, malicious, defamatory, and libelous against him.

Issue:

Whether or Not accused is entitled to constitutional protection by virtue of his right to free speech
and free press.

Held:

Yes. The guaranties of a free speech and a free press include the right to criticize judicial conduct.
The administration of the law is a matter of vital public concern. Whether the law is wisely or badly
enforced is, therefore, a fit subject for proper comment. If the people cannot criticize a justice of the
peace or a judge the same as any other public officer, public opinion will be effectively suppressed.
It is a duty which every one owes to society or to the State to assist in the investigation of any
alleged misconduct. It is further the duty of all who know of any official dereliction on the part of a
magistrate or the wrongful act of any public officer to bring the facts to the notice of those whose
duty it is to inquire into and punish them.

The right to assemble and petition is the necessary consequence of republican institutions and the
complement of the part of free speech. Assembly means a right on the part of citizens to meet
peaceably for consultation in respect to public affairs. Petition means that any person or group of
persons can apply, without fear of penalty, to the appropriate branch or office of the government for
a redress of grievances. The persons assembling and petitioning must, of course, assume
responsibility for the charges made. All persons have an interest in the pure and efficient
administration of justice and of public affairs.

Public policy, the welfare of society, and the orderly administration of government have demanded
protection for public opinion. The inevitable and incontestable result has been the development and
adoption of the doctrine of privilege. All persons have an interest in the pure and efficient
administration of justice and of public affairs. The duty under which a party is privileged is sufficient
if it is social or moral in its nature and this person in good faith believes he is acting in pursuance
thereof although in fact he is mistaken. Although the charges are probably not true as to the justice
of the peace, they were believed to be true by the petitioners. Good faith surrounded their action.
Probable cause for them to think that malfeasance or misfeasance in office existed is apparent. The
ends and the motives of these citizens— to secure the removal from office of a person thought to be
venal — were justifiable. In no way did they abuse the privilege.

In the usual case malice can be presumed from defamatory words. Privilege destroys that
presumption. A privileged communication should not be subjected to microscopic examination to
discover grounds of malice or falsity.

Page 133
PITA VS. COURT OF APPEALS
[178 SCRA 362; G.R. NO.80806; 5 OCT 1989]

Facts:

On December 1 and 3, 1983, pursuing an Anti-Smut Campaign initiated by the Mayor of the City of
Manila, Ramon D. Bagatsing, elements of the Special Anti-Narcotics Group, Auxilliary Services
Bureau, Western Police District, INP of the Metropolitan Police Force of Manila, seized and
confiscated from dealers, distributors, newsstand owners and peddlers along Manila sidewalks,
magazines, publications and other reading materials believed to be obscene, pornographic and
indecent and later burned the seized materials in public at the University belt along C.M. Recto
Avenue, Manila, in the presence of Mayor Bagatsing and several officers and members of various
student organizations.

Among the publications seized, and later burned, was "Pinoy Playboy" magazines published and
co-edited by plaintiff Leo Pita.

Plaintiff filed a case for injunction with prayer for issuance of the writ of preliminary injunction against
Mayor Bagatsing and Narcisco Cabrera, as superintendent of Western Police District of the City of
Manila, seeking to enjoin said defendants and their agents from confiscating plaintiff’s magazines or
from preventing the sale or circulation thereof claiming that the magazine is a decent, artistic and
educational magazine which is not per se obscene, and that the publication is protected by the
Constitutional guarantees of freedom of speech and of the press. Plaintiff also filed an Urgent
Motion for issuance of a temporary restraining order against indiscriminate seizure, confiscation and
burning of plaintiff's "Pinoy Playboy" Magazines, pending hearing on the petition for preliminary
injunction. The Court granted the temporary restraining order. The case was set for trial upon the
lapse of the TRO. RTC ruled that the seizure was valid. This was affirmed by the CA.

Issue:

Whether or Not the seizure violative of the freedom of expression of the petitioner.

Held:

Freedom of the press is not without restraint as the state has the right to protect society from
pornographic literature that is offensive to public morals, as indeed we have laws punishing the
author, publishers and sellers of obscene publications. However, It is easier said than done to say,
that if the pictures here in question were used not exactly for art's sake but rather for commercial
purposes, the pictures are not entitled to any constitutional protection. Using the Kottinger rule: the
test of obscenity is "whether the tendency of the matter charged as obscene, is to deprave or
corrupt those whose minds are open to such immoral influences and into whose hands a publication
or other article charged as being obscene may fall." Another is whether it shocks the ordinary and
common sense of men as an indecency. Ultimately "whether a picture is obscene or indecent must
depend upon the circumstances of the case and that the question is to be decided by the "judgment
of the aggregate sense of the community reached by it." The government authorities in the instant
case have not shown the required proof to justify a ban and to warrant confiscation of the literature
First of all, they were not possessed of a lawful court order: (1) finding the said materials to be
pornography, and (2) authorizing them to carry out a search and seizure, by way of a search
warrant. The court provides that the authorities must apply for the issuance of a search warrant from
a judge, if in their opinion an obscenity seizure is in order and that;

1. The authorities must convince the court that the materials sought to be seized are obscene
and pose a clear and present danger of an evil substantive enough to warrant State
interference and action;

Page 134
2. The judge must determine whether or not the same are indeed obscene. The question is to
be resolved on a case-to-case basis and on the judge’s sound discretion;

AYER PRODUCTIONS VS. CAPULONG


[160 SCRA 861; G.R. NO. L-82380; 29 APR 1988]

Facts:

Petitioner McElroy an Australian film maker, and his movie production company, Ayer Productions,
envisioned, sometime in 1987, for commercial viewing and for Philippine and international release,
the historic peaceful struggle of the Filipinos at EDSA. The proposed motion picture entitled "The
Four Day Revolution" was endorsed by the MTRCB as and other government agencies consulted.
Ramos also signified his approval of the intended film production.

It is designed to be viewed in a six-hour mini-series television play, presented in a "docu-drama"


style, creating four fictional characters interwoven with real events, and utilizing actual documentary
footage as background. David Williamson is Australia's leading playwright and Professor McCoy
(University of New South Wales) is an American historian have developed a script.

Enrile declared that he will not approve the use, appropriation, reproduction and/or exhibition of his
name, or picture, or that of any member of his family in any cinema or television production, film or
other medium for advertising or commercial exploitation. petitioners acceded to this demand and the
name of Enrile was deleted from the movie script, and petitioners proceeded to film the projected
motion picture. However, a complaint was filed by Enrile invoking his right to privacy. RTC ordered
for the desistance of the movie production and making of any reference to plaintiff or his family and
from creating any fictitious character in lieu of plaintiff which nevertheless is based on, or bears
substantial or marked resemblance to Enrile. Hence the appeal.

Issue:

Whether or Not freedom of expression was violated.

Held:

Yes. Freedom of speech and of expression includes the freedom to film and produce motion
pictures and exhibit such motion pictures in theaters or to diffuse them through television.
Furthermore the circumstance that the production of motion picture films is a commercial activity
expected to yield monetary profit, is not a disqualification for availing of freedom of speech and of
expression.

The projected motion picture was as yet uncompleted and hence not exhibited to any audience.
Neither private respondent nor the respondent trial Judge knew what the completed film would
precisely look like. There was, in other words, no "clear and present danger" of any violation of any
right to privacy. Subject matter is one of public interest and concern. The subject thus relates to a
highly critical stage in the history of the country.

At all relevant times, during which the momentous events, clearly of public concern, that petitioners
propose to film were taking place, Enrile was a "public figure:" Such public figures were held to have
lost, to some extent at least, their right to privacy.

The line of equilibrium in the specific context of the instant case between the constitutional freedom
of speech and of expression and the right of privacy, may be marked out in terms of a requirement
that the proposed motion picture must be fairly truthful and historical in its presentation of events.
Page 135
LOPEZ VS. SANDIGANBAYAN
[34 SCRA 116; L-26549; 31 JUL 1970]

Facts:

In the early part of January, 1956, there appeared on the front page of The Manila Chronicle, of
which petitioner Lopez was the publisher, as well as on other dailies, a news story of a sanitary
inspector assigned to the Babuyan Islands, Fidel Cruz, sending a distress signal to a passing United
States Airforce plane which in turn relayed the message to Manila. An American Army plane
dropping on the beach of an island an emergency-sustenance kit containing, among other things, a
two-way radio set. He utilized it to inform authorities in Manila that the people in the place were
living in terror, due to a series of killings committed since Christmas of 1955. Losing no time, the
Philippines defense establishment rushed to the island a platoon of scout rangers. Upon arriving
Major Encarnacion and his men found, instead of the alleged killers, a man named Fidel Cruz who
merely wanted transportation home to Manila. In view of this finding, Major Encarnacion branded as
a "hoax," the report of respondent.

This Week Magazine of the Manila Chronicle, then edited by Gatbonton, devoted a pictorial article
to it in its issue of January 15, 1956. Mention was made that while Fidel Cruz’ story turned out to be
false it brought attention to the government that people in that most people in the area are sick sick,
only two individuals able to read and write, food and clothing being scarce.

The magazine carried photographs of the person purporting to be Fidel Cruz. Unfortunately, the
pictures that were published were that of private respondent Fidel G. Cruz, a businessman
contractor from Santa Maria, Bulacan. It turned out that the photographs of respondent Cruz and
that of Fidel Cruz, sanitary inspector, were on file in the library of the Manila Chronicle but when the
news quiz format was prepared, the two photographs were in advertently switched. However a
correction was published immediately.

Respondent sued petitioners in the Court of First Instance of Manila for the recovery of damages
alleging the defamatory character of the above publication of his picture. Defense interposed that
they are beating the deadline. The court ruled in his favor. Hence the appeal.

Issue:

Whether or Not petitioners abused the freedom of the press.

Held:

No. The SC, quoting Quisumbing v. Lopez, found for plaintiff, but with reduced damages, since the
error in this case could have been checked considering that this was a weekly magazine and not a
daily. The ruling: "there is no evidence in the record to prove that the publication of the news item
under consideration was prompted by personal ill will or spite, or that there was intention to do
harm,' and that on the other hand there was 'an honest and high sense of duty to serve the best
interests of the public, without self-seeking motive and with malice towards none.' Every citizen of
course has the right to enjoy a good name and reputation, but we do not consider that the
respondents, under the circumstances of this case, had violated said right or abused the freedom of
the press. The newspapers should be given such leeway and tolerance as to enable them to
courageously and effectively perform their important role in our democracy. In the preparation of
stories, press reporters and editors usually have to race with their deadlines; and consistently with
good faith and reasonable care, they should not be held to account, to a point of suppression, for
honest mistakes or imperfection in the choice of words.

“No inroads on press freedom should be allowed in the guise of punitive action visited on what
otherwise could be characterized as libel whether in the form of printed words or a defamatory
Page 136
imputation resulting from the publication of respondent's picture with the offensive caption as in the
case here complained of. This is merely to underscore the primacy that freedom of the press
enjoys.”

PRIMICIAS VS. FUGOSO


[80 PHIL 71; L-1800; 27 JAN 1948]

Facts:

An action was instituted by the petitioner for the refusal of the respondent to issue a permit to them
to hold a public meeting in Plaza Miranda for redress of grievances to the government. The reason
alleged by the respondent in his defense for refusing the permit is, "that there is a reasonable
ground to believe, basing upon previous utterances and upon the fact that passions, specially on
the part of the losing groups, remains bitter and high, that similar speeches will be delivered tending
to undermine the faith and confidence of the people in their government, and in the duly constituted
authorities, which might threaten breaches of the peace and a disruption of public order." Giving
emphasis as well to the delegated police power to local government. Stating as well Revised
Ordinances of 1927 prohibiting as an offense against public peace, and penalizes as a
misdemeanor, "any act, in any public place, meeting, or procession, tending to disturb the peace or
excite a riot; or collect with other persons in a body or crowd for any unlawful purpose; or disturb or
disquiet any congregation engaged in any lawful assembly." Included herein is Sec. 1119, Free use
of Public Place.1

Issue:

Whether or Not the freedom of speech was violated.

Held:

Yes. Dealing with the ordinance, specifically, Sec. 1119, said section provides for two constructions:
(1) the Mayor of the City of Manila is vested with unregulated discretion to grant or refuse, to grant
permit for the holding of a lawful assembly or meeting, parade, or procession in the streets and
other public places of the City of Manila; (2) The right of the Mayor is subject to reasonable
discretion to determine or specify the streets or public places to be used with the view to prevent
confusion by overlapping, to secure convenient use of the streets and public places by others, and
to provide adequate and proper policing to minimize the risk of disorder. The court favored the
second construction. First construction tantamount to authorizing the Mayor to prohibit the use of
the streets. Under our democratic system of government no such unlimited power may be validly
granted to any officer of the government, except perhaps in cases of national emergency.

The Mayor’s first defense is untenable. Fear of serious injury cannot alone justify suppression of
free speech and assembly. It is the function of speech to free men from the bondage of irrational
fears. To justify suppression of free speech there must be reasonable ground to fear that serious
evil will result if free speech is practiced. There must be reasonable ground to believe that the
danger apprehended is imminent. There must be reasonable ground to believe that the evil to be
prevented is a serious one . The fact that speech is likely to result in some violence or in destruction
of property is not enough to justify its suppression. There must be the probability of serious injury to
the state.

ZALDIVAR VS. SANDIGANBAYAN


[170 SCRA 1; G.R. NO. 79690-707; 1 FEB 1989]

Facts:

Page 137
The case stemmed from the resolution of the Supreme Court stopping the respondent from
investigating graft cases involving Antique Gov. Enrique Zaldivar. The Court ruled that since the
adoption of the 1987 Constitution, respondent’s powers as Tanodbayan have been superseded by
the creation of the Office of the Ombudsman, he however becomes the Special Prosecutor of the
State, and can only conduct an investigation and file cases only when so authorized by the
Ombudsman. A motion for reconsideration was filed by the respondent wherein he included
statements which were unrelated in the Issue raised in the Court. This include: (a)That he had been
approached twice by a leading member of the court and he was asked to 'go slow on Zaldivar and
'not to be too hard on him; (b) That he "was approached and asked to refrain from investigating the
COA report on illegal disbursements in the Supreme Court because 'it will embarass the Court;" and
(c) that in several instances, the undersigned respondent was called over the phone by a leading
member of the Court and was asked to dismiss the cases against two Members of the Court."
Statements of the respondent saying that the SC’s order '"heightens the people's apprehension over
the justice system in this country, especially because the people have been thinking that only the
small fly can get it while big fishes go scot-free” was publicized in leading newspapers.

Now, the Court Resolved to require respondent to explain in writing why he should not be punished
for contempt of court for making such public statements reported in the media. Respondent then
sought to get some members of the Court to inhibit themselves in the resolution of the Zaldivar case
for alleged bias and prejudice against him. A little later, he in effect asked the whole Court to inhibit
itself from passing upon the Issue involved in proceeding and to pass on responsibility for this
matter to the Integrated Bar of the Philippines, upon the ground that respondent cannot expect due
process from this Court, that the Court has become incapable of judging him impartially and fairly.
The Court found respondent guilty of contempt of court and indefinitely suspended from the practice
of law. Now, he assails said conviction, invoking his freedom of speech. Counsel for respondent
urges that it is error "for this Court to apply the "visible tendency" rule rather than the "clear and
present danger" rule in disciplinary and contempt charges."

Issue:

Whether or Not there was a violation of the freedom of speech/expression.

Held:

There was no violation. The Court did not purport to announce a new doctrine of "visible tendency,"
it was simply paraphrasing Section 3 (d) of Rule 71 of the Revised Rules of Court which penalizes a
variety of contumacious conduct including: "any improper conduct tending, directly or indirectly, to
impede, obstruct or degrade the administration of justice."

Under either the "clear and present danger" test or the "balancing-of-interest test," the Court held
that the statements made by respondent Gonzalez are of such a nature and were made in such a
manner and under such circumstances, as to transcend the permissible limits of free speech. What
is here at stake is the authority of the Supreme Court to confront and prevent a "substantive evil"
consisting not only of the obstruction of a free and fair hearing of a particular case but also the
avoidance of the broader evil of the degradation of the judicial system of a country and the
destruction of the standards of professional conduct required from members of the bar and officers
of the courts, which has some implications to the society.
REYES VS. BAGATSING
[125 SCRA 553; L-65366; 9 NOV 1983]

Facts:

Petitioner sought a permit from the City of Manila to hold a peaceful march and rally on October 26,
1983 from 2:00 to 5:00 in the afternoon, starting from the Luneta to the gates of the United States
Embassy. Once there, and in an open space of public property, a short program would be held. The
march would be attended by the local and foreign participants of such conference. That would be
followed by the handing over of a petition based on the resolution adopted at the closing session of
Page 138
the Anti-Bases Coalition. There was likewise an assurance in the petition that in the exercise of the
constitutional rights to free speech and assembly, all the necessary steps would be taken by it "to
ensure a peaceful march and rally. However the request was denied. Reference was made to
persistent intelligence reports affirming the plans of subversive/criminal elements to infiltrate or
disrupt any assembly or congregations where a large number of people is expected to attend.
Respondent suggested that a permit may be issued if it is to be held at the Rizal Coliseum or any
other enclosed area where the safety of the participants themselves and the general public may be
ensured. An oral argument was heard and the mandatory injunction was granted on the ground that
there was no showing of the existence of a clear and present danger of a substantive evil that could
justify the denial of a permit. However Justice Aquino dissented that the rally is violative of
Ordinance No. 7295 of the City of Manila prohibiting the holding of rallies within a radius of five
hundred (500) feet from any foreign mission or chancery and for other purposes. Hence the Court
resolves.

Issue:

Whether or Not the freedom of expression and the right to peaceably assemble violated.

Held:

Yes. The invocation of the right to freedom of peaceable assembly carries with it the implication that
the right to free speech has likewise been disregarded. It is settled law that as to public places,
especially so as to parks and streets, there is freedom of access. Nor is their use dependent on who
is the applicant for the permit, whether an individual or a group. There can be no legal objection,
absent the existence of a clear and present danger of a substantive evil, on the choice of Luneta as
the place where the peace rally would start. Time immemorial Luneta has been used for purposes
of assembly, communicating thoughts between citizens, and discussing public questions.
Such use of the public places has from ancient times, been a part of the privileges, immunities,
rights, and liberties of citizens.

With regard to the ordinance, there was no showing that there was violation and even if it could be
shown that such a condition is satisfied it does not follow that respondent could legally act the way
he did. The validity of his denial of the permit sought could still be challenged.

A summary of the application for permit for rally: The applicants for a permit to hold an assembly
should inform the licensing authority of the date, the public place where and the time when it will
take place. If it were a private place, only the consent of the owner or the one entitled to its legal
possession is required. Such application should be filed well ahead in time to enable the public
official concerned to appraise whether there may be valid objections to the grant of the permit or to
its grant but at another public place. It is an indispensable condition to such refusal or modification
that the clear and present danger test be the standard for the decision reached. Notice is given to
applicants for the denial.

BAYAN VS. EXECUTIVE SECRETARY ERMITA


[488 SCRA 226; G.R. NO. 169838; 25 APR 2006]

Facts:

Rallies of September 20, October 4, 5 and 6, 2005 is at issue. BAYAN’s rally was violently
dispersed. 26 petitioners were injured, arrested and detained when a peaceful mass action they
was preempted and violently dispersed by the police. KMU asserts that the right to peaceful
assembly, are affected by Batas Pambansa No. 880 and the policy of “Calibrated Preemptive
Response” (CPR) being followed to implement it. KMU, et al., claim that on October 4, 2005, a rally
KMU co-sponsored was to be conducted at the Mendiola bridge but police blocked them along C.M.
Recto and Lepanto Streets and forcibly dispersed them, causing injuries to several of their
members. They further allege that on October 6, 2005, a multi-sectoral rally which KMU also co-
sponsored was scheduled to proceed along España Avenue in front of the UST and going towards
Page 139
Mendiola bridge. Police officers blocked them along Morayta Street and prevented them from
proceeding further. They were then forcibly dispersed, causing injuries on one of them. Three other
rallyists were arrested.

All petitioners assail Batas Pambansa No. 880 The Public Assembly Act of 1985, some of them in
toto and others only Sections 4, 5, 6, 12, 13(a), and 14(a), as well as the policy of CPR. They seek
to stop violent dispersals of rallies under the “no permit, no rally” policy and the CPR policy
announced on Sept. 21, 2005.

Petitioners Bayan, et al., contend that BP 880 is clearly a violation of the Constitution and the
International Covenant on Civil and Political Rights and other human rights treaties of which the
Philippines is a signatory.

They argue that B.P. No. 880 requires a permit before one can stage a public assembly regardless
of the presence or absence of a clear and present danger. It also curtails the choice of venue and
is thus repugnant to the freedom of expression clause as the time and place of a public assembly
form part of the message for which the expression is sought.

Petitioners Jess del Prado, et al., in turn, argue that B.P. No. 880 is unconstitutional as it is a
curtailment of the right to peacefully assemble and petition for redress of grievances because it puts
a condition for the valid exercise of that right. It also characterizes public assemblies without a
permit as illegal and penalizes them and allows their dispersal. Thus, its provisions are not mere
regulations but are actually prohibitions. Regarding the CPR policy, it is void for being an ultra vires
act that alters the standard of maximum tolerance set forth in B.P. No. 880, aside from being void
for being vague and for lack of publication.

KMU, et al., argue that the Constitution sets no limits on the right to assembly and therefore B.P.
No. 880 cannot put the prior requirement of securing a permit. And even assuming that the
legislature can set limits to this right, the limits provided are unreasonable: First, allowing the Mayor
to deny the permit on clear and convincing evidence of a clear and present danger is too
comprehensive. Second, the five-day requirement to apply for a permit is too long as certain events
require instant public assembly, otherwise interest on the issue would possibly wane.As to the CPR
policy, they argue that it is preemptive, that the government takes action even before the rallyists
can perform their act, and that no law, ordinance or executive order supports the policy.
Furthermore, it contravenes the maximum tolerance policy of B.P. No. 880 and violates the
Constitution as it causes a chilling effect on the exercise by the people of the right to peaceably
assemble.

Respondents argued that petitioners have no standing. BP 880 entails traffic re-routing to prevent
grave public inconvenience and serious or undue interference in the free flow of commerce and
trade. It is content-neutral regulation of the time, place and manner of holding public assemblies.
According to Atienza RA. 7160 gives the Mayor power to deny a permit independently of B.P. No.
880. and that the permit is for the use of a public place and not for the exercise of rights; and that
B.P. No. 880 is not a content-based regulation because it covers all rallies.

Issue:

Whether or Not BP 880 and the CPR Policy unconstitutional.

Held:

No question as to standing. Their right as citizens to engage in peaceful assembly and exercise the
right of petition, as guaranteed by the Constitution, is directly affected by B.P. No. 880. B.P. 880 is
not an absolute ban of public assemblies but a restriction that simply regulates the time, place and
manner of the assemblies. It refers to all kinds of public assemblies that would use public places.
The reference to “lawful cause” does not make it content-based because assemblies really have to
be for lawful causes, otherwise they would not be “peaceable” and entitled to protection. Maximum
Page 140
tolerance1 is for the protection and benefit of all rallyists and is independent of the content of the
expressions in the rally. There is, likewise, no prior restraint, since the content of the speech is not
relevant to the regulation.

The so-called calibrated preemptive response policy has no place in our legal firmament and must
be struck down as a darkness that shrouds freedom. It merely confuses our people and is used by
some police agents to justify abuses. Insofar as it would purport to differ from or be in lieu of
maximum tolerance, this was declared null and void.

The Secretary of the Interior and Local Governments, are DIRECTED to take all necessary steps for
the immediate compliance with Section 15 of Batas Pambansa No. 880 through the establishment
or designation of at least one suitable freedom park or plaza in every city and municipality of the
country. After thirty (30) days from the finality of this Decision, subject to the giving of advance
notices, no prior permit shall be required to exercise the right to peaceably assemble and petition in
the public parks or plazas of a city or municipality that has not yet complied with Section 15 of the
law.

FERNANDO VS. ESTORNINOS


[G.R. NO 159751; 6 DEC 2006]

Facts:

Acting on reports of sale and distribution of pornographic materials, officers of the PNP Criminal
Investigation and Detection Group in the National Capital Region conducted police surveillance on
the store Gaudencio E. Fernando Music Fair (Music Fair) in Quiapo. A Search Warrant for violation
of Article 201 of RPC against petitioner and a certain Warren Tingchuy and the seizure of the
following items:

a. Copies of New Rave Magazines with nude obscene pictures;


b. Copies of IOU Penthouse Magazine with nude obscene pictures;
c. Copies of Hustler International Magazine with nude obscene pictures; and
d. Copies of VHS tapes containing pornographic shows.

The police searched the premises and confiscated twenty-five VHS tapes(among of which is “Kahit
sa Pangarap Lang” with Myra Manibog as actress who is naked) and ten different
magazines(Dalaga, Penthouse, Swank, Erotic, Rave, Playhouse, Gallery, QUI), which they deemed
pornographic. Petitioners were charged and convicted. CA affirmed the decision hence this appeal.

Issue:

Whether or Not the CA erred in affirming RTC’s decision.

Held:

No. As obscenity is an unprotected speech which the State has the right to regulate, the State in
pursuing its mandate to protect the public from obscene, immoral and indecent materials must
justify the regulation or limitation. (Kottinger Rule Applied).

MALABANAN VS. RAMENTO


[129 SCRA 359; G.R. NO.62270; 21 MAY 1984]

Facts:

Petitioners were officers of the Supreme Student Council of respondent University. They sought and
were granted by the school authorities a permit to hold a meeting from 8:00 A.M. to 12:00 P.M, on
Page 141
August 27, 1982. Pursuant to such permit, along with other students, they held a general assembly
at the Veterinary Medicine and Animal Science basketball court (VMAS), the place indicated in such
permit, not in the basketball court as therein stated but at the second floor lobby. At such gathering
they manifested in vehement and vigorous language their opposition to the proposed merger of the
Institute of Animal Science with the Institute of Agriculture. The same day, they marched toward the
Life Science Building and continued their rally. It was outside the area covered by their permit. Even
they rallied beyond the period allowed. They were asked to explain on the same day why they
should not be held liable for holding an illegal assembly. Then on September 9, 1982, they were
informed that they were under preventive suspension for their failure to explain the holding of an
illegal assembly. The validity thereof was challenged by petitioners both before the Court of First
Instance of Rizal against private respondents and before the Ministry of Education, Culture, and
Sports. Respondent Ramento found petitioners guilty of the charge of illegal assembly which was
characterized by the violation of the permit granted resulting in the disturbance of classes and oral
defamation. The penalty was suspension for one academic year. Hence this petition.

Issue:

Whether on the facts as disclosed resulting in the disciplinary action and the penalty imposed, there
was an infringement of the right to peaceable assembly and its cognate right of free speech.

Held:

Yes. Student leaders are likely to be assertive and dogmatic. They would be ineffective if during a
rally they speak in the guarded and judicious language of the academe. But with the activity taking
place in the school premises and during the daytime, no clear and present danger of public disorder
is discernible. This is without prejudice to the taking of disciplinary action for conduct, "materially
disrupts classwork or involves substantial disorder or invasion of the rights of others."

The rights to peaceable assembly and free speech are guaranteed students of educational
institutions. Necessarily, their exercise to discuss matters affecting their welfare or involving public
interest is not to be subjected to previous restraint or subsequent punishment unless there be a
showing of a clear and present danger to a substantive evil that the state, has a right to present. As
a corollary, the utmost leeway and scope is accorded the content of the placards displayed or
utterances made. The peaceable character of an assembly could be lost, however, by an advocacy
of disorder under the name of dissent, whatever grievances that may be aired being susceptible to
correction through the ways of the law. If the assembly is to be held in school premises, permit must
be sought from its school authorities, who are devoid of the power to deny such request arbitrarily or
unreasonably. In granting such permit, there may be conditions as to the time and place of the
assembly to avoid disruption of classes or stoppage of work of the non-academic personnel. Even
if, however, there be violations of its terms, the penalty incurred should not be disproportionate to
the offense.

NON VS. DAMES


[185 SCRA 523; G.R. NO. 89317; 20 MAY 1990]

Facts:

Petitioners, students in private respondent Mabini Colleges, Inc. in Daet, Camarines Norte, were not
allowed to re-enroll by the school for the academic year 1988-1989 for leading or participating in
student mass actions against the school in the preceding semester. The subject of the protests is
not, however, made clear in the pleadings.

Petitioners filed a petition in the court seeking their readmission or re-enrollment to the school, but
the trial court dismissed the petition. They now petition the court to reverse its ruling in Alcuaz vs.
PSBA1, which was also applied in the case. The court said that petitioners waived their privilege to
be admitted for re-enrollment with respondent college when they adopted, signed, and used its
Page 142
enrollment form for the first semester of school year 1988-89, which states that: The Mabini College
reserves the right to deny admission of students whose scholarship and attendance are
unsatisfactory and to require withdrawal of students whose conduct discredits the institution and/or
whose activities unduly disrupts or interfere with the efficient operation of the college. Students,
therefore, are required to behave in accord with the Mabini College code of conduct and discipline.

Issue:

Whether or Not the students’ right to freedom of speech and assembly infringed.

Held:

Yes. The protection to the cognate rights of speech and assembly guaranteed by the Constitution is
similarly available to students is well-settled in our jurisdiction. However there are limitations. The
permissible limitation on Student Exercise of Constitutional Rights within the school presupposes
that conduct by the student, in class or out of it, which for any reason whether it stems from time,
place, or type of behavior should not materially disrupt classwork or must not involve substantial
disorder or invasion of the rights of others.

IN RE: TULFO
[A.M. NO. 90-4-1545-0; 17 APR 1990]

Facts:

In Oct. 13, 1989, Tulfo wrote an article in his column in PDI 'On Target' stating that the Supreme
Court rendered an idiotic decision in legalizing checkpoints, and again on Oct. 16, 1989, where he
called the Supreme Court stupid and "sangkatutak na mga bobo justices of the Philippine Supreme
Court". Tulfo was required to show cause why he should not be punished for contempt. Tulfo said
that he was just reacting emotionally because he had been a victim of harassment in the
checkpoints, and "idiotic" meant illogical and unwise, and "bobo" was just quoted from other
attorneys, and since the case had been decided and terminated, there was not contempts. Lastly,
the article does not pose any clear and present danger to the Supreme court.

Issue:

Whether or Not Tulfo is in contempt.

Held:

Yes. At the time Tulfo wrote the article, the checkpoints case had not yet been decided upon, and
the Supreme Court was still acting on an MR filed from the CA. The power to punish is inherent as it
is essential for self-preservation. Contempt of court is defiance of the authority, justice and dignity of
the courts. It brings disrepute to the court. There are two kinds of publications which can be
punished for contempt:

a. those whose object is to affect the decision in a pending case.


b. those whose object is to bring courts to discredit. Tulfo's article constituted both.

It should have been okay to criticize if respectful language was used, but if its object is only to
degrade and ridicule, then it is clearly an obstruction of justice. Nothing constructive can be gained
from them. Being emotional is no excuse for being insulting. Quoting is not an excuse also, because
at the end of his article, Tulfo said, "So you bobo justices, watch out!" Also, he said he was not sorry
for having written the articles.

Tulfo is found in contempt of court and is gravely censured.


Page 143
PBM EMPLOYEES VS. PBM
[51 SCRA 189; G.R. NO. L-31195; 5 JUN 1993]

Facts:

The petitioner Philippine Blooming Mills Employees Organization (PBMEO) is a legitimate labor
union composed of the employees of the respondent Philippine Blooming Mills Co., Inc., and
petitioners. Benjamin Pagcu and Rodulfo Munsod are officers and members of the petitioner Union.
Petitioners claim that on March 1, 1969, they decided to stage a mass demonstration at
Malacañang on March 4, 1969, in protest against alleged abuses of the Pasig police. PBMEO thru
Pagcu confirmed the planned demonstration and stated that the demonstration or rally cannot be
cancelled because it has already been agreed upon in the meeting. Pagcu explained further that the
demonstration has nothing to do with the Company because the union has no quarrel or dispute
with Management. The Management, thru Atty. C.S. de Leon, Company personnel manager,
informed PBMEO that the demonstration is an inalienable right of the union guaranteed by the
Constitution but emphasized that any demonstration for that matter should not unduly prejudice the
normal operation of the Company. Workers who without previous leave of absence approved by the
Company, particularly , the officers present who are the organizers of the demonstration, who shall
fail to report for work the following morning shall be dismissed, because such failure is a violation of
the existing CBA and, therefore, would be amounting to an illegal strike. Because the petitioners
and their members numbering about 400 proceeded with the demonstration despite the pleas of the
respondent Company that the first shift workers should not be required to participate in the
demonstration and that the workers in the second and third shifts should be utilized for the
demonstration from 6 A.M. to 2 P.M. on March 4, 1969, filed a charge against petitioners and other
employees who composed the first shift, for a violation of Republic Act No. 875(Industrial Peace
Act), and of the CBA providing for 'No Strike and No Lockout.' Petitioners were held guilty in by CIR
for bargaining in bad faith, hence this appeal.

Issue:

Whether or Not the petitioners right to freedom of speech and to peaceable assemble violated.

Held:

Yes. A constitutional or valid infringement of human rights requires a more stringent criterion,
namely existence of a grave and immediate danger of a substantive evil which the State has the
right to prevent. This is not present in the case. It was to the interest herein private respondent firm
to rally to the defense of, and take up the cudgels for, its employees, so that they can report to work
free from harassment, vexation or peril and as consequence perform more efficiently their
respective tasks enhance its productivity as well as profits. Herein respondent employer did not
even offer to intercede for its employees with the local police. In seeking sanctuary behind their
freedom of expression well as their right of assembly and of petition against alleged persecution of
local officialdom, the employees and laborers of herein private respondent firm were fighting for
their very survival, utilizing only the weapons afforded them by the Constitution — the untrammelled
enjoyment of their basic human rights. The pretension of their employer that it would suffer loss or
damage by reason of the absence of its employees from 6 o'clock in the morning to 2 o'clock in the
afternoon, is a plea for the preservation merely of their property rights. The employees' pathetic
situation was a stark reality — abused, harassment and persecuted as they believed they were by
the peace officers of the municipality. As above intimated, the condition in which the employees
found themselves vis-a-vis the local police of Pasig, was a matter that vitally affected their right to
individual existence as well as that of their families. Material loss can be repaired or adequately
compensated. The debasement of the human being broken in morale and brutalized in spirit-can
never be fully evaluated in monetary terms. As heretofore stated, the primacy of human rights —
freedom of expression, of peaceful assembly and of petition for redress of grievances — over
property rights has been sustained. To regard the demonstration against police officers, not against
Page 144
the employer, as evidence of bad faith in collective bargaining and hence a violation of the collective
bargaining agreement and a cause for the dismissal from employment of the demonstrating
employees, stretches unduly the compass of the collective bargaining agreement, is "a potent
means of inhibiting speech" and therefore inflicts a moral as well as mortal wound on the
constitutional guarantees of free expression, of peaceful assembly and of petition. Circulation is one
of the aspects of freedom of expression. If demonstrators are reduced by one-third, then by that
much the circulation of the Issue raised by the demonstration is diminished. The more the
participants, the more persons can be apprised of the purpose of the rally. Moreover, the absence
of one-third of their members will be regarded as a substantial indication of disunity in their ranks
which will enervate their position and abet continued alleged police persecution.

Page 145
THE IMPAIRMENT CLAUSE

Art 3, Sec. 10. “No law impairing the obligation of contracts shall be passed.”

RUTTER VS. ESTEBAN


[93 PHIL 68; NO.L-3708; 18 MAY 1953]

Facts:

On August 20,1941 Rutter sold to Esteban two parcels of land situated in the Manila for P9,600 of
which P4,800 were paid outright, and the balance was made payable as follows: P2,400 on or
before August 7, 1942, and P2,400 on or before August 27, 1943, with interest at the rate of 7
percent per annum. To secure the payment of said balance of P4,800, a first mortgage has been
constituted in favor of the plaintiff. Esteban failed to pay the two installments as agreed upon, as
well as the interest that had accrued and so Rutter instituted an action to recover the balance due,
the interest due and the attorney's fees. The complaint also contains a prayer for sale of the
properties mortgaged in accordance with law. Esteban claims that this is a prewar obligation
contracted and that he is a war sufferer, having filed his claim with the Philippine War Damage
Commission for the losses he had suffered as a consequence of the last war; and that under
section 2 of RA 342(moratorium law), payment of his obligation cannot be enforced until after the
lapse of eight years. The complaint was dismissed. A motion for recon was made which assails the
constitutionality of RA 342.

Issue:

Whether or Not RA 342 unconstitutional on non-impairment clause grounds.

Held:

Yes. The moratorium is postponement of fulfillment of obligations decreed by the state through the
medium of the courts or the legislature. Its essence is the application of police power. The economic
interests of the State may justify the exercise of its continuing and dominant protective power
notwithstanding interference with contracts. The question is not whether the legislative action affects
contracts incidentally, or directly or indirectly, but whether the legislation is addressed to a legitimate
end and the measures taken are reasonable and appropriate to that end.

However based on the President’s general SONA and consistent with what the Court believes to be
as the only course dictated by justice, fairness and righteousness, declared that the continued
operation and enforcement of RA 342 at the present time is unreasonable and oppressive, and
should not be prolonged should be declared null and void and without effect. This holds true as
regards Executive Orders Nos. 25 and 32, with greater force and reason considering that said
Orders contain no limitation whatsoever in point of time as regards the suspension of the
enforcement and effectivity of monetary obligations.

ORTIGAS VS. FEATI BANK


[94 SCRA 533; NO.L-24670; 14 DEC 1979]

Facts:

Plaintiff is engaged in real estate business, developing and selling lots to the public, particularly the
Highway Hills Subdivision along EDSA. On March 4, 1952, plaintiff, as vendor, and Augusto Padilla
and Natividad Angeles, as vendees, entered into separate agreements of sale on installments over
Page 146
two parcels of land of the Subdivision. On July 19, 1962, the said vendees transferred their rights
and interests over the aforesaid lots in favor of one Emma Chavez. Upon completion of payment of
the purchase price, the plaintiff executed the corresponding deeds of sale in favor of Emma Chavez.
Both the agreements (of sale on installment) and the deeds of sale contained the stipulations or
restrictions that:

1. The parcel of land shall be used exclusively for residential purposes, and she shall not be
entitled to take or remove soil, stones or gravel from it or any other lots belonging to the
Seller.
2. All buildings and other improvements (except the fence) which may be constructed at any
time in said lot must be, (a) of strong materials and properly painted, (b) provided with
modern sanitary installations connected either to the public sewer or to an approved septic
tank, and (c) shall not be at a distance of less than two (2) meters from its boundary lines.

Eventually said lots were bought by defendant. Lot 5 directly from Chavez and Lot 6 from Republic
Flour Mills by deed of exchange, with same restrictions. Plaintiff claims that restriction is for the
beautification of the subdivision. Defendant claimed of the commercialization of western part of
EDSA. Defendant began constructing a commercial bank building. Plaintiff demand to stop it, which
forced him to file a case, which was later dismissed, upholding police power. Motion for recon was
denied, hence the appeal.

Issue:

Whether or Not non-impairment clause violated.

Held:

No. Resolution is a valid exercise of police power. EDSA, a main traffic artery which runs through
several cities and municipalities in the Metro Manila area, supports an endless stream of traffic and
the resulting activity, noise and pollution are hardly conducive to the health, safety or welfare of the
residents in its route. Health, safety, peace, good order and general welfare of the people in the
locality are justifications for this. It should be stressed, that while non-impairment of contracts is
constitutionally guaranteed, the rule is not absolute, since it has to be reconciled with the legitimate
exercise of police power.

LOZANO VS. MARTINEZ


[146 SCRA 323; NO.L-63419; 18 DEC 1986]

Facts:

A motion to quash the charge against the petitioners for violation of the BP 22 was made,
contending that no offense was committed, as the statute is unconstitutional. Such motion was
denied by the RTC. The petitioners thus elevate the case to the Supreme Court for relief. The
Solicitor General, commented that it was premature for the accused to elevate to the Supreme
Court the orders denying their motions to quash. However, the Supreme Court finds it justifiable to
intervene for the review of lower court's denial of a motion to quash.

Issue:

Whether or Not BP 22 impairs freedom of contract. Whether or not BP 22 transgresses the


constitutional inhibition against imprisonment for debt.

Held:

Page 147
The freedom of contract which is constitutionally protected is freedom to enter into "lawful"
contracts. Contracts which contravene public policy are not lawful. Checks can not be categorized
as mere contracts. It is a commercial instrument which, in this modem day and age, has become a
convenient substitute for money; it forms part of the banking system and therefore not entirely free
from the regulatory power of the state.

The offense punished by BP 22 is the act of making and issuing a worthless check or a check that is
dishonored upon its presentation for payment. It is not the non-payment of an obligation which the
law punishes. The law is not intended or designed to coerce a debtor to pay his debt. The thrust of
the law is to prohibit, under pain of penal sanctions, the making of worthless checks and putting
them in circulation.

Page 148
EX POST FACTO LAWS

Art 3, Sec. 22. “No ex post facto law or bill of attainder shall be enacted.”

PEOPLE VS. FERRER


[48 SCRA 382; NOS.L-32613-14; 27 DEC 1972]

Facts:

Hon. Judge Simeon Ferrer is the Tarlac trial court judge that declared RA1700 or the Anti-
Subversive Act of 1957 as a bill of attainder. Thus, dismissing the information of subversion against
the following: 1.) Feliciano Co for being an officer/leader of the Communist Party of the Philippines
(CPP) aggravated by circumstances of contempt and insult to public officers, subversion by a band
and aid of armed men to afford impunity. 2.) Nilo Tayag and 5 others, for being members/leaders of
the NPA, inciting, instigating people to unite and overthrow the Philippine Government. Attended by
Aggravating Circumstances of Aid or Armed Men, Craft, and Fraud. The trial court is of opinion that
1.) The Congress usurped the powers of the judge 2.) Assumed judicial magistracy by pronouncing
the guilt of the CPP without any forms of safeguard of a judicial trial. 3.) It created a presumption of
organizational guilt by being members of the CPP regardless of voluntariness.

The Anti Subversive Act of 1957 was approved 20June1957. It is an act to outlaw the CPP and
similar associations penalizing membership therein, and for other purposes. It defined the
Communist Party being although a political party is in fact an organized conspiracy to overthrow the
Government, not only by force and violence but also by deceit, subversion and other illegal means.
It declares that the CPP is a clear and present danger to the security of the Philippines. Section 4
provided that affiliation with full knowledge of the illegal acts of the CPP is punishable. Section 5
states that due investigation by a designated prosecutor by the Secretary of Justice be made prior
to filing of information in court. Section 6 provides for penalty for furnishing false evidence. Section
7 provides for 2 witnesses in open court for acts penalized by prision mayor to death. Section 8
allows the renunciation of membership to the CCP through writing under oath. Section 9 declares
the constitutionality of the statute and its valid exercise under freedom if thought, assembly and
association.

Issue:

Whether or not RA1700 is a bill of attainder/ ex post facto law.

Whether or Not RA1700 violates freedom of expression.

Held:

The court holds the VALIDITY Of the Anti-Subversion Act of 1957.

A bill of attainder is solely a legislative act. It punishes without the benefit of the trial. It is the
substitution of judicial determination to a legislative determination of guilt. In order for a statute be
measured as a bill of attainder, the following requisites must be present: 1.) The statute specifies
persons, groups. 2.) the statute is applied retroactively and reach past conduct. (A bill of attainder
relatively is also an ex post facto law.)

In the case at bar, the statute simply declares the CPP as an organized conspiracy for the
overthrow of the Government for purposes of example of SECTION 4 of the Act. The Act applies
not only to the CPP but also to other organizations having the same purpose and their successors.
The Act’s focus is on the conduct not person.
Page 149
Membership to this organizations, to be UNLAWFUL, it must be shown that membership was
acquired with the intent to further the goals of the organization by overt acts. This is the element of
MEMBERSHIP with KNOWLEDGE that is punishable. This is the required proof of a member’s
direct participation. Why is membership punished. Membership renders aid and encouragement to
the organization. Membership makes himself party to its unlawful acts.

Furthermore, the statute is PROSPECTIVE in nature. Section 4 prohibits acts committed after
approval of the act. The members of the subversive organizations before the passing of this Act is
given an opportunity to escape liability by renouncing membership in accordance with Section 8.
The statute applies the principle of mutatis mutandis or that the necessary changes having been
made.

The declaration of that the CPP is an organized conspiracy to overthrow the Philippine Government
should not be the basis of guilt. This declaration is only a basis of Section 4 of the Act. The
EXISTENCE OF SUBSTANTIVE EVIL justifies the limitation to the exercise of “Freedom of
Expression and Association” in this matter. Before the enactment of the statute and statements in
the preamble, careful investigations by the Congress were done. The court further stresses that
whatever interest in freedom of speech and association is excluded in the prohibition of membership
in the CPP are weak considering NATIONAL SECURITY and PRESERVATION of DEMOCRACY.

The court set basic guidelines to be observed in the prosecution under RA1700. In addition to
proving circumstances/ evidences of subversion, the following elements must also be established:

1. Subversive Organizations besides the CPP, it must be proven that the organization
purpose is to overthrow the present Government of the Philippines and establish a
domination of a FOREIGN POWER. Membership is willfully and knowingly done by
overt acts.
2. In case of CPP, the continued pursuance of its subversive purpose. Membership is
willfully and knowingly done by overt acts.

The court did not make any judgment on the crimes of the accused under the Act. The Supreme
Court set aside the resolution of the TRIAL COURT.

BAYOT VS. SANDIGANBAYAN


[128 SCRA 383; NO.L-61776 TO NO.L-61861; 23 MAR 1984]

Facts:

Bayot is one of the several persons who was accused in more than 100 counts of estafa thru
falsification of Public documents before the Sandiganbayan. The said charges started from his
alleged involvement as a government auditor of the commission on audit assigned to the Ministry
of education and culture, with some other employees from the said ministry. The bureau of treasury
and the teacher’s camp in Baguio City for the preparation and encashment of fictitious TCAA
checks for the nom-existent obligations of the teacher’s camp resulting in damage to the
government of several millions. The 1st 32 cases were filed on july 25, 1987, while Bayot ran for
municipal mayor of Amadeo Cavite and was elected on January 1980. but on May 1980
Sandiganbayan promulgated a decision convicting the accused together with his other co-accused
in all but one of the thirty two cases filed against them.
Page 150
On Mach 16, 1982 Batas Pambansa Blg 195 was passed amending RA 3019.

Issue:

Whether or Not it would be violative of the constitutional guarantee against an ex post facto law.

Held:

The court finds no merit in the petitioner’s contention that RA 3019 as amended by Batas
Pambansa Blg 195, which includes the crime of estafa through falsification of Public Documents as
among crimes subjecting the public officer charged therewith with suspension from public office
pending action in court, is a penal provision which violates the constitutional prohibition against the
enactment of ex post facto law. Accdg to the RPC suspension from employment and public office
during trial shall not be considered as a penalty. It is not a penalty because it is not a result of a
judicial proceeding. In fact, if acquitted the official who is suspended shall be entitled to
reinstatement and the salaries and benefits which he failed to receive during suspension. And does
not violate the constitutional provision against ex post facto law.

The claim of the petitioner that he cannot be suspended because he is currently occupying a
position diffren tfrom that under which he is charged is untenable. The amendatory provision clearly
states that any incumbent public officer against whom any criminal prosecution under a valid
information under RA 3019 for any offense involving fraud upon the government or public funds or
property or whatever stage of execution and mode of participation shall be suspended from office.
The use of the word “office” applies to any office which the officer charged may be holding and not
only the particular office under which he was charged.

PEOPLE VS. SANDIGANBAYAN


[211 SCRA 241; G.R. NO. 101724; 3 JUL 1992]

Facts:

Two letter complaints were filed with the Tanodbayan by Teofilo Gelacio on October 28,1986 and
December 9, 1986, a political leader of Governor Valentina Plaza, wife of Congressman Democrito
Plaza of Agusan del Sur, shortly after private respondent had replaced Mrs. Plaza as OIC/provincial
Governor of Agusan del Sur on March 1986 The complaint questioned the issuance to Governor
Paredes, when he was still the provincial attorney in 1976 of a free patent title for a lot in the
Rosario public land subdivision in San Francisco, Agusan del Sur. He misrepresented to a Lands
Inspector of the Bureau of Lands that the lands subject herein are disposable lands, thereby
inducing said inspector to recommend approval of his application for free patent. On August 10,
1989 an information for violation of RA 3019 Anti-Graft and Corrupt Practices Act was then filed in
the Sandiganbayan after an ex parte preliminary investigation. A motion to quash the information
was filed by the private respondent contending among others that he is charged for an offence
which has prescribed. Said motion was granted. The crime was committed on January 21, 1976,
period of prescription was 10 years, therefore it has prescribed in 1986. Now the motion to quash
was being assailed.

Issue:

Whether or Not the motion to quash validly granted.

Held:

Page 151
Yes. RA 3019, being a special law the computation of the period for the prescription of the crime is
governed by Sec. 29 of Act No. 3326, which begins to run from the day of the commission of the
crime and not the discovery of it. Additionally, BP 195 which was approved on March 16, 1982,
amending Sec. 11 of RA 3019 by increasing ten to fifteen years of the period for the prescription or
extinguishment of a violation of RA 3019 may not be given retroactive application to the crime which
was committed by Paredes, as it is prejudicial to the accused. To apply BP 195 to Paredes would
make it an ex post facto law1 for it would alter his situation to his disadvantage by making him
criminally liable for a crime that had already been extinguished under the law existing when it was
committed.

Page 152
NON-IMPRISONMENT FOR DEBT

Art 3, Sec. 20. “No person shall be imprisoned for debt or non-payment of a poll tax.”

SERAFIN VS. LINDAYAG


[67 SCRA 166; ADM. MATTER. NO. 297-MJ; 30 SEPT 1975]

Facts:

Plaintiff failed to pay a simple indebtedness for P1500 Carmelito Mendoza, then municipal secretary
and his wife Corazon Mendoza and therefore an estafa case was filed against her. Complainant
admitted complaint. Now complainant filed a case against respondent Judge for not dismissing the
case and issuing a warrant of arrest as it falls on the category of a simple indebtedness, since
elements of estafa are not present. Further she contended that no person should be imprisoned for
non-payment of a loan of a sum of money. Two months after respondent dismissed plaintiff’s case.
(Judge here committed gross ignorance of law. Even if complainant desisted case was pursued.)

Issue:

Whether or Not there was a violation committed by the judge when it ordered the imprisonment of
plaintiff for non-payment of debt?

Held:

Yes. Since plaintiff did not commit any offense as, his debt is considered a simple loan granted by
her friends to her. There is no collateral or security because complainant was an old friend of the
spouses who lent the money and that when they wrote her a letter of demand she promised to pay
them and said that if she failed to keep her promise, they could get her valuable things at her home.
Under the Constitution she is protected. Judge therefore in admitting such a "criminal complaint"
that was plainly civil in aspects from the very face of the complaint and the "evidence" presented,
and issuing on the same day the warrant of arrest upon his utterly baseless finding "that the
accused is probably guilty of the crime charged," respondent grossly failed to perform his duties
properly.

LOZANO VS. MARTINEZ


[146 SCRA 323; NO.L-63419; 18 DEC 1986]

Facts:

A motion to quash the charge against the petitioners for violation of the BP 22 was made,
contending that no offense was committed, as the statute is unconstitutional. Such motion was
denied by the RTC. The petitioners thus elevate the case to the Supreme Court for relief. The
Solicitor General, commented that it was premature for the accused to elevate to the Supreme
Court the orders denying their motions to quash. However, the Supreme Court finds it justifiable to
intervene for the review of lower court's denial of a motion to quash.

Issue:

Whether or not BP 22 is constitutional as it is a proper exercise of police power of the State.

Held:

The enactment of BP 22 a valid exercise of the police power and is not repugnant to the
constitutional inhibition against imprisonment for debt.
Page 153
The offense punished by BP 22 is the act of making and issuing a worthless check or a check that is
dishonored upon its presentation for payment. It is not the non-payment of an obligation which the
law punishes. The law is not intended or designed to coerce a debtor to pay his debt.

The law punishes the act not as an offense against property, but an offense against public order.
The thrust of the law is to prohibit, under pain of penal sanctions, the making of worthless checks
and putting them in circulation. An act may not be considered by society as inherently wrong,
hence, not malum in se but because of the harm that it inflicts on the community, it can be outlawed
and criminally punished as malum prohibitum. The state can do this in the exercise of its police
power.

Page 154
INVOLUNTARY SERVITUDE

Art 3, Sec. 18. “(2) No involuntary servitude in any form shall exist except as a punishment for a
crime whereof the party shall have been duly convicted.”

CAUNCA VS. SALAZAR


[82 PHIL 851; NO.L-2690; 1 JAN 1949]

Facts:

This is an action for habeas corpus brought by Bartolome Caunca in behalf of his cousin Estelita
Flores who was employed by the Far Eastern Employment Bureau, owned by Julia Salazar,
respondent herein. An advanced payment has already been given to Estelita by the employment
agency, for her to work as a maid. However, Estelita wanted to transfer to another residence, which
was disallowed by the employment agency. Further she was detained and her liberty was
restrained. The employment agency wanted that the advance payment, which was applied to her
transportation expense from the province should be paid by Estelita before she could be allowed to
leave.

Issue:

Whether or Not an employment agency has the right to restrain and detain a maid without returning
the advance payment it gave?

Held:

An employment agency, regardless of the amount it may advance to a prospective employee or


maid, has absolutely no power to curtail her freedom of movement. The fact that no physical force
has been exerted to keep her in the house of the respondent does not make less real the
deprivation of her personal freedom of movement, freedom to transfer from one place to another,
freedom to choose one’s residence. Freedom may be lost due to external moral compulsion, to
founded or groundless fear, to erroneous belief in the existence of an imaginary power of an
impostor to cause harm if not blindly obeyed, to any other psychological element that may curtail the
mental faculty of choice or the unhampered exercise of the will. If the actual effect of such
psychological spell is to place a person at the mercy of another, the victim is entitled to the
protection of courts of justice as much as the individual who is illegally deprived of liberty by duress
or physical coercion.

Page 155
THE WRIT OF HABEAS CORPUS

Art 3, Sec. 15. “The privilege of the writ of habeas corpus shall not be suspended except in cases
of invasion or rebellion when the public safety requires it.”

LANSANG VS. GARCIA


[42 SCRA 448; L-33964; 11 Dec 1971]

Facts:

In the evening of August 21, 1971, at about 9 p.m., while the Liberal Party of the Philippines was
holding a public meeting at Plaza Miranda, Manila, for the presentation of its candidates in the
general elections scheduled for November 8, 1971, two hand grenades were thrown at the platform
where said candidates and other persons were. Eight persons were killed and many more injured.
Proclamation 889 was issued by the President suspending privilege of writ of habeas corpus stating
that there is a conspiracy of rebellion and insurrection in order to forcibly seize political power.
Petitions for writ of habeas corpus were filed by persons (13) who have been arrested without a
warrant.

It was stated that one of the safeguards of the proclamation was that it is to be applied to persons
caught in flagrante delicto. Incidentally, Proc. 889-A was issued as an amendment, inserting the
word “actually staging”. Proc. 889-B was also issued lifting the suspension of privilege in 27
provinces, 3 sub-provinces and 26 cities. Proc. 889-C was issued restoring the suspension in 13
provinces and cities(mostly in Mindanao). Proc. 889-D further lifted the suspension in 7 provinces
and 4 cities. Only 18 provinces and sub-provinces and 2 cities whose privilege was suspended.
Petitioners maintained that Proclamation No. 889 did not declare the existence of actual "invasion
insurrection or rebellion or imminent danger thereof, however it became moot and academic since it
was amended. Petitioners further contend that public safety did not require the issuance of
proclamations stating: (a) that there is no rebellion; (b) that, prior to and at the time of the
suspension of the privilege, the Government was functioning normally, as were the courts; (c) that
no untoward incident, confirmatory of an alleged July-August Plan, has actually taken place after
August 21, 1971; (d) that the President's alleged apprehension, because of said plan, is non-
existent and unjustified; and (e) that the Communist forces in the Philippines are too small and weak
to jeopardize public safety to such extent as to require the suspension of the privilege of the writ of
habeas corpus.

A resolution was issued by majority of the Court having tentatively arrived at a consensus that it
may inquire in order to satisfy itself of the existence of the factual bases for the proclamations. Now
the Court resolves after conclusive decision reached by majority.

Issue:

Whether or Not the authority to decide whether the exigency has arisen requiring suspension (of the
privilege of the writ of habeas corpus) belongs to the President and his decision is final and
conclusive upon the courts and upon all other persons.

Whether or Not public safety require the suspension of the privilege of the writ of habeas corpus
decreed in Proclamation No. 889-A.

Held:

The President has authority however it is subject to judicial review. Two conditions must concur for
the valid exercise of the authority to suspend the privilege to the writ (a) there must be "invasion,
insurrection, or rebellion" or "imminent danger thereof," and (b) "public safety" must require the
suspension of the privilege. President has three (3) courses of action: (a) to call out the armed
Page 156
forces; (b) to suspend the privilege of the writ of habeas corpus; and (c) to place the Philippines or
any part thereof under martial law. He had, already, called out the armed forces, proved inadequate.
Of the two other alternatives, the suspension of the privilege is the least harsh.

Petitioners contention that CPP-NPA has no ability, is negatived by the killing of 5 mayors, 20 barrio
captains and 3 chiefs of police; that there were fourteen (14) meaningful bombing incidents in the
Greater Manila Area in 1970. CPP has managed to infiltrate or establish and control nine major
labor organizations; has exploited the (11) major student or youth organizations; about thirty (30)
mass organizations actively advancing the CPP.

Page 157
RIGHTS OF THE ACCUSED

Art 3, Sec. 12. “(1) Any person under investigation for the commission of an offense shall have the
right to be informed of his right to remain silent and to have competent and independent counsel
preferably of his own choice. If the person cannot afford the services of counsel, he must be
provided with one. These rights cannot be waived except in writing and in the presence of counsel.
(2) No torture, force, violence, threat, intimidation, or any other means which vitiate the free will
shall be used against him. Secret detention places, solitary, incommunicado, or other similar forms
of detention are prohibited.
(3) Any confession or admission obtained in violation of this or Section 17 hereof shall be
inadmissible in evidence against him.
(4) The law shall provide for penal and civil sanctions for violations of this section as well as
compensation to and rehabilitation of victims of torture or similar practices, and their families.”

Art 3, Sec. 14. “(1) No person shall be held to answer for a criminal offense without due process of
law.
(2) In all criminal prosecutions, the accused shall be presumed innocent until the contrary is proved,
and shall enjoy the right to be heard by himself and counsel, to be informed of the nature and cause
of the accusation against him, to have a speedy, impartial, and public trial, to meet the witnesses
face to face, and to have compulsory process to secure the attendance of witnesses and the
production of evidence in his behalf. However, after arraignment, trial may proceed notwithstanding
the absence of the accused provided that he has been duly notified and his failure to appear is
unjustifiable.”

Art 3, Sec. 11. “Free access to the courts and quasi-judicial bodies and adequate legal assistance
shall not be denied to any person by reason of poverty.”

Art 3, Sec. 16. “All persons shall have the right to a speedy disposition of their cases before all
judicial, quasi-judicial, or administrative bodies.”

Art 3, Sec. 17. “No person shall be compelled to be a witness against himself.”

Art 3, Sec. 19. “(1) Excessive fines shall not be imposed, nor cruel, degrading or inhuman
punishment inflicted. Neither shall death penalty be imposed, unless, for compelling reasons
involving heinous crimes, the Congress hereafter provides for it. Any death penalty already imposed
shall be reduced to reclusion perpetua.
(2) The employment of physical, psychological, or degrading punishment against any prisoner
or detainee or the use of substandard or inadequate penal facilities under subhuman conditions
shall be dealt with by law.”

Art 3, Sec. 21. “No person shall be twice put in jeopardy of punishment for the same offense. If an
act is punished by a law and an ordinance, conviction or acquittal under either shall constitute a bar
to another prosecution for the same act.”

GAMBOA VS. CRUZ


[162 SCRA 642;L-56291; 27 JUN 1988]

Facts:

Petitioner was arrested for vagrancy without a warrant. During a line-up of 5 detainees including
petitioner, he was identified by a complainant to be a companion in a robbery, thereafter he was
charged. Petitioner filed a Motion to Acquit on the ground that the conduct of the line-up, without
notice and in the absence of his counsel violated his constitutional rights to counsel and to due
process. The court denied said motion. Hearing was set, hence the petition.
Page 158
Issue:

Whether or Not petitioner’s right to counsel and due process violated.

Held:

No. The police line-up was not part of the custodial inquest, hence, petitioner was not yet entitled, at
such stage, to counsel. He had not been held yet to answer for a criminal offense. The moment
there is a move or even an urge of said investigators to elicit admissions or confessions or even
plain information which may appear innocent or innocuous at the time, from said suspect, he should
then and there be assisted by counsel, unless he waives the right, but the waiver shall be made in
writing and in the presence of counsel.

On the right to due process, petitioner was not, in any way, deprived of this substantive and
constitutional right, as he was duly represented by a counsel. He was accorded all the opportunities
to be heard and to present evidence to substantiate his defense; only that he chose not to, and
instead opted to file a Motion to Acquit after the prosecution had rested its case. What due process
abhors is the absolute lack of opportunity to be heard.

PEOPLE VS. JUDGE AYSON


[175 SCRA 216; G.R. NO. 85215; 7 JUL 1989]

Facts:

Felipe Ramos was a ticket freight clerk of the Philippine Airlines, assigned at its Baguio City station.
It was alleged that he was involved in irregularities in the sales of plane tickets, the PAL
management notified him of an investigation to be conducted. That investigation was scheduled in
accordance with PAL's Code of Conduct and Discipline, and the Collective Bargaining Agreement
signed by it with the Philippine Airlines Employees' Association (PALEA) to which Ramos pertained.
A letter was sent by Ramos stating his willingness to settle the amount of P76,000. The findings of
the Audit team were given to him, and he refuted that he misused proceeds of tickets also stating
that he was prevented from settling said amounts. He proffered a compromise however this did not
ensue. Two months after a crime of estafa was charged against Ramos. Ramos pleaded not guilty.
Evidence by the prosecution contained Ramos’ written admission and statement, to which
defendants argued that the confession was taken without the accused being represented by a
lawyer. Respondent Judge did not admit those stating that accused was not reminded of his
constitutional rights to remain silent and to have counsel. A motion for reconsideration filed by the
prosecutors was denied. Hence this appeal.

Issue:

Whether or Not the respondent Judge correct in making inadmissible as evidence the admission
and statement of accused.

Held:

No. Section 20 of the 1987 constitution provides that the right against self-incrimination (only to
witnesses other than accused, unless what is asked is relating to a different crime charged- not
present in case at bar).

This is accorded to every person who gives evidence, whether voluntarily or under compulsion of
subpoena, in any civil, criminal, or administrative proceeding. The right is not to "be compelled to be
a witness against himself.” It prescribes an "option of refusal to answer incriminating questions and
not a prohibition of inquiry." the right can be claimed only when the specific question, incriminatory
in character, is actually put to the witness. It cannot be claimed at any other time. It does not give a
Page 159
witness the right to disregard a subpoena, to decline to appear before the court at the time
appointed, or to refuse to testify altogether. It is a right that a witness knows or should know. He
must claim it and could be waived.

Rights in custodial interrogation as laid down in miranda v. Arizona: the rights of the accused
include:

1) he shall have the right to remain silent and to counsel, and to be informed of such right.
2) nor force, violence, threat, intimidation, or any other means which vitiates the free will shall
be used against him.
3) any confession obtained in violation of these rights shall be inadmissible in evidence.

The individual may knowingly and intelligently waive these rights and agree to answer or make a
statement. But unless and until such rights and waivers are demonstrated by the prosecution at the
trial, no evidence obtained as a result of interrogation can be used against him.

PEOPLE VS. MAQUEDA


[242 SCRA 565; G.R. NO.112983; 22 MAR 1994]

Facts:

British Horace William Barker (consultant of WB) was slain inside his house in Tuba, Benguet while
his Filipino wife, Teresita Mendoza was badly battered with lead pipes on the occasion of a robbery.
Two household helpers of the victims identified Salvamante (a former houseboy of the victims) and
Maqueda as the robbers. Mike Tabayan and his friend also saw the two accused a kilometer away
from the house of the victims that same morning, when the two accused asked them for directions.

Maqueda was then arrested in Guinyangan, Quezon. He was taken to Calauag, Quezon where he
signed a Sinumpaang Salaysay wherein he narrated his participation in the crime. According to
SPO3 Molleno, he informed Maqueda of his constitutional rights before he signed such document.
Afterwards he was brought to the Benguet Provincial Jail. While he was under detention, Maqueda
filed a Motion to Grant Bail. He stated therein that "he is willing and volunteering to be a State
witness in the above entitled case, it appearing that he is the least guilty among the accused in this
case."

Maqueda also admitted his involvement in the commission of the robbery to Prosecutor Zarate and
to Salvosa.

Issue:

Whether or Not the trial court was correct in holding that the Sinumpaan Salaysay is admissible as
evidence.

Held:

No. The Sinumpaang Salaysay is inadmissible because it was in clear violation of the constitutional
rights of the accused. First, he was not informed of his right to remain silent and his right to
counsel. Second, he cannot be compelled to be a witness against himself. At the time of the
confession, the accused was already facing charges in court. He no longer had the right to remain
silent and to counsel but he had the right to refuse to be a witness and not to have any prejudice
whatsoever result to him by such refusal. And yet, despite his knowing fully well that a case had
already been filed in court, he still confessed when he did not have to do so.

The contention of the trial court that the accused is not entitled to such rights anymore because the
information has been filed and a warrant of arrest has been issued already, is untenable. The
exercise of the rights to remain silent and to counsel and to be informed thereof under Section 12(1)
Page 160
of the Bill of Rights are not confined to that period prior to the filing of a criminal complaint or
information but are available at that stage when a person is "under investigation for the commission
of an offense."

Pursuant to Section 12(3) of the Bill of Rights therefore, such extra-judicial admission is
inadmissible as evidence.

As to the admissions made by Maqueda to Prosecutor Zarate and Ray Dean Salvosa, the trial court
admitted their testimony thereon only to prove the tenor of their conversation but not to prove the
truth of the admission because such testimony was objected to as hearsay. Maqueda voluntarily
and freely made them to Prosecutor Zarate not in the course of an investigation, but in connection
with Maqueda's plea to be utilized as a state witness; and as to the other admission (Salvosa), it
was given to a private person therefore admissible.

Note: a distinction between a confession and admission has been made by the SC:
Admission of a party. — The act, declaration or omission of party as to a relevant fact may be given
in evidence against him.

Confession. — The declaration of an accused acknowledging his guilt of the offense charged, or of
any offense necessarily included therein, may be given in evidence against him.

PEOPLE VS. BANDULA


[232 SCRA 566; G.R. NO. 89223; 27 MAY 1994]

Facts:

Six armed men barged into the compound of Polo Coconut Plantation in Tanjay, Negros Oriental.
The armed men were identified by Security Guard, including accused. Salva and Pastrano, security
guards were hogtied and accused proceeded to the Atty. Garay, counsel of plantation. They
ransacked the place and took with them money and other valuables. Atty. Garay was killed.
Accused-appellant is charged with robbery with homicide along with 3 others who were acquitted for
insufficiency of evidence. Appellant was convicted.

Now, appellant argues that the extrajudicial confessions he and accused Dionanao executed suffer
from constitutional infirmities, hence, inadmissible in evidence considering that they were extracted
under duress and intimidation, and were merely countersigned later by the municipal attorney who,
by the nature of his position, was not entirely an independent counsel nor counsel of their choice.
Consequently, without the extrajudicial confessions, the prosecution is left without sufficient
evidence to convict him of the crime charged.

Issue:

Whether or Not extrajudicial confessions of appellant is admissible as evidence against him.

Held:

No. When accused-appellant Bandula and accused Dionanao were investigated immediately after
their arrest, they had no counsel present. If at all, counsel came in only a day after the custodial
investigation with respect to accused Dionanao, and two weeks later with respect to appellant
Bandula. And, counsel who supposedly assisted both accused was Atty. Ruben Zerna, the
Municipal Attorney of Tanjay. On top of this, there are telltale signs that violence was used against
the accused. Certainly, these are blatant violations of the Constitution which mandates in
Sec. 12, Art. III. Irregularities present include:

Page 161
1. The investigators did not inform the accused of their right to remain silent and to have
competent and independent counsel, preferably of their own choice, even before attempting
to elicit statements that would incriminate them.
2. Investigators continuously disregard the repeated requests of the accused for medical
assistance. Reason for Accused Sedigo’s "black eye" which even
Pat. Baldejera admitted is not established, as well as Bandula’s fractured rib.
3. Counsel must be independent. He cannot be a special counsel, public or private prosecutor,
counsel of the police, or a municipal attorney whose interest is admittedly adverse to the
accused.

PEOPLE VS. LUCERO


[244 SCRA 425; G.R. NO.97936; 29 MAY 1995]

Facts:

Alejandro Lucero, Bienvenido Echavez, Balbino Echavez, Peter Doe, Richard Doe and John Doe
were charged with the crime of robbery with homicide.

The prosecution:

Accused-appellant (alighted from a gray-reddish car), armed with handgun, blocked the way of the
said complainant who was on board a Mercedes Benz passing along Road 14, Mindanao Avenue,
Pag-asa, QC, rob and carry away cash money; one gold necklace with cross pendant, 7 karat; one
gold Rolex watch; one 3 karat gold ring; one 2 karat gold ring, domino style; one solid gold bracelet;
all worth P363,600.00, belonging to DR. DEMETRIO Z. MADRID. Accused shot LORENZO
BERNALES y ALERIA, a driver of the said offended party, thus inflicting upon him mortal wounds,
which resulted to the instantaneous death of ALERIA.

Only the accused Echavez brothers and Alejandro Lucero were apprehended.

When Lucero told him that he had no lawyer, in due time, Atty. Diosdado Peralta conferred with
Lucero. He apprised Lucero of his constitutional rights. He observed no reaction from Lucero.
Nonetheless, Atty. Peralta gathered the impression that Lucero understood his advice.

When the investigator started asking the preliminary questions, Atty. Peralta left to attend the wake
of his friend. The next morning, Lucero was accompanied by CIS agents to Atty. Peralta's house.
The extrajudicial statement of Lucero was presented to Atty. Peralta. It was already signed by
Lucero.

The three accused denied complicity in the crime charged.

Appellant Lucero's defense is alibi. He testified that he was at his house in Caloocan City.

He said he was surprised when several unidentified men accosted him while he was walking
towards his house. They chased him, handcuffed and blindfolded him and pushed him into a jeep.
He was blindfolded the whole night and did not know where he was taken. The men turned out to be
police officers.
The next day, he learned he was in Camp Crame. He claimed that he was tortured. He was not
informed of the offense for which he was being investigated. Neither did they reveal the identity of
the complainant.

Lucero denied knowing Dr. Madrid, the Echavez brothers and the other accused in this case. He
said he only met Dr. Madrid at the CIS Office during the police line-up. He was made to line-up four
(4) times before Dr. Madrid finally identified him on the fourth time.

Page 162
Lucero also claimed he signed the extrajudicial confession under duress. He denied engaging the
services of Atty, Peralta. He likewise confirmed that Atty. Peralta was not present during his actual
custodial interrogation.

After trial, the court a quo acquitted the Echavez brothers for insufficient evidence. The trial court,
however, convicted accused Lucero GUILTY as principal by direct participation of Robbery with
Homicide and sentenced to suffer an imprisonment term of RECLUSION PERPETUA.

Issue:

Whether or Not the lower court erred in convicting accused-appellant.

Held:

Appellant's conviction cannot be based on his extrajudicial confession.

Constitution requires that a person under investigation for the commission of a crime should be
provided with counsel. The Court have constitutionalized the right to counsel because of hostility
against the use of duress and other undue influence in extracting confessions from a suspect. Force
and fraud tarnish confessions can render them inadmissible.

The records show that Atty. Peralta, who was not the counsel of choice of appellant. Atty. Peralta
himself admitted he received no reaction from appellant although his impression was that appellant
understood him. More so, it was during his absence that appellant gave an uncounselled
confession.

Constitution requires the right to counsel, it did not mean any kind of counsel but effective and
vigilant counsel. The circumstances clearly demonstrate that appellant received no effective
counseling from Atty. Peralta.

Whereof, Decision convicting appellant Alejandro Lucero y Cortel is hereby reversed.

PEOPLE VS. AGUSTIN


[240 SCRA 541; G.R. NO. 110290; 25 JAN 1995]

Facts:

Dr. Bayquen, a dentist, together with his son, Anthony; Anthony's girlfriend, Anna Theresa; his
daughter, Dominic; and Danny, a family friend, were on their way aboard their Brasilia to the
doctor's residence at Malvar Street, Baguio City. While they were cruising along Malvar Street and
nearing the Baptist church, a man came out from the right side of a car parked about two meters to
the church. The man approached the Brasilia, aimed his armalite rifle through its window, and fired
at the passengers. The Brasilia swerved and hit a fence. The gunman immediately returned to the
parked car which then sped away. All those in the car were hit and Dr. Bayquen and Anna Theresa
died on the spot. Dominic was bale to get out of the Brasilia to run to the Alabanza store where she
telephoned her mother. Later, she and her mother brought her father and Anthony to the hospital.
Danny went home and was then brought to the Hospital for treatment.

Accused Quiaño, an alleged former military agent who had been picked up by the police authorities,
confessed during the investigation conducted by Baguio City Fiscal Erdolfo Balajadia in his office
that he was the triggerman. He implicated Abenoja, Jr., who engaged him to kill Dr. Bayquen for a
fee, Cartel, who provided the armalite, and a certain "Jimmy." During the investigation, Wilfredo
Quiaño was assisted by Atty. Reynaldo Cajucom. Stenographic notes of the proceedings during the
investigation as transcribed with the sworn statement of Quiaño was signed, with the assistance of
Atty. Cajucom, and swore to before City Fiscal Balajadia. The following day, Agustin was
apprehended, and was investigated and was afforded the privileges like that of Quijano. Agustin’s
Page 163
defense interpose that he was forced to admit involvement at gunpoint in the Kennon Road. He
further declared that although he was given a lawyer, Cajucom (a law partner of the private
prosecutor), he nevertheless, asked for his uncle Atty. Oliver Tabin, and that Atty. Cajucom
interviewed him from only two minutes in English and Tagalog but not in Ilocano, the dialect he
understands. The promise that he would be discharged as a witness did not push through since
Quijano escaped. However the RTC convicted him, since conspiracy was established. Hence the
appeal.

Issue:

Whether or Not accused-appellant’s extrajudicial statements admissible as evidence.

Held:

No. Extrajudicial statement is not extrajudicial confession. In a confession, there is an


acknowledgment of guilt of the accused, while an admission is a statement direct or implied of facts
pertinent to the issue. The rule on inadmissibility, however expressly includes admissions, not just
confessions.The extrajudicial admission of the appellant, contained in twenty-two pages appear to
be signed by him and Atty. Cajucom but for reasons not explained in the records, the transcript of
the notes which consists of twelve pages was not signed by the appellant. Since the court cannot
even read or decipher the stenographic notes it cannot be expected that appellant, who is a farmer
and who reached only the fourth grade, to read or decipher its contents. The appellant, therefore
was deprived of his rights under Section 12(1), Article III of the Constitution. Firstly, he was not fully
and properly informed of his rights. The appellant was not explicitly told of his right to have a
competent and independent counsel of his choice, specifically asked if he had in mind any such
counsel and, if so, whether he could afford to hire his services, and, if he could not, whether he
would agree to be assisted by one to be provided for him. He was not categorically informed that he
could waive his rights to remain silent and to counsel and that this waiver must be in writing and in
the presence of his counsel. He had, in fact, waived his right to remain silent by agreeing to be
investigated. Yet, no written waiver of such right appears in the transcript and no other independent
evidence was offered to prove its existence. In short, after the appellant said that he wanted to be
assisted by counsel, the City fiscal, through suggestive language, immediately informed him that
Atty. Cajucom was ready to assist him. Moreso said counsel is not independent since he is an
associate of the private prosecutor.

PEOPLE VS. BOLANOS


[211 SCRA 262; G.R. NO. 101808; 3 JUL 1992]

Facts:

Oscar Pagdalian was murdered in Marble Supply, Balagtas Bulacan. According to Pat. Rolando
Alcantara and Francisco Dayao, deceased was with two companions on the previous night, one of
whom the accused who had a drinking spree with the deceased. When they apprehended the
accused they found the firearm of the deceased on the chair where the accused was allegedly
seated. They boarded accused along with Magtibay, other accused on the police vehicle and
brought them to the police station. While in the vehicle Bolanos admitted that he killed the
deceased. RTC convicted him hence the appeal.

Issue:

Whether or Not accused-appellant deprived of his constitutional right to counsel.

Held:

Yes. Being already under custodial investigation while on board the police patrol jeep on the way to
the Police Station where formal investigation may have been conducted, appellant should have
Page 164
been informed of his Constitutional rights under Article III, Section 12 of the 1987 Constitution, more
particularly par. 1 and par. 3.

PEOPLE VS. MACAM


[238 SCRA 306; G.R. NOS. 91011-12; 24 NOV 1994]

Facts:

Prosecution’s version:

On Aug 18,1987, Eduardo Macam, Antonio Cedro, Eugenio Cawilan Jr., Danilo Roque and Ernesto
Roque went to the house of Benito Macam (uncle of Eduardo Macam) located at 43 Ferma Road
QC. Upon the arrival of the accused, Benito invited the former to have lunch. Benito asked his maid
Salvacion Enrera to call the companions of Eduardo who were waiting in a tricycle outside the
house. A. Cedro, E. Cawilan and D. Roque entered the house while E. Roque remained in the
tricycle. After all the accused had taken their lunch, Eduardo Macam grabbed the clutch bag of
Benito Macam and pulled out his uncle’s gun then declared a hold-up. They tied up the wife (Leticia
Macam), children, maid (Salvacion) and Nilo Alcantara and brought them to the room upstairs. After
a while Leticia was brought to the bathroom and after she screamed she was stabbed and killed by
A. Cedro. Benito, Nilo and Salvacion was also stabbed but survived. The total value of the items
taken was P536, 700.00.

Defense’s version:

Danilo Roque stated that he being a tricycle driver drove the 4 accused to Benito’s house for a fee
of P50.00. Instead of paying him, he was given a calling card by Eduardo Macam so that he can be
paid the following day. Upon arriving, he went with the accused inside the house to have lunch.
Thereafter he washed the dishes and swept the floor. When Eugenio Cawilan pulled a gun and
announced the hold-up, he was asked to gather some things and which he abided out of fear. While
putting the said thins inside the car of Benito (victim) he heard the accused saying “kailangan
patayin ang mga taong yan dahil kilala ako ng mga yan”. Upon hearing such phrase he escaped
and went home using his tricycle. He also testified that his brother Ernesto Roque has just arrived
from the province and in no way can be involved in the case at bar. On the following day, together
with his brother, they went to the factory of the Zesto Juice (owned by the father of Eduardo
Macam) for him to get his payment (50.00) . He and his brother was suddenly apprehended by the
security guards and brought to the police headquarters in Q.C. They were also forced to admit
certain things.

After which, he together with all the accused, in handcuffs and bore contusions on their faces
caused by blows inflicted in their faces during investigation, was brought to the QC General Hospital
before each surviving victims and made to line-up for identification. Eugenio Cawilan was also
charged with Anti-fencing Law but was acquitted in the said case.

Issue:

Whether or Not their right to counsel has been violated. WON the arrest was valid. WON the
evidence from the line-up is admissible.

Held:

It is appropriate to extend the counsel guarantee to critical stages of prosecution even before trial. A
police line-up is considered a “critical” stage of the proceedings. Any identification of an
uncounseled accused made in a police line-up is inadmissible. HOWEVER, the prosecution did not
present evidence regarding appellant’s identification at the line-up. The witnesses identified the
accused again in open court. Also, accused did not object to the in-court identification as being
tainted by illegal line-up.
Page 165
The arrest of the appellants was without a warrant. HOWEVER, they are estopped from questioning
the legality of such arrest because they have not moved to quash the said information and therefore
voluntarily submitted themselves to the jurisdiction of the trial court by entering a plea of not guilty
and participating in trial.

The court believed the version of the prosecution. Ernesto Roque, while remaining outside the
house served as a looked out.

Wherefore, decision of lower court is Affirmed. Danilo Roque and Ernesto Roque is guilty of the
crime of robbery with homicide as co-conspirators of the other accused to suffer reclusion perpetua.

Things taken: 2 toygun, airgun riffle, CO2 refiller, TV, betamax tapes, betamax rewinder, Samsonite
attache case, typewriter, chessboard, TOYOTA Crown Car Plate No. CAS-997, assorted
jewelry. .22 gun and money.

PEOPLE VS. DY
[158 SCRA 111; G.R. 74517; 23 FEB 1988]

Facts:

Pat. Padilla reported along with Benny Dy, with caliber .38 as suspect to the shooting incident at
"Benny's Bar," at Sitio Angol, Manoc-Manoc Malay, Aklan (Boracay) situated on the Island which
caused the death of Christian Langel Philippe, tourist, 24 years old and a Swiss nationale. He was
charged with the Murder With the Use of Unlicensed firearms. Appellant alleges that he carried the
victim to the shore to be brought to the hospital to save the latter, and who facilitated the surrender
to Pat. Padilla a gun which his helper found the following morning while cleaning the bar. Accused
posted bail which was granted. The accused denied having made any oral confession alleging that
he went to Pat. Padilla not to report the incident but to state that a boy helper in the bar had found a
gun on the sand floor while cleaning and that Pat. Padilla picked up the gun from the bar at his
request. The Accused argues that even if he did make such a confession, the same would be
inadmissible in evidence. He was found guilty in the RTC. Hence the appeal.

Issue:

Whether or Not the lower court correct in saying that the constitutional procedure on custodial
interrogation is not applicable in the instant case.

Held:

YES. Appellant's assertion that the gun he had surrendered was merely found by a boy helper while
cleaning the bar deserves no credence for, if it were so, it would have been absurd for him to have
placed himself under police custody in the early morning after the incident. Sworn Complaint for
"Murder with Use of Unlicensed Firearm" signed by the Chief of Police also attests to Appellant's
oral confession. That Complaint forms part of the record of the proceedings before the Municipal
Circuit Trial Court of Buruanga, Aklan, and is prima facie evidence of the facts therein stated.
Appellant's voluntary surrender implies no violation as "no warrant of arrest is issued for the
apprehension of the accused for the reason that he is already under police custody before the filing
of the complaint." What was told by the Accused to Pat, Padilla was a spontaneous statement not
elicited through questioning, but given in ordinary manner. No written confession was sought to be
presented in evidence as a result of formal custodial investigation.

NAVALLO VS. SANDIGANBAYAN


[234 SCRA 177; G.R. NO. 97214; 18 JUL 1994]
Page 166
Facts:

Accused was the Collecting and Disbursing Officer of the Numancia National Vocational School,
which school is also located at del Carmen, Surigao del Norte. His duties included the collection of
tuition fees, preparation of vouchers for salaries of teachers and employees, and remittance of
collections exceeding P500.00 to the National Treasury. An information for malversation of public
funds was filed. A warrant of arrest was issued, but accused-petitioner could not be found. on 10
December 1978, Presidential Decree No. 1606 took effect creating the Sandiganbayan and
conferring on it original and exclusive jurisdiction over crimes committed by public officers embraced
in Title VII of the Revised Penal Code. On 15 November 1984, Navallo was finally arrested. He was
released on provisional liberty upon the approval of his property bail bond. When arraigned by the
RTC on 18 July 1985, he pleaded not guilty. Upon motion of the prosecution, the RTC transferred
the case and transmitted its records to the Sandiganbayan. Special Prosecutor Luz L. Quiñones-
Marcos opined that since Navallo had already been arraigned before the case was transferred to
the Sandiganbayan, the RTC should continue taking cognizance of the case. The matter was
referred to the Office of the Ombudsman which held otherwise. The information was then docketed
with the Sandiganbayan. A new order for Navallo's arrest was issued by the Sandiganbayan. The
warrant was returned with a certification by the RTC Clerk of Court that the accused had posted a
bail bond. Navallo filed a motion to quash, contending (1) that the Sandiganbayan had no
jurisdiction over the offense and the person of the accused and (2) that since the accused had
already been arraigned by the RTC, the attempt to prosecute him before the Sandiganbayan would
constitute double jeopardy. However this was denied and trial ensued and he was found guilty.

Issue:

Whether or Not the constitutional right against double jeopardy and in custodial investigations in
favor of the accused violated.

Held:

No. Double jeopardy requires the existence of the following requisites:

(1) The previous complaint or information or other formal charge is sufficient in form and
substance to sustain a conviction;
(2) The court has jurisdiction to try the case;
(3) The accused has been arraigned and has pleaded to the charge; and
(4) The accused is convicted or acquitted or the case is dismissed without his express consent.

The RTC was devoid of jurisdiction when it conducted an arraignment of the accused which by then
had already been conferred on the Sandiganbayan. Moreover, neither did the case there terminate
with conviction or acquittal nor was it dismissed.

No. Appellant is not in custodial investigation. A person under a normal audit examination is not
under custodial investigation. An audit examiner himself can hardly be deemed to be the law
enforcement officer contemplated in the above rule. In any case, the allegation of his having been
"pressured" to sign the Examination Report prepared by Dulguime (examined cash, as ordered by
Espino, the provincial auditor) appears to be belied by his own testimony.

PEOPLE VS. ALICANDO


[251 SCRA 293; G.R. NO. 117487; 2 DEC 1995]

Facts:

Appellant was charged with the crime of rape with homicide of Khazie Mae Penecilla, a minor, four
years of age, choking her with his right hand. The incident happened after appellant drank liquor. A
Page 167
neighbor, Leopoldo Santiago found the victim’s body and the parents and police were informed.
Appellant was living in his uncle's house some five arm's length from Penecilla's house. Appellant
was arrested and interrogated by PO3 Danilo Tan. He verbally confessed his guilt without the
assistance of counsel. On the basis of his uncounselled verbal confession and follow up
interrogations, the police came to know and recovered from appellant's house, Khazie Mae's green
slippers, a pair of gold earrings, a buri mat, a stained pillow and a stained T-shirt all of which were
presented as evidence for the prosecution. He was arraigned with the assistance of Atty. Rogelio
Antiquiera of the PAO. Appellant pleaded guilty. The RTC convicted him. Hence an automatic
review for the imposition of death penalty.

Issue:

Whether or Not the death penalty proper.

Held:

No. The records do not reveal that the Information against the appellant was read in the language or
dialect known to him. The Information against the appellant is written in the English language. It is
unknown whether the appellant knows the English language. Neither is it known what dialect is
understood by the appellant. Nor is there any showing that the Information couched in English was
translated to the appellant in his own dialect before his plea of guilt. The RTC violated section 1(a)
of Rule 116, the rule implementing the constitutional right of the appellant to be informed of the
nature and cause of the accusation against him. It also denied appellant his constitutional right to
due process of law. It is urged that we must presume that the arraignment of the appellant was
regularly conducted. When life is at stake, we cannot lean on this rebuttable presumption. There
could be no presumption. The court must be sure.

The trial court violated section 3 of Rule 116 when it accepted the plea of guilt of the appellant. Said
section requires that the court shall conduct a searching inquiry the voluntariness and full
comprehension of the consequences of his plea and require the prosecution to prove his guilt and
the precise degree of culpability. The accused may also present evidence in his behalf. The trial
court simply inquired if appellant had physical marks of maltreatment. It did not ask the appellant
when he was arrested, who arrested him, how and where he was interrogated, whether he was
medically examined before and after his interrogation, etc. It limited its efforts trying to discover late
body marks of maltreatment as if involuntariness is caused by physical abuse alone.

Further, there are physical evidence to prove Khazie was raped. These consists of a pillow with
bloodstains in its center 14 and the T-shirt 15 of the accused colored white with bloodstains on its
bottom. These physical evidence are evidence of the highest order. They strongly corroborate the
testimony of Luisa Rebada that the victim was raped.These are inadmissible evidence for they were
gathered by PO3 Danilo Tan of the Iloilo City PNP as a result of custodial interrogation where
appellant verbally confessed to the crime without the benefit of counsel.

PEOPLE VS. DE GUZMAN


[224 SCRA 93; G.R. NOS. 98321-24; 30 JUN 1993]

Facts:

All the accused were charged before the Regional Trial Court of Cebu with three counts of murder
and one count of frustrated murder in four Informations. The victim Jose Bantug was found with
gunshots in the head, body, and skull. The other three informations charged them with the murder
of Francisco Carteciano y Sorilla and Antonio S. Carteciano, and the frustrated murder of Lorna V.
Carteciano. The other 8 accused were acquitted on the ground of reasonable doubt, while Victor
Nuñez was found guilty. The facts shown by evidence are: One morning, Major Antonio Carteciano
was driving his private jeep Camp General Arcadio Maxilom in Lahug, Cebu City where he was
stationed as medical officer of the PC/INP Provincial Command. In the front seat with him is his wife
Page 168
Lorna, and at the backseat are his mother in law, son, brother Francisco, neighbor Bantug, and
Bantug’s son. Near the intersection, gunshots were heard from the left side of the street. Major
Carteciano took his .45 cal pistol and fired. However, gunshots were fired in succession, and Major
Carteciano, his brother Francisco, Jose Bantug, and his wife Lorna were hit. When the jeep
stopped, several gunmen approached them. Nuñez demanded Lorna to give Nuñez her husband’s
pistol. Lorna asked to take her valuables instead. Then, Nuñez shot Major Carteciano’s head point
blank. Then the gunmen hijacked another jeep and took off. Lorna, her mother Juanita Ricaplaza,
and her son Reiser Carteciano positively identified the accused. Lorna identified Nuñez as the one
who shot her husband. Nuñez claimed that his arrest was illegal and that he was deprived of his
right to counsel when he was subjected to a paraffin test without the assistance of counsel.

Issue:

Whether or not the accused Nuñez’s constitutional right was violated

Held:

No. Nuñez pleaded not guilty at the arraignment. Therefore, he is estopped from questioning the
validity of his arrest. Furthermore, the illegal arrest of an accused is not sufficient cause for setting
aside a valid judgment rendered upon a sufficient complaint after trial free from error. The witnesses
also positively identified the accused, so he cannot question the credibility of the witnesses.
Regarding his right to counsel, the Supreme Court held that-- the right to counsel attaches only
upon the start of an investigation, that is, when the investigating officer starts to ask questions to
elicit information and/or confessions or admissions from the accused. At such point or stage, the
person being interrogated must be assisted by counsel to avoid the pernicious practice of extorting
false or coerced admissions or confessions from the lips of the person undergoing interrogation. In
the case at bar, when accused was subjected to a paraffin test, he was not then under custodial
investigation. Accused-appellant also argued that since his co-accused were acquitted, then their
acquittal negates conspiracy among them, and he should not be convicted with the charges filed.
However, the Court held that conspiracy was still proven by the evidence, and the other co-accused
were acquitted only because there was reasonable doubt. Therefore, accused-appellant is still
convicted of the four charges against him.

We, therefore, find that the conviction of accused-appellant for the crimes charged has been
established beyond reasonable doubt and the penalty imposed is in accordance with law. However,
the civil indemnity imposed by the trial court should be increased to P50,000 in conformity with our
recent rulings on the matter.

WHEREFORE, except for the modification that the civil indemnity to be paid by accused-appellant
Victor Nuñez, Jr. to the heirs of each victim who died is hereby increased to P50,000, the appealed
decision is hereby affirmed in all other respects, with costs against accused-appellant

PEOPLE VS. JUDGE DONATO


[198 SCRA 130; G.R. NO.79269; 5 JUN 1991]

Facts:

Private respondent and his co-accused were charged of rebellion on October 2, 1986 for acts
committed before and after February 1986. Private respondent filed with a Motion to Quash alleging
that: (a) the facts alleged do not constitute an offense; (b) the Court has no jurisdiction over the
offense charged; (c) the Court has no jurisdiction over the persons of the defendants; and (d) the
criminal action or liability has been extinguished. This was denied. May 9, 1987 Respondent filed a
petition for bail, which was opposed that the respondent is not entitled to bail anymore since
rebellion became a capital offense under PD 1996, 942 and 1834 amending ART. 135 of RPC. On 5
June 1987 the President issued Executive Order No. 187 repealing, among others, P.D. Nos. 1996,
Page 169
942 and 1834 and restoring to full force and effect Article 135 of the Revised Penal Code as it
existed before the amendatory decrees. Judge Donato now granted the bail, which was fixed at
P30,000.00 and imposed a condition that he shall report to the court once every two months within
the first ten days of every period thereof. Petitioner filed a supplemental motion for reconsideration
indirectly asking the court to deny bail to and to allow it to present evidence in support thereof
considering the "inevitable probability that the accused will not comply with this main condition of his
bail. It was contended that:

1. The accused has evaded the authorities for thirteen years and was an escapee from
detention when arrested; (Chairman of CPP-NPA)
2. He was not arrested at his residence as he had no known address;
3. He was using the false name "Manuel Mercado Castro" at the time of his arrest and
presented a Driver's License to substantiate his false identity;
4. The address he gave "Panamitan, Kawit, Cavite," turned out to be also a false address;
5. He and his companions were on board a private vehicle with a declared owner whose
identity and address were also found to be false;
6. Pursuant to Ministry Order No. 1-A dated 11 January 1982 , a reward of P250,000.00 was
offered and paid for his arrest.

This however was denied. Hence the appeal.

Issue:

Whether or Not the private respondent has the right to bail.

Held:

Yes. Bail in the instant case is a matter of right. It is absolute since the crime is not a capital offense,
therefore prosecution has no right to present evidence. It is only when it is a capital offense that the
right becomes discretionary. However it was wrong for the Judge to change the amount of bail from
30K to 50K without hearing the prosecution.

Republic Act No. 6968 approved on 24 October 1990, providing a penalty of reclusion perpetua to
the crime of rebellion, is not applicable to the accused as it is not favorable to him.

Accused validly waived his right to bail in another case(petition for habeas corpus). Agreements
were made therein: accused to remain under custody, whereas his co-detainees Josefina Cruz and
Jose Milo Concepcion will be released immediately, with a condition that they will submit
themselves in the jurisdiction of the court. Said petition for HC was dismissed. Bail is the security
given for the release of a person in custody of the law. Ergo, there was a waiver. We hereby rule
that the right to bail is another of the constitutional rights which can be waived. It is a right which is
personal to the accused and whose waiver would not be contrary to law, public order, public policy,
morals, or good customs, or prejudicial to a third person with a right recognized by law.

CARPIO VS. MAGLALANG


[196 SCRA 41; G.R. NO. 78162; 19 APR 1991]

Facts:

On January 8, 1987, information for the murder of Mayor Jose Payumo of Dinalupihan Bataan was
filed against Escaño and ten other unindentified persons by the provincial fiscal in the RTC of
Bataan at Balanga. Four days later, the Acting Executive Judge of said court issued an order of
arrest against Escaño recommending no bail for his provisional liberty. Pat. Cesar Diego who acted
on the warrant returned to the court with a certification issued by NBI agent Gonzales, stating
therein that accused was still under investigation.

Page 170
Through counsel Rolando T. Cainoy, Escaño filed in court an urgent ex-parte motion for his
commitment at the provincial jail of Bataan on the ground that he wanted to be where his family and
counsel could have easy access to him. He alleged therein that his detention at the NBI
headquarters in Manila was irregular and in defiance of the warrant of arrest issued by the court.
This was granted.

A motion for reconsideration was filed by Director Carpio stating that the NBI needed physical
custody of Escaño for the identification of the other accused in the case who were still the objects of
a manhunt by NBI agents; that in view of the finding of NBI agents that the other accused and
suspects in the case were subversive elements or members of the New People's Army, it was for
the best interest of Escaño that he be detained at the NBI lock-up cell where security measures
were adequate; and that the NBI would produce the person of Escaño before the court whenever
required and every time that there would be a hearing on the case. However another motion was
executed by Escaño stating that he now wants to be detained in the NBI, alleging that he did not
authorize his counsel to execute the first motion. Also, Escaño's counsel Rolando T. Cainoy filed an
application for bail stating that Escaño was arrested by NBI agents on December 7, 1986 without a
warrant having been presented to him and that since then he had been detained in the lock-up cell
of the NBI; that said agents, also without a warrant, searched his house when he was arrested; that
he was subjected to inhuman torture and forced to admit participation in the killing of Mayor Payumo
and to implicate other persons, and that during the custodial investigation, he was not represented
by counsel. In opposing said application, the public prosecutor averred that the accused was
charged with a capital offense for which no bail may be availed of, that the reasons advanced in
said application would be overcome by strong and sufficient evidence; and that during the custodial
investigation, he was represented by counsel. The court granted the application for bail fixing the
same at P30,000, having found no sufficient evidence against accused. Director Carpio was
ordered to justify his actions and so as not to be considered in contempt.

Issue:

Whether or Not the order granting right to bail was proper.

Held:

No. The order granting bail had been rendered moot not only by the fact that he had been released
from NBI custody, but also because Escaño jumped bail and did not appear on the date set for his
arraignment. Notwithstanding, the Court resolved the issue of the legality of the order granting bail
to Escaño. Although the right to bail is principally for the benefit of the accused, in the judicial
determination of the availability of said right, the prosecution should be afforded procedural due
process. Thus, in the summary proceeding on a motion praying for admission to bail, the
prosecution should be given the opportunity to present evidence and, thereafter, the court should
spell out at least a resume of the evidence on which its order granting or denying bail is based.
Otherwise, the order is defective and voidable. In the case at bar the RTC erred in not summarizing
the factual basis of its order granting bail, the court merely stated the number of prosecution
witnesses but not their respective testimonies, and concluded that the evidence presented by the
prosecution was not "sufficiently strong" to deny bail to Escaño.

The facts, however, that Mayor Payumo was killed on August 20, 1986 when the 1973 Constitution
allowing the death penalty was still in force and that the application for bail was made on March 5,
1987 during the effectivity of the 1987 Constitution which abolished the death penalty, should not
have gotten in the way of resolving the application for bail in accordance with the Constitution and
procedural rules. Section 13, Article III of the Constitution explicitly provides that "(a)ll persons,
except those charged with offenses punishable by reclusion perpetua when evidence of guilt is
strong, shall before conviction, be bailable by sufficient sureties, or be released on recognizance as
may be provided by law." As the phrase "capital offenses" has been replaced by the phrase
"offenses punishable by reclusion perpetua, 25 crimes punishable by reclusion perpetua instead of
those punishable by the death penalty, when evidence of guilt is strong, are the exceptions to the
rule that the right to bail should be made available to all accused. As the court itself acknowledged
Page 171
in its order of April 2, 1987 that "capital punishment" in Section 4, Rule 114 has been amended to
reclusion perpetua, the court should have proceeded accordingly: i.e., resolved the application for
bail pursuant to Section 13, Article III of the Constitution. It did not have to invoke the abolition of the
death penalty and the lack of legislative enactment restoring it in justifying the grant of bail. All it had
to do was to determine whether evidence of guilt is strong in the light of the provision of Section 13,
Article III.

The RTC has the discretion in the consideration of the strength of the evidence at hand. However,
in the exercise of said discretion, the court is controlled by the following: first, the applicable
provisions of the Constitution and the statutes; second, by the rules which this Court may
promulgate; and third, by those principles of equity and justice that are deemed to be part of the
laws of the land. 27 The lower court not only failed to properly apply the pertinent provisions of the
Constitution and the Rules but it also disregarded equity and justice by its failure to take into
account the factual milieu surrounding the detention of Escaño

PEOPLE VS. FORTES


[223 SCRA 619; G.R. NO. 90643; 25 JUN 1993]

Facts:

Agripino Gine of Barangay Naburacan, Municipality of Matnog, Province of Sorsogon, accompanied


his 13-year old daughter, Merelyn, to the police station of the said municipality to report a rape
committed against the latter by the accused. Following this, the accused was apprehended and
charged. A bond of P25000 was granted for accused’s provisional release. The MCTC found him
guilty. An appeal to RTC was filed, the request for the fixing of bond was denied. Now accused
assails denial of bail on the ground that the same amounted to an undue denial of his constitutional
right to bail.

Issue:

Whether or Not the accused’s right to bail violated.

Held:

No. It is clear from Section 13, Article III of the 1987 Constitution and Section 3, Rule 114 of the
Revised Rules of Court, as amended, that before conviction bail is either a matter of right or of
discretion. It is a matter of right when the offense charged is punishable by any penalty lower than
reclusion perpetua. To that extent the right is absolute. If the offense charged is punishable by
reclusion perpetua bail becomes a matter of discretion. It shall be denied if the evidence of guilt is
strong. The court's discretion is limited to determining whether or not evidence of guilt is strong. But
once it is determined that the evidence of guilt is not strong, bail also becomes a matter of right. If
an accused who is charged with a crime punishable by reclusion perpetua is convicted by the trial
court and sentenced to suffer such a penalty, bail is neither a matter of right on the part of the
accused nor of discretion on the part of the court.

COMMENDADOR VS. DE VILLA


[200 SCRA 80; G.R. NO. 93177; 2 AUG 1991]

Facts:

The petitioners in G.R. Nos. 93177 and 96948 who are officers of the AFP were directed to appear
in person before the Pre-Trial Investigating Officers for the alleged participation the failed coup on
December 1 to 9, 1989. Petitioners now claim that there was no pre-trial investigation of the charges
as mandated by Article of War 71. A motion for dismissal was denied. Now, their motion for
reconsideration. Alleging denial of due process.
Page 172
In G.R. No. 95020, Ltc Jacinto Ligot applied for bail on June 5, 1990, but the application was denied
by GCM No.14. He filed with the RTC a petition for certiorari and mandamus with prayer for
provisional liberty and a writ of preliminary injunction. Judge of GCM then granted the provisional
liberty. However he was not released immediately. The RTC now declared that even military men
facing court martial proceedings can avail the right to bail.

The private respondents in G.R. No. 97454 filed with SC a petition for habeas corpus on the ground
that they were being detained in Camp Crame without charges. The petition was referred to RTC.
Finding after hearing that no formal charges had been filed against the petitioners after more than a
year after their arrest, the trial court ordered their release.

Issue:

Whether or Not there was a denial of due process.

Whether or not there was a violation of the accused right to bail.

Held:

NO denial of due process. Petitioners were given several opportunities to present their side at the
pre-trial investigation, first at the scheduled hearing of February 12, 1990, and then again after the
denial of their motion of February 21, 1990, when they were given until March 7, 1990, to submit
their counter-affidavits. On that date, they filed instead a verbal motion for reconsideration which
they were again asked to submit in writing. They had been expressly warned in the subpoena that
"failure to submit counter-affidavits on the date specified shall be deemed a waiver of their right to
submit controverting evidence." Petitioners have a right to pre-emptory challenge. (Right to
challenge validity of members of G/SCM)

It is argued that since the private respondents are officers of the Armed Forces accused of
violations of the Articles of War, the respondent courts have no authority to order their release and
otherwise interfere with the court-martial proceedings. This is without merit. * The Regional Trial
Court has concurrent jurisdiction with the Court of Appeals and the Supreme Court over petitions for
certiorari, prohibition or mandamus against inferior courts and other bodies and on petitions for
habeas corpus and quo warranto.

The right to bail invoked by the private respondents has traditionally not been recognized and is not
available in the military, as an exception to the general rule embodied in the Bill of Rights. The right
to a speedy trial is given more emphasis in the military where the right to bail does not exist.

On the contention that they had not been charged after more than one year from their arrest, there
was substantial compliance with the requirements of due process and the right to a speedy trial.
The AFP Special Investigating Committee was able to complete the pre-charge investigation only
after one year because hundreds of officers and thousands of enlisted men were involved in the
failed coup.

Accordingly, in G.R. No. 93177, the petition is dismissed for lack of merit. In G.R. No. 96948, the
petition is granted, and the respondents are directed to allow the petitioners to exercise the right of
peremptory challenge under article 18 of the articles of war. In G.R. Nos. 95020 and 97454, the
petitions are also granted, and the orders of the respondent courts for the release of the private
respondents are hereby reversed and set aside. No costs.

MANOTOC VS. COURT OF APPEALS


[142 SCRA 149; G.R. NO. L-62100; 30 MAY 1986]

Facts:
Page 173
Petitioner was charged with estafa. He posted bail. Petitioner filed before each of the trial courts a
motion entitled, "motion for permission to leave the country," stating as ground therefor his desire to
go to the United States, "relative to his business transactions and opportunities." The prosecution
opposed said motion and after due hearing, both trial judges denied the same. Petitioner thus filed a
petition for certiorari and mandamus before the then Court of Appeals seeking to annul the orders
dated March 9 and 26, 1982, of Judges Camilon and Pronove, respectively, as well as the
communication-request of the Securities and Exchange Commission, denying his leave to travel
abroad. He likewise prayed for the issuance of the appropriate writ commanding the Immigration
Commissioner and the Chief of the Aviation Security Command (AVSECOM) to clear him for
departure. The Court of Appeals denied the petition.
Petitioner contends that having been admitted to bail as a matter of right, neither the courts which
granted him bail nor the Securities and Exchange Commission which has no jurisdiction over his
liberty could prevent him from exercising his constitutional right to travel.

Issue:

Whether or Not the right to bail a matter of right.

Held:

The right to bail is a matter of right if the charge is not a capital offense or punishable by reclusion
perpetua to death.

A court has the power to prohibit a person admitted to bail from leaving the Philippines. This is a
necessary consequence of the nature and function of a bail bond.

The condition imposed upon petitioner to make himself available at all times whenever the court
requires his presence operates as a valid restriction on his right to travel.

Indeed, if the accused were allowed to leave the Philippines without sufficient reason, he may be
placed beyond the reach of the courts.

Petitioner has not shown the necessity for his travel abroad. There is no indication that the business
transactions cannot be undertaken by any other person in his behalf.

CALLANTA VS. VILLANUEVA


[77 SCRA 377; G.R. NOS. 24646 & L-24674; 20 JUN 1977]

Facts:

Two complaints for grave oral defamation were filed against Faustina Callanta. The City Judge of
Dagupan City, Felipe Villanueva, denied the motions to quash the complaints. Thus, petitioner
Callanta brought the suits for certiorari in the Supreme Court. Petitioner questions the validity of the
issuance of warrant of arrest by respondent, arguing that the City Fiscal should have conducted the
preliminary investigation. According to petitioner’s counsel, there was jurisdictional infirmity. After
the issuance of the warrants of arrest and the bail fixed at P600, petitioner posted the bail bond,
thus obtaining her provisional liberty. The City Fiscal in this case did not disagree with the judge’s
investigation, and agreed with the complaints filed.

Issue:

Whether or Not petitioner’s contentions are to be given merit.

Held:

Page 174
Based on many precedent cases of the Supreme Court, “where the accused has filed bail and
waived the preliminary investigation proper, he has waived whatever defect, if any, in the
preliminary examination conducted prior to the issuance of the warrant of arrest”. In the case at bar,
it is futile for the petitioner to question the validity of the issuance of the warrant of arrest, because
she posted the bail bond. Petitioner also erred in arguing that only the City Fiscal can conduct a
preliminary investigation. According to the Charter of the City of Dagupan, “the City Court of
Dagupan City may also conduct preliminary investigation for any offense, without regard to the limits
of punishment, and may release, or commit and bind over any person charged with such offense to
secure his appearance before the proper court”. Petition for certiorari is denied. Restraining order
issued by the Court is lifted and set aside.

TATAD VS. SANDIGANBAYAN


[159 SCRA 70; G.R. NOS. L-72335-39; 21 MAR 1988]

Facts:

The complainant, Antonio de los Reyes, originally filed what he termed "a report" with the Legal
Panel of the Presidential Security Command (PSC) on October 1974, containing charges of alleged
violations of Rep. Act No. 3019 against then Secretary of Public Information Francisco S. Tatad.
The "report" was made to "sleep" in the office of the PSC until the end of 1979 when it became
widely known that Secretary (then Minister) Tatad had a falling out with President Marcos and had
resigned from the Cabinet. On December 12, 1979, the 1974 complaint was resurrected in the form
of a formal complaint filed with the Tanodbayan. The Tanodbayan acted on the complaint on April 1,
1980 which was around two months after petitioner Tatad's resignation was accepted by Pres.
Marcos by referring the complaint to the CIS, Presidential Security Command, for investigation and
report. On June 16, 1980, the CIS report was submitted to the Tanodbayan, recommending the
filing of charges for graft and corrupt practices against former Minister Tatad and Antonio L.
Cantero. By October 25, 1982, all affidavits and counter-affidavits were in the case was already for
disposition by the Tanodbayan. However, it was only on June 5, 1985 that a resolution was
approved by the Tanodbayan. Five criminal informations were filed with the Sandiganbayan on June
12, 1985, all against petitioner Tatad alone. (1) Section 3, paragraph (e) of RA. 3019 for giving D'
Group, a private corporation controlled by his brother-in-law, unwarranted benefits, advantage or
preference in the discharge of his official functions; (2) Violation of Section 3, paragraph (b) for
receiving a check of P125,000.00 from Roberto Vallar, President/General Manager of Amity Trading
Corporation as consideration for the release of a check of P588,000.00 to said corporation for
printing services rendered for the Constitutional Convention Referendum in 1973; (3) Violation of
Section 7 on three (3) counts for his failure to file his Statement of Assets and Liabilities for the
calendar years 1973, 1976 and 1978. A motion to quash the information was made alleging that the
prosecution deprived accused of due process of law and of the right to a speedy disposition of the
cases filed against him. It was denied hence the appeal.

Issue:

Whether or not petitioner was deprived of his rights as an accused.

Held:

YES. Due process (Procedural) and right to speedy disposition of trial were violated. Firstly, the
complaint came to life, as it were, only after petitioner Tatad had a falling out with President Marcos.
Secondly, departing from established procedures prescribed by law for preliminary investigation,
which require the submission of affidavits and counter-affidavits by the complainant and the
respondent and their witnesses, the Tanodbayan referred the complaint to the Presidential Security
Command for finding investigation and report. The law (P.D. No. 911) prescribes a ten-day period
for the prosecutor to resolve a case under preliminary investigation by him from its termination.
Page 175
While we agree with the respondent court that this period fixed by law is merely "directory," yet, on
the other hand, it can not be disregarded or ignored completely, with absolute impunity. A delay of
close to three (3) years can not be deemed reasonable or justifiable in the light of the circumstance
obtaining in the case at bar.

GALMAN VS. SANDIGANBAYAN


[144 SCRA 43; G.R. NO.72670; 12 SEP 1986]

Facts:

Assassination of former Senator Benigno "Ninoy" Aquino, Jr. He was killed from his plane that had
just landed at the Manila International Airport. His brain was smashed by a bullet fired point-blank
into the back of his head by an assassin. The military investigators reported within a span of three
hours that the man who shot Aquino (whose identity was then supposed to be unknown and was
revealed only days later as Rolando Galman) was a communist-hired gunman, and that the military
escorts gunned him down in turn.

President was constrained to create a Fact Finding Board to investigate due to large masses of
people who joined in the ten-day period of national mourning yearning for the truth, justice and
freedom.

The fact is that both majority and minority reports were one in rejecting the military version stating
that "the evidence shows to the contrary that Rolando Galman had no subversive affiliations. Only
the soldiers in the staircase with Sen. Aquino could have shot him; that Ninoy's assassination was
the product of a military conspiracy, not a communist plot. Only difference between the two reports
is that the majority report found all the twenty-six private respondents above-named in the title of the
case involved in the military conspiracy; " while the chairman's minority report would exclude
nineteen of them.

Then Pres. Marcos stated that evidence shows that Galman was the killer.

Petitioners pray for issuance of a TRO enjoining respondent court from rendering a decision in the
two criminal cases before it, the Court resolved by nine-to-two votes 11 to issue the restraining
order prayed for. The Court also granted petitioners a five-day period to file a reply to respondents'
separate comments and respondent Tanodbayan a three-day period to submit a copy of his 84-
page memorandum for the prosecution.

But ten days later, the Court by the same nine-to-two-vote ratio in reverse, resolved to dismiss the
petition and to lift the TRO issued ten days earlier enjoining the Sandiganbayan from rendering its
decision. The same Court majority denied petitioners' motion for a new 5-day period counted from
receipt of respondent Tanodbayan's memorandum for the prosecution (which apparently was not
served on them).

Thus, petitioners filed a motion for reconsideration, alleging that the dismissal did not indicate the
legal ground for such action and urging that the case be set for a full hearing on the merits that the
people are entitled to due process.

However, respondent Sandiganbayan issued its decision acquitting all the accused of the crime
charged, declaring them innocent and totally absolving them of any civil liability. Respondents
submitted that with the Sandiganbayan's verdict of acquittal, the instant case had become moot and
academic. Thereafter, same Court majority denied petitioners' motion for reconsideration for lack of
merit.

Hence, petitioners filed their motion to admit their second motion for reconsideration alleging that
respondents committed serious irregularities constituting mistrial and resulting in miscarriage of

Page 176
justice and gross violation of the constitutional rights of the petitioners and the sovereign people of
the Philippines to due process of law.

Issue:

Whether or not petitioner was deprived of his rights as an accused.

Whether or not there was a violation of the double jeopardy clause.

Held:

Petitioners' second motion for reconsideration is granted and ordering a re-trial of the said cases
which should be conducted with deliberate dispatch and with careful regard for the requirements of
due process.

Deputy Tanodbayan Manuel Herrera (made his expose 15 months later when former Pres. was no
longer around) affirmed the allegations in the second motion for reconsideration that he revealed
that the Sandiganbayan Justices and Tanodbayan prosecutors were ordered by Marcos to
whitewash the Aquino-Galman murder case. Malacañang wanted dismissal to the extent that a
prepared resolution was sent to the Investigating Panel. Malacañang Conference planned a
scenario of trial where the former President ordered then that the resolution be revised by
categorizing the participation of each respondent; decided that the presiding justice, Justice
Pamaran, (First Division) would personally handle the trial. A conference was held in an inner room
of the Palace. Only the First Lady and Presidential Legal Assistant Justice Lazaro were with the
President. The conferees were told to take the back door in going to the room where the meeting
was held, presumably to escape notice by the visitors in the reception hall waiting to see the
President. During the conference, and after an agreement was reached, Pres. Marcos told them
'Okay, mag moro-moro na lamang kayo;' and that on their way out of the room Pres. Marcos
expressed his thanks to the group and uttered 'I know how to reciprocate'.

The Court then said that the then President (code-named Olympus) had stage-managed in and
from Malacañang Palace "a scripted and predetermined manner of handling and disposing of the
Aquino-Galman murder case;" and that "the prosecution in the Aquino-Galman case and the
Justices who tried and decided the same acted under the compulsion of some pressure which
proved to be beyond their capacity to resist. Also predetermined the final outcome of the case" of
total absolution of the twenty-six respondents-accused of all criminal and civil liability. Pres. Marcos
came up with a public statement aired over television that Senator Aquino was killed not by his
military escorts, but by a communist hired gun. It was, therefore, not a source of wonder that
President Marcos would want the case disposed of in a manner consistent with his announced
theory thereof which, at the same time, would clear his name and his administration of any
suspected guilty participation in the assassination. such a procedure would be a better arrangement
because, if the accused are charged in court and subsequently acquitted, they may claim the
benefit of the doctrine of double jeopardy and thereby avoid another prosecution if some other
witnesses shall appear when President Marcos is no longer in office.

More so was there suppression of vital evidence and harassment of witnesses. The disappearance
of witnesses two weeks after Ninoy's assassination. According to J. Herrera, "nobody was looking
for these persons because they said Marcos was in power. The assignment of the case to
Presiding Justice Pamaran; no evidence at all that the assignment was indeed by virtue of a regular
raffle, except the uncorroborated testimony of Justice Pamaran himself. The custody of the accused
and their confinement in a military camp, instead of in a civilian jail. The monitoring of proceedings
and developments from Malacañang and by Malacañang personnel. The partiality of
Sandiganbayan betrayed by its decision: That President Marcos had wanted all of the twenty-six
accused to be acquitted may not be denied. In rendering its decision, the Sandiganbayan overdid
itself in favoring the presidential directive. Its bias and partiality in favor of the accused was clearly
obvious. The evidence presented by the prosecution was totally ignored and disregarded.

Page 177
The record shows that the then President misused the overwhelming resources of the government
and his authoritarian powers to corrupt and make a mockery of the judicial process in the Aquino-
Galman murder cases. "This is the evil of one-man rule at its very worst." Our Penal Code penalizes
"any executive officer who shall address any order or suggestion to any judicial authority with
respect to any case or business coming within the exclusive jurisdiction of the courts of justice."

Impartial court is the very essence of due process of law. This criminal collusion as to the handling
and treatment of the cases by public respondents at the secret Malacañang conference (and
revealed only after fifteen months by Justice Manuel Herrera) completely disqualified respondent
Sandiganbayan and voided ab initio its verdict. The courts would have no reason to exist if they
were allowed to be used as mere tools of injustice, deception and duplicity to subvert and suppress
the truth. More so, in the case at bar where the people and the world are entitled to know the truth,
and the integrity of our judicial system is at stake.

There was no double jeopardy. Courts' Resolution of acquittal was a void judgment for having been
issued without jurisdiction. No double jeopardy attaches, therefore. A void judgment is, in legal
effect, no judgment at all. By it no rights are divested. It neither binds nor bars anyone. All acts and
all claims flowing out of it are void.

Motion to Disqualify/Inhibit should have been resolved ahead. In this case, petitioners' motion for
reconsideration of the abrupt dismissal of their petition and lifting of the TRO enjoining the
Sandiganbayan from rendering its decision had been taken cognizance of by the Court which had
required the respondents', including the Sandiganbayan's, comments. Although no restraining order
was issued anew, respondent Sandiganbayan should not have precipitately issued its decision of
total absolution of all the accused pending the final action of this Court. All of the acts of the
respondent judge manifest grave abuse of discretion on his part amounting to lack of jurisdiction
which substantively prejudiced the petitioner.

With the declaration of nullity of the proceedings, the cases must now be tried before an impartial
court with an unbiased prosecutor. Respondents accused must now face trial for the crimes
charged against them before an impartial court with an unbiased prosecutor with all due process.

The function of the appointing authority with the mandate of the people, under our system of
government, is to fill the public posts. Justices and judges must ever realize that they have no
constituency, serve no majority nor minority but serve only the public interest as they see it in
accordance with their oath of office, guided only the Constitution and their own conscience and
honor.

PEOPLE VS. DRAMAYO


[42 SCRA 60; G.R. L-21325; 29 OCT 1971]

Facts:

Dramayo brought up the idea of killing Estelito Nogaliza so that he could not testify in the robbery
case where he is an accused. The idea was for Dramayo and Ecubin to ambush Estelito, who was
returning from Sapao. The others were to station themselves nearby. Only Dramayo and Ecubin
were convicted in the RTC for murder. Hence the appeal

Issue:

Whether or not the accused’s criminal liability proved beyond reasonable doubt.

Held:
Yes. It is to be admitted that the starting point is the Presumption of innocence. So it must be,
according to the Constitution. That is a right safeguarded both appellants. Accusation is not,
according to the fundamental law, synonymous with guilt. It is incumbent on the prosecution
Page 178
demonstrate that culpability lies. Appellants were not even called upon then to offer evidence on
their behalf. Their freedom is forfeit only if the requisite quantum of proof necessary for conviction
be in existence. Their guilt be shown beyond reasonable doubt. What is required then is moral
certainty. "By reasonable doubt is meant that which of possibility may arise, but it is doubt
engendered by an investigation of the whole proof and an inability, after such investigation, to let the
mind rest easy upon the certainty of guilt. Absolute certain of guilt is not demanded by the law to
convict of any carnal charge but moral certainty is required, and this certainty is required as to every
proposition of proof regular to constitute the offense."

The judgment of conviction should not have occasioned any surprise on the part of the two
appellants, as from the evidence deserving of the fullest credence, their guilt had been more than
amply demonstrated. The presumption of innocence could not come to their rescue as it was more
than sufficiently overcome by the proof that was offered by the prosecution. The principal contention
raised is thus clearly untenable. It must be stated likewise that while squarely advanced for the first
time, there had been cases where this Court, notwithstanding a majority of the defendants being
acquitted, the element of conspiracy likewise being allegedly present, did hold the party or parties,
responsible for the offense guilty of the crime charged, a moral certainty having arisen as to their
capability.

DUMLAO VS. COMELEC


[95 SCRA 392; L-52245; 22 JAN 1980]

Facts:

Petitioner Dumlao questions the constitutionality of Sec. 4 of Batas Pambansa Blg 52 as


discriminatory and contrary to equal protection and due process guarantees of the Constitution.
Sec. 4 provides that any retired elective provicial or municipal official who has received payments of
retirement benefits and shall have been 65 years of age at the commencement of the term of office
to which he seeks to be elected, shall not be qualified to run for the same elective local office from
which he has retired. According to Dumlao, the provision amounts to class legislation. Petitioners
Igot and Salapantan Jr. also assail the validity of Sec. 4 of Batas Pambansa Blg 52, which states
that any person who has committed any act of disloyalty to the State, including those amounting to
subversion, insurrection, rebellion, or other similar crimes, shall not be qualified for any of the offices
covered by the act, or to participate in any partisan activity therein: provided that a judgment of
conviction of those crimes shall be conclusive evidence of such fact and the filing of charges for the
commission of such crimes before a civil court or military tribunal after preliminary investigation shall
be prima facie evidence of such fact.

Issue:

Whether or not the aforementioned statutory provisions violate the Constitution and thus, should be
declared null and void

Held:

In regards to the unconstitutionality of the provisions, Sec. 4 of BP Blg 52 remains constitutional and
valid. The constitutional guarantee of equal protection of the laws is subject to rational classification.
One class can be treated differently from another class. In this case, employees 65 years of age are
classified differently from younger employees. The purpose of the provision is to satisfy the “need
for new blood” in the workplace. In regards to the second paragraph of Sec. 4, it should be declared
null and void for being violative of the constitutional presumption of innocence guaranteed to an
accused. “Explicit is the constitutional provision that, in all criminal prosecutions, the accused shall
be presumed innocent until the contrary is proved, and shall enjoy the right to be heard by himself
and counsel (Article IV, section 19, 1973 Constitution). An accusation, according to the fundamental
law, is not synonymous with guilt. The challenged proviso contravenes the constitutional
presumption of innocence, as a candidate is disqualified from running for public office on the ground
Page 179
alone that charges have been filed against him before a civil or military tribunal. It condemns before
one is fully heard. In ultimate effect, except as to the degree of proof, no distinction is made
between a person convicted of acts of dislotalty and one against whom charges have been filed for
such acts, as both of them would be ineligible to run for public office. A person disqualified to run for
public office on the ground that charges have been filed against him is virtually placed in the same
category as a person already convicted of a crime with the penalty of arresto, which carries with it
the accessory penalty of suspension of the right to hold office during the term of the sentence (Art.
44, Revised Penal Code).”

And although the filing of charges is considered as but prima facie evidence, and therefore, may be
rebutted, yet. there is "clear and present danger" that because of the proximity of the elections, time
constraints will prevent one charged with acts of disloyalty from offering contrary proof to overcome
the prima facie evidence against him.
Additionally, it is best that evidence pro and con of acts of disloyalty be aired before the Courts
rather than before an administrative body such as the COMELEC. A highly possible conflict of
findings between two government bodies, to the extreme detriment of a person charged, will thereby
be avoided. Furthermore, a legislative/administrative determination of guilt should not be allowed to
be substituted for a judicial determination.
Being infected with constitutional infirmity, a partial declaration of nullity of only that objectionable
portion is mandated. It is separable from the first portion of the second paragraph of section 4 of
Batas Pambansa Big. 52 which can stand by itself.

Wherefore, the first paragraph of section 4 of Batas pambansa Bilang 52 is hereby declared valid
and that portion of the second paragraph of section 4 of Batas Pambansa Bilang 52 is hereby
declared null and void, for being violative of the constitutional presumption of innocence guaranteed
to an accused.

PEOPLE VS. ALCANTARA


[240 SCRA 122; G.R. NO. 91283; 17 JAN 1995]

Facts:

On July 19, 1988, Venancio Patricio, accompanied by Larry Salvador, drove a ten-wheeler truck a
Coca-Cola plant in Antipolo to load cases of softdrinks. They were about to leave the plant at
10:00pm when several men approached them to hitch for a ride. Ascertaining that Salvador knew
appellant, Venancio accommodated appellant's request. Appellant had four companions. At Ortigas
Ave., one of them poked a gun at Venancio and grabbed the steering wheel. At the North Diversion
Road, Venancio and Salvador(helper) were brought down from the vehicle and tied to the fence of
the expressway, thereafter they were stabbed and left bleeding to death. Venancio survived but
Salvador did not.Appellant was arrested in the vicinity of Otis Street in Pandacan, Manila. A few
days later, he was turned over to the Constabulary Highway Patrol Group. Sgt. Alberto Awanan
brought the appellant to the MCU hospital and was presented to Venancio for identification.
Appellant was brought to the Headquarters at Camp Crame where he confessed.

Appelant’s Defense: Denial and alibi. He said that he was just applying to be a driver and stayed
there even if he was told that no work was available, to confirm with the truck drivers. While he was
applying for CONCEPCION TRUCKING located across Otis street from the Coca-cola plant. He
was arrested. He denied any knowledge of the "hit" on the Coca-cola delivery truck. He remained in
the custody of the police for two days and two nights. On the third day of his detention, he was
turned over to the Constabulary Highway Patrol Group. Appellant was the brought to the MCU
Page 180
hospital. He was made to confront Venancio whom he saw for the first time. CHPG Sgt. Awanan
asked Venancio twice if appellant was among those who hijacked the truck he was driving. On both
times, Venancio did not respond. Undaunted, Sgt. Awanan, called to a photographer present, forced
appellant to stand about a foot from Venancio, and told the latter to just point at the suspect. " Basta
ituro mo lang," Sgt. Awanan directed. Venancio obeyed, and pictures of him pointing to the suspect
were taken. From the hospital, appellant was brought to the Constabulary Highway Patrol Group
headquarters at Camp Crame. Without being apprised of his rights nor provided with counsel, he
was interrogated and urged to confess his guilt. He balked. At ten o'clock that night, hours after
questioning began, appellant's interrogators started boxing him and kicking him. He was also hit on
the back with a chair, and electrocuted. Still, he refused to admit to the crime. In the midst of his
ordeal, appellant heard someone say, "Tubigan na iyan." He was then blindfolded and brought to
another room where he was made to lie down. Water was slowly and continuously poured on his
face, over his mouth. Appellant could no longer bear the pain caused by the water treatment.
Finally, he confessed to being one of the hijackers. He was led to another room, where he was
handcuffed and left until the following day. Later, he was made to sign prepared statements
containing his full confession.

Alcantara was arraigned under an information charging him and four others (at large) with the crime
of robbery with Homicide and Frustrated Homicide.

The trial court convicted the accused despite the following inconsistency between Venancio’s
affidavit and testimony:

Affidavit Testimony
1. mentioned 5 assailants - only Alcantara was identified
2. stabbing was preceded by a - only 3 assailants had a
3. conference by all assailants - conference
4. claimed to have allowed assailants - failed to identify Alcantara
5. to hitch a ride because Alcantara - at the hospital and in open
6. was familiar to them - court (pointed to another person)

Issue:

Whether or not the rights of the accused was violated.

Held:

YES. The people’s evidence failed to meet the quantum required to overcome the presumption. The
second identification which correctly pointed to accused by Venancio should not be credited. There
is no reason for him to err as they know each other for 3 years. It was also incorrect to give too
much weight to Police Sgt. Awanan’s testimony as to the “previous identification” at the hospital.
The testimony of Sgt. Awanan was not corroborated by Venancio.

The identification procedure was irregular. Due process demands that the identification procedure of
criminal suspects must be free from impermissible suggestions as the influence of improper
suggestion probably accounts for more miscarriages of justice than any other single factor.
Conviction must be based on the strength of the prosecution and not the weakness of the defense.
There was blatant violation of the constitutional rights of appellant as an accused. Appellant belongs
to the economically deprived in our society. He is nearly illiterate(third grade education). Our
Constitution and our laws strictly ordain their protection following the Magsaysay desideratum that
those who have less in life should have more in law.

CORPUZ VS. REPUBLIC


[194 SCRA 73; G.R. NO. 74259; 14 FEB 1991]
Page 181
Facts:

Generoso Corpuz is the Supervising Accounting Clerk in the Office of the Provincial Treasurer of
Nueva Viscaya. He was designated Acting Supervising Cashier in the said office. In this capacity,
he received collections, disbursed funds and made bank deposits and withdrawals pertaining to
government accounts. On April 13, 1981 his designation as Acting Supervising Cashier was
terminated and a transfer of accountabilities was effected between him and his successor. The
Certificate of turnover revealed a shortage of P72,823.00. He was able to pay only P10,159.50.
After a final demand letter for the total of P50,596.07 which was not met, a case of malversation
was filed against him. Corpuz did not deny such facts but he insists that the shortage was
malversed by other persons. He alleged that Paymaster Diosdado Pineda through 1 of 4 separate
checks (PNB) issued and encashed such checks while he was of leave. Also, Acting Deputy
Provincial Treasurer Bernardo Aluning made to post the amount on his cashbook although he had
not received the said amount. He was convicted in Sandiganbayan.

Issue:

Whether or Not Corpuz is guilty of malversation.

Held:

It is a subtle way of camouflaging the embezzlement of the money equivalent when 1 of the 4
checks issued and encashed in the same day was entered in the accused’s cash book 3 months
after such encashments. Also, Corpuz claim that he was absent when Paymaster Diosdado Pineda
through 1 of 4 separate checks (PNB) issued and encashed such checks, was not proven.

Post-Audit is not a preliminary requirement to filing a malversation case. The failure of the public
officer to have duly forthcoming any public funds with which he is chargeable, upon demand by an
authorized officer shall be a prima facie evidence that he has put such missing funds to personal
use.

The equipoise rule(balancing test) which is the presumption of innocence is applicable only where
the evidence of the parties is evenly balance, in which case the scale of justice should be tilt in favor
of the accused. There is no such balance in the case at bar. The evidence of the prosecution is
overwhelming and has not been overcome by the petitioner with his claims. The presumed
innocence must yield to the positive finding that he is guilty of malversation.

Wherefore his petition is denied. He is guilty as principal of Malversation of Public Funds.

PEOPLE VS. HOLGADO


[85 PHIL 752; G.R.L-2809; 22 MAR 1950]

Facts:

Appellant Frisco Holgado was charged in the court of First Instance of Romblon with slight illegal
detention because according to the information, being a private person, he did "feloniously and
without justifiable motive, kidnap and detain one Artemia Fabreag in the house of Antero Holgado
for about eight hours thereby depriving said Artemia Fabreag of her personal liberty. He pleaded
guilty (without a counsel) and said that he was just instructed by Mr. Ocampo, which no evidence
was presented to indict the latter.

Issue:

Whether or Not there was any irregularity in the proceedings in the trial court.

Page 182
Held:

Yes. Rule 112, section 3 of ROC that : “If the defendant appears without attorney, he must be
informed by the court that it is his right to have attorney being arraigned., and must be asked if he
desires the aid of attorney, the Court must assign attorney de oficio to defend him. A reasonable
time must be allowed for procuring attorney.” This was violated. Moreso the guarantees of our
Constitution that "no person shall be held to answer for a criminal offense without due process of
law", and that all accused "shall enjoy the right to be heard by himself and counsel." In criminal
cases there can be no fair hearing unless the accused be given the opportunity to be heard by
counsel.

The trial court failed to inquire as to the true import of the qualified plea of accused. The record does
not show whether the supposed instructions of Mr. Ocampo was real and whether it had reference
to the commission of the offense or to the making of the plea guilty. No investigation was opened by
the court on this matter in the presence of the accused and there is now no way of determining
whether the supposed instruction is a good defense or may vitiate the voluntariness of the
confession. Apparently the court became satisfied with the fiscal's information that he had
investigated Mr. Ocampo and found that the same had nothing to do with this case. Such attitude of
the court was wrong for the simple reason that a mere statement of the fiscal was not sufficient to
overcome a qualified plea of the accused. But above all, the court should have seen to it that the
accused be assisted by counsel especially because of the qualified plea given by him and the
seriousness of the offense found to be capital by the court.

PEOPLE VS. MAGSI


[124 SCRA 64; G.R. NO.L-32888; 12 AUG 1983]

Facts:

Soon after appellant was apprehended on August 20, 1970, his arraignment was scheduled before
the Criminal Circuit Court of San Fernando, La Union. The case was actually set and rescheduled
for six (6) times, first of which was on August 1, 1970. On that date, despite appointment by the
court of Atty. Mario Rivera as de officio counsel for the accused, hearing was re-set to September 8,
1970 on motion of Atty. Rivera, who was prompted to ask for it because of accused desire to be
represented by a de parte counsel. Prior to the next hearing, Atty. Rivera moved to withdraw as de
officio counsel and it was favorably acted on by the court on September 7, 1970. At the second
hearing on September 8, 1970, for failure of the de officio and de parte counsels to appear, despite
a second call of the case, the hearing was re-set for the next day and the court appointed Atty.
Dominador Cariaso de officio counsel for the accused. On the third hearing date, neither the de
parte nor the de officio counsel was in Court, so Atty. Rivera was reappointed that day as de officio
counsel for arraignment purposes only. The accused del Rosario entered a plea of guilty but
qualified it with the allegation that he committed the crime out of fear of his co-accused Eloy Magsi
and the other coaccused. Appellant was found guilty of murder and made to suffer the death
penalty.

Issue:

Whether or not there was a violation of the rights of the accused.

Held:

YES. The desire to speed up the disposition of cases should not be effected at the sacrifice of the
basic rights of the accused. Citing People vs. Domingo (55 SCRA 243-244): the trial courts should
exercise solicitous care before sentencing the accused on a plea of guilty especially in capital
offenses by first insuring that the accused fully understands the gravity of the offense, the severity of
the consequences attached thereto as well as the meaning and significance of his plea of guilty;
and that the prudent and proper thing to do in capital cases is to take testimony, to assure the court
Page 183
that the accused has not misunderstood the nature and effect of his plea of guilty. Mere pro-forma
appointment of de officio counsel, who fails to genuinely protect the interests of the accused,
resetting of hearing by the court for alleged reception of evidence when in fact none was conducted,
perfunctory queries addressed to the accused whether he understands the charges and the gravity
of the penalty, are not sufficient compliance.

SORIANO VS. SANDIGANBAYAN


[131 SCRA 184; G.R. NO.L-65952; 31 JUL 1984]

Facts:

Tan was accused of qualified theft. The petitioner, who was an Asst. Fiscal, was assigned to
investigate. In the course of the investigation, petitioner demanded Php.4000 from Tan as price for
dismissing the case. Tan reported it to the NBI which set up an entrapment. Tan was given a
Php.2000, marked bill, and he had supplied the other half. The entrapment succeeded and an
information was filed with the Sandiganbayan. After trial, the Sandiganbayan rendered a decision
finding the petitioner guilty as a principal in violating the Anti Graft and Corrupt Practices Act
(R.A.3019). A motion for reconsideration was denied by the Sandiganbayan, hence this instant
petition.

Issue:

Whether or Not the investigation conducted by the petitioner can be regarded as contract or
transaction within the purview of .RA.3019.

Held:

R.A. 3019 Sec.3. Corrupt practices of public officers - In addition to acts or omissions of public
officers already penalized by existing laws, the following shall constitute corrupt practices of any
public officer and are hereby declared to be unlawful: xxx b. Directly or indirectly requesting or
receiving any gift, present, share percentage or benefit, for himself or for other person, in
connection with any contract or transaction between the Govt. and any other party wherein the
public officer in his official capacity has to intervene under the law.

The petitioner stated that the facts make out a case of direct bribery under Art.210 of the RPC and
not a violation of R.A. 3019 sec.3 (b). The offense of direct bribery is not the offense charged and is
not included in the offense charged which is violation of R.A.3019 sec.3 (b).

The respondent claimed that, transaction as used hereof, is not limited to commercial or business
transaction, but includes all kinds of transaction whether commercial, civil, or administrative in
nature.

The court agrees with the petitioner. It is obvious that the investigation conducted by the petitioner
was neither a contract nor transaction. A transaction like a contract is one which involves some
consideration as in credit transactions. And this element is absent in the investigation conducted by
the petitioner.

Judgment modified. Petitioner is guilty of direct bribery under Art.210 of the RPC.

Page 184
BORJA VS. MENDOZA
[77 SCRA 422; G.R. NO.L-45667; 20 JUN 1977]

Facts:

Borja was accused of slight physical injuries in the City of Cebu. However, he was not arraigned.
That not withstanding, respondent Judge Senining proceeded with the trial in absentia and rendered
a decision finding petitioner guilty of the crime charged. The case was appealed to the Court o First
Instance in Cebu presided by respondent Judge Mendoza. It was alleged that the failure to arraign
him is a violation of his constitutional rights. It was also alleged that without any notice to petitioner
and without requiring him to submit his memorandum, a decision on the appealed case was
rendered The Solicitor General commented that the decision should be annulled because there was
no arraignment.

Issue:

Whether or Not petitioner’s constitutional right was violated when he was not arraigned.

Held:

Yes. Procedural due process requires that the accused be arraigned so that he may be informed as
to why he was indicted and what penal offense he has to face, to be convicted only on a showing
that his guilt is shown beyond reasonable doubt with full opportunity to disprove the evidence
against him. It is also not just due process that requires an arraignment. It is required in the Rules
that an accused, for the first time, is granted the opportunity to know the precise charge that
confronts him. It is imperative that he is thus made fully aware of possible loss of freedom, even of
his life, depending on the nature of the crime imputed to him. At the very least then, he must be fully
informed of why the prosecuting arm of the state is mobilized against him. Being arraigned is thus a
vital aspect of the constitutional rights guaranteed him. Also, respondent Judge Senining convicted
petitioner notwithstanding the absence of an arraignment. With the violation of the constitutional
right to be heard by himself and counsel being thus manifest, it is correct that the Solicitor General
agreed with petitioner that the sentence imposed on him should be set aside for being null. The
absence of an arraignment can be invoked at anytime in view of the requirements of due process to
ensure a fair and impartial trial.

Wherefore, the petition for certiorari is granted. The decision of respondent Judge Romulo R.
Senining dated December 28, 1973, finding the accused guilty of the crime of slight physical
injuries, is nullified and set aside. Likewise, the decision of respondent Judge Rafael T. Mendoza
dated November 16, 1976, affirming the aforesaid decision of Judge Senining, is nullified and set
aside. The case is remanded to the City Court of Cebu for the prosecution of the offense of slight
physical injuries, with due respect and observance of the provisions of the Rules of Court, starting
with the arraignment of petitioner.

CONDE VS. RIVERA


[45 PHIL 650; G.R. NO. 21741; 25 JAN 1924]

Facts:

Aurelia Conde, formerly a municipal midwife in Lucena, Tayabas, has been forced to respond to no
less the five information for various crimes and misdemeanors, has appeared with her witnesses
Page 185
and counsel at hearings no less than on eight different occasions only to see the cause postponed,
has twice been required to come to the Supreme Court for protection, and now, after the passage of
more than one year from the time when the first information was filed, seems as far away from a
definite resolution of her troubles as she was when originally charged.

Issue:

Whether or Not petitioner has been denied her right to a speedy and impartial trial.

Held:

Philippine organic and statutory law expressly guarantee that in all criminal prosecutions the
accused shall enjoy the right to have a speedy trial. Aurelia Conde, like all other accused persons,
has a right to a speedy trial in order that if innocent she may go free, and she has been deprived of
that right in defiance of law. We lay down the legal proposition that, where a prosecuting officer,
without good cause, secures postponements of the trial of a defendant against his protest beyond a
reasonable period of time, as in this instance for more than a year, the accused is entitled to relief
by a proceeding in mandamus to compel a dismissal of the information, or if he be restrained of his
liberty, by habeas corpus to obtain his freedom.

PEOPLE VS. TAMPAL


[244 SCRA 202; G.R. NO. 102485; 22 MAY 1995]

Facts:

Luis Tampal, Domingo Padumon, Arsenio Padumon, Samuel Padumon, Pablito Suco, Dario Suco
and Galvino Cadling were charged of robbery with homicide and multiple serious physical injuries in
the Regional Trial Court of Zamboanga with Hon. Wilfredo Ochotorena as presiding judge.
However, only private respondents, Luis Tampal, Domingo Padumon, Arsenio Padumon, and
Samuel Padumon were arrested, while the others remained at large.

The case was set for hearing on July 26, 1991, but Assistant Provincial Prosecutor Wilfredo
Guantero moved for postponement due to his failure to contact the material witnesses. The case
was reset without any objection from the defense counsel. The case was called on September 20,
1991 but the prosecutor was not present. The respondent judge considered the absence of the
prosecutor as unjustified, and dismissed the criminal case for failure to prosecute. The prosecution
filed a motion for reconsidereation, claiming that his absence was because such date was a Muslim
holiday and the office of the Provincial prosecutor was closed on that day. The motion was denied
by respondent judge.

Issue:

Whether or Not the postponement is a violation of the right of the accused to a speedy disposition of
their cases.

Whether or Not the dismissal serves as a bar to reinstatement of the case.

Held:

In determining the right of an accused to speedy disposition of their case, courts should do more
than a mathematical computation of the number of postponements of the scheduled hearings of the
case. What are violative of the right of the accused to speedy trial are unjustified postponements
which prolong trial for an unreasonable length of time. In the facts above, there was no showing that
there was an unjust delay caused by the prosecution, hence, the respondent judge should have
given the prosecution a fair opportunity to prosecute its case.

Page 186
The private respondents cannot invoke their right against double jeopardy. In several cases it was
held that dismissal on the grounds of failure to prosecute is equivalent to an acquittal that would bar
another prosecution for the same offense, but in this case, this does not apply, considering that the
rights of the accused to a speedy trial was not violated by the State. Therefore, the order of
dismissal is annulled and the case is remanded to the court of origin for further proceedings.

REPUBLIC ACT NO. 8493 “THE SPEEDY TRIAL ACT”

The arraignment of an accused shall be held within 30 days from filing of the information, or from
the date the accused has appeared before the justice, judge or court in which the charge is pending,
whichever date last occurs. Thereafter, where a plea of not guilty is entered, the accused shall have
at least 15 days to prepare for trial. Trial shall commence within 30 days from arraignment as fixed
by the court. In no case shall the entire trial period exceed 180 days from the 1 st day of trial, except
as otherwise authorized by the Chief Justice of the Supreme Court.

RE: REQUEST FOR LIVE TV OF TRIAL OF JOSEPH ESTRADA


[360 SCRA 248; A.M. NO 01-4-03-SC; 29 JUN 2001]

Facts:

The Kapisanan ng mga Brodkaster ng Pilipinas (KBP) sent a letter requesting the Court to allow live
media coverage of the anticipated trial of the plunder and other criminal cases filed against former
President Joseph E. Estrada before the Sandiganbayan in order "to assure the public of full
transparency in the proceedings of an unprecedented case in our history." The request was
seconded by Mr. Cesar N. Sarino and, still later, by Senator Renato Cayetano and Attorney Ricardo
Romulo.

Issue:

Whether or Not live media coverage of the trial of the plunder and other criminal cases filed against
former President Joseph E. Estrada should be permitted by the court.

Held:

The propriety of granting or denying the instant petition involve the weighing out of the constitutional
guarantees of freedom of the press and the right to public information, on the other hand, along with
the constitutional power of a court to control its proceedings in ensuring a fair and impartial trial.
When these rights race against one another, jurisprudence tells us that the right of the accused
must be preferred to win.

Due process guarantees the accused a presumption of innocence until the contrary is proved in a
trial that is not lifted about its individual settings nor made an object of public’s attention and where
the conclusions reached are induced not by any outside force or influence but only be evidence and
argument given in open court, where fitting dignity and calm ambiance is demanded.

An accused has a right to a public trial but it is a right that belongs to him, more than anyone else,
where his life or liberty can be held critically in balance. A public trial aims to ensure that he is fairly
dealt with and would not be unjustly condemned and that his rights are not compromised in secret
conclaves of long ago. A public trial is not synonymous with publicized trial, it only implies that the
court doors must be open to those who wish to come, sit in the available seats, conduct themselves
with decorum and observe the trial process.

The courts recognize the constitutionally embodied freedom of the press and the right to public
information. It also approves of media's exalted power to provide the most accurate and
comprehensive means of conveying the proceedings to the public. Nevertheless, within the
Page 187
courthouse, the overriding consideration is still the paramount right of the accused to due process
which must never be allowed to suffer diminution in its constitutional proportions.

PEOPLE VS. SALAS


[143 SCRA 163; G.R. NO. L-66469; 29 JUL 1986]

Facts:

At about 6:00 o'clock in the morning of March 6, 1992, a 60 year old woman, identified as Virginia
Talens was found lying dead in a canal at Bo. San Nicolas, Mexico, Pampanga; she was last seen
alive at about 3:00 o'clock early morning of March 6, 1992 by Orlando Pangan and Richard Pangan
who were with her going home coming from the wake of one Leonardo Flores; both Orlando and
Richard Pangan testified that accused was with them in going home at about 3:00 o'clock in the
morning of March 6, 1992; Orlando and Richard Pangan reached first their house and left the two
on the way and that was the last time Virginia was seen alive; just a few minutes after reaching his
house and while inside his house, Orlando Pangan heard a shout; another woman, one Serafia
Gutierrez, testified that she likewise was awakened by a shout at about 3:00 in the morning; Dr.
Aguda who autopsied the victim found hematoma on the head and chest, an abrasion on the left
chin and stabwound on the neck which stabwound, the doctor claims, was the cause of death of the
victim; Police Investigator Gonzales who immediately responded upon report, recovered at the
scene a pin, the victim's wristwatch, earring, a ring and P135.00 money; he likewise found on March
9, 1992 when he continued his investigation bloodstain on the front door of the house of the
accused which bloodstain when submitted for examination was found to be of human blood; one
Resultay was with Virginia Talens at about 5:00 afternoon of March 5, 1992 in going to the wake,
who claims that Virginia had money on a purse as while they were on the way Virginia bet on a
jueteng she saw Virginia got money from her purse a P500.00 bill but as she had no change she
instead took P8.00 from her other pocket; one Ramil Talens, a son of the victim corroborated the
claim of Resultay that Virginia had with her at that time money worth P2,000.00 as in the morning of
March 5, 1992 he gave her mother for safekeeping the sum of P1,500.00 which he claims his
mother placed in her purse and claims further that at the wake, he asked and was given P50.00 by
his mother as he also participated in the gambling thereat, however, the purse of Virginia containing
about P2,000.00 was no longer to be found when she was found dead; Orlando Pangan saw the
accused gambled in the wake; Virginia likewise gambled at the wake; accused had been working for
three days before March 6 at Sta. Ana, Pampanga and up to March 5, 1992, but the following day,
he did not anymore report for work at Sta. Ana, Pampanga, was no longer to be found and was last
seen at about 3:00 morning together with Virginia Talens on their way home coming from the wake;
the parents of [the] accused were informed by Investigator Gonzales that their son was the suspect
and adviced them to surrender him, but since March 6, 1992 when accused left Mexico, Pampanga,
he returned only on September 19, 1992 at Arayat, Pampanga, not at Mexico, Pampanga where he
was ultimately apprehended by the Mexico Police on September 22, 1992 after chancing on a radio
message by the police of Arayat to their Provincial commander that a vehicular incident occurred at
Arayat, Pampanga where one Elmer Salas was the victim and was hospitalized at the district
hospital at Arayat, Pampanga where he used the name of Rommel Salas and not Elmer Salas. The
trial court rendered convicting Salas for Robbery with Homicide

Issue:

Whether or Not there is evidence sufficient to sustain a conviction of the appellant of the crime of
Robbery with Homicide.

Whether or Not the appellant’s crime homicide or robbery with homicide.

Held:

There was no eyewitness or direct evidence; either to the robbery or to the homicide and none of
the things allegedly stolen were ever recovered. However, direct evidence is not the only matrix
Page 188
from which the trial court may draw its findings and conclusion of culpability. Resort to circumstantial
evidence is essential when to insist on direct testimony would result in setting felons free.

For circumstantial evidence to be sufficient to support a conviction, all the circumstances must be
consistent with each other, consistent with the theory that the accused is guilty of the offense
charged, and at the same time inconsistent with the hypothesis that he is innocent and with every
other possible, rational hypothesis excepting that of guilt. All the circumstances established must
constitute an unbroken chain which leads to one and fair and reasonable conclusion pointing solely
to the accused, to the exclusion of all other persons, as the author of the crime. The facts and
circumstances consistent with the guilt of the accused and inconsistent with his innocence can
constitute evidence which, in weight and probative value, may be deemed to surpass even direct
evidence in its effect on the court.

The fatal stabbing of Virginia Talens occurred at around 3:00 a.m. of March 6, 1992. Appellant
hastily abandoned his house in Barrio San Nicolas, Mexico, Pampanga, his residence since
childhood, on that very date. Appellant was nowhere when his co-worker and barrio mate, Eduardo
Bagtas, came to appellant's house to fetch him for work at around 6:30 to 7:00 a.m. of March 6,
1992. Appellant also abandoned his job as a painter in Sta. Ana, Pampanga, on March 6, 1992, the
date of the crime, leaving behind an unfinished painting project. He was not seen again from said
date. Police investigators found human bloodstains on the front door of appellant's house, on his
clothing, and on his yellow slippers after the victim was killed. Despite efforts of the police to find
appellant as the principal suspect, a fact known to appellant's family and neighbors, appellant did
not present himself to the authorities. Appellant was apprehended only a full six months after the
date of the crime, following his confinement in a hospital in Arayat, Pampanga because he was
sideswiped by a Victory Liner bus in Arayat. When hospitalized, appellant used the alias Rommel
Salas, instead of his true name Elmer Salas. These circumstances denote flight, which when
unexplained, has always been considered by the courts as indicative of guilt.

Both appellant and victim gambled at the wake they attended. The victim was, in fact, enjoying a
winning streak when her son, Ramil Talens, came to fetch her but which he failed to do because his
mother was winning, and she refused to leave. The purse of Talens containing cash was gone when
her corpse was found in the canal with a stab wound and bruises. What was left was a safety pin
which victim used to fasten the missing purse to her clothes.

Denial is an inherently weak defense which must be buttressed by strong evidence of non-
culpability to merit credibility. Denial is negative and self-serving and cannot be given greater
evidentiary weight over the testimonies of credible witnesses who positively testified that appellant
was at the locus criminis and was the last person seen with the victim alive.

The absence of evidence showing any improper motive on the part of the principal witness for the
prosecution to falsely testify against the appellant strongly tends to buttress the conclusion that no
such improper motive exists and that the testimony of said witnesses deserve full faith and credit.

The essence of voluntary surrender is spontaneity and the intent of the accused to give himself up
and submit himself unconditionally to the authorities either because he acknowledges his guilt or he
wants to save the State the trouble of having to effect his arrest. Spontaneity and intent to give
one's self up are absent where the accused went into hiding for six months after the incident and
had to resort to an alias when he was involved in an accident being investigated by the police
authorities.

Robbery with Homicide is a special complex crime against property. Homicide is incidental to the
robbery which is the main purpose of the criminal. In charging Robbery with Homicide, the onus
probandi is to establish: "(a) the taking of personal property with the use of violence or intimidation
against a person; (b) the property belongs to another; (c) the taking is characterized with animus
lucrandi; and (d) on the occasion of the robbery or by reason thereof, the crime of homicide, which
is used in the generic sense, was committed." Although there was no witness as to the actual

Page 189
robbing of the victim, there is testimony that the victim had more or less P2,000.00; and wore gold
earrings valued at P750.00. These were never recovered.

While there is indeed no direct proof that Virginia Talens was robbed at the time she was killed, we
may conclude from four circumstances that the robbery occasioned her killing: (1) Both appellant
and victim gambled at the wake. (2) The appellant knew that victim was winning. (3) The victim was
last seen alive with appellant. (4) The victim's purse containing her money and earrings were
missing from her body when found.

The decision of the regional trial court is affirmed. Costs against appellant. So ordered.

PEOPLE VS. MAGPALAO


[197 SCRA 79; G.R. NO. 92415; 14 MAY 1991]

Facts:

Eleven (11) people rode in a Ford Fiera going to Baguio. Namely they are: Felizardo Galvez, Jimmy
Jetwani, Simeon Calama, Rene Salonga, Eduardo Lopez, Adolfo Quiambao, Aliman Bara-akal,
Anwar Hadji Edris, Gumanak Ompa and defendant-appelants in this case, Omar Magpalao and Rex
Magumnang.

After an hour of driving, the car stopped so that one of the passengers could urinate. While the car
was stopped the Bara-akal, Edris, Ompa, Magpalao and Magumnang pointed guns and knives at
the other passengers and divested them of their properties.

On of the robbers then ordered Galvez to drive the car towards the precipice (bangin). When the car
was near the precipice, Galvez then stepped to the brakes. The other passengers jumped out of the
car and went to different directions to escape. Galvez however, was left in side the car and was
stabbed by one of the robbers. The robbers then escaped. Quiambao, who owned the car helped
Galvez to get to a hospital. Galvez died in the hospital. The robbers were then apprehended with
the exception of Edris who remain at large. Mangumnang however escaped while being in detention
and Bara-akal died inside the jail. Since Mangumnang was not arrested, the trial in absentia
continued as to him. Ompa, Magpalao, and Magumnang were all held guilty as principal by direct
participation of the crime of Robbery with Homicide.

Issue:

Whether or Not the lower court erred in failing to apply the Constitutional mandate on the
presumption of innocence and proof beyond reasonable doubt when it allowed the trial in absentia
to push through on the part of defendant-appellant Magumnang.

Held:

The Court affirmed the decision of the lower court. The reason is that the lower court has jurisdiction
over Magumnang the moment the latter was in custody. Jurisdiction once acquired is not lost upon
the instance of parties but until the case is terminated. Since all the requisites of trial in absentia are
complete, the court has jurisdiction over Magumnang.

In addition, Magumnang was presumed innocent during his trial in absentia. The prosecution had
strong evidence against him as proof beyond reasonable doubt that he is a principal by direct
participation in the crime of Robbery with Homicide. Thus, the Constitutional mandate was not
violated.

PEOPLE VS. ACABAL


[226 SCRA 694 ; G.R. NO. 103604, 23 SEP 1993]
Page 190
Facts:

The accusatory portion in the information for murder. Facts are as follows:

"That sometime in the evening of the 28th of January, 1980, at Nagbinlod, Municipality of Sta.
Catalina, Province of Negros Oriental, Philippines, and within the jurisdiction of this Honorable
Court, the accused, including several 'John Does', conspiring and confederating with one another,
with intent to kill, and with treachery and evident premeditation and being then armed with bolos and
'pinuti', did then and there willfully, unlawfully and feloniously attack, assault and use personal
violence on the person of one Rizalina Apatan Silvano while the latter was about to leave her house
and inflicting upon her injuries, to wit: 'right leg amputated below the knee; left leg hacked behind
the knee; abdomen hacked with viscerae evacerated,' and did then and there set the house on fire
while the aforementioned Rizalina Apatan Silvano was inside said house trying to escape therefrom,
and allowing her to be burned inside said house which was burned to the ground, thereby causing
upon said Rizalina Apatan Silvano her death and burning her beyond recognition.

But on 16 May 1987, a fire gutted the building where Branch 37 was located and the records of
these two cases were burned. The records were subsequently reconstituted upon petition of the
prosecuting fiscal. The testimonies of the witnesses were retaken, however, before it could
commence, accused Engracio Valeriano jumped bail and the warrant for his arrest issued on 16
November 1987 was returned unserved because he could not be found. An alias warrant for his
arrest was issued on 26 June 1989, but he remains at large up to the present.

After the completion of the re-taking of the testimonies of the witnesses in Branch 37, Criminal
Cases Nos. 4584 and 4585 were re-raffled to Branch 33 of the trial court, then presided over by
Judge Pacifico S. Bulado.

The decision of the trial court, per Judge Pacifico S. Bulado, dated 31 October 1991 but
promulgated on 20 December 1991, contained no specific dispositive portion. Its rulings are found
in the last two paragraphs which read as follows:

"The elements of murder in this case, Criminal Case No. 4585 for the killing of Rizalina Apatan-
Silvano having been proved by the prosecution beyond doubt, the accused JUANITO RISMUNDO,
MACARIO ACABAL and ABUNDIO NAHID, considering the attendant qualifying aggravating
circumstances of nighttime, use of fire by burning the house of victim Rizalina Apatan-Silvano in
order to forcibly drive her out of her house and hack her to death, the abuse of superior strength,
the penalty impossable [sic] here will be in its maximum degree, that is reclusion perpetua taking
into account Article 248 of the Revised Penal Code, the penalty now for murder is Reclusion
Temporal to Reclusion Perpetua, and for all the accused to indemnify the heirs of the victim the sum
of Thirty Thousand (P30,000.00) Pesos since this case occurred [sic] in 1980. For the wounding of
the victim Wilson A. Silvano, this Court believes that simple frustrated homicide only is committed by
the accused Engracio Valeriano only.

But since the person who actually inflicted the injuries of victim Wilson Silvano, accused Engracio
Valeriano only is nowhere to be found, hence, not brought to the bar of justice, he being a fugitive or
at large, no penalty could be imposed on him since he is beyond the jurisdiction of this court to
reach. All the other two (2) accused, JUANITO RISMUNDO and ABUNDIO NAHID are hereby
ordered and declared absolved from any criminal responsibility from frustrated homicide.

The bail bond put up by the three accused, namely: Juanito Rismundo, Macario Acabal and
Abundio Nahid are hereby ordered cancelled and let a warrant of arrest be issued for their
immediate confinement."

Issue:

Whether or not the judgment complied with the Rules of Court.


Page 191
Whether or not the cancellation of the bail bonds of the accused is valid.

Whether or not the accused may be tried in absentia.

Whether or not the accused is guilty of the crime of frustrated murder.

Held:

We find that the decision substantially complies with the Rules of Court on judgments as it did
sentence the accused-appellants to reclusion perpetua. A judgment of conviction shall state (a) the
legal qualification of the offense constituted by the acts committed by the accused, and the
aggravating or mitigating circumstances attending the commission, if there are any; (b) the
participation of the accused in the commission of the offense, whether as principal, accomplice or
accessory after the fact; (c) the penalty imposed upon the accused; and (d) the civil liability or
damages caused by the wrongful act to be recovered from the accused by the offended party, if
there is any, unless the enforcement of the civil liability by a separate action has been reserved or
waived.

It is obvious that they clearly understood that they were found guilty beyond reasonable doubt of the
crime of murder and were sentenced to suffer the penalty of reclusion perpetua in Criminal Case
No. 4585. Were it otherwise, they would not have declared in open court their intention to appeal
immediately after the promulgation of the decision and would not have subsequently filed their
written notice of appeal.

Accused-appellants contend that the trial court did not impose any sentence and so cannot cancel
anymore their bail bonds and direct their arrest and immediate commitment because it already lost
jurisdiction over their persons when they perfected their appeal.

The decision did impose the penalty of reclusion perpetua. Since the order cancelling their bail
bonds and directing their arrest is contained in the decision itself, it is apparent that their
abovementioned contention is highly illogical. At the time the order in question was made, the trial
court still had jurisdiction over the persons of the accused-appellants.

The trial court further erred in holding that no penalty could be imposed on accused Engracio
Valeriano in Criminal Case No. 4584 because he "is nowhere to be found, hence, not brought to the
bar of justice, he being a fugitive or at large." The court ignored the fact that Engracio jumped bail
after he had been arraigned, just before the retaking of evidence commenced. Paragraph (2),
Section 14, Article III of the Constitution permits trial in absentia after the accused has been
arraigned provided he has been duly notified of the trial and his failure to appear thereat is
unjustified. One who jumps bail can never offer a justifiable reason for his non-appearance during
the trial.

Accordingly, after the trial in absentia, the court can render judgment in the case and promulgation
may be made by simply recording the judgment in the criminal docket with a copy thereof served
upon his counsel, provided that the notice requiring him to be present at the promulgation is served
through his bondsmen or warden and counsel.

In conclusion, because of reasonable doubt as to their guilt, the accused-appellants must be


acquitted. Every accused is presumed innocent until the contrary is proved; that presumption is
solemnly guaranteed by the Bill of Rights. The contrary requires proof beyond reasonable doubt, or
that degree of proof which produces conviction in an unprejudiced mind. Short of this, it is not only
the right of the accused to be freed; it is even the constitutional duty of the court to acquit him.

US VS. TAN TENG


[23 PHIL 145; G.R. NO. 7081; 7 SEP 1912]
Page 192
Facts:

The defendant herein raped Oliva Pacomio, a seven-year-old girl. Tan Teng was gambling near the
house of the victim and it was alleged that he entered her home and threw the victim on the floor
and place his private parts over hers. Several days later, Pacomio was suffering from a disease
called gonorrhea. Pacomio told her sister about what had happened and reported it to the police.

Tan Teng was called to appear in a police line-up and the victim identified him. He was then
stripped of his clothing and was examined by a policeman. He was found to have the same
symptoms of gonorrhea. The policeman took a portion of the substance emitting from the body of
the defendant and turned it over to the Bureau of Science. The results showed that the defendant
was suffering from gonorrhea.

The lower court held that the results show that the disease that the victim had acquired came from
the defendant herein. Such disease was transferred by the unlawful act of carnal knowledge by the
latter. The defendant alleged that the said evidence should be inadmissible because it was taken in
violation of his right against self-incrimination.

Issue:

Whether or Not the physical examination conducted was a violation of the defendant’s rights against
self-incrimination.

Held:

The court held that the taking of a substance from his body was not a violation of the said right. He
was neither compelled to make any admissions or to answer any questions. The substance was
taken from his body without his objection and was examined by competent medical authority.

The prohibition of self-incrimination in the Bill of Rights is a prohibition of the use of physical or
moral compulsion to extort communications from him, and not an exclusion of his body as evidence,
when it may be material. It would be the same as if the offender apprehended was a thief and the
object stolen by him may be used as evidence against him.

VILLAFLOR VS. SUMMERS


[41 PHIL 62; G.R. NO. 16444; 8 SEP 1920]

Facts:

Petitioner Villaflor was charged with the crime of adultery. The trial judge ordered the petitioner to
subject herself into physical examination to test whether or not she was pregnant to prove the
determine the crime of adultery being charged to her. Herein petitioner refused to such physical
examination interposing the defense that such examination was a violation of her constitutional
rights against self-incrimination.

Issue:

Whether or Not the physical examination was a violation of the petitioner’s constitutional rights
against self-incrimination.

Held:

No. It is not a violation of her constitutional rights. The rule that the constitutional guaranty, that no
person shall be compelled in any criminal case to be a witness against himself, is limited to a

Page 193
prohibition against compulsory testimonial self-incrimination. The corollary to the proposition is that,
an ocular inspection of the body of the accused is permissible.

BELTRAN VS. SAMSON


[53 PHIL 570; G.R. NO. 32025; 23 SEPT 1929]

Facts:

Beltran, as a defendant for the crime of Falsification, refused to write a sample of his handwriting as
ordered by the respondent Judge. The petitioner in this case contended that such order would be a
violation of his constitutional right against self-incrimination because such examination would give
the prosecution evidence against him, which the latter should have gotten in the first place. He also
argued that such an act will make him furnish evidence against himself.

Issue:

Whether or not the writing from the fiscal's dictation by the petitioner for the purpose of comparing
the latter's handwriting and determining whether he wrote certain documents supposed to be
falsified, constitutes evidence against himself within the scope and meaning of the constitutional
provision under examination.

Held:

The court ordered the respondents and those under their orders desist and abstain absolutely and
forever from compelling the petitioner to take down dictation in his handwriting for the purpose of
submitting the latter for comparison. Writing is something more than moving the body, or the hands,
or the fingers; writing is not a purely mechanical act, because it requires the application of
intelligence and attention; and in the case at bar writing means that the petitioner herein is to furnish
a means to determine whether or not he is the falsifier, as the petition of the respondent fiscal
clearly states. Except that it is more serious, we believe the present case is similar to that of
producing documents or chattels in one's possession. We say that, for the purposes of the
constitutional privilege, there is a similarity between one who is compelled to produce a document,
and one who is compelled to furnish a specimen of his handwriting, for in both cases, the witness is
required to furnish evidence against himself. It cannot be contended in the present case that if
permission to obtain a specimen of the petitioner's handwriting is not granted, the crime would go
unpunished. Considering the circumstance that the petitioner is a municipal treasurer, it should not
be a difficult matter for the fiscal to obtained genuine specimens of his handwriting. But even
supposing it is impossible to obtain specimen or specimens without resorting to the means
complained herein, that is no reason for trampling upon a personal right guaranteed by the
constitution. It might be true that in some cases criminals may succeed in evading the hand of
justice, but such cases are accidental and do not constitute the raison d' etre of the privilege. This
constitutional privilege exists for the protection of innocent persons.

PASCUAL VS. BME


[28 SCRA 345; G.R. NO. 25018; 26 MAY 1969]

Facts:

Petitioner Arsenio Pascual, Jr. filed an action for prohibition against the Board of Medical
Examiners. It was alleged therein that at the initial hearing of an administrative case for alleged
immorality, counsel for complainants announced that he would present as his first witness the
petitioner. Thereupon, petitioner, through counsel, made of record his objection, relying on the
constitutional right to be exempt from being a witness against himself. Petitioner then alleged that to
compel him to take the witness stand, the Board of Examiners was guilty, at the very least, of grave
abuse of discretion for failure to respect the constitutional right against self-incrimination.
Page 194
The answer of respondent Board, while admitting the facts stressed that it could call petitioner to the
witness stand and interrogate him, the right against self-incrimination being available only when a
question calling for an incriminating answer is asked of a witness. They likewise alleged that the
right against self-incrimination cannot be availed of in an administrative hearing.

Petitioner was sustained by the lower court in his plea that he could not be compelled to be the first
witness of the complainants, he being the party proceeded against in an administrative charge for
malpractice. Hence, this appeal by respondent Board.

Issue:

Whether or Not compelling petitioner to be the first witness of the complainants violates the Self-
Incrimination Clause.

Held:

The Supreme Court held that in an administrative hearing against a medical practitioner for alleged
malpractice, respondent Board of Medical Examiners cannot, consistently with the self-incrimination
clause, compel the person proceeded against to take the witness stand without his consent. The
Court found for the petitioner in accordance with the well-settled principle that "the accused in a
criminal case may refuse, not only to answer incriminatory questions, but, also, to take the witness
stand." If petitioner would be compelled to testify against himself, he could suffer not the forfeiture of
property but the revocation of his license as a medical practitioner. The constitutional guarantee
protects as well the right to silence: "The accused has a perfect right to remain silent and his silence
cannot be used as a presumption of his guilt." It is the right of a defendant "to forego testimony, to
remain silent, unless he chooses to take the witness stand — with undiluted, unfettered exercise of
his own free genuine will."

The reason for this constitutional guarantee, along with other rights granted an accused, stands for
a belief that while crime should not go unpunished and that the truth must be revealed, such
desirable objectives should not be accomplished according to means or methods offensive to the
high sense of respect accorded the human personality. More and more in line with the democratic
creed, the deference accorded an individual even those suspected of the most heinous crimes is
given due weight. The constitutional foundation underlying the privilege is the respect a government
... must accord to the dignity and integrity of its citizens.

PEOPLE VS. BALISACAN


[17 SCRA 1119; G.R. NO. L-26376; 31 AUG 1966]

Facts:

Aurelio Balisacan was charged with homicide in the CFI of Ilocos Norte. Upon being arraigned, he
entered into a plea of guilty. In doing so, he was assisted y counsel. At his counsel de officio, he
was allowed to present evidence and consequently testified that he stabbed the deceased in self-
defense. In addition, he stated that he surrendered himself voluntarily to the police authorities. On
the basis of the testimony of the accused, he was acquitted. Thus, the prosecution appealed.

Issue:

Whether or Not the appeal placed the accused in double jeopardy.

Held:

The Supreme Court held that it is settled that the existence of plea is an essential requisite to
double jeopardy. The accused had first entered a plea of guilty but however testified that he acted in
Page 195
complete self-defense. Said testimony had the effect of vacating his plea of guilty and the court a
quo should have required him to plead a new charge, or at least direct that a new plea of not guilty
be entered for him. This was not done. Therefore, there has been no standing of plea during the
judgment of acquittal, so there can be no double jeopardy with respect to the appeal herein.

PEOPLE VS. OBSANIA


[23 SCRA 1249; G.R. L-24447; 29 JUN 1968]

Facts:

The accused was charged with Robbery with Rape before the Municipal Court of Balungao,
Pangasinan. He pleaded not guilty. His counsel moved for the dismissal of the charge for failure to
allege vivid designs in the info. Said motion was granted. From this order of dismissal the
prosecution appealed.

Issue:

Whether or Not the present appeal places the accused in Double Jeopardy.

Held:

In order that the accused may invoke double jeopardy, the following requisites must have obtained
in the original prosecution, a) valid complaint, b) competent court, c) the defendant had pleaded to
the charge, d) defendant was acquitted or convicted or the case against him was dismissed or
otherwise terminated without his express consent.

In the case at bar, the converted dismissal was ordered by the Trial Judge upon the defendant's
motion to dismiss. The “doctrine of double jeopardy” as enunciated in P.vs. Salico applies to
wit when the case is dismissed with the express consent of the defendant, the dismissal will not be
a bar to another prosecution for the same offense because his action in having the case is
dismissed constitutes a waiver of his constitutional right/privilege for the reason that he thereby
prevents the Court from proceeding to the trial on the merits and rendering a judgment of conviction
against him.

In essence, where a criminal case is dismissed provisionally not only with the express consent of
the accused but even upon the urging of his counsel there can be no double jeopardy under Sect. 9
Rule 113, if the indictment against him is revived by the fiscal.

PAULIN VS. GIMENEZ


[217 SCRA 386; G.R. NO. 103323; 21 JAN 1993]

Facts:

Respondent and Brgy Capt. Mabuyo, while in a jeep, were smothered with dust when they were
overtaken by the vehicle owned by Petitioner Spouses. Irked by such, Mabuyo followed the vehicle
until the latter entered the gate of an establishment. He inquired the nearby security guard for the
identity of the owner of the vehicle. Later that day, while engaged in his duties, petitioners allegedly
pointed their guns at him. Thus, he immediately ordered his subordinate to call the police and block
road to prevent the petitioners’ escape. Upon the arrival of the police, petitioners put their guns
down and were immediately apprehended.

A complaint “grave threats” was filed against the petitioners (Criminal Case No. 5204). It was
dismissed by the court acting on the motion of the petitioners. Mabuyo filed a MOR thus the
dismissal was reversed. Thereafter, petitioners filed for “certiorari, prohibition, damages, with relief
Page 196
of preliminary injunction and the issuance of a TRO” (CEB-9207). Petition is dismissed for lack of
merit and for being a prohibited pleading and ordered to proceed with the trial of the case. Hence,
this instant petition.

Issue:

Whether or Not the dismissal of 5204 was a judgment of acquittal.

Whether or Not the judge ignored petitioner’s right against double jeopardy by dismissing CEB-
9207.

Held:

For double jeopardy to attach, the dismissal of the case must be without the express consent of the
accused. Where the dismissal was ordered upon motion or with the express assent of the accused,
he has deemed to have waived his protection against double jeopardy. In the case at bar, the
dismissal was granted upon motion of the petitioners. Double jeopardy thus did not attach.

Furthermore, such dismissal is not considered as an acquittal. The latter is always based on merit
that shows that the defendant is beyond reasonable doubt not guilty. While the former, in the case
at bar, terminated the proceedings because no finding was made as to the guilt or innocence of the
petitioners.

The lower court did not violate the rule when it set aside the order of dismissal for the reception of
further evidence by the prosecution because it merely corrected its error when it prematurely
terminated and dismissed the case without giving the prosecution the right to complete the
presentation of its evidence. The rule on summary procedure was correctly applied.

PEOPLE VS. COURT OF SILAY


[74 SCRA 248; G.R. NO. L-43790; 9 DEC 1976]

Facts:

That sometime on January 4,1974, accused Pacifico Sensio, Romeo Millan and Wilfredo Jochico
who were then scalers at the Hawaiian-Philippine Company, weighed cane cars No.1743,1686 and
1022 loaded with sugar canes which were placed in tarjetas (weight report cards), Apparently, it
was proven and shown that there was padding of the weight of the sugar canes and that the
information on the tarjetas were to be false making it appear to be heavier than its actual weight.
The three accused then were charged with “Falsification by private individuals and use of falsified
document”. After the prosecution had presented, the respondent moved to dismiss the charge
against them on the ground that the evidences presented were not sufficient to establish their guilt
beyond reasonable doubt. Acting on the motion, respondent court issued its order dismissing the
case on the ground that the acts committed by the accused do not constituted the crime of
falsification as strictly enumerated in the revised penal code defining the crime of falsification which
was charged earlier and that their case be dismissed. People asserts that the plea of double
jeopardy is not tenable even if the case at bar was dismissed because according to them, it was
done with the consent of the accused therefore waiving there defense of double jeopardy. The
accused on the other hand, reiterated the fact that the dismissal was due to lack of merits of the
prosecution which would have the same effect as an acquittal which will bar the prosecution from
prosecuting the accused for it will be unjust and unconstitutional for the accused due to double
jeopardy rule thus the appeal of the plaintiff.

Issue:

Whether or Not the grant of petition by the court would place the accused Sensio, Millan and
Jochico in double jeopardy
Page 197
Held:

Yes the revival of the case will put the accused in double jeopardy for the very reason that the case
has been dismissed earlier due to lack of merits. It is true that the criminal case of falsification was
dismissed on a motion of the accused however this was a motion filed after the prosecution had
rested its case, calling for the evidence beyond reasonable ground which the prosecution had not
been able to do which would be tantamount to acquittal therefore will bar the prosecution of another
case. As it was stated on the requirements of a valid defense of double jeopardy it says: That
there should be a valid complaint, second would be that such complaint be filed before a competent
court and to which the accused has pleaded and that defendant was previously acquitted, convicted
or dismissed or otherwise terminated without express consent of the accused in which were all
present in the case at bar. There was indeed a valid, legitimate complaint and concern against the
accused Sensio, Millan and Jochico which was filed at a competent court with jurisdiction on the
said case. It was also mentioned that the accused pleaded not guilty and during the time of trial, it
was proven that the case used against the accused were not sufficient to prove them guilty beyond
reasonable doubt therefore dismissing the case which translates to acquittal. It explained further
that there are two instances when we can conclude that there is jeopardy when first is that the
ground for the dismissal of the case was due to insufficiency of evidence and second, when the
proceedings have been reasonably prolonged as to violate the right of the accused to a speedy trial.
In the 2 requisites given, it was the first on that is very much applicable to our case at bar where
there was dismissal of the case due to insufficiency of evidence which will bar the approval of the
petition in the case at bar for it will constitute double jeopardy on the part of the accused which the
law despises.

PEOPLE VS. RELOVA


[149 SCRA 292; G.R. NO.L-45129; 6 MAR 1987]

FACTS: In this petition for certiorari and mandamus, People of the Philippines seeks to set aside
the orders of Respondent Judge Hon. Relova quashing an information for theft filed against Mr.
Opulencia on the ground of double jeopardy and denying the petitioner’s motion for
reconsideration.. On Feb.1 1975, Batangas police together with personnel of Batangas Electric
Light System, equipped with a search warrant issued by a city judge of Batangas to search and
examine the premises of the Opulencia Carpena Ice Plant owned by one Manuel Opulencia. They
discovered electric wiring devices have been installed without authority from the city government
and architecturally concealed inside the walls of the building. Said devices are designed purposely
to lower or decrease the readings of electric current consumption in the plant’s electric meter. The
case was dismissed on the ground of prescription for the complaint was filed nine months prior to
discovery when it should be 2months prior to discovery that the act being a light felony and
prescribed the right to file in court. On Nov 24, 1975, another case was filed against Mr. Opulencia
by the Assistant City Fiscal of Batangas for a violation of a Batangas Ordinance regarding
unauthorized electrical installations with resulting damage and prejudice to City of Batangas in the
amount of P41,062.16. Before arraignment, Opulencia filed a motion to quash on the ground of
double jeopardy. The Assistant fiscal’s claim is that it is not double jeopardy because the first
offense charged against the accused was unauthorized installation of electrical devices without the
approval and necessary authority from the City Government which was punishable by an ordinance,
where in the case was dismissed, as opposed to the second offense which is theft of electricity
which is punishable by the Revised Penal Code making it a different crime charged against the 1 st
complaint against Mr.Opulencia.

Issue:

Whether or Not the accused Mr. Opulencia can invoke double jeopardy as defense to the second
offense charged against him by the assistant fiscal of Batangas on the ground of theft of electricity
punishable by a statute against the Revised Penal Code.

Page 198
Held:

Yes, Mr. Opulencia can invoke double jeopardy as defense for the second offense because as
tediously explained in the case of Yap vs Lutero, the bill of rights give two instances or kinds of
double jeopardy. The first would be that “No person shall be twice put in jeopardy of punishment for
the same offense and the second sentence states that “If an act is punishable by a law or an
ordinance, the conviction or acquittal shall bar to another prosecution for the same act”. In the case
at bar, it was very evident that the charges filed against Mr. Opulencia will fall on the 2 nd kind or
definition of double jeopardy wherein it contemplates double jeopardy of punishment for the same
act. It further explains that even if the offenses charged are not the same, owing that the first
charge constitutes a violation of an ordinance and the second charge was a violation against the
revised penal code, the fact that the two charges sprung from one and the same act of conviction or
acquittal under either the law or the ordinance shall bar a prosecution under the other thus making it
against the logic of double jeopardy. The fact that Mr. Opulencia was acquitted on the first offense
should bar the 2nd complaint against him coming from the same identity as that of the 1 st offense
charged against Mr.Opulencia.

ESMENA VS. POGOY


[102 SCRA 861; G.R. NO. L-54110; 20 FEB 1981]

Facts:

Petitioners Esmeña and Alba were charged with grave coercion in the Court of Cebu City for
allegedly forcing Fr. Thomas Tibudan to withdraw a sum of money worth P5000 from the bank to be
given to them because the priest lost in a game of chance. During arraignment, petitioners pleaded
“Not Guilty”. No trial came in after the arraignment due to the priest’s request to move it on another
date. Sometime later Judge Pogoy issued an order setting the trial Aug.16,1979 but the fiscal
informed the court that it received a telegram stating that the complainant was sick. The accused
invoked their right to speedy trial. Respondent judge dismissed the case because the trial was
already dragging the accused and that the priest’s telegram did not have a medical certificate
attached to it in order for the court to recognize the complainant’s reason to be valid in order to
reschedule again another hearing. After 27 days the fiscal filed a motion to revive the case and
attached the medical certificate of the priest proving the fact that the priest was indeed sick of
influenza. On Oct.24,1979, accused Esmeña and Alba filed a motion to dismiss the case on the
ground of double jeopardy.

Issue:

Whether or Not the revival of grave coercion case, which was dismissed earlier due to
complainant’s failure to appear at the trial, would place the accused in double jeopardy

Held:

Yes, revival of the case will put the accused in double jeopardy for the very reason that the case
has been dismissed already without the consent of the accused which would have an effect of an
acquittal on the case filed. The dismissal was due to complainant’s incapability to present its
evidence due to non appearance of the witnesses and complainant himself which would bar further
prosecution of the defendant for the same offense. For double jeopardy to exist these three
requisites should be present, that one, there is a valid complaint or information filed second, that it is
done before a court of competent jurisdiction and third, that the accused has been arraigned and
has pleaded to the complaint or information. In the case at bar, all three conditions were present,
as the case filed was grave coercion, filed in a court of competent jurisdiction as to where the
coercion took place and last the accused were arraigned and has pleaded to the complaint or the
information. When these three conditions are present then the acquittal, conviction of the accused,
and the dismissal or termination of the case without his express consent constitutes res judicata and
is a bar to another prosecution for the offense charged. In the case, it was evidently shown that the
Page 199
accused invoked their right to a speedy trial and asked for the trial of the case and not its
termination which would mean that respondents had no expressed consent to the dismissal of the
case which would make the case filed res judicata and has been dismissed by the competent court
in order to protect the respondents as well for their right to speedy trial which will be equivalent to
acquittal of the respondents which would be a bar to further prosecution.

PEOPLE VS. DE LA TORRE


[380 SCRA 586; G.R. NOS. 137953-58; 11 MAR 2002]

Facts:

Wilfredo dela Torre, appellee, has three children from a common-law relationship, the eldest of
which is Mary Rose. When Mary Rose was 7 yearsold, her mother left them together with her
youngest brother so she and her other brother were left to the care of her father.

Mary Rose was the brightest in her class despite their poverty. However, in January 1997, a
sudden change in Mary Rose’s behavior behavior was noticed. She was twelve years old at that
time. She appeared sleepy, snobbish and she also urinated on her panty. When confronted by her
head teacher, Mary Rose admitted that she was abused repeatedly by her father. Her father,
however, denied vehemently the charges being imputed to him by her daughter.

The RTC convicted appellee of two counts of acts of lasciviousness and four counts of murder.
However, the RTC refused to impose the supreme penalty of death on appellee. It maintained that
there were circumstances that mitigated the gravity of the offenses such as the absence of any
actual physical violence or intimidation on the commission of the acts, that after the mother of Mary
Rose left the conjugal home, for more than five years, Wilfredo, Mary Rose and her brother were
living together as a family and Mary Rose was never molested by her father.

The prosecution seeks to modify the RTC Decision by imposing the supreme penalty of death of the
accused. It argues that it has proven that the victim is the daughter of the accused, and that she
was below eighteen years old when the rapes took place. As a consequence, the trial court should
have been imposed the penalty of death pursuant to Section 11 of R.A. 7659. .

Issue:

Whether or Not the Court erred in penalizing the appellee with reclusion perpetua in each of the four
indictments of rape, instead of imposing the supreme penalty of death as mandated by R.A. 7659.

Held:

Under Section 1, Rule 122 of the 2000 Rules of Criminal Procedure, any party may appeal from a
judgment or final order unless the accused will be put in double jeopardy. In People vs. Leones, it
declared that:

“while it is true that this Court is the Court of last resort, there are allegations of error
committed by a lower court which we ought not to look into to uphold the right of the
accused. Such is the case in an appeal by the prosecution seeking to increase the
penalty imposed upon the accused for this runs afoul of the right of the accused
against double jeopardy…When the accused after conviction by the trial court did not
appeal his decision, an appeal by the government seeking to increase the penalty
imposed by the trial court places the accused in double jeopardy and should
therefore be dismissed.”

The ban on double jeopardy primarily prevents the State from using its criminal processes as an
instrument of harassment to wear out the accused by a multitude of cases with accumulated trials.
It also serves as a deterrent from successively retrying the defendant in the hope of securing a
Page 200
conviction. And finally, it prevents the State, following conviction, from retrying the defendant again
in the hope of securing a greater penalty.

Being violative of the right against double jeopardy, the appeal of the prosecution cannot prosper.

Page 201
CITIZENSHIP

Page 202
CITIZENSHIP

Art. 4

Sec. 1. The following are citizens of the Philippines:


(1) Those who are citizens of the Philippines at the time of the adoption of this Constitution;
(2) Those whose fathers or mothers are citizens of the Philippines;
(3) Those born before January 17, 1973, of Filipino mothers, who elect Philippine citizenship
upon reaching the age of majority; and
(4) Those who are naturalized in accordance with law.

Sec. 2. Natural-born citizens are those who are citizens of the Philippines from birth without having
to perform any act to acquire or perfect their Philippine citizenship. Those who elect Philippine
citizenship in accordance with paragraph (3), Section 1 hereof shall be deemed natural-born
citizens.

Sec. 3. Philippine citizenship may be lost or reacquired in the manner provided by law.

Sec. 4. Citizens of the Philippines who marry aliens shall retain their citizenship, unless by their act
or omission they are deemed, under the law, to have renounced it.

Sec. 5. Dual allegiance of citizens is inimical to the national interest and shall be dealt with by law.

FRIVALDO VS. COMELEC


[174 SCRA 245; G.R. NO. 87193; 23 JUN 1989]

Facts:

Petitioner Juan G. Frivaldo was proclaimed governor-elect of the province of Sorsogon on January
22, 1988, and assumed office in due time. On October 27, 1988, the League of Municipalities,
Sorsogon Chapter, represented by its President, Estuye, who was also suing in his personal
capacity, filed with the COMELEC a petition for the annulment of Frivaldo; election and
proclamation on the ground that he was not a Filipino citizen, having been naturalized in the United
States on January 20, 1983. In his answer dated May 22, 1988, Frivaldo admitted that he was
naturalized in the United States as alleged but pleaded the special and affirmative defenses that he
had sought American citizenship only to protect himself against President Marcos. His
naturalization, he said, was "merely forced upon himself as a means of survival against the
unrelenting persecution by the Martial Law Dictator's agents abroad." He added that he had
returned to the Philippines after the EDSA revolution to help in the restoration of democracy. In their
Comment, the private respondents reiterated their assertion that Frivaldo was a naturalized
American citizen and had not reacquired Philippine citizenship on the day of the election on January
18, 1988. He was therefore not qualified to run for and be elected governor. They also argued that
their petition in the Commission on Elections was not really for quo warranto under Section 253 of
the Omnibus Election Code. The ultimate purpose was to prevent Frivaldo from continuing as
governor, his candidacy and election being null and void ab initio because of his alienage. Speaking
for the public respondent, the Solicitor General supported the contention that Frivaldo was not a
citizen of the Philippines and had not repatriated himself after his naturalization as an American
citizen. As an alien, he was disqualified from public office in the Philippines. His election did not cure
this defect because the electorate of Sorsogon could not amend the Constitution, the Local
Government Code, and the Omnibus Election Code. He also joined in the private respondent's
argument that Section 253 of the Omnibus Election Code was not applicable because what the
League and Estuye were seeking was not only the annulment of the proclamation and election of
Frivaldo. He agreed that they were also asking for the termination of Frivaldo's incumbency as
governor of Sorsogon on the ground that he was not a Filipino.

Page 203
Issue:

Whether or Not petitioner Juan G. Frivaldo was a citizen of the Philippines at the time of his election
on January 18, 1988, as provincial governor of Sorsogon.

Held:

The reason for this inquiry is the provision in Article XI, Section 9, of the Constitution that all public
officials and employees owe the State and the Constitution "allegiance at all times" and the specific
requirement in Section 42 of the Local Government Code that a candidate for local elective office
must be inter alia a citizen of the Philippines and a qualified voter of the constituency where he is
running. Section 117 of the Omnibus Election Code provides that a qualified voter must be, among
other qualifications, a citizen of the Philippines, this being an indispensable requirement for suffrage
under Article V, Section 1, of the Constitution.

In the certificate of candidacy he filed on November 19, 1987, Frivaldo described himself as a
"natural-born" citizen of the Philippines, omitting mention of any subsequent loss of such status. The
evidence shows, however, that he was naturalized as a citizen of the United States in 1983 per the
following certification from the United States District Court, Northern District of California, as duly
authenticated by Vice Consul Amado P. Cortez of the Philippine Consulate General in San
Francisco, California, U.S.A.

The Court sees no reason not to believe that the petitioner was one of the enemies of the Marcos
dictatorship. Even so, it cannot agree that as a consequence thereof he was coerced into
embracing American citizenship. His feeble suggestion that his naturalization was not the result of
his own free and voluntary choice is totally unacceptable and must be rejected outright.

This Court will not permit the anomaly of a person sitting as provincial governor in this country while
owing exclusive allegiance to another country. The fact that he was elected by the people of
Sorsogon does not excuse this patent violation of the salutary rule limiting public office and
employment only to the citizens of this country. The qualifications prescribed for elective office
cannot be erased by the electorate alone. The will of the people as expressed through the ballot
cannot cure the vice of ineligibility, especially if they mistakenly believed, as in this case, that the
candidate was qualified. Obviously, this rule requires strict application when the deficiency is lack of
citizenship. If a person seeks to serve in the Republic of the Philippines, he must owe his total
loyalty to this country only, abjuring and renouncing all fealty and fidelity to any other state.

It is true as the petitioner points out that the status of the natural-born citizen is favored by the
Constitution and our laws, which is all the more reason why it should be treasured like a pearl of
great price. But once it is surrendered and renounced, the gift is gone and cannot be lightly
restored. This country of ours, for all its difficulties and limitations, is like a jealous and possessive
mother. Once rejected, it is not quick to welcome back with eager arms its prodigal if repentant
children. The returning renegade must show, by an express and unequivocal act, the renewal of his
loyalty and love.

Petition Dismissed. Petitioner JUAN G. FRIVALDO is hereby declared not a citizen of the
Philippines and therefore disqualified from serving as Governor of the Province of Sorsogon.
Accordingly, he is ordered to vacate his office and surrender the same to the duly elected Vice-
Governor of the said province once this decision becomes final and executory.

MERCADO VS. MANZANO


[307 SCRA 630; G.R. NO. 135083; 26 MAY 1999]

Facts:

Petitioner Ernesto Mercado and Private respondent Eduardo Manzano are candidates for the
Page 204
position of Vice-Mayor of Makati City in the May, 1998 elections. Private respondent was the winner
of the said election but the proclamation was suspended due to the petition of Ernesto Mamaril
regarding the citizenship of private respondent. Mamaril alleged that the private respondent is not a
citizen of the Philippines but of the United States. COMELEC granted the petition and disqualified
the private respondent for being a dual citizen, pursuant to the Local Government code that
provides that persons who possess dual citizenship are disqualified from running any public
position. Private respondent filed a motion for reconsideration which remained pending until after
election. Petitioner sought to intervene in the case for disqualification. COMELEC reversed the
decision and declared private respondent qualified to run for the position. Pursuant to the ruling of
the COMELEC, the board of canvassers proclaimed private respondent as vice mayor. This petition
sought the reversal of the resolution of the COMELEC and to declare the private respondent
disqualified to hold the office of the vice mayor of Makati.

Issue:

Whether or Not private respondent is qualified to hold office as Vice-Mayor.

Held:

Dual citizenship is different from dual allegiance. The former arises when, as a result of the
concurrent application of the different laws of two or more states, a person is simultaneously
considered a national by the said states. For instance, such a situation may arise when a person
whose parents are citizens of a state which adheres to the principle of jus sanguinis is born in a
state which follows the doctrine of jus soli. Private respondent is considered as a dual citizen
because he is born of Filipino parents but was born in San Francisco, USA. Such a person, ipso
facto and without any voluntary act on his part, is concurrently considered a citizen of both states.
Considering the citizenship clause (Art. IV) of our Constitution, it is possible for the following classes
of citizens of the Philippines to posses dual citizenship: (1) Those born of Filipino fathers and/or
mothers in foreign countries which follow the principle of jus soli; (2) Those born in the Philippines of
Filipino mothers and alien fathers if by the laws of their fathers’ country such children are citizens of
that country; (3) Those who marry aliens if by the laws of the latter’s country the former are
considered citizens, unless by their act or omission they are deemed to have renounced Philippine
citizenship. Dual allegiance, on the other hand, refers to the situation in which a person
simultaneously owes, by some positive act, loyalty to two or more states. While dual citizenship is
involuntary, dual allegiance is the result of an individual’s volition.

By filing a certificate of candidacy when he ran for his present post, private respondent elected
Philippine citizenship and in effect renounced his American citizenship. The filing of such certificate
of candidacy sufficed to renounce his American citizenship, effectively removing any disqualification
he might have as a dual citizen.

By declaring in his certificate of candidacy that he is a Filipino citizen; that he is not a permanent
resident or immigrant of another country; that he will defend and support the Constitution of the
Philippines and bear true faith and allegiance thereto and that he does so without mental
reservation, private respondent has, as far as the laws of this country are concerned, effectively
repudiated his American citizenship and anything which he may have said before as a dual citizen.
On the other hand, private respondent’s oath of allegiance to the Philippine, when considered with
the fact that he has spent his youth and adulthood, received his education, practiced his profession
as an artist, and taken part in past elections in this country, leaves no doubt of his election of
Philippine citizenship.

TECSON VS. COMELEC


[424 SCRA 277; G.R. No. 161434; 3 Mar 2004]

Facts:

Page 205
Victorino X. Fornier, petitioner initiated a petition before the COMELEC to disqualify FPJ and to
deny due course or to cancel his certificate of candidacy upon the thesis that FPJ made a material
misrepresentation in his certificate of candidacy by claiming to be a natural-born Filipino citizen
when in truth, according to Fornier, his parents were foreigners; his mother, Bessie Kelley Poe, was
an American, and his father, Allan Poe, was a Spanish national, being the son of Lorenzo Pou, a
Spanish subject. Granting, petitioner asseverated, that Allan F. Poe was a Filipino citizen, he could
not have transmitted his Filipino citizenship to FPJ, the latter being an illegitimate child of an alien
mother. Petitioner based the allegation of the illegitimate birth of respondent on two assertions -
first, Allan F. Poe contracted a prior marriage to a certain Paulita Gomez before his marriage to
Bessie Kelley and, second, even if no such prior marriage had existed, Allan F. Poe, married Bessie
Kelly only a year after the birth of respondent.

Issue:

Whether or Not FPJ is a natural born Filipino citizen.

Held:

It is necessary to take on the matter of whether or not respondent FPJ is a natural-born citizen,
which, in turn, depended on whether or not the father of respondent, Allan F. Poe, would have
himself been a Filipino citizen and, in the affirmative, whether or not the alleged illegitimacy of
respondent prevents him from taking after the Filipino citizenship of his putative father. Any
conclusion on the Filipino citizenship of Lorenzo Pou could only be drawn from the presumption that
having died in 1954 at 84 years old, Lorenzo would have been born sometime in the year 1870,
when the Philippines was under Spanish rule, and that San Carlos, Pangasinan, his place of
residence upon his death in 1954, in the absence of any other evidence, could have well been his
place of residence before death, such that Lorenzo Pou would have benefited from the "en masse
Filipinization" that the Philippine Bill had effected in 1902. That citizenship (of Lorenzo Pou), if
acquired, would thereby extend to his son, Allan F. Poe, father of respondent FPJ. The 1935
Constitution, during which regime respondent FPJ has seen first light, confers citizenship to all
persons whose fathers are Filipino citizens regardless of whether such children are legitimate or
illegitimate.

But while the totality of the evidence may not establish conclusively that respondent FPJ is a
natural-born citizen of the Philippines, the evidence on hand still would preponderate in his favor
enough to hold that he cannot be held guilty of having made a material misrepresentation in his
certificate of candidacy in violation of Section 78, in relation to Section 74, of the Omnibus Election
Code.

BENGZON VS. HRET


[357 SCRA 545; G. R. No. 142840; 7 May 2001]

Facts:

Respondent Teodoro Cruz was a natural-born citizen of the Philippines. He was born in San
Clemente, Tarlac, on April 27, 1960, of Filipino parents. The fundamental law then applicable was
the 1935 Constitution. On November 5, 1985, however, respondent enlisted in the United States
Marine Corps and without the consent of the Republic of the Philippines, took an oath of allegiance
to the United States. As a Consequence, he lost his Filipino citizenship for under Commonwealth
Act No. 63, section 1(4), a Filipino citizen may lose his citizenship by, among other, "rendering
service to or accepting commission in the armed forces of a foreign country.” He was naturalized in
US in 1990. On March 17, 1994, respondent Cruz reacquired his Philippine citizenship through
repatriation under Republic Act No. 2630. He ran for and was elected as the Representative of the

Page 206
Second District of Pangasinan in the May 11, 1998 elections. He won over petitioner Antonio
Bengson III, who was then running for reelection.

Issue:

Whether or Not respondent Cruz is a natural born citizen of the Philippines in view of the
constitutional requirement that "no person shall be a Member of the House of Representative unless
he is a natural-born citizen.”

Held:

Respondent is a natural born citizen of the Philippines. As distinguished from the lengthy process of
naturalization, repatriation simply consists of the taking of an oath of allegiance to the Republic of
the Philippine and registering said oath in the Local Civil Registry of the place where the person
concerned resides or last resided. This means that a naturalized Filipino who lost his citizenship will
be restored to his prior status as a naturalized Filipino citizen. On the other hand, if he was originally
a natural-born citizen before he lost his Philippine citizenship, he will be restored to his former status
as a natural-born Filipino.

Page 207

You might also like